Download as pdf or txt
Download as pdf or txt
You are on page 1of 225

lOMoARcPSD|20277325

TA10.TỔNG TẬP ĐỀ THI Olympic 30-4 2010-2014 P4

Academic English (Trường Đại học Kinh tế Thành phố Hồ Chí Minh)

Scan to open on Studocu

Studocu is not sponsored or endorsed by any college or university


Downloaded by Nhung Nguy?n (nhungnth2004@gmail.com)
lOMoARcPSD|20277325

MỤC LỤC
ĐỀ THI OLYMPIC TRUYỀN THỐNG 30/4 – NĂM 2013 2
ĐỀ THI CHÍNH THỨC 2
CÁC ĐỀ THI ĐỀ NGHỊ
1. TRƯỜNG THPT CHUYÊN MẠC ĐĨNH CHI – TP HỒ CHÍ MINH 10
2. TRƯỜNG THPT CHUYÊN NGUYỄN THƯỢNG HIỀN – TP HỒ CHÍ MINH 18
3. TRƯỜNG THPT CHUYÊN TRẦN ĐẠI NGHĨA – TP HỒ CHÍ MINH 26
4. TRƯỜNG THPT CHUYÊN LƯƠNG THẾ VINH – ĐỒNG NAI 35
5. TRƯỜNG THPT CHUYÊN NGUYỄN TẤT THÀNH – KON TUM 40
6. TRƯỜNG THPT CHUYÊN NGUYỄN ĐÌNH CHIỂU – ĐỒNG THÁP 48
7. TRƯỜNG THPT CHUYÊN PHAN NGỌC HIỂN – CÀ MAU 55
8. TRƯỜNG THPT CHUYÊN TRẦN HƯNG ĐẠO – BÌNH THUẬN 62
9. TRƯỜNG THPT CHUYÊN QUANG TRUNG – BÌNH PHƯỚC 71
10. TRƯỜNG THPT CHUYÊN HUỲNH THÚC KHÁNG – QUẢNG NAM 79
11. TRƯỜNG THPT CHUYÊN NGUYỄN THỊ MINH KHAI – SÓC TRĂNG 86
12. TRƯỜNG THPT CHUYÊN NGUYỄN DU – ĐẮC LĂK 94
13. TRƯỜNG THPT CHUYÊN LÊ QUÝ ĐÔN – ĐÀ NẴNG 101
14. TRƯỜNG THPT CHUYÊN BẾN TRE – BẾN TRE 108
ĐÁP ÁN 117

ĐỀ THI OLYMPIC TRUYỀN THỐNG 30/4


1

Downloaded by Nhung Nguy?n (nhungnth2004@gmail.com)


lOMoARcPSD|20277325

NĂM 2013
ĐỀ THI CHÍNH THỨC
A. MULTIPLE CHOICE
I. PHONOLOGY
Choose the word whose underlined part is pronounced differently from the others
1. A. choreograph B. christianity C. archaic D. chivalry
2. A. slaughter B. borough C. drought D. laughter
3. A. humane B. locate C. strategy D. rabies
4. A. wretched B. blessed C. allegedly D. knackered
5. A. realm B. cleanse C. heather D. meager
Choose the word which is stressed differently from the other three.
6. A. architecture B. comparison C. championship D. communism
7. A. propaganda B. influential C. mediocre D. obligatory
8. A. addressee B. referee C. employee D. nominee
9. A. malice B. leopard C. phenom D. cohort
10. A. unaffected B. unanimous C. unaccountable D. unambiguous
II. WORD CHOICE:
Choose the best options to complete the following sentences.
11. When I finish writing this composition, I’m going to……………and go to bed.
A. make time B. hit the hay C. hit the big time D. call it a day
12. Mary usually buys her clothes……………. It’s cheaper than going to a dressmaker.
A. in public B. on the shelf C. off the peg D. on the house
13. You are at……………to do what you like.
A. freedom B. odds C. disposal D. liberty
14. Without written evidence, we don’t have a……………on.
A. leg to stand B. foot to stand C. leg to lean D. foot to lean
15. It was very strange but I had a(n)……………that the plane would crash.
A. intuition B. omen C. premonition D. prediction
16. Although he spoke slowly, I found it difficult at times to follow the……………of his argument.
A. spool B. track C. thread D. path
17. I intend to……………an official complaint to the Director.
A. write B. lodge C. place D. take
18. The transport strikes a real……………, which will make it very difficult for me to get to work.
A. pain in the back B. nuisance C. last straw D. frustration
19. ……………benefits include a new car and free health insurance.
A. Well B. Fringe C. Edge D. Verge
20. Mind that the baby shouldn’t touch the knife; it’s as sharp as a …………….
A. blade B. sword C. cut D. razor
II. GRAMMAR AND STRUCTURES
Choose the best options to complete the following sentences.
21. He prefers to attend Economics University rather than…………….
A. going to Polytechnic B. to be accepted to Polytechnic
C. studying Polytechnic D. to attend Polytechnic
22. It pays……………some professional advice before you make a decision.
A. get B. getting C. to getting D. io get
23. It……………able to finish it in an hour.
A. can’t have been too hard if you had been C. couldn’t be too hard if you are
B. can’t have been too hard if you were D. couldn’t' be too hard if you had been
24. It could have been a lot worse……………there.
2

Downloaded by Nhung Nguy?n (nhungnth2004@gmail.com)


lOMoARcPSD|20277325

A. when he had not been C. had he not been


B. for he had not been D. whether or not he had been
25.……………believed to be over 300 species of trees in El Yunque rain forest in Puerto Rico.
A. There are B. They are C. It is D. Is has been
26. ……………classified as a carnivore, the North American Grizzly bear eats berries and even grass.
A. Just as B. Because of C. Although D. Either
27. I’m……………my brother is.
A. nothing near as ambitious C. nowhere like so ambitious
B. nothing as ambitious than D. nowhere near as ambitious as
8. On the island……………the only representation of the Indians’ handicraft.
A. does it remain B. remains C. did it remain D. remains it
29. It is advised that not only Tam but also his friends……………punctual.
A. be B. are C. is D. were
10. Many a boy……………come up with many new ideas.
A. have B. has C. have been D. has been
V. PHRASAL VERBS AND PREPOSITIONS
31. If you pay thé restaurant bill with your credit card, I’ll …………… with you later.
A. settle down B. pay back C. settle up D. pay up
32. Demand for the products is expected to peak 5 years from now and then to……………
A. taper off B. fall down C. set back D. drift away
33. It was an embarrassing situation, but she managed to……………
A. laugh at it B. laugh on it C. laugh it off D. laugh it out
34. The police……………a good deal of criticism over their handling of the demonstration.
A. came in for B. brought about C. went down with D. opened up
35. The rain was simply……………down on the deserted street.
A. pelting B. spraying C. showering D. dripping
36. As we were waiting on the pavement, a black Mercedes……………beside us.
A. pulled up B. pulled down C. pulled off D. pulled through
37. I was……………the impression that you liked Indian food.
A. on B. with C. over D. under
38. Ms. Dung, your form teacher,……………as a very sympathetic person. She can be a shoulder to cry on.
A. comes out B. comes at C. comes over D. comes
about
39. She brought……………the deal even though nobody thought she was capable of doing it.
A. in B. off C. out D. down
40. It took the parents a long time to……………their children’s games.
A. catch up with B. catch up C. catch on to D. catch out
V. GUIDED CLOZE
Read the text below and decide which answer best fits each space.
(41)…………Christmas evolved in the United States, new customs were (42) …………and many old
ones were reworked. The legend of Santa Claus, for example, had (43) …………in Europe and was brought
by Dutch settlers to New York in the early 18th century. Traditionally, Santa Clause - from the Dutch Sinter
Klaas - was depicted (44) …………a tall, dignified, religious figure riding a white horse (45) …………the air.
Known as Saint Nicholas in Germany, he was usually accompanied by Black Peter, an elf who punished
disobedient children. In North America, he (46) …………developed into a fat, jolly old gentleman who had
neither the religious (47) …………of Saint Nicholas nor the (48) …………disciplinarian character of Black
Peter.
Santa’s transformation began in 1823, when a New York newspaper published the poem “A Visit from-
Saint Nicholas”, which Clement Clark Moore had written to amuse his daughter. The poem introduced many
3

Downloaded by Nhung Nguy?n (nhungnth2004@gmail.com)


lOMoARcPSD|20277325

Americans to the story of a kindly saint who flew over housetops in a reindeer-drawn sleigh. Portraits and
drawings of Santa Claus by American illustrator Thomas Nast further (49) …………the legend
during the second half of the 19th century. Living at the North Pole and assisted by elves, the modern Santa
produced and delivered toys to all good children. By the late 19th century, he had become such a (50)
…………figure of American folklore that in 1897, when Virginia O’Hanlon wrote to the New York Sun
newspaper asking if Santa was real, she received a direct answer: “Yes, Virginia, there is a Santa Claus.”
41. A. As B. Since C. Through D. Now that
42. A. adapted B. acclaimed C. adopted D. assumed
43. A. roots B. stems C. origins D. backgrounds
44. A. by B. as C. for D. into
45.A. up B. on C. at D. through
4Í. A. eventually B. at last C. finally D. ultimately
47. A. hopes B. roles C. temperaments D. attributes
48. A. severe B. strict C. harsh D. austere
49. A. strengthened B. consolidated C. confirmed D. bettered
50. A. prominent B. prime C. domineering D. principle
VI. GUIDED CLOZE 2
Read the text below and decide which answer best fits each space.
CARNIVOROUS PLANTS
All plants rely on nutrients taken from the soil in order to survive. However, in areas where the soil
does not contain enough (51)………….nutrients, some plants have adapted to (52) …………. their diets from
another source: living organisms. Though they are few in number, carnivorous plants are (53) ………….
fascinating beings that “eat” anything from one-celled organisms to insects in order to survive. They are
commonly found in marshlands. Carnivorous plants feature one of several types of “traps” to ensnare prey,
which they consume to make up for nutrients that may be missing from the soil. While there are over 400
species of carnivorous plants in the world today, some are more (54) ………….than others.
The most well known of these plants are the snap traps, which include the Venus fly trap. Snap traps
are easily identified by , their leaves, which are separated into two lobes that have the ability to fold together.
Inside the lobes, the surface is covered with tiny hairs that are (55) ………….to movement. When the plant’s
prey brushes against the hairs, it triggers a closing mechanism that rapidly brings the two lobes together,
trapping the prey (56) ………….inside. The response of the traps is phenomenal (57) ………….speed: the
time between triggering the hairs and snapping shut is less than a second. As the prey struggles inside the trap,
it only triggers more hairs, causing the leaves to tighten their (58) …………..The plant then secretes liquid
chemicals from special glands into the trap to dissolve the prey and absorb all of its nutrients. Besides the
Venus fly trap, only one other type of snap trap exists today, (59) ………….to as the waterwheel plant. The
two share a common ancestor and differ only in a few ways. For instance, the waterwheel is an aquatic plant,
while the fly trap is exclusively terrestrial. In addition, the fly trap feeds primarily on arthropods like spiders,
while the waterwheel lives (60) ………….simple invertebrates, like certain types of plankton.
51. A. vital B. essential C. crucial D. indispensable
52. A. enlarge B. modify C. supplement D. augment
53. A. nevertheless B. nonetheless C. though D. contradictorily
54, A. prevailing B. prevalent C. current D. domineering
55. A. liable B. prone C. sensitive D. vulnerable
56. A. closely B. irreplaceably C. steadily D. securely
57. A. in regard to B. in accordance with C. in preference to D. on merits of
58. A. hold B. fist C. grip D. seizure
59. A. denoted B. indicated C. referred D. implicated
60. A. off B. onto C. with D. through
VII. READING PASSAGE 1
4

Downloaded by Nhung Nguy?n (nhungnth2004@gmail.com)


lOMoARcPSD|20277325

Read the text below and choose the best answer to each question.
GERTRUDE STEIN
One of the most influential literary figures of the twentieth century was American author Gertrude
Stein. Her literary style and vision was often a radical departure from traditional methods, which relied on a
more linear plotline. Instead, Stein focused on language itself by employing methods of repetition and
spontaneity in an attempt to mirror human consciousness. But, Stein’s influence did not stop with her writing.
As an expatriate in Paris, she was responsible for bringing some of the greatest minds in art and literature
together at her apartment, Salon 27. The Spanish painter Pablo Picasso and American writer Ernest
Hemingway were two frequent visitors. Indeed, Stein was the one who coined the phrase “Lost Generation” to
denote up-and-coming American writers living in Paris at the time mainly due to their disillusionment with art
as a whole back home. While Gertrude Stein may not be the most recognizable figure in literature, her
personal and literary influence on artists was invaluable.
Stein began living in Paris in 1903. Eventually, her flat, Salon 27, became a center of intellectual
exchange for writers and artists. Before long, she became an integral part of the artistic and literary scene in
the city and befriended numerous important figures, such as Picasso and Matisse. They would meet there
weekly, where they could expound on new theories of art, philosophy, literature, politics, and social issues in
the stimulating, prỏductive environment provided by Stein. Without Stein’s Salon 27, it is quite possible that
many of the artists and writers of the day never would have crossed paths, and the individual disciplines
would not have become as diverse or fully developed as they eventually did. These interactions also became a
major influence on Stein’s own literary style.
Stein became enamored with Picasso’s cubist style, and, as a result, many of his earliest works adorned
the walls of her apartment. But they were not simply decorative. Cubism attempts to reduce the subject from
its natural form into an abstract, geometrical shape capable of numerous angles of perception. In a similar
fashion, Stein attempted to interpret cubism through literature and writing. Like cubist painters, Stein wrote in
a style that took into consideration every possible angle of her subject matter. She wanted to give readers the
opportunity to view her work on many different levels, not just a single, flat surface. But, unlike the cubists
who relied on formal structure to some degree, Stein took it a step further and placed less emphasis on formal
writing structures such as grammar and syntax and focused on what she called “automatic” language, which
was spontaneous and repetitive and relied more on the spoke word.
In many ways, Stein’s style was a reaction against nineteenth century authors’ tendency to place order
and structure above all other considerations. Stein revolutionized the style of the twentieth century by
undermining traditional, expected methods that came before the modern era. Stein focused on the process of
writing, not its ultimate result. She also believed that all parts of a sentence were equally important. For
example, to her, an article was as important as a noun, and a conjunction was as vital as a verb. In essence,
every part of a sentence was related. She learned this from painters, who believed every brushstroke on a
canvass contributed to the whole and could not be left out Furthermore, the act of perception, whether it be art
or literature, was crucial, not the ultimate outcome because to Stein, consciousness never ceases. It is
continuous, so it became her task to try to embody the movement of consciousness in her work through
techniques such as the repetition of words or spontaneity rather than depicting events or a plotline in a linear
fashion.
In many ways, Stein’s writing mirrors the chaotic, detached atmosphere of post-Word War I Paris, yet
it also marks the moment when literature began to leave the nineteenth century behind in favor of a more
modernist style of imagination and innovation. It is also a clear precursor of stream of consciousness, which
dominated writing styles during the rest of the twentieth century in America. For instance, Hemingway’s style
in many of his early novels is clearly influenced by Stein. While Stein was able to capitalize on artistic
renditions of life and translate them into literature, other artists benefited form her willingness to reject the
accepted traditions and create her own.
61. The word “radical” is closest in meaning to
A. gradual B. abrupt C. swift D. drastic
5

Downloaded by Nhung Nguy?n (nhungnth2004@gmail.com)


lOMoARcPSD|20277325

62. The author’s description of Gertrude Stein mentions which of the following?
A. She was originally from Paris and later moved to the US.
B. Her writing style was centered on a structured plotline.
C. She attempted to focus on the linguistic side of writing.
D. Her work was quite controversial due to its extreme nature.
63. Which of the following can be inferred from Paragraph 1 about American expatriates in Paris?
A. They did not completely agree with Picasso’s style at first.
B. They were more satisfied with the creative capacity of Europe.
C. They attempted to forge a new style of literary consciousness.
D. They rarely met together in Stein’s apartment to converse.
64. The author discusses Salon 27 in paragraph 2 in order to
A. illustrate its relevance as a place of artistic1 interaction of the time.
B. note it is where the painters Matisse and Picasso first met one another.
C. indicate it was an adequate place for such great minds to mingle.
D. show that artists of the day lacked a meeting place at’which to gather.
65. According to paragraph 3, Stein followed the cubist style because
A. it focused on the geometric forms inherent in most subject matter
B. it gave her a way to present her reader with various perspectives.
C. it was the simplest form with which to interpret and understand her work.
D. it eschewed the secondary parts in fayor of the major parts of art.
66. The word “undermining” in the passage is closest in meaning to
A. simplifying B. overshadowing C. promoting D. weakening
67. The author’s description of Stein’s revolutionary style mentions all of the following EXCEPT
A. She placed reason, order and logic above all other facets of her technique.
B. She considered each part of the sentence to be essential to its weight and meaning.
C. She did not believe that the finality of a work of literature was very important
D. She attempted to capture the movement, of consciousness through various, techniques.
68. The word “embody” in the passage is closest in meaning to
A. evade B. express C. simplify D. replicate
69. According to paragraph 5, Stein’s style is a product of the era because
A. it translates the sentiment of Europeans during a time of reconstruction.
B. it is more imaginative and innovative than anything the world had seen
C. it continues the former structural traditions present before World War II.
D. it rejects the past and is fashioned out of the sentiment of post-war Europe.
70. The word “renditions” in the passage is closest in meaning to
A. interpretations B. outlooks C. perspectives D. concepts
VIII. READING PASSAGE 2
Read the text below and choose the best answer to each question.
According to accounts, when the first astronauts in space looked down and saw the Earth floating in
the vast black void, they had what can only be described as a profound spiritual experience; in an instant they
had attained a 'global consciousness' in which all national and international boundaries disappeared, and they
were left with the awesome realisation that they were mere 'planetary citizens'. To the astronauts, the planet
looked as if it were some huge single living system. The photographs they brought back touched us all in
some way, and the blue sphere in space came to symbolise the oneness of all humanity and life on Earth. The
idea that the planet might be alive, strange though it sounds, was soon to gain credence, even among the
scientific community.
Not long afterwards in the 1970s, the hypothesis that the Earth's biosphere actually functions as a
single living system was put forward by Dr James Lovelock, a British scientist and inventor who had been
commissioned by NASA to help determine whether or not there was life on Mars. By comparing the
6

Downloaded by Nhung Nguy?n (nhungnth2004@gmail.com)


lOMoARcPSD|20277325

atmospheres of both planets, he soon realised that, while Mars had a stable, unchanging, 'dead' atmosphere,
Earth had no such equilibrium, and that there were some complex processes going on. It was this imbalance
that made the planet suitable for sustaining life. He postulated that: 'the physical and chemical condition of the
surface of the Earth, of the atmosphere and of the oceans has been, and is, actively made fit and comfortable
by the presence of life itself... in contrast to the conventional wisdom which held that life adapted to the
planetary conditions as it, and they, evolved their separate ways.'
Suffice it to say, Lovelock knew that when looking at the Earth in this way, what he was seeing was
not so much a planet that just happened to be suitable for sustaining life, but a self-evolving, and self-
regulating system that adjusted itself to support life. This seemed to qualify the Earth as a living entity in her
own right, so he named her 'Gaia' - after the Greek goddess who was said to have drawn the living world forth
from Chaos - and the 'Gaia hypothesis' was born.
Lovelock first published his idea in 1979 in his book, Gaia, a New Look at Life on Earth, although the
science behind the hypothesis was still imprecise. The ideas in the book provoked a storm of criticism, but
also generated a lot of research, which has since led to profound new insights about life on Earth. For
instance, Lovelock knew that the heat of the sun has increased by 25% since life began on Earth, yet he did
not understand by which process the temperature on the surface had been kept at the optimum conditions
suitable for sustaining life.
Since that time, many of the mechanisms by which Gaia regulates her systems have been identified.
For example, it has been shown that cloud formation over the open ocean is almost entirely a function of the
metabolism of oceanic algae. Previously, it was thought that this cloud formation was a purely chemical
phenomenon. Further research suggested that Gaia has automatically been controlling global temperature,
atmospheric content, ocean salinity, and other factors in order to 'maintain the conditions suitable for its own
survival', in much the same way that any individual organism regulates its body temperature, blood salinity
etc.
Similarly, all the life forms on the planet are a part of Gaia, in a way analogous to the different organs
in a body, each with its own function. The oceans and atmosphere act as the planet's circulatory and
temperature control systems, while the tropical rainforests could be compared to the liver, cleansing the body
of toxins. In their diversity, the myriad life forms of earth co-evolve and contribute interactively to produce
and sustain the system as a whole.
Some of Lovelock's critics took his hypothesis to imply that the Earth was behaving with a sense of
purpose, that it was being a teleological* being, actively controlling the climate and so on. However, Lovelock
had never stated that planetary self-regulation was purposeful, only that it was a living, highly complex
system. No one doubts that plants or bacteria are alive, yet they do not produce processes nearly as
complicated as the Earth's.
The Gaia Theory has already had a huge impact on science and has inspired many leading figures of
the past 20 years, who have written and spoken eloquently about how we can model human activities that are
beneficial to the living systems of our planet. By making us more aware of the damage we are doing to the
eco-system, Gaia theory may also help us to survive. We are just one part of a larger system, and are reliant on
that system for our continued existence. As Lovelock said: 'if we see the world as a superorganism of which
we are a part - not the owner, nor the tenant, not even a passenger - we could have a long time ahead of us and
our species might survive for its 'allotted span'. It all depends on you and me.'
*A being with an ultimate purpose/design towards an end.
71. The first astronauts in space were
A. conscious of the lack of physical boundaries between nations.
B. forced to adjust their perspective of their place in the cosmos.
C. profoundly affected by the symbolism of the Earth.
D. made aware of the life-forces operating on Earth.
72. The word “credence” is closest in meaning to
A. acceptance B. concordance C. protection D. rejection
7

Downloaded by Nhung Nguy?n (nhungnth2004@gmail.com)


lOMoARcPSD|20277325

73. Dr. James Lovelock had originally


A. been arv inventor in Britain.
C. been employed to compare Mars with Earth.
B. been looking for Martian life.
D. proved Mars was a dead planet.
74. The word “they” in paragraph 2 refers to
A. life on Earth B. planetary conditions
C. oceans . D. atmosphere and temperatures
75. Lovelock surmised that
A. Earth’s inconstant atmosphere was a bi-product of life on the planet.
B. the chemical condition of the Earth had come about by accident.
C. the imbalance of gases on Earth had created life.
D. life had evolved to survive in Earth’s planetary conditions.
76. Research has shown that
A. the various planetary systems are regulated by different mechanisms.
B. clouds are formed by metabolic chemical changes in the sky.
C. the saltiness of the seas is due to the presence of oceanic algae.
D. Gaia can ultimately control her own survival.
77. The word “analogous” is closest in meaning to
A. consistent B. comparable C. related D. traceable
78. According to Gaia theory
A. the planet has physical biological organs similar to a person’s.
B. the oceans control the temperature of the Earth’s atmosphere.
C. the rainforests can remove all the pollution from the Earth.
D. each species on Earth has a part to play in the planet’s survival.
79. According to Lovelock,
A. higher forces are at work behind the Earth’s existence and survival.
B. the Earth had developed senses and was conscious of its purpose.
C. the complex life-forces on earth are equal to the sum of a living being.
D. bacteria and plants are alive but can only produce simple processes.
80. The Gaia hypothesis may ultimately ensure the immediate survival of
A. the planet Earth B. the human species
C. the ecosystem D. all life on Earth
B. WRITTEN TEST
I. CLOZE TEST: Read the text below and think of the word which best fits eack space. Use only ONE
WORD for each space.
OPEN CLOZE 1
Quite different from storm surges are the giant sea waves called tsunamis, which derive their (1)
………….from the Japanese expression for “high water in a harbor”. These waves are also referred to by the
general public as tidal waves, although they have relatively (2) …………. to do with tides. Scientists often
refer to them as seismic sea waves, far more appropriate in (3) ………….they do result from undersea seismic
activity.
Tsunamis are caused when the sea bottom suddenly moves and the water above the moving earth is
suddenly displaced. This sudden shift of water sets off a series of waves, which can travel great distances at
speeds close to 700 kilometers per hour. In the (4) ………….ocean, tsunamis have little noticeable amplitude,
often no more than one or two meters. It is when they hit the shallow water (5) ………….they increase in(6)
…………….possibly up to 40 meters. Tsunamis often occur in the Pacific because the Pacific is an area of (7)
………….seismic activity. Two areas of the Pacific (8) ………….accustomed to the threat of tsunamis are
Japan and Hawaii. Because the seismic activity that causes tsunamis in Japan often (9) ………….on the ocean
8

Downloaded by Nhung Nguy?n (nhungnth2004@gmail.com)


lOMoARcPSD|20277325

bottom quite close to the islands, the tsunamis that hit Japan often come with little (10) ………….and can
therefore prove disastrous.
OPEN CLOZE 2
The response of most animals when suddenly faced with a predator is to flee.(11) ………….selection
has acted in a variety of ways in different (12) ………….to enhance the efficacy of the behaviors, known as
“flight behaviors” or escape behaviors that are used by prey in fleeing predators. Perhaps the most direct
adaptation is enhanced flight (13) ………….and agility.
Adaptations for speed, however, are likely to require sacrifices, so we might expect only some species
to adopt a simple fast flight strategy. Another way of enhancing the effectiveness of flight is to move in an
erratic and unpredictable (14) ………….. Many species, like ptarmigans, snipes, and various antelopes and
gazelles, flee from predators in a characteristic zigzag fashion. Rapid unexpected changes in flight direction
(15) ………….it difficult for a predator to track (16) ………….. In some species, like the European hare,
erratic zigzag flight might be more effective in the (17) ………….of predators that are faster than they are and
straight flight more effective against predators that are (18) …………. A quite different way of enhancing
escape by flight is to use so-called “flash” behavior. Here, the alarmed grey flees for a short distance and then
“freezes”. Some predators are unexcited by (19) …………. prey, and a startling flash of activity followed by
immobility may confuse them. “Flash behavior” is used in (20) ………….by frog and orthopteran insects.
II. WORD FORMATION
PART 1: Complete each sentence, using the correct form of the word in parentheses.
1. This involved some…………..dealings with the chief of the police. (HAND)
2. Ensure your screen colors are not causing eye strain. Avoid fluorescent…………..screens. (COLOR)
3. You can travel from one end of the park to the other on a…………..railway. (MINIMIZE)
4. Mr. Madill stated that the…………..against him were unfounded. (ALLEGE)
5. The lingering war in the country has brought about nothing but the…………..effects of poverty and squalor.
(HUMAN)
6. The…………..between blacks and whites are more and more common. (MARRY)
7. I don’t think Tom’s getting too much sleep lately. His. eyes are terribly………….. (BLEED)
8. He has unwavering belief in something unreal such as…………..forces. (NORMALITY).
9. …………..can be dangerously close to racism. (NATION)
10. The world champion was…………..by a younger Russian challenger. (THRONE)
PART 2: Complete the passage with appropriate forms from the words given in the box.
KICK LEAVER IKELIHOOD ECSTASY RELATION
ADVENTUROUS NURTURE COUNT ATTEND REMARK

Over the years, there have been (11) …………. fans of the classic Hanna-Barbera cartoon character
Yogi Bear. The cartoon series enjoyed by young and old alike revolved mostly around the (12) ………….of
this loveable bear and his (13) …………. Boo-Boo as they tried unsuccessfully to snag “pic-a-nic” baskets in
the made-up land of Jellystone Park. It is not often that people think about where the ideas for these cartoon
characters come from, which brings up an interesting point: do bears actually search for food (14) ………….
in Picnic baskets and (15) …………. campsites?
(16) ………….enough, bears have been known to seek out food from some (17) ………….sources,
including picnic baskets, on top of their usual diet of berries, insects, and fish. Bears work throughout the
summer and fall to build up fat stores so as to have energy enough to last them through their winter
hibernations. (18) …………. to this is their need to replenish their depleted reserves when they wake up in the
spring. Food is generally scarce in the early spring, and consequently they will (19) ………….indulge in any
foods that are (20) ………….. This is the main reason for many incidents involving bears entering campsites
in search of food.
III. ERROR CORRECTION
The following passage contains 10 errors. Identify and correct them.
9

Downloaded by Nhung Nguy?n (nhungnth2004@gmail.com)


lOMoARcPSD|20277325

1 Many flowering plants woo insect pollinators and gently direct them to their most fertile blossoms by
changing the colors of individual flowers from day to day. Through color cues, the plant signals for the
insect that it would be better off visiting one flower on its bush than other. The particular hue tells the
pollination that the flower is full of far more pollen than are
5 neighboring blooms. That nectar-rich flower also happens to be fertile and ready to disperse its pollen
or to receive pollen the insect has picked out from another flower. Plants do not have to spend precious
resources remaining reservoirs of nectar in all their flowers. Thus, the colorcode communication
system benefits both plants and insects.
For example, on the lantana, a flower starts out on the first day as yellow, when it is rich
10 with pollen and nectar. Influenced by an as-yet-unidentified environmental signal, the flower changes
its color of triggering the production of the pigment anthromyacin. It turns orange on the second day
and red on the third. By the third day, it has no pollen to offer insects and is no longer fertile. On any
given lantana bush, only 10 to 15 percent of the blossoms are likely to be yellow and fertile. But in the
tests measuring the response of butterflies, it’s discovered that
15 the insects visited the yellow flowers at least 100 times more than would be expected from the
haphazard visitation. Experiments with paper flowers demonstrated that the butterflies-were
responding with color cues rather than, say, the scent of the nectar. In other types of plants, blossoms
change from white to red, others from yellow to red, and so on. These color changes have been
observed in 74 families of plants.
IV. SENTENCE TRANSFORMATION
Rewrite the following sentences using the words given.
1. This liver condition is common among those who drink a lot.
→ This liver condition……………………………………………………………. drinkers.
2. “ I think the whole idea is ridiculous,” he said.
→ He dismissed…………………………………………………………………………….
3. He threatened the officers with violence.
→ He made …………………………………………………………………………………
4. It shouldn t have surprised me that my children didn’t like the new, cheaper icecream.
→ I might …………………………………………………………………………………...
5. Her lateness made the boss angry. (BANANAS)
→ It was because she………………………………………………………………………..
6. She wore headphones in order not to disturb anyone.
→ She wore headphones lest………………………………………………………………..
7. She is not very good at arranging flowers. (FLAIR)
→…………………………………………………………………………………………….
8. Bill changed his ways when he came out of prison. (LEAF)
→ Bill has…………………………………………………………………………………....
9. You must not, I repeat not, open this box until Christmas Day. (TO)
→ Under ………………………………………………………opened until Christmas Day.
10. Her early success has made her very conceited. (HEAD)
→…………………………………………………………………………………………….

TRƯỜNG THPT MẠC ĐĨNH CHI - TP. HỒ CHÍ MINH


A. MULTIPLE CHOICE QUESTIONS
I. PHONOLOGY
Choose the word whose underlined part is pronounced differently from the other three.
1. A blessed B minced C wicked D wretched
2. A slaughter B throughout C drought D laughter
3, A trainer B fairness C impairment D dairy-maid
10

Downloaded by Nhung Nguy?n (nhungnth2004@gmail.com)


lOMoARcPSD|20277325

4. A subtle B timber C plumber D doubtful


5. A terrible B sentimental C terrific D memory
Choose the word whose stress pattern is different from that of the other three.
6. A. popularize B apologize C maximize D pasteurize
7. A best-selling B high-pitched C multi-purpose D red-hot
8. A statistics B ecotourism C consciousness D maintenance
9. A painstakingly B fascinatingly C interestingly D temporarily
10. A disposable B documentary C original D nonprofit
II. WORD CHOICE
Choose the best answer to complete eạch of the following sentence.
1. We welcome the new regulations, which become……………on the first of next month.
A. effective B. efficient. C. efficacious D. effete
2. Although he spoke slowly, I found it difficult at times to follow the…………… of his argument.
A. spool B. track C. thread D. path
3. The Red Cross is ……………an international aid organization.
A. intriguingly B. intrusively C. intrinsically D. intrepidly
4. If I were you, I would regard their offer with considerable……………because it seems too good to be true.
A. suspicion B doubt C. reservation D. disbelief
5. When his accomplices failed to turn up at the meeting point, it............ on him that he had been tricked.
A. dawned B. broke C. awoke D. becathe clear
6. Tourism provides people with jobs - albeit often rather …………… ones!
A. superficial B. menial C. trivial D. remedial
7. Anna is a very nervous child and she’s very……………of strangers.
A. terrified B. frightened C. petrified D. horrified
8. He was……………with an extraordinary musical ability.
A. ensures B. entruysted C. entreated D. enddwed
9. I’m opting out of the ……………race and going to live on a small farm in the countryside.
A. rat B. dog C. horse D. cat and mouse
10. Champagne is a……………wine, which originally came from the north-east of France.
A. glittering B. sparkling C. glistening D. gleaming
III. STRUCTURES AND GRAMMAR
Choose the best answer to complete each of the following sentence.
1. Jacob won’t pass the course without studying,……………else he does.
A. whatever B. wherever C. whenever D. however much
2. They……………the loan because they already owed too much money elsewhere.
A. weren’t successful to get B. didn’t succeed to get
C. weren’t succeeded in getting D. didn’t succeed in getting
3. It……………been Max you saw last night because he’s been out of town for a week.
A. will not have B. wasn’t to have C. Should not have D. couldn’t have
4. ‘Is this the suitcase you want to take on the trip?’
‘No, I wanted ……………brown one.’
A. other than B. that other C. that another D. another than
5. Irene…………… asked to the party.
A. pleased to be B. pleased being
C. was please for being D. was pleased to be
6. The new highway could not be completed because ……………money.
A. lack of B. of lack C. is lacking of D. of a lack of
7. You can take either a math or a physics course this semester,…………….what you decide.
A. dependent in B. depending on C. depends for D. to depend for
11

Downloaded by Nhung Nguy?n (nhungnth2004@gmail.com)


lOMoARcPSD|20277325

8. Where are my car keys? I can’t remember where……………


A. last they were put B. did I last put them
C. were they to be put last D. I last put them
9. Sarah had to take……………lessons before she got her license.
A. driver B. driver’s C. driven D. driving
10. Mary was having trouble deciding whether or…………… take the new job.
A. not should she B. not she should
C. should she not D. should not she
IV. PREPOSITIONS AND PHRASAL VERBS
Choose the correct phrase to complete each sentence.
1. No matter how often I explain it, he doesn’t seem to ............
A. put it through B. take it in C. take it on D. put it in
2. That is the proposal which I shall…………… to the Managing Director.
A. put forward B. take down C. bring up D. put on
3. Leila……………the information she wanted in her reference books.
A. looked after B. looked up C. took out D. took up
4. Local shopkeepers are……………their prices for the summer sales.
A. taking down B. looking on C. taking off D. bringing down
5. Lucia was surprised when her guests……………late for the party.
A. came up B. turned up C. looked up D. put up
6. Sergio noticed how cold it was when he……………the plane
A. got off B. took off C. went off D. went out of
7. All the ideas were good, but Michael……………the best plan of all.
A. put on B. got on with C. came up D. came up with
8. I think a plain blouse would……………better……………that skirt.
A. go - with B. put - with C. come - with D. go - to
9. She……………the flat three times, before deciding to buy it.
A. came round B. brought round C. looked round D. got round
10. Although he’s my friend, I find it hard to ……………his selfishness.
A. get out of B. come up with C. take on D. put up with
V. READING COMPREHENSION
Read the following passages and choose the best answer for each of the questions below.
PASSAGE A
The modern comic strip started out as ammunition in a newspaper war between giants of the American
press in the late nineteeth century. The first full-color comic strip appeared in January 1894 in the New York
World, owned by Joseph Pulitzer. The first regular weekly full-color comic supplement, similar to today’s
Sunday funnies, appeared two years later, in William Randolph Hearst’s rival New York paper, the Morning
Journal.
Both were immensely popular, and publishers realized that supplementing the news with comic relief
boosted the sale of papers. The Morning Journal started another feature in 1896, the “Yellow Kid,” the first
continuous comic character in the United States, whose creator, Richard Outcaulthad been lured away from
the World by the ambitious Hearst. The “Yellow Kid’ was in many ways a pioneer. Its comic dialogue was
the strictly urban farce that came to characterize later strips, and it introduced the speech balloon inside the
strip, usually placed above the characters heads.
The first strip to incorporate all the elements of later comics was Rudolph Dirks’s “Katzenjammer
Kids,” based on Wilheim Busch’s Max and Moritz, a European satire of the nineteenth century. The “Kids”
strip, first published in 1897, served as the prototype for future American strips. It contained not only speech
balloons, but a continuous cast of characters, and was divided into small regular panels that did away with the
larger panoramic scenes of most earlier comics.
12

Downloaded by Nhung Nguy?n (nhungnth2004@gmail.com)


lOMoARcPSD|20277325

Newspaper syndication played a major role in spreading the popularity of comic strips throughout the
country. Though weekly colored comics came first, daily black- and-white strips were not far behind. They
first appeared in the Chicago American in 1904. It was followed by many imitators, and by 1915 black-and-
white comic strips had become a staple of daily newspapers around the country.
1. What does the passage mainly discuss?
A. A comparison of two popular comic strips
B. The differences between early and modem comic strips
C. The effects of newspapers on comic strip stories
D. Features of early comic-Strips in the United States
2. Why does the author mention Joseph Pulitzer and William Randolph Hearst?
A. They established New York’s first newspaper.
B. They published comic strips about the newspaper war.
C. Their comic strips are still published today.
D. They owned major competitive newspapers.
3. The passage suggests that comic strips were popular for which of the following reasons?
A. They provided a break from serious news stories.
B. Readers enjoyed the unusual drawings.
C. Readers could identify with the characters.
D. They were about real-life situations.
4. To say that Richard Outcault had been “lured away from the “world” by Hearst (line 9) means which of the
following?
A. Hearst convinced Outcault to leave the World.
B. Hearst fired Outcault from the World.
C. Hearst warned Outcault to leave the World.
D. Hearst wanted Outcault to work for the World.
5. The word “it” in line 10 refers to…………..
A. The “Yellow Kid” B. dialogue C. farce D. balloon
6. According to the passage, the “Yellow Kid” was the first comic strip to do all of the following
EXCEPT…………..
A. feature the same character in each episode
B. include dialogue inside a balloon
C. appear in a Chicago newspaper
D. charactertize city life in a humorous way
7. The word “incorporate” in line 12 is closest in meaning to…………..
A. affect B. create C. combine D. mention
8. The word “prototype” in line 14 is closest in meaning to…………..
A. story B. humor C. drawing D. model
9. The word “staple” in line 20 is closest in meaning to…………..
A. regular feature B. popular edition C. new version D. huge success
10. In what order does the author discuss various comic strips in the passage?
A. Alphabetical order by title
B. In the order in which they were created
C. According to the newspaper in which they appeared
D. From most popular to least popular
PASSAGE B
Life originated in the early seas less than a billion years after the Earth was formed. Yet another three
billion years were to pass before the first plants and animals appeared on the continents. Life’s transition from
the sea to the land was perhaps as much of an evolutionary challenge as was the genesis of life.
What forms of life were able to make such a drastic change in lifestyle? The traditional view of the
13

Downloaded by Nhung Nguy?n (nhungnth2004@gmail.com)


lOMoARcPSD|20277325

first terrestrial organisms is based on megafossils - relatively large specimens of essentially whole plants and
animals. Vascular plants, related to modern seed plants and ferns, left the first comprehensive megafossil
record. Because of this, it has been commonly assumed that the sequence of terrestrialization reflected the
evolution of modern terrestrial ecosystems. In this view, primitive vascular plants first colonized the margins
of continental waters, followed by animals that fed on the plants, and lastly by animals that preyed on the
plant-eaters. Moreover, the megafossils suggest that terrestrial life appeared and diversified explosively near
the boundary between the Silurian and the Devonian periods, a little more than 400 million years ago.
Recently, however, paleontologists have been taking a closer look at the sediments below this Silurian-
Devonian geological boundary. It turns out that some fossils can be extracted from these sediments by putting
the rocks in an acid bath. The technique has uncovered new evidence from sediments that were deposited near
the shores of the ancient oceans - plant microfossils and microfossils and microscopic pieces of small animals.
In many instances the specimens are less than one-tenth of a millimeter in diameter. Although they were
entombed in the rocks for hundreds of millions of years, many of the fossils consist of the organic remains of
the organism.
These newly discovered fossils have not only revealed the existence of previously unknown
organisms, but have also pushed back these dates for the invasipn of land by multicellular organisms. Our
views about the nature of the early plant and animal communities are now being revised. And with those
revisions come new speculations about the first terrestrial life-forms.
11. The word “drastic” in line 5 is closest in meaning to
A. widespread B. radical C. progressive D. risky
12. According to the theory that the author calls “the traditional view,” what was the first form of life to
appear on land?
A. Bacteria B. Meat-eating animals
C. Plant-eating animals D. Vascular plants
13. According to the passage, what happened about 400 million years ago?
A. Many terrestrial life-forms died out.
B. New life-forms on land developed at a rapid rate.
C. The megafossils were destroyed by floods.
D. Life began to develop in the ancient seas.
14. The word “extracted” in line 15 is closest in meaning to
A. located B. preserved C. removed D. studied
15. What can be inferred from the passage about the fossils mentioned in lines 15-18?
A. They have not been helpful in understanding the evolution of terrestrial life.
B. They were found in approximately the same numbers as vascular plant fossils
C. They are older than the megafossils.
D. They consist of modem life-forms.
16. The word “instances” in line 18 is closest in meaning to
A. methods B. processes C. cases D. reasons
17. The word “they” in line 19 refers to
A. rocks B. shores C. oceans D. specimens
18. The word “entombed” in line 19 is closest in meaning to
A. crushed B. trapped C. produced D. excavated
19. Which of the following resulted from the discovery of microscopic fossils?
A. The time estimate for the first appearance of terrestrial life-forms was revised.
B. Old techniques for analyzing fossils were found to have new uses.
C. The origins of primitive sea life were explained.
D. Assumptions about the locations of ancient seas were changed.
20. With which of the following conclusions would the author probably agree?
A. The evolution of terrestrial life was as complicated as the origin of life itself.
14

Downloaded by Nhung Nguy?n (nhungnth2004@gmail.com)


lOMoARcPSD|20277325

B. The discovery of microfossilssupports the traditional viewof how terrestrial life


C. New species have appeared at the same rate over the course of the last 400 million years.
D. The technology used by paleontologists is too primitive to make accurato determinations about ages of
fossils.
VI. CLOZE TESTS
PASSAGE A
When it became quite common for people to own a television in the USA in the 1950s, producers were
forced to (1) ………….up with different kinds of programmes. One type that rapidly (2) ………….popularity
was the ‘game show’, in which contestants had to answer questions. One such programme, which was the
subject of a film called Quiz Short, caused a scandal of nationwide proportions over forty years ago.
Like all commercial television stations, the one that (3) …………. this show was interested in the
number and opinions of viewers who watched the programme (4) …………., the sponsors were paying a great
deal of money to get their (5) ................advertised. This put pressure on the producers. They had to make sure
people (6) .......... on watching so they carried out surveys about the contestants who (7) ………….on the
show. When the public grew tired of the champion, there was a danger of people watching another television
(8) ………….. which, in turn, would put their jobs at risk.
In order to keep everybody happy, they had to ‘fix’ the show. They did this by giving either the
champion or the challenger the answers to the questions, (9) ………….the public’s preferences and wishes.
The plan worked well for a while, but when it was (10) …………., the popularity of game shows decreased.
1. A. find B. get C. think D. come
2. A. won B. earned C. took D. gained
3. A. sent B. broadcast C. announced D.spread
4. A. To sum up B. In total C. After all D. In fact
5. A. stock B. items C. products D. commodities
6. A. carried B. insisted C. stayed D. kept
7. A. participated B. appeared C. contested D. opposed
8. A. canal B. side C. channel D. station
9. A. owing to B. according to C. on account of D. as regards
10. A. found B. realised C. discovered D. understood
PASSAGE B
Almost all governments in sub-Saharan Africa faced with the problem of a (11) ………….human
population and limited financial resources. In this situation, many other topics are (12) ………….to be higher
on their list of priorities than the protection of the many species in danger of extinction. Nevertheless, much
can be done. South Africa is home to an astonishing array of wild life, (13) ………….from elephants and
rhinoceros to thousands of species of insects. Herds of animals that are (14) ………….for extinction
elsewhere are free to roam over 70,000 square kilometres of nature (15) ………….Yet these areas only
account for six per cent of the country so that, increasingly, private land and farms are playing a major role in
conserving wildlife. As a result, some progress has been (16) ………….The black rhinoceros, numbering
100,000 only fifty years ago, was decimated by the trade in its horn for medicines in Asia and ornamental (17)
………….for weapons in the Middle East but the signs are that the trend towards the (18) ………….
of the species has at least been halted, if not reserved. The change has come about largely because of greater
co-operation between government and private agencies. The law has been much more (19) ………….
Enforced. Poachers now face heavy fines and sentences of ten years in prison. Those who live (20) ………….
to protected areas now feel they have a stake in the preservation of the country’s wild life. This is an immense
advantage in ensuring the survival of the species.
11. A. developing B. gaining C. growing D. lifting
12. A. assured B. bound C. granted D. undoubted
13. A. crossing B. ranging C. spreading D. stretching
14. A. directing B. going C. heading D. proceeding
15

Downloaded by Nhung Nguy?n (nhungnth2004@gmail.com)


lOMoARcPSD|20277325

15. A. enclosures B. folds C. parks D. reserves


16. A. brought B. done C. got D. made
17. A. grips B. handles C. holds D. levers
18. A. abolition B. elimination C. removal D. suppression
19. A. extremely B. firmly C. strictly D. strongly
20. A. alongside B. approximate C. close D. nearby
B. WRITTEN TEST
I. OPEN CLOZE TESTS
PASSAGE A
The seasonal movement of animals, especially of birds, fish and some mammals such as porpoises, is
still not (1) ………….understood. Climatic conditions are thought to trigger off migration where perhaps
lower temperatures (2) ………….in less food being available. Some animals, particularly (3) ………….travel
vast distances. Golden Plovers are just one example, as they annually fly 8000 miles from the Arctic to South
America.
Migrating animals (4) ………….to use three mechanisms fox finding their way. Over short distances
an animal moyes to successive familiar landmarks and this is called piloting. In orientation, a straight line path
is taken, (5) …………. on the animal adopting a particular compass direction. Navigation is the most complex
process as the animal must first (6) ………….its present position before taking a direction relative to that. It
seems that some birds (7) ………….the sun, stars (often the North star which moves very little), and an
‘internal clock’ which (8) …………. allowances for the relative position of these heavenly (9) ………….
Even when the sun is hidden behind a cloud, many birds are (10) …………. to continue their migration quite
accurately by plotting their direction with respect to the Earth’s magnetic field.
PASSAGE B
Football fans who insist that officials need their eyes tested may not be far off the mark. Scientists
have discovered that in the case of offside, (11) ………….in every ten rulings is wrong. The rule states that a
player is offside if he is nearer the goal than the last defender, (12) ………….from the goalkeeper. It is
designed to prevent players hanging around the goal, waiting for the ball to come to them. Linesmen must
raise a flag indicating that a player is offside and the referee must then stop the game but they often (13)
………….it difficult to decide (14) …………. all the players on the field are in (15) ………….to each other.
Experts have argued that the reason for this is that the linesman is looking at the player who passes the ball,
not at the one who receives it, and when he looks back, the other player has moved. But the research (16)
…………out suggests that the mistake is due to an optical illusion. If the linesman is (17) …………. to the
goal than the last defender, it is very hard to judge the players’ position. The study also showed that a linesman
who is in (18) …………. is not very likely to raise the flag if the players are beside him, but will often raise
it by (19) ………….if they are on the other side of the field. The ideal (20) ………….to the problem would be
to employ a video eye to check the decisions but in that case the referee would have to stop the game
continually to ask for a second opinion.
II. WORD FORMS
Complete the following sentences with the correct forms of the words given.
1. It was thanks to the ………….of the medical staff that she recovered from her injuries. (DEDICATE)
2 …………., the hole in the ozone layer has doubled in size this year. (ALARM)
3. It’s………….to see the friendship and enthusiasm these kids display on the sports field. (HEART)
4. Some groups of football fans have reputation for………….(DESTROY)
5. The fans’ bad behaviour has resulted in the………….of their football team from the championship.
(QUALIFY)
6. John works completely independently - he is............... to nobody but himself. (ANSWER)
7. The spectators were............ debating the likely result of the tennis final. (ANIMATE)
8. She runs so………….as if it’s the easiest things in the world. (EFFORT)
9. You can trust Samantha- she always manages to remain………….in a crisis. (COOL)
16

Downloaded by Nhung Nguy?n (nhungnth2004@gmail.com)


lOMoARcPSD|20277325

10. It was very.………….of you to admit your shot had gone out after the umpire had said it was in.
(SPORTS)
Put the words given in the correct blanks. You have to use their correct forms to make a meaningful
passage.
fertility - help - initial - retrieve - sleep
special - success - surgery - swell - treat
One of the things people think about when a young woman is diagnosed with cancer is that the
chemotherapy will almost certainly leave her (11).………….afterwards. In other words, she will be unable to
have a baby. One woman, Theresa, explained her experience. She told how one day she had found a strange
(12) ………….lump. She had gone to the hospital and, to her horror, found she had cancer. She had recently
married and had been thinking of starting a family. She had had many (13) …………. Nights worrying about
how she and her husband would cope without children. (14) ………….the doctors did not even broach the
baby issue. Although Theresa found her own oncologist (15) …………., one enlightened (16) …………. told
her about the possibility of egg (17) ………….This is when the woman’s eggs are removed and kept safe
while she has chemotherapy. She was determined to recover from cancer and have children. She had to go to a
(18) ………….to find out that there was a hormone (19) …………. that could help protect the ovaries during
chemotherapy. Happily, Theresa’s cancer was (20) ………….treated and now she is expecting her first child.
III. ERROR IDENTIFICATION
There are 10 mistakes in the following passage. Identify the mistakes and correct them.
People commonly complain that they never have time enough to accomplish tasks. The hours and
minutes seem slipping away before many planned chores get done. According to time management experts,
the main reason for this is that most people fail to set priorities about how to do first. They get tied down of
trivial, timeconsuming matters and never complete the important ones.
One simple solution often used by those at the top is to keep lists of tasks being accomplished daily.
These lists order jobs from most essential to least essential and checked regularly through the day to assess
progress. Not only this is an effective way to manage time, but also it serves giving individuals a much-
deserved sense of satisfaction over their achievements. People not keep lists often face the end of the work
day with uncertainty over the significance of their accomplishments, which over time can contribute with
serious problems in mental and physical health.
1…………………….. 2……………………..
3…………………….. 4……………………..
5 .................................. 6……………………..
7.…………………….. 8……………………..
9…………………….. 10.……………………..

IV. SENTENCE TRANSFORMATION


1. Mr. Tipper’s wife was very sorry she couldn’t celebrate the New Year with her husband.
→ Mrs. Tipper greatly……………………………………………………………………………..
2. The journalists only heard about the changes to the wedding plans when they arrived at the venue.
→ It was only ………………………………………………………………………………………
3. Success in the academic field depends on your ability to amass qualifications.
→ The more ……………………………………………………………………………………….
4. We finished dinner and, a few moments later, Mrs. Jones arrived on the doorstep.
→ Hardly …………………………………………………………………………………………….
5. Do you have any idea about how Jack made enough money to buy that new sports car. (light)
→ Can you……………………………………………………………………………………………
6. Kate has finally accepted that their friendship is over. (terms)
→ Kate has finally…………………………………………………………………………………….
7. You should not lock this door for any reason when the building is open to the public. (circumstances)
17

Downloaded by Nhung Nguy?n (nhungnth2004@gmail.com)


lOMoARcPSD|20277325

→ Under……………………………………………………………………………………………….
8. In Colin’s opinion, he hasn’t done anything he should apologise for. (concerned)
→As……………………………………………………………………………………………………
9. The stranded climber would never have been rescued if his brother hadn’t had an ingenious plan.
(ingenuity)
→ But ………………………………………………………………………………………………….
10. When I started work I was so inexperienced that I couldn’t send a fax. (clue)
→ I didn’t………………………………………………………………………………………………

TRƯỜNG THPT NGUYỄN THƯỢNG HIỀN - TP. HỒ CHÍ MINH


A. MULTIPLE CHOICE QUESTIONS
I. PHONOLOGY
Choose the word whose underlined part is pronounced differently from the others
1. A. cantaloupe B. catastrophe C. recipe D. apostrophe
2. A. message B. beverage C. massage D. cottage
3. A. chronicle B. orchid C. orchard D. orchestral
4. A. fathom B. feather C. anthem D. within
5 A. odor B. opera C. oxygen D. operate
Choose the word whose stress pattern is different from the other three of each group
1. A. independent B. altogether C. beneficial D. considerate
2. A. intimacy B. contaminate C. outnumber D. mechanic
3. A. exempt B. hygiene C. neglected D. inspire
4. A. mausoleum B. sanctuary C. chimpanzee D. politician
5. A. nutritious B. chameleon C. historic D. momentary
II. VOCABULARY AND STRUCTURE
Choose the best answer A, B, V or D to complete each blank.
1. Ann’s encouraging words gave me to……………undertake the demanding task once again.
A. a point B. an incentive C. a resolution D. a target
2. The neighbours said they hadn’t got a……………who could have broken into our garage.
A. clue B. view C. point D. hint
3. I don’t need any medicine. I’m as right as ……………
A. clouds B. a ray C. rain D. a haze
4. Don’t be angry with Sue. All that she did was in good……………
A. hope B. belief C. idea D. faith
5. A military junta has taken over power in the country after the democratic administration ……………
A. collapsed B. stumbled C. vanished D. abandoned
5. Hats like this may have been fashionable in the 60’s, but now they are……………the times.
A. beneath B. under C. over D. behind
7. Just change your approach towards the assignment. The way you are dealing with it now will
certainly…………… you nowhere.
A. pass B. put C. get D. reach
8. Anything he does is in……………with the law and that’s why I have suggested him for the post.
A. compliance B. obedience C. commitment D. responsibility
9. I hope you won’t take……………if I say that your project needs more improvement to be accepted.
A. hatred B. nerve C. offence D. anger
10. Of course, we don’t need this dictionary at present, but in the long……………it may prove useful.
A. run B. time C. future D. perspective
11. There are geographic, economic, and cultural reasons why……………around the world.
A. diets differ B. do diets differ C. are diets different D. to differ a diet
18

Downloaded by Nhung Nguy?n (nhungnth2004@gmail.com)


lOMoARcPSD|20277325

12. The wedges……………dart board are worth from one to twenty points each.
A. they are on a B. are on a C. are they on a D. on a
13. The Dewey Decimal System, currently used in libraries throughout the world,……………all written
works into ten classes according to subject.
A. dividing B. divides C. it would divide D. was divided
14. Individual differences in brain-wave activity may shed light on why some people are more prone to
emotional stress disorders……………
A. that others are B. and others are C. others are D. than are others
15……………a cheese shop has since grown into a small conglomerate consisting of a catering business and
two retail stores.
A. In the beginning of B. It began as
C. its beginning which was D. What began as
16……………definitive study of a western hard - rock mining community cemetery appears to have been
done is in Silver City, Nevada.
A. Most B. The most C. Where most D. Where the most
17……………in the first draft of the budget will not necessarily be in the final draft.
A. Although it appears B. It appears
C. What appears D. Despite its appearance
18. During free fall, ……………up to a full minute, a skydiver will fall at a constant speed of 120m.p.h.
A. it is B. which is C. being D. is
19. Like Thomas Berger’s fictional character Little Big Man, Lauderdale managed to find himself
where……………of important events took place.
A. it was an extraordinary number B. there was an extraordinary number
C. an extraordinary number D. an extraordinary number existed
20.This course is one of……………few English courses offered by the college.
A. the B. only C. mostly D. almost
III. PREPOSITION
Choose the best answer A, B, C or D to complete each blank.
1. During tough negotiations, such as the ones we have witnessed over the last week or so, neither side wants
to……………
A. give in B. get by C. take hold D. come on
2. It is regrettable that two items were……………of the invoice and need to be added immediately.
A. left out B. stood out C. found out D. cut up
3. He brought……………the business deal through sheer persistence.
A. out B. in C. off D. up
4. The bully……………when he saw the teacher approaching.
A. cleared off B. called at C. brought about D. caught up
5. You cannot……………violence by using violence.
A. break away B. do away with C. keep away D. pass away
6. He……………the schedule one more time just for my benefit.
A. went back over B. set back C. went back on D. fell back on
7. You shouldn’t……………other people’s problems, even if you don’t consider them to be very serious.
A. shed light upon B. come to light C. make light of D. see the light
8. Whatever happens tomorrow, she’ll……………in history……………a great stateswoman.
A. go down as B. put down to C. come down to D. talk down to
9. She……………to the temptation of lying around in bed all day.
A. gave in B. took in C. joined in D. put in
10. The effect of the drug won’t……………until tonight.
A. break off B. let off C. wear off D. put off
19

Downloaded by Nhung Nguy?n (nhungnth2004@gmail.com)


lOMoARcPSD|20277325

IV. READING
Read the following passage and choose the best answer for each of the questions below.
DESERT PLANTS
On the surface, a desert appears to be one of the harshest environments on Earth, yet a remarkable
variety of plants have adapted to thrive in this dry climate. Despite their origins in different locations around
the globe, desert plants have developed similar strategies for surviving in arid environments. Some plants have
adapted in such a way that it is hard to tell them apart, even though they belong to very different biological
families. One useful way to classify, and thus better understand, desert plants is to examine the evolution of
certain survival strategies, which are shared across geographic and biological boundaries.
In general, the survival strategies of desert plants can be divided into two kinds: adaptation for quick
use of ephemeral resources (“maximum variance behavior’’) and adaptation for the best use of poor but more
permanent resources (“minimum variance behavior”). The former strategy involves adaptation to
environmental changes, such as seasonal availability of water. This is observed in desert annuals and
perennials alikes. Such plants tend to grow rapidly and produce many seeds under the right conditions. The
latter strategy involves adaptation to the worst possible conditions, which can be seen in succulents, true
xerophytes, and grasses. These plants usually grow slowly, use water efficiently, and exhibit passive cooling.
Water scarcity and heat are the key factors limiting the survival of desert plants. Plants that have
adapted to the worst possible conditions have evolved ways for quickly acquiring and storing water to ensure
their survival. Depending on strategies and physical adaptations, they can be divided into either succulents or
drought-resistant plants. For the most part, succulents have evolved extensive, shallow root systems to quickly
absorb water during brief periods of heavy rainfall. Water is then stored in the fleshy tissue of their thick
trunks or lobes, as well as in the roots. The cactus is a good example of a succulent. To retain water,
succulents have waxy coatings on their skin and a modified system of respiration. The stomata (surface
pores), through which the plant takes in carbon dioxide and releases oxygen, open only at night when
temperatures are cooler and less water from the plant will evaporate. Unlike most other plants, a succulent
stores all or most of its chlorophyll, the chemical essential to photosynthesis, in its stem, skin, or other outer
tissues, rather than concentrating it in the leaves. Doing this places it in a strong, well-watered part of the
plant, rather than in an appendage susceptible to drying out and dropping off.
Drought-resistant plants, or xerophytes, come in many forms including true xerophytes, deciduous
plants, and grasses. Xerophytic shrubs, such as the five species of creosote bush, are the most abundant type
of vegetation in most arid environments. They are able to withstand desiccation in severe droughts without
dying. Some have evolved small leaves with special coatings to reduce loss of water through evaporation.
Others have replaced leaves with thorns or spines. While succulent roots form shallow, dense webs,
xerophytes tend to develop deep root networks that pull water out of soil other desert plants cannot reach.
For example, the roots of the mesquite bush, said to have the deepest root systems among desert plants, can
reach depths of up to eighty feet.
Succulents and xerophytes, on one hand, have physically adapted to gather and retain water to survive
long periods of drought. Drought-avoiding plants, on the other hand, escape unfavorable conditions by
perishing. These include annuals and perennials. Because profuse seed development is crucial to the survival
of most annual species, they tend to produce far more flowers than other types of desert plants. The desert
marigold of the American southwest, for example, has adapted to seasonal changes in rainfall by growing
rapidly, bursting into a brilliant display of yellow flowers, and then dropping a cascade of seeds before dying.
In some cases, annuals complete their entire life cycle in a matter of a few weeks or months. Their seeds may
remain inactive for up to ten years while waiting for wetter conditions. Perennials, such as the ocotillo, may
go dormant during dry periods, spring to life when it rains, and then return to dormancy in a process that may
occur up to five times per year.
1. According to paragraph 1, which of the following is true of desert plants?
A. They are hard to see in the harsh desert landscape.
B. They have evolved similar features, regardless of geographic origins.
20

Downloaded by Nhung Nguy?n (nhungnth2004@gmail.com)


lOMoARcPSD|20277325

C. They have evolved from different families that exchanged survival strategies.
D. They all look essentially the same.
2. The word ephemeral in the passage is closest in meaning to
A. minimal B. yearly C. short-lived D. abundant
3. The word respiration in the passage is closest in meaning to
A. nutrition B. breathing C. growing D. coloration
4. Why does the author mention the action of stomata in the passage?
A. To explain how chlorophyll works
B. To emphasize a unique adaptation of succulents
C. To illustrate that cacti have no leaves
D. To describe the basic parts of a succulent
5. The word desiccation in the passage is closest in meaning to
A. destruction B. growth C. adaptation D. drying
6. The word others in the passage refers to
A. grasses B. shrubs C. coatings D. leaves
7. Which of the following best expresses the essential information in the highlighted sentence in the passage?
Incorrect choices change the meaning in important ways or leave out essential information.
A. At the same time that xerophytes evolved their thick webs of low-lying roots, succulents formed thin
networks to reach surface water.
B. Both succulents and xerophytes form long, dense networks of roots in order to reach water within the soil
that non-desert plants could not reach.
C. Because of their long roots systems developed to reach water deep within the soil, xerophytes have become
more dominant in desert environments than succulents.
D. In contrast to xerophytes, which produce long root systems to tap water deep within the soil, succulents
develop a thick web of roots just below the soil surface.
8. According to paragraph 4, thorns on xerophytes
A. are similar to blades of grass
B. can reach water very far underground
C. are what some plants have instead of leaves
D. help keep the plants from being eaten by animals
9. In what form do drought-avoiding annuals wait for wetter weather?
A. stems B. flowers C. roots D. seeds
10. According to the author, all of the following are associated with plants that exhibit "minimum variance
behavior" EXCEPT
A. slow growth, B. modified respiration
C. rapid seed production D. deep root systems
Passage 2 MAKING EVERY DROP COUNT
A. The history of human civilisation is entwined with the history of the ways we have learned to
manipulate water resources. As towns gradually expanded, water, was brought from increasingly
remote sources, leading to sophisticated engineering efforts such as dams and aqueducts. At the height
of the Roman Empire, nine major systems, with an innovative layout of pipes and well-built sewers,
supplied the occupants of Rome with as much water per person as is provided in many parts of the
industrial world today.
B During the industrial revolution and population explosion of the 19th and 20th centuries, the demand
for water rose dramatically. Unprecedented construction of tens of thousands of monumental
engineering projects designed to control floods, protect clean water supplies, and provide water for
irrigation and hydropower brought great benefits to hundreds of millions of people. Food production
has kept pace with soaring populations mainly because of the expansion of artificial irrigation systems
that make possible the growth of 40% of the world's food. Nearly one fifth of all the electricity
21

Downloaded by Nhung Nguy?n (nhungnth2004@gmail.com)


lOMoARcPSD|20277325

generated worldwide is produced by turbines spun by the power of falling water.


C. Yet there is a dark side to this picture: despite our progress, half of the world's population still suffers,
with water services inferior to those available to the ancient Greeks and Romans. As the United
Nations report on access to water reiterated in November 2001, more than one billion people lack
access to clean drinking water; some two and a half billion do not have adequate sanitation services.
Preventable water-related diseases kill an estimated 10,000 to 20,000 children every day, and the latest
evidence suggests that we are falling behind in efforts to solve these problems.
D. The consequences of our water policies extend beyond jeopardising human health. Tens of millions of
people have been forced to move from their homes - often with little warning or compensation - to
make way for the reservoirs behind dams. More than 20% of all freshwater fish species are now
threatened or endangered because dams and water withdrawals have destroyed the free-flowing river
ecosystems where they thrive. Certain irrigation practices degrade soil quality and reduce agricultural
productivity. Groundwater aquifers are being pumped down faster than they are naturally replenished
in parts of India, China, the USA and elsewhere. And disputes over shared water resources have led to
violence and continue to raise local, national and even international tensions.
E. At the outset of the new millennium, however, the way resource planners think about water is
beginning to change. The focus is slowly shifting back to the provision of basic human and
environmental needs as top priority - ensuring ‘some for all,’ instead of 'more for some’. Some water
experts are now demanding that existing infrastructure be used in smarter ways rather than building
new facilities, which is increasingly considered the option of last, not first, resort. This shift in
philosophy has not been universally accepted, and it comes with strong opposition from some
established water organisations. Nevertheless, it may be the only way to address successfully the
pressing problems of providing everyone with clean water to drink, adequate water to grow food and a
life free from preventable water-related illness.
F. Fortunately - and unexpectedly the demand for water is not rising as rapidly as some predicted. As a
result, the pressure to build new water infrastructures has diminished over the past two decades.
Although population, industrial output and economic productivity have continued to soar in developed
nations, the rate at which people withdraw water from aquifers, rivers and lakes has slowed. And in a
few parts of the world, demand has actually fallen.
G. What explains this remarkable turn of events? Two factors: people have figured out how to use water
more efficiently, and communities are rethinking their priorities for water use. Throughout the first
three-quarters of the 20th century, the quantity of freshwater consumed per person doubled on average;
in the USA, water withdrawals increased tenfold while the population quadrupled. But since 1980, the
amount of water consumed per person has actually decreased, thanks to a range of new technologies
that help to conserve water in homes and industry. In 1965, for instance, Japan used approximately 13
million gallons of water to produce $1 million of commercial output; by 1989 this had dropped to 3.5
million gallons (even accounting for inflation) - almost a quadrupling of water productivity. In the
USA, water withdrawals have fallen by more than 20% from their peak in 1980.
H. On the other hand, dams, aqueducts and other kinds of infrastructure will still have to be built,
particularly in developing countries where basic human needs have not been met. But such projects
must be built to higher specifications and with more accountability to local people and their
environment than in the past. And even in regions where new projects seem warranted, we must find
ways to meet demands with fewer resources, respecting ecological criteria and to a smaller budget.
Choose the correct heading for paragraph B - H from the list of the headings below.
List of headings
I Scientists’ call for a revision of policy
II An explanation for reduced water use
III How a global challenge was met
IV Irrigation systems fall into disuse
22

Downloaded by Nhung Nguy?n (nhungnth2004@gmail.com)


lOMoARcPSD|20277325

V Environmental effects
VI The financial cost of recent technological improvements
VII The relevance to health
VIII Addressing the concern over increasing populations
IX A surprising downward trend in demand for water
X The need to raise standards
XI A description of ancient water supplies
Example: Paragraph A: XI
1.Paragraph B 2. Paragraph C 3. Paragraph D
4.Paragraph E 5. Paragraph F 6. Paragraph G 7. Paragraph H
Do the following statements agree with the information given in Reading Passage 2? For question 8-10
write down
YES if the statement agrees with the claims of the writer
NO if the statement contradicts the claims of the writer
NOT GIVEN if it is impossible to say what the writer thinks about this
8. Feeding increasing populations is possible due primarily to improved irrigation systems.
9. Modern water systems imitate those of the ancient Greeks and Romans.
10. Industrial growth is increasing the overall demand for water.
V. GUIDED CLOZE TEST
Read the following passage and choose the options that best complete the blanks.
PASSAGE 1
The idea that only an Italian is (1)………….of making the world’s greatest cup of coffee seems to
have been around forever, so universally is it (2) …………. today. (3) …………. it is actually a fairy recent
phenomenon. Emilio Lavazza, who died in 2010 at the age of 78, can (4) ………….much of the credit. He
taught the world not only how to make coffee, but also how to drink it. That may explain why Italy has still
not been invaded by the American coffee - bar chains so ubiquitous elsewhere in Europe.
Emilio Lavazza was born in 1932, and was a leading figure in the generation of Italian businessmen
who (5) …………. their family firms in the 1950s. These began to expand rapidly, first around the country
and then abroad as Italy (6) …………. its long post - war economic expansion. This was the generation that
(7) ………….the seeds for what has (8) …………. to be known as ‘Made in Italy’, the (9) ………….of
companies and brands that make high - quality household and consumer products, from fashion to food to
furniture. These products are identified with a (10) ………….of craftsmanship on the one hand, and the
elegant Italian lifestyle on the other. Emilio Lavazza made sure that coffee became an inextricable part of that
heritage.
1. A. capable B. skilled C. qualified D. competent
2. A. consented B. agreed C. accepted D. complied
3. A. Yet B. Though C. Whereas D. Whilst
4. A. insist B. claim C. demand D. uphold
5. A. enlisted B. joined C. enrolled D. participated
6. A. entertained B. appreciated C. benefited D. enjoyed
7. A. set B. sowed C. laid D. buried
8. A. ended B. come C. finished D. gone
9. A. cluster B. pile C. bundle D. heap
10. A. range B. connection C. variety D. combination
PASSAGE 2
The growth of cheap and readily available air travel has helped turn tourism into one of the world’s
largest industries. But behind the image projected by the glossy brochures, there is a less positive (1) ............
to the situation. In Goa, for example, indigenous peoples have been displaced to (2) ............... way for hotels
and golf courses, and local fishermen have been (3) .............. access to the beaches from which they set off for
23

Downloaded by Nhung Nguy?n (nhungnth2004@gmail.com)


lOMoARcPSD|20277325

work. One (4) ............ study has shown that 60 per cent of foreign capital (5) ................ in tourism in
developing countries returns to first - world countries, and although tourism often provides employment,
much of it is low - skilled and very poorly paid.
Recently, however, a British organization, Tourism Concern, surprised everyone by mounting an
(6) .......... vigorous campaign for tourism which respects the rights of local people. They argue that local
people are at present nowhere near as involved as they should be in decisions about how to (7) ............. their
area for tourism. Furthermore, they believe tourists should be actively encouraged to use what is often
perfectly satisfactory accommodation offered by local communities, and to fly with airlines (8) ………….in
the host countries. This would (9) ………….that jobs are provided at all (10) ………….and that capital does
not disappear out of the country.
1. A. face B. section C. edge D. side
2. A. leave B. make C. get D. move
3. A. rejected B. denied C. disallowed D. stopped
4. A. comprehensive B. inclusive C. complete D. total
5. A. provided B. placed C. speculated D. invested
6. A. extensively B. extremely C. utterly D. actually
7. A. grow B. inflate C. reproduce D. develop
8. A. based B. created C. stood D. laid
9. A. promise B. agree C. undertake D. ensure
10. A. heights B. planes C. levels D. steps

B. WRITTEN TEST
I. OPEN CLOZE
Fill in each gap with one suitable word.
Passage 1
THE TROUBLE WITH SCHOOL
In the first few years at school all appears to (1)………….. very well. There is much concern, (2)
………….. the part of the teachers, with high educational standard, and the children, even those who are (3)
………….. from being socially privileged in other ways, seem eager and happy. However, by the time the
children reach adolescence, the promise of the early years frequently remains unfulfilled. Many leave school
(4)………….. having mastered those basic skills which society demands, let (5)………….. having developed
the ability to exercise any sort of creative intelligence.
There is no denying that, in spite of the enlightened concern of our primary schools with happiness,
schooling (6)………….. or other turns into a distinctly unhappy experience for many of our children. Large
numbers of them emerge from it well (7)………….. that they are ill - equipped for life in our society. So then
they either regard (8)………….. as stupid for failing or else, quite understandably, they regard the
activities at (9) ............ they have failed as stupid. In any event they want no (10) ………….. of them. How
can we justify a long period of compulsory education which ends like that?
Passage 2
RELAXATION
True relaxation is most certainly not a matter of flopping down in front of the television with a
welcome drink. Nor is it about drifting (1)…………….an exhausted sleep. Useful though these responses to
tension and over - tiredness (2) ……………be, we should distinguish between them and conscious relaxation
in (3) ……………of quality and effect. (4) ……………of the level of tiredness, real relaxation is a state of
alert yet at the same time passive awareness, in which our bodies are at rest while our minds are awake.
Moreover, it is as natural (5) ……………a healthy person to be relaxed when moving as resting.
Being relaxed in action means we bring the appropriate energy to everything we do, (6) ……………as to have
a feeling of healthy tiredness by the end of the day, rather than one of exhaustion.
Unfortunately, as a (7) …………… of living in today’s competitive world, we are under constant
24

Downloaded by Nhung Nguy?n (nhungnth2004@gmail.com)


lOMoARcPSD|20277325

strain and have difficulty in coping, (8) …………… alone nurturing our body’s abilities. (9)………….needs
to be rediscovered is conscious relaxation. With that in mind we must apply ourselves to understanding stress
and the nature of its causes, (10) ……………deep - seated.
II. WORD FORM
Part 1. Supply the correct form of the word in the bracket in each sentence.
1. You’d better ask Frank, I’m not in………….. ..(POSSESS) of the manual you are looking for.
2. Thanks to the operation my ………….. .(SEE) has improved considerably. I hope that after the next one I
won’t have to wear glasses at all.
3. The English term café, borrowed from French, is ultimately a………….. (DERIVE) of the Turkish kahve,
meaning coffee.
4. Cats are not considered to be social animals in the sense that they have never………….. (HABIT) travelled
in packs or adopted leaders.
5. With two ………….. (LYRIC) Charles Hart and Richard Stilgoe, Lloyd Webber composed a hugely
successful version of‘The Phantom Of The Opera’ .
6. The Thompsons have helped us a lot in these hard times. We do feel………….. (DEBT) to them for their
great support.
7. ‘Have you got any………….. (GIVE) about the cooperation?’ - ‘Oh no, I’m sure it will be successful.’
8. I was overcome with great………….. (BITTER) when my invitation was rejected by most of my friends.
9. Cigarettes, coffee, alcohol and other ………….. (ADD) are known to have an adverse influence upon
human health.
10. The Bulgarian sportsman was immediately………….. (QUALIFY) after the traces of steroids were
discovered in his blood samples.
Part 2
Complete the following passage with the correct forms of the given words.
NECESSARY RELY VISION EXCEED GRADE
SIGNIFICANT MODE DESCEND ADVANTAGE PERSUADE

Ancient man used sticks of charcoal to draw pictures on cave walls in order to communicate (with,
probably, their deities and trainee huntsmen). Today, some of their direct (1) ……………are still using ‘chalk
and talk’ and other (2) ……………equipment to make presentations to sophisticated business audiences.
Now, there’s nothing wrong with whiteboards, flip charts and overhead projectors. In their right
context, they are still (3) …………… useful presentation tools. But in a business environment in which the
presentation of clear, easily understandable information is a (4) …………… and in which memorability is
key, managers should be constantly (5) ……………their equipment to keep pace with developments.
Audiences are coming to expect high - quality presentations that are (6) ……………stimulating and get the
message across without wasting time. Professionally - made presentations clearly (7) ……………that the
person giving them has thought through the issues and knows what they are talking about. They can put a (8)
……………case that wins over an audience in a way that pieces of paper can’t. And they can put you, or your
company, in the most (9) ……………light possible by delivering a well thought - out message (10)
………….every time.
Ill ERROR IDENTIFICATION
There are ten mistakes in the following paragraph. Find them then give the correction.
Adolescence always has been and always will be a very difficult time in life. You are lost where
between childhood and adulthood, but still, this is the time in life when you have to break freely from the
conformity of your peers to find yourself. Some people argue that it’s more difficult to be young today than it
used to be. Is this true, and in that case, why?
In modern society teenagers pressure to mature much more quickly than one or two hundred years ago.
Today, minors on a very early stage have to make decisions regard education, often bearing upon their future
careers. In the past children were expected to follow-in the footsteps of their parents, that is to say, the son was
25

Downloaded by Nhung Nguy?n (nhungnth2004@gmail.com)


lOMoARcPSD|20277325

supposed to take in the profession of his father, while the daughter was expected to stay at home to take care
of domestic duties such as cooking and cleaning.
Furthermore, today it’s much more difficult to find your place in society. As cities grow, crime
increases, and the aponymity people experiment grow as well. It becomes more difficult to find and cultivate
your own ideals and values.
On the other hand, the adolescent ọf today have great opportunities than ever before. In the past, if
your father was a blacksmith or a farmer, in ten years, so you would be. Today, teenagers have the possibility
to fulfil in all their dreams and ambitions.
IV. SENTENCE TRANSFORMATION
Rewrite the sentences with the given words or beginning in such a way that their meanings remain
unchanged.
1. Nobody knows what caused the collapse of the building.
It’s …………………………………………………………………………………………………….
2. We aren’t willing to support the strike, we don’t approve of it.
Far from………………………………………………………………………………………………..
3. His mood began to improve as soon as he drank a glass of wine.
Scarcely ………………………………………………………………………………………………..
4. Were Mike not so affluent a man, Ann wouldn’t be dating him.
But………………………………………………………………………………………………………
5. Apart from a few minor mistakes, you did a good job on the whole.
By and …………………………………………………………………………………………………..
6. I expected the book to be far better because it had been written by such a good novelist. (SHORT)
The book ………………………………………………………………………………………………..
7. She was concentrating so hard on her work that she didn’t notice when I came in. (WRAPPED)
She………………………………………………………………………………………………………
8. She’s not very good at arranging flowers. (FLAIR)
She………………………………………………………………………………………………………
9. Diane finds that creating things stops her from thinking about her work. (MIND)
Diane finds that being ………………………………………………………………………………….
10.The manager praised one particular player. (SINGLED)
One particular player……………………………………………………………………………………
TRƯỜNG THPT CHUYÊN TRẦN ĐẠI NGHĨA - TP. HỒ CHÍ MINH
A. MULTIPLE CHOICE QUESTIONS
I. PHONOLOGY
A. Choose the word whose underlined part is pronounced differently from that of the others of the
group
1. A. enthusiast B. tsunami C. optimistic D. philosopher
2. A. destroyed B. developed C. conquered D. closed
3. A. sociable B. species C. specimen D. specialist
4. A. rose B. house C. mouse D. practise
5. A. language B. grammar C. international D. many
B. Choose the word that has the main stress placed differently from that of the other three
6. A. intellectual B. accelerate C. eliminate D. eradicate
7. A. refugee B. referee C. attendee D. committee
8. A. determine B. institute C. industry D. negligence
9. A. apartheid B. abundant C. estimate D. embarrass
10. A. vulnerable B. incredible C. abominable D. responsible
II. USE OF ENGLISH: Word choice/ Structure and Grammar
1. Mary usually buys her clothes…………. It’s cheaper than going to a dressmaker.
26

Downloaded by Nhung Nguy?n (nhungnth2004@gmail.com)


lOMoARcPSD|20277325

A. in public B. on the shelf C. off the peg D. on the house


2. Kathy was as pleased as………….when she heard she had passed the exam.
A. punch B. a poppy C. a sunflower D. a dungeon
3. I’ve heard that argument before and quite frankly it just doesn’t………….!
A. hold water B. face the music C. carry weight D. hit the roof
4. When I tell my parents that we’ve decided to call off the wedding, they’re going to have………….
A. goose B. kitten C. rat D. donkey
5. It is advisable to ………….any contact with potential rabit animals.
A. escape B. avoid C. prevent D. evade
6. Please hand in a concise report on this issue to me by tomorrow morning.
A. brief B. bountiful C. modified D. abundant
7. I couldn’t tolerate my mother’s incessant nagging.
A. endless B. bothersome C. annoying D. repetitive
8. It took me quite ạ long time to………….what the aim of the game was.
A. ponder B. consider C. grasp D. reckon
9. Nancy has the………….to be a great dancer.
A. possibility B. prospect C. quality D. ability
10. The car was………….speed.
A. gathering B. collecting C. consuming D. firing
11.…………., I would give a party.
A. Were she to come B. Were she coming
C. Were she come D. She were coming
12. Jane was not………….the landlady had imagined she was.
A. what B. that C. which D. at all
13.I am no………….able to operate this machine than he is.
A. far B. more C. much D. less
14. The weather forecast was for rain, but the day was fine and dry,………….it turned out
A. when B. though C. like D. as
15. What………….make is your car? It’s………….Toyota Altis.
A. the/a B. the/the C. …/…. D. …/ a
16. He………….upon himself the responsibility for protecting her.
A. looked B. took C. got D. brought
17. There…………..
A. does my bus come B. my bus comes
C. Comes my bus D. my bus does come
18. You were the last one………….the office, weren’t you?
A. leaving B. left C. to leave D. leave
19. Our plane tickets never arrived,………….in the post.
A. must have been B. having been lost
C. having lost D. could have been lost
20. When Kevin arrived, he soon………….at all his jokes.
A. had everyone laughed B. had laughing everyone
C. had laughed everyone D. had everyohe laughing
Prepositions and Phrasal Verbs:
1. The boss was away so his assistant had to………….him and make a speech.
A. make off with B. do away with C. stand in for D. take up on
2. They………….that movie so much. It’s bound to be a letdown.
A. brought in B. played up C. hyped up D. ran through
3. I hate the way Carol………….people………….behind their backs.
27

Downloaded by Nhung Nguy?n (nhungnth2004@gmail.com)


lOMoARcPSD|20277325

A. runs...down B. plays...down C. lays...down D. batters...down


4. I’m so………….with work at the moment - it’s awful.
A. iced under B. rained under C. snowed under D. fogged under
5. Keep………….and you’ll finish your novel eventually.
A. taking on B. tiding over C. lying ahead D. plodding away
6. The area has been………….following a bomb threat
A. cordoned off B. headed off C. stopped off D. knocked off
7. Rudy was really opposed to the idea at the start, but we managed to………….him………….in the end.
A. catch...on B. talk..round C. put...across D. blurt...out
8. If you ignore it, you’re just………….problems for the future.
A. taking in B. storing up C. setting out D. moving over
9. I’ve got to………….the French Revolution for tomorrow’s test.
A. patch up B. dive in C. bump into D. swot up
10. Could you lend me $50 to………….me………….until pay day?
A. push...around B. mount.. .up C. grow...on D.. tide.. .over
III. READING COMPREHENSION
Reading 1:
IMAGE AND THE CITY
In the city, we are barraged with images of the people we might become. Identity is presented as
plastic, a matter of possessions and appearances; and a very large proportion of the urban landscape is taken
up by slogans, advertisements, flatly photographed images of folk heroes - the man who turned into a
sophisticated dandy overnight by drinking a particular brand of drink, the girl who transformed herself into a
femme fatale with a squirt of cheap scent. The tone of the wording of these advertisements is usually pert and
facetious, comically drowning in its own hyperbole. But the pictures are brutally exact: they reproduce every
detail of a style of life, down to the brand of cigarette-lighter, the stone in the ring and the economic row of
books on the shelf.
Yet, if one studies a line of ads across from where one is sitting on a tube train, these images radically
conflict with each other. Swap the details about between the pictures, and they are instantly made illegible if
the characters they represent really are heroes, then they clearly have no individual claim to speak for society
as a whole. The clean-cut and the shaggy, rakes, innocents, brutes, home-lovers, adventurers, clowns all
compete for our attention and invite emulation. As a gallery, they do provide a glossy mirror of the aspirations
of a representative city crowd: but it is exceedingly hard to discern a single dominant style an image of how
most people would like to see themselves.
Even in the business of the mass-production of images of identity, this shift from the general to the
diverse and particular is quite recent. Consider another line of stills: the back-lit, soft-focus portraits of the
first and second generations of great movie stars. There is a degree of romantic unparticularity in the face of
each one, as if they were communal dream-projections of society at large. Only in the specialised genres of
Westerns, farces and gangster movies were stars allowed to have odd, knobbly cadaverous faces. ,The hero as
loner belonged to history or the underworld: he spoke from the perimeter of society, reminding us of its
dangerous edges.
The stars of the last decade have looked quite different. Soft-focus photography has gone, to be
replaced by a style which searches out warts and bumps, emphasises the uniqueness not the generality of the
face. Voices, too, are strenuously idiosyncratic: whines, stammers and low rumbles are exploited as features
of'star quality'. Instead of romantic heroes and heroines, we have a brutalist, hard-edged style in which
isolation and egotism are assumed as natural social conditions.
In the movies, as in the city, the sense of stable hierarchy has become increasingly exhausted; we no
longer live in a world where we can all share the same values, the same heroes. (It is doubtful whether this
world, so beloved of nostalgia moralists, ever existed; but lip-service was paid to it, the pretence, at least was
kept up.) The isolate and the eccentric push towards the centre of the stage; their fashions and mannerisms are
28

Downloaded by Nhung Nguy?n (nhungnth2004@gmail.com)


lOMoARcPSD|20277325

presented as having as good a claim to the limelight and the future as those of anyone else. In the crowd on the
underground platform, one may observe a honeycomb of fully-worked-out worlds, each private, exclusive,
bearing little comparison with its nearest neighbour. What is prized in one is despised in another. There are no
clear rules about how one is supposed to manage one’s body, dress, talk, or think. Though there are elaborate
protocols and etiquettes among particular cults and groups within the city, they subscribe to no common
standard.
For the new arrival, this disordered abundance; is the city’s most evident and alarming quality. He
feels as if he has parachuted into a funfair of contradictor imperatives. There are so many'people he might
become' and a suit of clothes, a make of car, a brand of cigarettes' will go some way towards turning him into
a personage even before he has discovered who that personage is. Personal identity has always been deeply
rooted in property, but hitherto the relationship has been a simple one - a question of buying what you could
afford, and leaving your wealth to announce your status. In the modern city, there are so many things to buy,
such a quantity of different kinds of status, that the choice and its attendant anxieties have created a new
pornography of taste.
The leisure pages of the Sunday newspapers, fashion magazines, TV plays, popular novels, cookbooks,
window displays all nag at the nerve of our uncertainty and snobbery. Should we like American cars, hard-
rock hamburger joints, Bauhaus chairs...? Literature and art are promoted as personal accessories: the
paintings of Mondrian or the novels of Samuel Beckett 'go' with certain styles like matching handbags., There
is in the city a creeping imperialism of taste, in which more and more commodities are made over to being
mere expressions of personal identity. Tire piece of furniture, the pair of shoes, the book, the film, are
important not so much in themselves but for what they communicate about their owners; and ownership is
stretched to include what one likes or believes in as well as what one can buy.
1. What does the word ‘barraged’ mean?
A. manipulated B. bombarded C. impressed D. obsessed
2. What does the writer say about advertisements in the first paragraph?
A. Certain kinds are considered more effective in cities than others
B. The way in which some of them are worded is cleverer than it might appear.
C. They often depict people that most other people would not care to be like.
D. The pictures in them accurately reflect the way that some people really live.
3. The writer says that if you look at a line of advertisements on a tube train, it is clear that
A. city dwellers have very diverse ideas about what image they would like to have.
B. some images in advertisements have a general appeal that others lack.
C. city dwellers are more influenced by images on advertisements than other people are.
D. some images are intended to be representative of everyone’s aspirations.
4. What does the writer imply about portraits of old movie stars?
A. Their tried to disguise the less attractive features of their subjects.
B. Most people did not think they were accurate representations of the stars in them.
C. They made people feel that their own faces were rather unattractive.
D. They reflected an era in which people felt basically safe.
5. What does the word ‘cadaverous’ mean?
A. extremely pale and thin B. energetic and enthusiastic
C. dangerous D. skeptical
6. What does the writer suggest about the stars of the last decade?
A. Some of them may be uncomfortable about the way they come across.
B. They make an effort to speak in a way that may not be pleasant on the ear.
C. They make people wonder whether they should become more selfish.
D. Most people accept that they are not typical of society as a whole.
7. What does the word ‘hierarchy’ mean?
A. methodology B. hypothesis C. ideology D. system
29

Downloaded by Nhung Nguy?n (nhungnth2004@gmail.com)


lOMoARcPSD|20277325

8. The writer uses the crowd on an underground platform to exemplify his belief that
A. no single attitude to life is more common than another in a city.
B. no one in a city has strict attitudes towards the behaviour of others.
C. views of what society was like in the past are often inaccurate.
D. people in cities would like to have more in common with each other’
9. The writer implies that new arrivals in a city may
A. change the image they wish to have too frequently’
B. underestimate the importance of wealth.
C. acquire a certain image without understanding what that involves.
D. decide that status is of little importance.
10. What point does the writer make about city dwellers in the final paragraph?
A. They are unsure as to why certain things are popular with others.
B. They are aware that judgements are made about them according to what they buy.
C. They want to acquire more and more possessions.
D. They are keen to be the first to appreciate new styles.
Reading 2:
PARENTHOOD
Paul watched the television above the bar. An army of turtles waddled up a beach, cumbersome
helmets dragged through the fine sand to deposit a clutch of smooth, white eggs in the dunes. He saw the
wriggling reptilian babies emerge sticky from the broken shells and repeat the journey in reverse, thousands of
tiny helmets trundling inexorably over the moonlit dunes towards the breakers. Those who escaped being
flipped over on their backs and pecked to death by wading birds were finally swallowed up in the surf. There
was no pleasure involved in this reptilian cycle of birth and death. The turtles survived purely because there
were so many of them, and the Oceans were so vast, that one or two were bound to slip through unnoticed.
He wondered why they bothered, and presumed it could only be because they had no choice. Their
genes forced them ever onwards - life would not be denied. Previous generations had imposed their will upon
their distant descendants, and the descendants wearily obeyed. If, by chance, a turtle was born in whom this
instinct towards multiplication was misformed or absent, a turtle whose instincts directed them not towards
reproduction but towards reflection on the purpose of reproduction, say, or towards seeing how long it could
stay underwater on one breath, then this instinct would die with the turtle. The turtles were condemned to
multiply purely by the breeding success of their own ancestors. There was no escape for them. Multiplication,
once set in motion, was unstoppable.
At the present moment, the balance of his own inclinations tilted more towards sleep, the cessation of
thought, hibernation, vegetation. Had he been one of those tiny helmets, he would, at that moment, have
flipped over belly-up in the sand and simply awaited the releasing beak. Parenthood had taken him by
surprise. The books, the articles, the classes, had not prepared him for the intensity of it all. Snap decisions to
be made, everybody looking to him for the answers, and no way of knowing if he had made the correct guess,
no way of finding his way back to the main track if he took a wrong turning. Last night he had been half a
couple. He had lived with others all his life. It was easy -you had rows, you had resentments, but if they
became too frequenij or too boring, or if the compensations ceased to be adequate, you just left, and tried
again with someone else until you found someone you could put up with. He could noti remember how it had
all changed. Perhaps it had been the doors of youth and liberty creaking shut behind him, or the demands that
were suddenly being made of him, the faces turning towards him when a decision was required. Or perhaps it
was just the steaming concoction of his emotions, his hormones, his thoughts slopping around his veins with
the coffee and nicotine. Whatever it was, something had obliged him to seek out a tranquil place in order to
restore some order to his metabolism.
Then there was the feeling that he had been duped - the one feeling that he hadn't been warned of -
when he saw mother and baby together and realized that the reason why everyone made such a big deal of
fatherhood these days was simply because it was such an implausible state. Mothers and babies were the
30

Downloaded by Nhung Nguy?n (nhungnth2004@gmail.com)


lOMoARcPSD|20277325

world. Fathers werej optional extras, accessories. If some strange virus colonized the Y-chromosome and
poisoned all the men, the world would carry on. It would not be a very exciting world perhaps, rather bland
and predictable, but women would find some way to reproduce, and within a generation or two it would be
difficult to believe that there had ever been men at all. They would appear in the encyclopaedias somewhere
between dinosaurs and Romans. Future generations of little girls would try, in vain, to understand what it had
been that men had done, how they had contributed. What use had they been? He had suddenly seen his role
exposed as that of a footnote. The books had warned him of this feeling, of jealousy, of irrelevance and
superfluity. They had said it was natural, that he would get over it. What they had not said was that it was
natural because it was so manifestly, poignantly true, or that he would get over it only by stopping thinking
about it. Fathers deceived themselves. Mothers and babies held it all together. The men came and went,
interchangeably, causing trouble and bringing presents to make up for it.
He turned his attention to the television. The tiny helmets he had watched clawing their way down
towards the surf had become parents themselves now. You could tell they were the same turtles, because the
scientists had painted fluorescent hieroglyphics on their shells. They returned to the beach on which they had
hatched, and the credits rolled.
1.What did Paul notice about the turtles in the first paragraph?
A. their reluctance to return to the sea
B. their behaviour with their young
C. the effort they made to survive
D. the tiny proportion of young who survived
2. What does the word ‘inexorably’ mean?
A. unstoppably B. inexplicably C. inevitably D. inadvisably
3. Paul assumed that if a turtle did not wish to reproduce,
A. it is would be punished by other turtles. B. it would end up doing so anyway.
C. this attitude would not spread to other turtles. D. this would not come as a surprise.
4. His thoughts turned towards going to sleep because
A. he knew that he was unlikely to get much in the near future.
B. he had been left mentally exhausted by becoming a parent.
C. he had become weary of his actions being criticized.
D. he felt that thạt was what many o{ the turtles probably wanted to do.
5. What does the word ‘resentment’ mean?
A. anger B. suspicion C. jealousy D. confusion
6. What did he feel he had been forced to do since last night?
A. accept that he was not really cut out for living with other people
B. find a way pf making himself feel better physically
C. identify precisely what had caused his life to change so radically
D. remind himself of howr he had felt prior to this
7. In what way did he feel that he had been duped?
A. He had expected his role to be one that differed from that of most men.
B. He had not been informed about how women changed when they became mothers.
C. He had not been told the truth by women about how they really regarded men.
D. He had thought fatherhood was treated as a major subject because fathers were important.
8. He felt that the books had failed to warn him that his feeling of irrelevance
A. would not fade away naturally. B. would not be shared by others.
C. would be replaced by worse feelings. D. would reduce him to inactivity.
9. What does the word ‘duped’ mean?
A. shocked B. fascinated C. cheated D. appealed
10. What is implied about events on the television programme?
A. They made him more depressed than he would otherwise have been.
31

Downloaded by Nhung Nguy?n (nhungnth2004@gmail.com)


lOMoARcPSD|20277325

B. They made him feel that turtles were better off than humans.
C. They reflected his own lack of joy at becoming a father.
D. They gave him a Chance to escape{rom his own thoughts.
IV. CLOZE TEST:
Cloze Test 1:
HOLIDAY COMPLAINTS
There are too many people in Britain today who are ready to complain rather than enjoy their holidays
and who ……………(1) the word about how easy it is to get money back from the travel industry which
appears to have become paralysed with fear over bad…………… (2)
I recently heard a middle-aged woman protesting bitterly that she had to pay for the …………… (3) of
a golf cart during an all-inclusive golfing holiday in Jamaica and demanded back all the money spent on the
cart together with more for the ‘distress’ she had been…………… (4). A friend of mine recently overheard a
group of young lads deciding, before they even…………… (5) the aircraft, what they were going to complain
about to get a refund on the cost of their holiday and which high-value item would be ‘lost’ during the
…………… (6).
Last month a Manchester man led a ‘mutiny’ of 200 passengers on a…………… (7) ship because, he
said, the bingo prize was not big enough. This week I heard details of a honeymoon…………… (8) who were
sent a big bouquet of flowers by the tour operator and then complained that it was too big and…………… (9)
them from moving freely about their hotel room. These types, who we have all met in airport………...(10) ,
just make the rest of us suffer, when they lie and fool tour operators into handing cash back.
1. A. spread B. give C. pass D. tell
2. A. review B. publicity C. media D. reputation
3. A. loan B. rent C. let D. hire
4. A. made B. suffered C. done D. caused
5. A. took B. landed C. boarded D. caught
6. A. travel B. ride C. excursion D. trip
7. A. ferry B. tour C. cruise D. cargo
8. A. couple B. pair C. duo D. double
9. A. harmed B. limited C. prevented D. reduced
10. A. spaces B. lounges C. halls D. sitting-rooms

Cloze Test 2:
ENGLISH SPELLING
Why does English spelling have a reputation for being difficult? English was first written down when
Christian monks came to England in Anglo-Saxon…………… (1). They used the 23 letters of Latin to write
down the sounds of Anglo-Saxon …………… (2) as they heard it. However, English has a wider range of
basic sounds (over 40) than Latin. The alphabet was too small, and so combinations of letters were needed
to…………… (3) the different sounds. Inevitably, there were inconsistencies in the way that letters were
combined.
With the Norman invasion of England, the English language was put at risk. English survived, but the
spelling of many English words changed to follow French…………… (4), and many French words
were…………… (5) into the language. The result was more irregularity.
When the printing press was invented in the fifteenth century, many early printers of English texts
spoke other first languages. They …………… (6) little effort to respect English spelling. Although one of the
short-term …………… (7) of printing was to produce a number of variant spellings, in the long term it
created fixed spellings. People became used to seeing words spelt in the same way. Rules were ……………
(8), and dictionaries were put together which printers and writers could refer to. However, spoken English was
not fixed and continued to change slowly - just as it still does now. Letters that were sounded in the Anglo-
Saxon period, like the ‘k’ in 'knife’, now became …………… (9). Also, the pronunciation of vowels then had
32

Downloaded by Nhung Nguy?n (nhungnth2004@gmail.com)


lOMoARcPSD|20277325

little in common with how they sound now, but the way they are spelt hasn’t changed. No…………… (10),
then, that it is often difficult to see the link between sound and spelling.
1. A. times B. centuries C. ages D. years
2. A. chat B. communication C. speech D. discussion
3. A. explain B. tell C. perform D. express
4. A. guides B. plans C. patterns D. types
5 A. announced B. found C. started D. introduced
6, A. made B. did C. brought D. put
7, A. actions B. effects C. conclusions D. meanings
8. A. handed out B. filled in C. got across D. drawn up
9. A. dumb B. quiet C. speechless D. silent
10. A. idea B. wonder C. problem D. mention
B. WRITTEN TEST
I. OPEN CLOZE TEST
Open Cloze Test 1:
When rainforests are cleared and [1] ……………,millions of carbon dioxide are released into the
atmosphere affecting climatic conditions and threatening us all [2] ……………severe flooding, drought and
drop failure. The rainforests [3] …………… at least half of the earth’s species. At the current rate of
devastation an [4] …………… 50 species worldwide become extinct every day.
One in four purchases from our chemists is derived from the rainforests. Scientists are [5] …………
caught in a race against time to find rainforest treatments for cancer, AIDS and heart disease before they are
[6] ………… forever. Tribal people in the rainforests have been shot, poisoned and infected with diseases to
which they have no resistance - to make room for logging, mining and dams. If this destruction continues,
only nine [7] …………… the 33 countries currently exporting rainforest timber will have any [8] ……………
by the end of the decade.
Almost everyone will have part of the rainforests in their home, as do-it-yourself stores still supply and
the construction industry still uses tropical hardwoods for doors, window [9] ……………and even toilet seats.
Please help us [10] ……………the tropical rainforests now, before it is too late.
Open Cloze Test 2:
Plants and animals hold medicinal, agricultural, ecological [1] …………… Endangered species must
be protected and saved so that future generations can experience their presence and value. Plants and animals
are responsible for a variety of useful medications. In fact, about forty percent of all prescriptions written
today are composed from the natural compounds of different species. These species not only save lives, but
they [2] …………… to a prospering pharmaceutical industry worth over $40 billion annually. Unfortunately,
only 5% of known plant species have been screened for their medicinal values, although we continue to lose
up to 100 species daily. The Pacific yew, a slow- growing tree found in the ancient forests of the Pacific
Northwest, was historically [3] ……………a “trash" tree. [4] ……………, a substance in its bark was
recently identified as one of the most promising treatments for ovarian and breast cancer. Additionally, more
than 3 million American heart disease sufferers would die within 72 hours of a heart attack without digitalis, a
drug [5] ……………from the purple foxglove.
Plant and animal species are the foundation of healthy ecosystems. Humans depend on ecosystems
such as coastal estuaries, prairie grasslands, and ancient forests to purify I their air, clean their water, and
supply them with food. When species become [6] …………… it is an indicator that the health of these vital
ecosystems is beginning to unravel. The U.S. Fish and Wildlife Service [7] …………… that losing one plant
species can trigger the loss of up to 30 other insect, plant and higher animal species. The northern spotted owl,
listed as threatened in 1990, is an indicator of the declining health of the ancient forests of the Pacific
Northwest. These forests are the home [8] …………… over 100 other old-growth dependent species, which
are at risk due to decades of unsustainable forest management practices. Pollution [9] ……………the coast
of Florida is killing the coral reefs along the Florida Keys, which serve as habitat for hundreds of species of
33

Downloaded by Nhung Nguy?n (nhungnth2004@gmail.com)


lOMoARcPSD|20277325

fish. Commercial fish species have begun to decline, [10] ……………a threat to the multi-million dollar
tourism industry; which depends on the quality of the environment.
II. WORD FORMS
A. Complete the sentences with the correct forms of the given word
1. He deposited his…………… gains in foreign bank accounts. (GET)
2. As young women…………… , they may perceive sex as an assertion of independence and gender equality.
(CULTURAL)
3. She…………… all the reasons why she wanted to leave him. (NUMBER)
4. He's been involved in some …………… dealings. (HAND)
5. Radium was one million times more…………… than its parent uranium. (ACT)
6. She found him arrogant and …………… (DOMINATE)
7. He claimed that the drugs had been planted to…………… him. (CRIME)
8. An anonymous…………… donated $2 million. (BENEFIT)
9. ……………of course, I'm much better off than I used to be. (MONEY)
10. The rain gave us a …………… excuse to stay at home. (MAKE)
B. Complete the following passage with the correct forms of the words given in the box
action fundamentalism imply plausibility disturbance
speculate punctual affect puzzle wake

It is fairly clear that the sleeping period must have some function, and because there is so much of it
the function would seem to be important. (1) ……………about its nature have been going on for literally
thousands of years, and one odd finding that makes the problem (2) …………… is that it looks very much as
if sleeping is not simply a matter of giving the body a rest. 'Rest', in terms of muscle relaxation and so on, can
be achieved by a brief period lying, or even sitting down. The body's tissues are self-repairing and self-
restoring to a degree, and function best when more or less continuously active. In fact a basic amount of
movement occurs during sleep which is specifically concerned with preventing muscle (3) …………….
If it is not a question of resting the body, then perhaps it is the brain that needs resting? This might be a
(4) ……………hypothesis were it not for two factors. First the electroencephalograph (which is simply a
device for recording the electrical activity of the brain by attaching electrodes to the scalp) shows that while
there is a change in the pattern of activity during sleep, there is no evidence that the total amount of activity is
any less. The second factor is more interesting and more (5) ……………In 1960 an American psychiatrist
named William Dement published experiments dealing with the recording of eye-movements during sleep. He
showed that the average individual's sleep cycle is (6) ……………with peculiar bursts of eye-movements,
some drifting and slow, others jerky and rapid. People woken during these periods of eyemovements generally
reported that they had been dreaming. When (7) ……………at other times they reported no dreams. If one
group of people were disturbed from their eye¬movement sleep for several nights on end, and another group
were (8) ……………for an equal period of time but when they were not exhibiting eye-movements, the first
group began to show some personality disorders while the others seemed more or less (9) …………… The
(10) ……………of all this were that it was not the disturbance of sleep that mattered, but the disturbance of
dreaming.
III. ERROR IDENTIFICATION
There are 10 errors ill the following passage. Identify and correct them.
Journeys are the midwife of thought. Few places are more conducive of internal conversations than a
moving plane, ship or train. There is an almost quaint correlation with what is in front of our eyes and the
thoughts that we are able to have in our heads: large thoughts at times require large views, new thoughts, new
places. Introspect reflections that are liable to stall are helped along by the flow of the landscape. The mind
may be reluctant to think properly when thinking is all it is supposed to do. The task can be as, paralyzing as
having to play a joke or minijic an accent in demand. Thinking improves when parts of the mind are given
other tasks, are charged with listening to music or following a line of trees.
34

Downloaded by Nhung Nguy?n (nhungnth2004@gmail.com)


lOMoARcPSD|20277325

Of all modes of transport, the train is perhaps the best aid to thought; the views have none of the
potential monotony of that on a ship or plane, they move fast enough for us not to get exasperated but slowly
enough to allow us to identify objects. They offered us brief, inspiring glimpses into private domains, letting
us see a woman at the exactly precise moment when she takes a cup from a shelf in her kitchen, then carrying
us on to a patio where a man is sleeping and then to a park where a child is catching a ball thrown by a figure
we cannot see.
IV. SENTENCE TRANSFORMATION
Finish each of the following sentences in such a way that it means exactly the same as the sentence
printed before it
1. He is said to be a very good pianist.
→ He has……………………………………………………………………………………………..
2. ‘I think the whole idea is ridiculous,’ he said.
→ He dismissed………………………………………………………………………………………
3. I have little confidence that he will be able to do the job.
→ I have serious………………………………………………………………………………………
4. I agreed to do the work because I understood that I would be paid for it.
→ My understanding when……………………………………………………………………………
5. I left without saying goodbye since I didn’t want to disturb her.
→ Rather ………………………………………………………………………………………………
Rewrite each sentence so that it contains the word in capitals, and so that the meaning stays the same.
6. Those of us at the back couldn’t hear most of the play. (inaudible)
→………………………………………………………………………………………………………
7. Sean did all the illustrations for the book but no one acknowledged his work. (credit)
→………………………………………………………………………………………………………
8. Having finished your studies, you must decide what to do next. (future)
→………………………………………………………………………………………………………
9. We didn’t phone you in case we woke you up. (so)
→………………………………………………………………………………………………………
10. Only Martha survived the crash, (sole)
→………………………………………………………………………………………………………

TRƯỜNG THPT CHUYÊN LƯƠNG THẾ VINH – ĐỒNG NAI


A. PHONETICS
I. Pick out the word whose underlined part is pronounced differently from the rest
1. A. heat B. unleaded C. peach D. teacher
2. A. business B. duke C. reduce D. student
3. A. apology B. deny C. empty D. simply
4. A. changed B. continued C. happened D.tossed
5. A. chapters B. horizons C. mammals D. ornaments
II. Pick out the word which has the main stressed syllable differently from the rest
1. A. Europe B. jumper C. survive D. tourist
2. A. active B. mountain C. tattoo D. travel
3. A. certain B. because C. prevent D. unless
4. A. decadence B. departure C. monument D. passenger
5. A. bankruptcy B. commercial C. ingenious D. ingredient
B. VOCABULARY
I. Choose the most suitable word to fill in each blank.
1. Xiuyun’s most striking ………… is her ability, to withstand stress and strain.
A. attitude B. characteristic C. character D. factor
35

Downloaded by Nhung Nguy?n (nhungnth2004@gmail.com)


lOMoARcPSD|20277325

2. The crowded city is so unpleasant compared to the ………… countryside.


A. outstretched B. abundant C. spacious D. large
3. A person was lying…………on the road at the scehe of the accident.
A. unaware B. unconscious C. insensible D. fainted
4. A child's existence seems so…………in the arms of its mother.
A. tight B. steady C. stable D. secure
5. Demeng was…………to court for violating the traffic law.
A. requested B. required C. summoned D. demanded
6. Yongle derived a lot of…………from his opponent’s confusion.
A. comfort B. ease C. gratification D. satisfaction
7. The soldiers …………when they heard that the war was over.
A. rejoiced B. gladdened C. revolted D. gloated
8.…………me of stealing your wallet is ridiculous, because I wasn’t even anywhere near you.
A. Incriminating B. Accusing C. Censuring D. Impeaching
9. The…………that the emperor was dead came as a shock to everybody.
A. suggestion B. implication C. announcement D. accusation
10. The teacher…………the students not to copy one another’s work or they would be punished.
A. rebuked B. warned C. asked D. censured
II. Choose the word to replace the underlined words in each sentence below.
1. Zhiwei has left a job from his job because he has just got a better-paying one.
A. withdrawn B. renounced C. retired D. resigned
2. Zaleha’s parents were against to her going out alone at night.
A. rejected B. rebuked C. refused D. objected
3. The fact that he would have to stay at the deserted old mansion all night filled with terror him.
A. appalled B. scared C. amazed D. astounded
4. The king’s reaching of a place was heralded by several short blasts of the trumpets.
A. advent B. departure C. arrival D. withdrawal
5. He was jailed on charges of culpable homicide not adding up to murder.
A. totaling B. amounting C. counting D. arising
6. The people who design buildings need to check the plans of the houses again.
A. engineers B. surveyors . C. architects D. designers
7. We have a large amount of proof of Mr Chen’s guilt, though he denies being involved.
A. abounding B. abundant C. plenteous D. surplus
8. We mustn’t allow to have way to these impudent demands.
A. allow B. entrust C. bestow D. give.
9. Nathan is often taken to be another for his twin brother, Shanker.
A. mistaken B. assumed C. misjudged D. wronged
10. The wealthy art collector came to get most of his antiques from the junk shop dealer.
A. required B. acquired C. fulfilled D. possessed
III. Choose the correct word or phrase to fill in each blank.

keep the ball rolling made a bee-line for nipped in the bud
mind your own business an all-rounder bolt from the blue
bombshell beat about the bush apple of his eye
a bear with a sore head in cold blood take up arms

1. Xiuzhen is……………She is able to do almost any type of job satisfactorily.


2. Mr Huang’s only grandson was the…………..
3. Many able-bodied young men were required to leave their families and …………..during the Japanese
36

Downloaded by Nhung Nguy?n (nhungnth2004@gmail.com)


lOMoARcPSD|20277325

occupation.
4. Mr Li had to…………..at the office when his partner was ill.
5. Haniff has been as bad-tempered as…………..all morning.
6. Karen…………..for the post-office because she had an important letter to post.
7. The murder was carried out …………...
8. Shanker’s visit was quite a …………...
9. She dropped a…………..about her secret marriage to her varsity mate.
10. Their friendship was…………..before it could develop.
IV. Fill in the blanks with the correct form of the word in capital.
1. The…………..little puppy looked up at us with its large brown eyes. ADORE
2. She pressed down on the…………..and raced off in her new car. ACCELERATE
3. “Glad to make your…………..,”said Weiming to his new classmate. ACQUAINT
4. Under the weight of a heavy conscience, he finally…………..to committing the crime. ADMIT
5. The…………..board has decided against implementing the new system. ADVISE
6. Mr Lin works at a ………….. CAN
7. The flowers that Meiying arranged made a lovely…………... CENTRE
8. Mr Brown was the…………..at the meeting. CHAIR
9. Elias gave a very…………..account of the incident in his report. DESCRIBE
10. He was…………..of making her acquaintance. DESIRE
C. GRAMMAR AND STRUCTURES
I. Choose the word or phrase which best fits each gap of the sentence.
1. The chaos in the house was starting to…………...
A. put him up B. get down him C. put up with him D. get him down
2. Don’t worry about paying; I’ll…………..
A. shoulder the bill B. foot the bill
C. foot the money D. shoulder the money
3. The journalist reported that the city was…………..the verge of a crisis.
A. on B. within C. into D. in
4. Tourism is a word…………..for the last 50 years.
A. using B. which was used C. that has been used D. that is used
5. Humans are the only animal that chooses what it…………..
A. will look B. will look like C. looks D. like
6.…………..their physical appearance, our ancestors distanced themselves…………..the rest of animals.
A. Customize / away B. Customizing / away
C. Customized / from D. By customizing / from
7. There…………..a drop in the sales of new houses since mortgages became more difficult…………..
A. has been / to obtain B. have been / to obtain
C. was / to be obtained D. were / to be obtained
8. I’m rather…………..for time at the moment ………….. we talk about it tomorrow?
A. pressing / Couldn’t B. pressing / Wouldn’t
C. pressed / Couldn’t D. pressed / Wouldn’t
9. When I…………..after the accident, I couldn’t remember what had happened.
A. brought round B. came round C. brought up D. came up
10. At first I found…………..to get used to…………..on the other side of the road.
A. difficult / drive B. it difficult / drive
C. difficult / driving D. it difficult / driving
II. The passage below contains 5 mistakes. UNDERLINE the mistakes and write their correct forms in
the space provided below. (0) has been done as an example.
The paintings of Leonardo da Vinci has always attracted controversy. Only 14 work have ever been
37

Downloaded by Nhung Nguy?n (nhungnth2004@gmail.com)


lOMoARcPSD|20277325

attributed by him and experts have questioned the authenticity of several. Not even the Mona Lisa is above
suspicion. The painting is neither signed nor dating and no record of payment to Leonardo has ever been
found. Believing to be the portrait of the wife of Florentine merchant Francesco del Gioconda dating from
1502, it has been in public display in the Louvre since 1804. Now housed in a bullet-proof glass case, it has
always been surrounded by tight security.
0. has → have
1…………. 3………….
2…………. 4…………. 5………….
D. READING COMPREHENSION
I. Cloze test: Fill each of the following numbered blanks with ONE suitable word
In the 21st century food will (1)…………..more than just feed you. A new range of products appearing
on shelves in shops and supermarkets (2) …………..designed to give you specific health benefits. The
demands of modern life make these foods very attractive. Not only do they provide proven ways to improve
health, but they are also very attractive (3) ………….. a quick and convenient way of making sure we enjoy a
healthy diet.
In some countries it is already possible to buy crisps that make you feel (4) …………..depressed,
chewing gum that increases your brain power and tea that helps you (5) ………….. over the tiredness
associated (6) ………….. long-distance air travel. In the future, experts promise biscuits that will keep your
heart healthy, and a hot chocolate drink to give you strong bones.
Despite the fact that these ‘functional’ foods cannot replace a balanced diet and regular exercise, they
can help the body perform at (7) …………..best a lot of the time. At (8) …………..these foods are more
expensive than other foods, but that is due to the ingredients they (9) ………….. of and the way they are
made. All the foods contain probiotics (10) ………….. increase the number of ‘good’ bacteria in your
stomach, helping to keep your digestive system healthy.
For questions 1-18, read the three texts below and decide which answer (A, B, C or D) kstfits each gap.
TELEVISION DOCUMENTARIES
Here's a new game for you. Watch a documentary with the sound turned down and make up your own
commentary. It's great for parties. You (1) …………..all your guests to stand outside and then they come in
one at a time and have a (2) …………..at guessing what it's about.
It's only when you turn down the sound that you realize just how (3) …………...the pictures are to
most documentaries. I expect you've noticed by now that television is primarily a visual medium. TV directors
get into a terrible (4) ………….. if there’s nothing to show you. They don't mind so much if there's nothing to
tell you - 80% of television has nothing to say - but no director has ever turned to a cameraman and asked:
'What are you doing here?'
The most insuperable problem with a large (5) …………..of documentaries is that they are working in
the wrong medium. They ought to be newsprint articles. You can say more in print. You can say it better. And
it's interactive. The readers can go at their own (6) …………..Television is hampered by having to fill the
screen and move at the approximate speed of the slowest member of the audience.
1. A. have B. sort C. get D. settle
2. A. bid B. stab C. venture D. speculation
3. A. irrelevant B. incompatible C. inconsistent D. incongruous
4. A. lather B. fuss C. ado D. stir
5. A. fraction B. proportion C. ratio D. bulk
6. A. time B. flow C. course D. pace
THE REJECTED NOVEL
“You’ve not had much luck with the book, I hear.'
That had to be the understatement of the year. My novel had been rejected four times (7) …………..
far. I’ve no doubt that behind my (8) …………..the family were having a good snigger. Rhona of course had
been the loyal exception, though I admit that her piteous expressions when the thing limped home battered by
38

Downloaded by Nhung Nguy?n (nhungnth2004@gmail.com)


lOMoARcPSD|20277325

franking stamps were harder to (9) …………..than her sister's outright sarcasm: 'Has your boomerang got
back yet, Patton?' she'd enquire, while her husband Jack would give the knife an extra twist by asking if I'd
managed to sell any of my daubs. Which meant that he presumed I'd (10) …………..my job on the railways to
pursue a painting career. Maybe I should have. The manuscript had begun to show bruises from its days,
weeks and months buried in the 'slush pile' of various publishing firms. Actual criticism of the novel by its
rejecters was very (11) …………..on the ground, although the consensus of opinion seemed to indicate that its
main weakness (12) …………..in its apparent 'lack of plot'.
7. A. yet B. thus C. hence D.by
8. A. back B. head C. ears D. face
9. A. bear B. defy C. cope D. resist
10. A. broken off B. wound up C. pulled out D. packed in
11. A. light B. shallow C. thin D. scant
12. A. stood B. revolved C. lay D. centred
LONELINESS IN THE CITY
For the really lonely individual in the city, life becomes a string of disconnected occasions. Eating by
himself in a restaurant he feels conspicuous; he tries to (13) ............... the eyes of other lone diners,
(14) ............... himself the subject of other people's conversations, sees a world divided into two groups the
majority; complacent couples, parties and families, and an envious minority of single people.. He takes (15)
…………. at the imagined snubs or cursory service of the waiter. He calls for his bill with the coffee,
knowing he has no further excuse to (16) ...............put. In a phone booth, he makes a cliffhanger out of the
ringing tone, and (17) ...............with relief when it is answered. When his own phone stays silent for a day, he
suspects a conspiracy to drop him, and (18) ...............the operator to check his line. At a party, he stays too
long, since there is nothing to follow it.
13. A. set B. make C. take D. catch
14. A. conceives B. infers C. imagines D. fantasizes
15. A. offence B. indignation C. outrage D. insult
16. A. stick B. hang C. stay D. hold
17. A. blows B. sighs C. yawns D. snorts
18. A. persists B. pesters C. perseveres D. pleads
You are going to read four extracts which are all about the writers’ school days. For questions 19 - 20,
choose the answer (A, B, C or D) which you think fits best according to the text.
THE ELEVEN PLUS EXAM
At school, particularly when I was ten or eleven, I was suffocated by the amount of ipormality I was
subjected to. I knew how to count, and yet they, the teachers, pushed all these complex forms of mathematics
down my throat. I could read, but I wasn't allowed to read what I wanted. I was force-fed the school syllabus,
and all because it was deemed to be the standard. And someone had decided that children should be judged at
the age of eleven so that they could be segregated for the rest of their lives. No consideration was given for
talents outside the limited range required by the examining board. The misfits were destined to become
factory fodder, farm workers or manual workers like my father, with no incentive to achieve or realize their
potential.
I sat at my desk on the day of the Eleven Plus exam and looked at my paper. I felt it was more than my
intelligence that was being tested, it was my whole being. I had to decide either to play the game their way,
and succeed or fail according to their rules, or take my own route. I decided to settle my own fate. I signed my
name at the top of the paper, and did nothing more for the rest of the exam. The room was silent, apart from
the anxious scratching of pencils, and yet inside my head was a triumphant explosion, like the opening cannon
shot of war. I had made my first statement to the world. But it was also like watching opportunity float away
on a piece of paper down the river. It would damage me, but at the same time it was a victory. For the first
time in my life I realized that it would be a battle between me and them.
19. The writer implies in the first paragraph that his school days were characterized by
39

Downloaded by Nhung Nguy?n (nhungnth2004@gmail.com)


lOMoARcPSD|20277325

A. a system which benefited none of the pupils at his school.


B. his belief that he would not end up in the same sort of job his father had.
C. his belief that he had abilities that the school system did not cater for.
D. a refusal on his part to do what teachers expected of him.
20. When the writer decided not to answer any of the questions in the exam, he
A. was fully aware of the consequences of what he was doing.
B. did so because he feared that the exam would suggest he was not very intelligent.
C. told himself that doing so would not have long-term consequences.
D. thought it was the logical thing to do in view of his behaviour at school.
E. WRITING
Use the word given in brackets and make any necessary additions to complete a new sentence in such a
way that it is as similar as possible in meaning to the original sentence. Do NOT change the form of the
given word. You must use between three and eight words, including the word given.
1. Now that their parents have consented to their marriage, their future happiness is guaranteed. (stands)
→ Now that they have their………………………………………………………………………………
way of their future happiness.
2. Puzzle-solving is one of her favorite pastimes and always has been. (passion)
→She ………………………………………………………………………………she was a child.
3. They all agreed that the new dress code was a complete success. (hailed)
→ The new dress code ……………………………………………………………………everyone.
4. She knew nothing of her husband’s impending promotion. (about)
→Little………………………………………………………………………………to be promoted.
5. She only ever has one chocolate at a time. (permits)
→ She never ……………………………………………………………………one chocolate at a time.

TRƯỜNG CHUYÊN NGUYỄN TẤT THÀNH - KON TUM


MULTIPLE CHOICE QUESTIONS
I. PHONOLOGY:
A. Choose the word which has the underlined part is pronounced differently from the rest:
1. A. tissue B. permission C. press D. pressure
2. A. negotiation B. think C. invent D. enlarge
3. A. campaign B. foreign C. gain D. sign
4. A. handcraft B. handbook C. handkerchief D. handbag
5. A. halfpenny B. lift C. soft D. gift
B. Choose the word which is stressed differently from the rest:
6. A. Australian B. European C. American D. Canadian
7. A. guarantee B. reinforce C. dispirited D. recommend
8. A. stringent B. urology C. testimony D. terrorist
9. A. legislature B. repository C. magnificent D. mistake
10. A. constituency B. constitution C. presentiment D. subsidiary
II. WORD CHOICE:
Choose the word which best completes each sentence:
11. Modern buildings should………….with the surrounding area.
A. suit B. fit C. blend D. join
12. The proposal has not met with………….agreement.
A. voluntary B. universal C. informal D. effective
13. In the old days, people believed that the world was flat and ships would fall off the………….
A. boundary B. edge C. border D. limit
14. You must forgive my………….in these matters.
40

Downloaded by Nhung Nguy?n (nhungnth2004@gmail.com)


lOMoARcPSD|20277325

A. experience B. inexperience C. unexperience D. imexperience


15. That book looks like an advanced …………..text to me.
A. economic B. economical C. economics D. economist
16. The police have asked for the………….. of the public in tracing the whereabouts of the escaped convict.
A. partnership B. co-operation C. association D. alliance
17. Although we have a large number of students, each one receives……………attention.
A. individual B. only C. alone D. single
18. When the electricity failed, he………….a match to find the candles.
A. rubbed B. scratched C. struck D. started
19. They wasted………….time searching for the car keys.
A. priceless B. costly C. expensive D. valuable
20. Whenever we organise social events or outings in our office, Janice remains ………….as if such things
are beneath her.
A. inaccessible B. outstanding C. aloof D. invisible
III. STRUCTURES AND GRAMMAR:
Choose the word or phrase which best completes each sentence:
21………….today, she would get home by Friday.
A. Would she leave B. Was she leaving C. Were she to leave D. If she leaves
22. “Did he make any comments?”
“Yes, he said it was imperative that we………….on time tomorrow.”
A. had been B. were C. have been D. be
23………….last Sunday.
A. I would sooner you had come here B. I would sooner you have come here
C. I would sooner you came here D. I would sooner you should have come here
24. I’m so sure that I passed the exam that I’m not going to wait for the results………….
A. being announced B. to announce
C. will be announced D. to be announced
25.………….snow that covers………….top of………….mountain is………….beautiful sight.
A. The-the-the-the B. Ø-the-a-a C. The-the-the-a D. Ø - Ø -a-the
26. It’s about time she apologized to me………….me a gossip in front of everybody.
A. to have called B. for having called
C. that she called D. from calling
27. You must refrain ………….tea or coffee while taking this medicine.
A. to drink B. from drinking C. drinking D. to drinking
28. Not until I arrived home…………..
A. did I remember about the umbrella
B. I remembered about the umbrella
C. that I could remember about the umbrella
D. then would I remember about the umbrella
29. You………….me, because I didn’t say that.
A. must misunderstand B. must have misunderstood.|
C. must be misunderstanding D. had to misunderstand
30. Your request that the test………….has been denied.
A. delayed B. delaying C. be delayed D. being delayed
IV. PREPOSITIONS AND PHRASAL VERBS:
Choose the word or phrase which best completes each sentence or replaces the underlined part:
31. He’ll be very upset if you………….his offer of help.
A. turn away B. turn from C. turn down D. turn against
32. Look………….! There is a rattlesnake under the picnic table!
41

Downloaded by Nhung Nguy?n (nhungnth2004@gmail.com)


lOMoARcPSD|20277325

A. on B. into C. for D. out


33. Someone broke into the house and made………….a lot of things.
A. by B. off C. up D. out
34. If you don’t pay your rent, your landlord is going to kick you out!
A. lend you some money B. play football with you
C. give you a kick D. force you to leave
35. We intend to………….with the old system as soon as we have developed a better one.
A. do up B. do away C. do down D. do in
36. I’m not surprised Margaret’s ill. With all the voluntary work she’s…………., she’s really been doing too
much.
A. taken off B. taken on C. taken in D. taken to
37. Its economy centers………….the production of coffee.
A. on B. with C. to D. at
38. Ben’s wife is a terrible snob. She………….almost all his friends because they have north country accents.
A. looks up to B. looks forward to C. looks out on D. looks down on
39. As we waited on the pavement, a black Mercedes………….beside us.
A. pulled up B. pulled down C. pulled off D. pulled through
40. I know David Fletcher………….sight, but I’ve never been introduced to him.
A. at B. in C. by D. on
41. I thought she was being serious, but she was only having me…………..
A. up B. on C. over D. round
42. The plans for the building were………….a few months ago.
A. drawn on B. drawn back C. drawn out D. drawn up
V. READING COMPREHENSION:
A. Read the following passage and choose the best answers to the questions:
It was once believed that being overweight was healthy, but nowadays few people subscribe to this
viewpoint. While many people are fighting the battle to reduce weight, studies are being conducted
concerning the appetite and how it is controlled by both emotional and biochemical factors. Some of the
conclusions of these studies may give insights into how to deal with weight problems. For example, when
several hundred people were asked about their eating habits in times of stress, 44 percent said they reacted to
stressful situations by eating. Further investigations with both humans and animals indicated that it is not food
which relieves tension but rather the act of chewing.
A test in which subjects were blindfolded showed that obese people have a keener sense of taste and
crave more flavorful food than nonobese people. When deprived of the variety and intensity of tastes, obese
people are not satisfied and consequently eat more to fulfill this need. Blood samples taken from people after
they were shown a picture of food revealed that overweight people reacted with an increase in blood insulin, a
chemical associated with appetite. This did not happen to average-weight people.
In another experiment, results showed that certain people have a specific, biologically induced hunger
for carbohydrates. Eating carbohydrates raises the level of serotonin, a neutrotransmitter in the brain. Enough
serotonin produces a sense of satiation, and hunger for carbohydrates subsides.
Exercise has been recommended as an important part of a weight-loss program. However, it has been
found that mild exercise, such as using the stairs instead of the elevator, is better in the long run than taking on
a strenuous program, such as jogging, which many people find difficult to continue over long periods of time
and which also increase appetite.
43. The word “crave” can best be replaced with ………….
A. devour B. absorb C. season D. desire
44. It can be inferred from the passage that………….
A. overweight people are tense
B. thin people don’t eat when under stress
42

Downloaded by Nhung Nguy?n (nhungnth2004@gmail.com)


lOMoARcPSD|20277325

C. weight watchers should chew on something inedible


D. 56 percent of the population isn’t overweight
45. It can be inferred from the passage that………….
A. thin people don’t enjoy food
B. a variety of foods and strong flavors satisfy heavy people
C. overweight people have an abnormal sense of taste
D. deprivation of food makes people fat
46. According to the passage, insulin ………….
A. increases in the bloodstream when people eat large amounts of food
B. can be used to lessen the appetite
C. causes a chemical reaction when food is seen
D. levels don’t change in average-weight people who see food
47. It can be inferred that for certain people…………..
A. eating carbohydrates eliminates hunger
B. carbohydrates biologically induce hunger
C. carbohydrates don’t satisfy a hungry person
D. carbohydrates subside when serotonin is produced
48. What can be said about serotonin?
A. It is a chemical that increases the appetite
B. Only certain people produce it in their brains
C. It tells the brain when a person is full
D. It neutrotransmits carbohydrates to the brain
49. In order to lose weight, it would be a good idea for heavy people to………….
A. jog 3 miles daily and chew on carrot sticks
B. walk up stairs and look at pictures of food
C. eat plenty of chewy carbohydrates
D. avoid stressful situations and eat spicy foods
50. Which one of the following exercises might be best for an overweight person to engage in daily?
A. an evening walk B. a long swim
C. cross-country skiing D. 10-mile bicycle rides
B. Read the following passage arid choose the best answers to the questions:
Recent technological advances in manned and unmanned vehicles, along with breakthroughs in
satellite technology and computer equipment, have overcome some of the limitations of divers and diving
equipment for scientists doing research on the great oceans of the world. Without a vehicle, divers often
became sluggish, and their mental concentration was severely limited. Because undersea pressure affects their
speech organs, communication among divers has always been difficult or impossible. But today, most
oceanographers avoid the use of vulnerable human divers, preferring to reduce the risk to human life and
make direct observations by means of instruments that are lowered into the ocean, from samples take from the
water, or from photographs made by orbiting satellites. Direct observations of the ocean floor can be made not
only by divers but also by deep-diving submarines in the water and even by the technology of sophisticated
aerial photography from vantage points above the surface of more than seven miles and cruise at depths of
fifteen thousand feet. In addition, radio-equipped buoys can be operated by remote control in order to transmit
information back to land-based laboratories via satellite. Particularly important for ocean study are data about
water temperature, currents, arid weather. Satellite photographs can show the distribution of sea ice, oil slicks,
and cloud formations over the ocean, maps created from satellite pictures can represent the temperature and
the color of the ocean’s surface, enabling researchers to study the ocean currents from laboratories on dry
land. Furthermore, computers help oceanographers to collect, organize, and analyze data from submarines and
satellites. By creating a model of the ocean’s movement and characteristics, scientists can predict the patterns
and possible effects of the ocean on the environment.
43

Downloaded by Nhung Nguy?n (nhungnth2004@gmail.com)


lOMoARcPSD|20277325

Recently, many oceanographers have been relying more on satellites and computers than on research
ships or even submarine vehicles because they can supply a greater range of information more quickly and
more effectively. Some of humankind’s most serious problems, especially those concerning energy and food,
may be solved with the help of observations made possible by this new technology.
51. With what topic is the passage primarily concerned?
A. Communication among drivers
B. Direct observation of the ocean floor
C. Undersea vehicles
D. Technological advances in oceanography
52. The word “sluggish” is closest in meaning to…………..
A. very weak B. nervous C. confused D. slow moving
53. This passage suggests that the successful exploration of the ocean depends upon…………..
A. vehicles as well as divers
B. controlling currents and the weather
C. radios that divers use to communicate
D. the limitations of diving equipment
54. Divers have had problems in communicating underwater because …………...
A. they did not pronounce clearly
B. the water destroyed their speech organs
C. the pressure affected their speech organs
D. the vehicles they used have not been perfected
55. The word “cruise” could best be replaced by…………...
A. travel at a constant speed B. function without problems
C. stay in communication D. remain still
56. Undersea vehicles …………...
A. have the same limitations that divers have
B. are too small for a man to fit inside
C. make direct observations of the ocean floor
D. are very slow to respond
57. The Word “information” is closest in meaning to…………...
A. samples B. photographs C. data D. articles
58. How is a radio-quipped buoy operated?
A. By operators outside the vehicle on a diving platform
B. By operators outside the vehicle in a laboratory on shore
C. By operators inside the vehicle in the part underwater
D. By operators outside thè vehicle on ship
59. Which of the following are NOT shown in satellite photographs?
A. The location of sea ice B. Cloud formations over the ocean
C. The temperature of the ocean’s surface D. A model of the ocean s movements
60. The words “those”'refers to…………..
A. vehicles B. problems C. ships D. computers
VI. GUIDED CLOZE TEST
A. Read the text below and decide which answer (A, B, C, or D) best fits each space.
UNIVERSAL WET WEEKEND
The weather across much of the British Isles (61) ……………settled last week, with a good (62)
……………of sunshine. On Saturday, the lunchtime temperature at Bridlington in the north-east of England
was 28.2 degree Celsius, which compared favourably with Alicante in southern Spain at 29 degree Celsius.
The rest of the world, however, was coping with some extreme conditions. A tropical storm, given the name
Helen, hit Hong Kong on Saturday morning, though her presence had been warned in advance. From noon on
44

Downloaded by Nhung Nguy?n (nhungnth2004@gmail.com)


lOMoARcPSD|20277325

Friday, the showers and (63) ……………of rain became more and more frequent so that by midnight on
Sunday, thirty-six hours later, there had been 333 mm of rainfall, not far off the average for the month of
August, at 367 mm. Even on Sunday there was a (64) ……………in Helen's tail. The town centre of Shanwei,
near Hong Kong, was flooded when 468 mm of rain fell in the sixty hours (65) ……………up to midday on
Sunday, (66) ……………twice the normal August rainfall. On the other (67) ……………of the globe, tropical
storm Gabrielle moved across the Gulf of Mexico and overnight rain (68) ……………the usual rainfall for the
whole month. Although most of Europe enjoyed sun, the high temperatures were sufficient to set off some
(69) ……………showers. On Tuesday morning, a thunderstorm at Lyons in eastern France (70) ……………
99 mm of rain in just six hours.
61. A. kept B. remained C. lasted D. held
62. A. extent B. quantity C. proportion D. deal
63. A. outbursts B. outbreaks C. outputs D. outlets
64. A. sting B. prick C. stab D. poke
65. A. going B. leading C. taking D. approaching
66. A. only B. fairly C. hardly D. nearly
67. A. section B. side C. face D. part
68. A. overtook B. exceeded C. passed D. beat
69. A. huge B. weighty C. heavy D. strong
70. A. deposited B. placed C. lay D. set
B. Read the text below and decide which answer (A, B, C, or D) best fits each space.
Mr. Gerrard owns a business that sells auto parts. He buys the parts (71) ……………he stocks from
many different manufactures, then sells them to repair shops and individuals. He must be very organized (72)
…………… of all the different parts hẹ buys and sells.
To ensure that his customers are happy and he (73) ……………lose money, Mr.Gerrard follows
specific (74) …………… The typical order takes about 1/2 hour to process. At 9:00 am Mr. Gerrard checks
his mail: (75) ……………and e-mail. He takes about 5 minutes to sort it into three types: orders, bills and
other business, and personal. He gives the order to Mr. Tanner (76)……………the orders into the computer.
About 5 minutes later, Mr. Tanner prints a "pick sheet" and gives it to Ms. Windly (77) ……………
"picks"the products off the shelves in the warehouse for shipping. This takes about 9 minutes. Mrs. Windly
(78) …………… the pick list and products to Mr.Hebner, who quickly rechecks that the right products have
been picked. If so, he boxes them and labels them for shipping. Within 10 minutes, Mr. Hebner sends the pick
sheet back to Mr. Tanner, noting if all items were shipped or if any were (79) ……………stock. Mr. Tanner
takes 5 minutes to adjust the invoice to match the shipment and enters the updated information in the
computer. (80) ……………there are backorders, Mr. Tanner lets Mr. Gerrard know so that he can order those
items from the manufacturer. By 9:30 the customer's order and invoice are sent.
71. A……….. B. where C. when D. who
72. A. tired B. to keep track C. send D. hear
73. A……….. B. does C. does not D. can
74. A.strategy B. time C.plan D. procedures
75. A. regular B. package C. junk D. spam
76. A. to place B. for entrance C. to enter D. to replace
77. A. that B. who C……….. D. when
78. A. brings B. gives C.buys D. makes
79 A. out of B. from C. made of D. to
80. A. If B. As if C. Whether D. Because
WRITTEN TEST
I. OPEN CLOZE TEST
A. Open cloze test 1:
21 Fill in each numbered blank with ONE suitable word.
45

Downloaded by Nhung Nguy?n (nhungnth2004@gmail.com)


lOMoARcPSD|20277325

ANIMAL IMPRINTING
Imprinting is a learning mechanism which occurs early in the life of certain animals. It is through this
process that they develop a positive attachment to members (1) ……………their own species. No sooner do
young birds hatch than they must learn to (2) ……………their mother in order to be able to follow and keep
close to her (3) ……………their safety. (4) ……………this process go wrong, newly-hatched chicks (5)
…………… the risk of becoming lost, and may come to harm. (6) ……………that they possess a high level
of mobility, it would be difficult for the mother to keep the chicks (7) ……………without the assistance of
imprinting. The process can take literally a matter of minutes. The first large moving object the chicks see (8)
……………automatically become “mother”. In normal circumstances, of course, it really is their mother, but
under experimental conditions it
can be almost (9) ……………. For instance, if the moving object happens to be an orange balloon on a piece
of string, then the balloon becomes “mother”. So powerful is this imprinting process that even after a few
days, and (10) ……………the presence of their real mother, the chicks will choose the balloon.
B. Open cloze test 2:
Fill in each numbered blank with ONE suitable word.
Each and every day we see hundreds of advertising images. (1) ……………other kind of image
confronts us with anything like the same frequency. Never in history (2) ……………there been such a
concentration of visual messages. The brain cannot help (3) ……………. take these messages in, and for a
moment they stimulate the imagination (4) ……………virtue of their appeal to memory or expectation.
Advertising is usually justified as a competitive medium of benefit to the public and efficient
manufacturers. (5) …………… it may be true that, in advertising, one particular brand competes against
another, it is also just (6) …………… true that such publicity images confirm and enhance others. That there
are choices to be made (7) ……………,without saying but, ultimately, advertising as a system makes a
single proposal-namely (8) ……………we transform ourselves or our lives, by buying something more. We
are led to believe that, by so doing, we will in one way or another become richer - but in fact we will become
poorer, (9) ……………spent our money.
Advertising shows us people who have apparently been transformed into a new state and are, as a
result, enviable. The state of being envied is (10) ……………constitutes glamour. And advertising is in the
business of manufacturing glamour.
II. WORD FORMS;
A. Supply the correct form of the word to fill in the blank.
ESTIMATE SUSPECT SIGNIFY FULFIL ANATOMY
ADHERE NOTIC ABLE SEDUCE EMPHATIC SUBSTANCE

There is little to disagree about in the notion that a good voice, whether in opera or rock music, is one
which moves its audience and brings a sense of release and (1) ……………to the singer. But contemporary
pop and rock music have come about due to (2) ……………advances in technology. Here, the impact of the
microphone should not be (3) ……………, as it has enabled the magnification of quiet, intimate sounds.
This, in turn, allows the singer to experiment with the (4) ……………on mood rather than on strict (5)
…………… to proper breathing and voice control. Donna Soto Morettin, a rock and jazz vocal trainer, feels
that (6) ……………reasons may account for the raspy sound produced by certain rock singers. Her (7)
…………… is that swollen vocal chords, which do not close properly, may allow singers to produce deeper
notes. She does not, however, regard this as detracting (8) ……………from the value of the sound produced.
Singing, she maintains, has an almost (9) ……………quality and so our responsd to it has more (10) ………
than its technical qualities.
B. Supply the correct form of the word provided in brackets.
1. There were no……………signs that the house was inhabited. (OUT)
2. Be careful. You may be……………to put all your eggs in one basket. (ADVICE)
3. Commentators on television or the radio should try to be informative, but should avoid sounding
46

Downloaded by Nhung Nguy?n (nhungnth2004@gmail.com)


lOMoARcPSD|20277325

…………….(OPINION)
4. Black clouds came and sheets of……………rain poured onto the parched landscape. (TORRENT)
5. The car rolled down a railway……………and hit a tree. (BANK)
6. One of the unexpected……………of the course was the forming of some really close friendships. (SPIN)
7. The little boy has displayed an amazing musical maturity, together with a virtually……………technique.
(FLAW)
8. It may be……………to force them to make a new decision, and if you upset them, they are quite likely to
overact. (PRODUCE)
9. As climate cooled gradually, almost……………, most visitors continued to climb in the mountain.
(PERCEPTIBLE)
10. The game was much……………when both teams scored with five minutes of each other. (LIFE)
III. ERROR IDENTIFICATION:
The passage below contains 10 mistakes. Underline the mistakes and correal them.
The horse and carriage is a thing of the past, but love and marriage are still with us and still closely
interrelated. Most American marriages, particular first, marriages uniting young people, are the result of
mutual attraction and affection rather than practical consideration.
In the United States, parents do not arrange marriages for their children. Teenagers begin date in high
school and usually find mates through their own academic and social contacts. Though young people feel free
to choose their friends from different groups, almost choose a mate of similar background. This is due in parts
to parental guidance. Parents cannot select spouses for their children, but they can usually influence choices
by voicing disapproval for someone they consider unsuitable.
However, marriages of members of different groups (interclass, interfaith, and interracial marriages)
are increasing, probably because of the greater mobile of today’s youth and the fact that they are restricted by
fewer prejudices as their parents. Many young people leave their hometowns to attend college, serve in the
armed forces, or pursue a career in a bigger city. One away from home and family, they are more likely to date
and marry outside their own social group.
1…………….→……………. 2…………….→…………….
3…………….→……………. 4.…………….→…………….
5…………….→……………. 6.…………….→…………….
7…………….→……………. 8.…………….→…………….
9…………….→……………. 10……………→…………….
IV. SENTENCE TRANSFORMATION
Finish each of the following sentences in such a way that it is as similar as possible in meaning to the
sentence printed before it:
1. The police had to let the suspect go because new evidence was produced. (LIGHT)
→…………………………………………………………………………………………………
2. Don’t let her relaxed manner deceive yo.q; she is an extremely shrewd woman! (TAKEN)
→…………………………………………………………………………………………………
3. Every possible effort was made by the orphanage to find the boy’s parents.(STONE)
→…………………………………………………………………………………………………
4. I feel that I don’t fit in with the people in the new office. (FISH)
→…………………………………………………………………………………………………
5. The two children began to argue fiercely about who had damaged the bicycle. (BROKE)
→…………………………………………………………………………………………………
6. Despite all their efforts, they could not get the old car to start.
→ No matter……………………………………………………………………………………..
7. It is important to be well prepared for an interview because if you make a mistake, you may not get the job.
→A mistake in an interview may cost…………………………………………………………….
8. “Nothing will persuade me to sleep in that haunted house”, she said.
47

Downloaded by Nhung Nguy?n (nhungnth2004@gmail.com)


lOMoARcPSD|20277325

→ She flatly……………………………………………………………………………………….
9. The news that the Prime Minister had resigned came as a great shock to everyone.
→ Everyone was taken……………………………………………………………………………
10. A computer breakdown was blamed for the delayed dispatch of the brochures.
→ A computer breakdown was given…………………………………………………………….

TRƯỜNG THPT CHUYÊN NGUYỄN ĐÌNH CHIỂU - ĐỒNG THÁP


PART A: MULTIPLE CHOICE QUESTIONS
I. PHONOLOGY
A. Find the word that has its underlined part pronounced differently from the other three in each
question.
1. A. archaic B. archive C. architect D. archery
2. A. camera B. stammer C. chamber D. hammer
3. A. idiot B. idol C. icon D. item
4. A. folk B. milk C. walk D. half
5. A. considerate B. narrate C. private D. pirate
B. Find the word with the stress pattern different from that of the other three words in each question.
6. A. phenomenon B. phonetic C. Pacific D. character
7. A. evaluate B. category C. investigate D. exception
8. A. diagram B. dynasty C. diagonal D. diamond
9. A. territory B. librarian C. respectable D. victorious
10. A. millennium B.lecture C. different D. grocery
II. VOCABULARY AND STRUCTURE
Choose the best answer.
1. One of the…………… for being a lifeguard is working emergency first aid for someone having a heart
attack
A. backgrounds B. requests C. requirements D. accomplishments
2. Mario was trying to…………… if there was any cinnamon in the apple he was eating.
A. favour B. detect C. sight D. spot
3. Mr. Patt did a/an…………… job on his presentation about recycling, inspiring those present to start
recycling programs.
A. outstanding B. mere C. advisable D. voluntary
4. We picked up a colorful travel…………… at the travel agent’s.
A. brochure B. handout C. prospectus D. leaflet
5. A Slovenian man battled piranhas and exhaustion …………… the first person …………… The entire
length of the Amazon river.
A. having become-swam B. has become-having swum
C. to become-to swim D. became-swimming
6 .His…………… to the safety regulations really can’t be ignored any longer.
A. inattention B. unfamiliarity C. carelessness D. disregard
7. Peter…………… Anne for her role in the play.
A. exercised B. studied C. practised D. coached
8. Sam thought that he could…………… across the water from the ferry to the quay, but he was wrong, and
he fell in.
A. leap B. rise C. climb D. drop
9. When his brakes failed, his car…………… over to the other side of the road.
A. swerved B. waved C. slammed D. collapsed
10. Some animals sefve as models in the study of human health because they suffer from almost……………
types of diseases…………… humans.
48

Downloaded by Nhung Nguy?n (nhungnth2004@gmail.com)


lOMoARcPSD|20277325

A. such-that B. either or C. as-as D. the same-as


11. You will have to…………… your holiday if you are too ill to travel.
A. call off B.cut down C. back out D. put aside
12. Meggie is not really looking forward to this weekend, when she…………… a family reunion.
A. has attended B. had been attending
C. will have to attend D. would be able tổ attend
13. People are…………… the pressure that they must drink eight glasses of water a day, but they don’t realize
they get water…………… other sources in their diet.
A. out of-at B. under-from C. of onto D. over-inside
14. Kenzie protected his handler from a/an…………… suspect.
A. technical B. track C. absolute D. armed
l5. Some police dogs work under cover…………… darkness
A. for B. with C. of D. at
16. The soldiers attacked the town …………… the danger of causing large numbers of civilian casualties
there.
A. despite B. in addition to C. besides D. for fear of
17. According to a study, women blink almost twice …………… men.
A. more than B. as much as C. many more D. so much that
18. Wildlife…………… are concerned about the environment.
A. scientists B. protectors C. activists D. members
19. About six million years ago, the Mediterranean sea is said…………… ,…………… thick layers of salt in
southern France.
A. to be evaporating-deposit B. to evaporate-deposited
C. having evaporated-to deposit D. to have evaporated-depositing
20. Betty is happy and…………… She always looks on the bright side of life.
A. confident B. honest C. flexible D. optimistic
III. PREPOSITIONS AND PARTICLES: Choose the best answer
1. Our holiday plans fell…………… due to lack of money.
A. over B. for C. through D. to
2. He wants to save up to buy a car so he’ll do…………… a holiday this year.
A. in B. for C. on D. with
3. You should cut…………… the number of cigarettes you smoke a day.
A. up to B. back with C. down on D. to with
4. This writer’s new book came…………… in time for Christmas.
A. in B. to C. with D. out
5. You will have to comply…………… these rules or you’ll lose your job.
A. to B.about C. for D. with
6. Sleeping Beauty will be…………… at the Theatre Royal for three weeks.
A. through B. for C. on D. in
7. The watch he bought from the man was identical…………… the one I had lost.
A. to B. with C. for D. upon
8. She held…………… some very important information which she refused to reveal.
A. from B. back C. to D. away
9. The fire will go…………… if you don’t put more wood on it.
A. down B. on C. over. D. out
10. I keep…………… current affairs by reading the newspapers.
A. upto B. up with C. up for D. up from
IV. READING COMPREHENSION
Reading 1: Read the passage carefully and select the best answer for each question from A, B, C, or D
49

Downloaded by Nhung Nguy?n (nhungnth2004@gmail.com)


lOMoARcPSD|20277325

Many flowering plants woo insect pollinators and gently direct them to their most fertile blossoms by
changing the color of individual flowers from day to day. Through color cues, the plant signals to the insect
that it would be better off visiting one flower on its bush than another. The particular hue tells the pollinator
that the flower is full of far more pollen than are neighboring blooms. That nectar-rich flower also happens to
be fertile and ready to disperse its pollen or to receive pollen the insect has picked up from another flower.
Plants do not have to spend precious resources maintaining reservoirs of nectar in all their flowers. Thus, the
color-coded communication system benefits both plant and insect.
For example, on the lantana, a flower starts out on the first day as yellow, when it is rich with pollen
and nectar. Influenced by an as-yet-unidentified environmental signal, the flower changes color by triggering
the production of the pigment anthromyacin. It turns orange on the second day and red on the third. By the
third day, it has no pollen to offer insects and is no longer fertile. On any givep lantana bush, only 10 to 15
percent of the blossoms are likely to be yellow and fertile. But in tests measuring the responsiveness of
butterflies, it was discovered that the insects visited the yellow flowers at least, 100 times more than would be
expected from the haphazard visitation. Experiments with paper flowers demonstrated that the butterflies were
responding to color cues rather than, say, the scent of the nectar.
In other types of plants, blossoms change from white to red, others from yellow to red, and so on.
These color changes have been observed in some 74 families of plant
1. The first paragraph of the passage implies that insects benefit from the color-coded communication system
because…………..
A. the colors hide them from predators
B. they can gather pollen efficiently
C. the bright colors attract fertile females
D. other insects species cannot understand the code
2. The word “woo” is closest in meaning to
A. frighten B. trap C. deceive D. attract
3. The word “it” refers to
A. a plant B. an insect C. a signal D. a blossom
4. Which of the following describes the sequence of color changes that lantana blossoms undergo?
A. Red to yellow to white B. White to red
C. Yellow to orange to red D. Red to purple
5. The word “trigger” is closest in meaning to
A. maintaining B. renewing C. limiting D. activating
6.The passage implies that insects would be most attracted to lantana blossoms…………...
A. on the first day that they bloom B. when they turn orange
C. on the third day that they bloom D. after they produce anthromacin
7. According to the passage, what is the purpose of the experiments involving paper flowers and painted
flowers?
A. to strengthen the idea that butterflies are attracted by the smell of flowers
B. to prove that flowers do not always need pollen to reproduce
C. to demonstrate how insects change color depending on the type of flowers they visit
D. to support the ideas that insects! respond to the changing color of flowers.
8. The word “haphazard” is closest in meaning to
A. Dangerous B. Random C. Fortunate D. Expected
9. What is known from the passage about the other types of plants?
A. They follow various sequences of color changes.
B. They use scent and other methods of attracting pollinators.
C. They have hot been studied as thoroughly as the lantana.
D. They have exactly the same pigments as the lantana.
10. According to the passage, m approximately how many families of plants has the color-changing
50

Downloaded by Nhung Nguy?n (nhungnth2004@gmail.com)


lOMoARcPSD|20277325

phenomenon described in the passage been observed?


A. 10 B. 15 C. 74 D.TOO
Reading 2: Fill the gaps in the passage by choosing the best phrases from A-K.
A WALK IN THE MIDDAY SUN
(1)…………..
(2) …………. Hot weather makes your heart pump harder, and if you’re not very fit, you start to
understand why the majority of mountain rescue statistics are made up from summer walkers suffering from
heart attacks. Heart exhaustion is quite easy to get when you’re making a great physical effort. It happens
where your body can’t produce enough sweat to keep you cool.
(3) …………. The answer is to keep up your water intAke. It’s a good idea to drink a pint of water for
every 10 degree Fahrenheit every 24 hours. So, if the temperature is in the 70s, and you’re doing a five-hour
walk, you’ll need a minimum of around one and a half pints of water. It’s vital that you don’t wait until you
develop a raging thirst be ore you stop for a drink-keep taking regular swigs from your water bottle.
(4) …………. Many walkers flavor their water with fruit juice, which makes it a lot more palatable.
You could even use one of the isotonic drinks made for athletes, which replace the body’s salts lost through
the sweating. Powders such as Doiralyte, which you may have in the house as a treatment for diarrhea, will do
the job just as well, as its main aim is also effective rehydration.
(5) ………….Give that evaporation is your body’s cooling mechanism; you can help things along with
an external application of water. Soaking your hat with water is a great way to cool the head, though if the sun
is beating down, it will probably dry off almost immediately. Better still then if you can plunge into the river
or the sea fully-clothed. And if that’s not possible, then at least take off your boots and socks and paddle in a
cool stream.
(6) ………….. Walking in the heat increases the rate at which your feet swell, which can lead to them
feeling tight in your boots. Cool water from a stream reduces any swelling and help general foot comfort. At
the same time, you can check out your feet for signs of blisters. Extra sweating makes the skin softer and
increases the chance of blisters forming, in the same way as when water leaks into your boots and gets to your
feet
(7) ………….. As for what clothing you wear, this should be lightweight and reasonably loose-fitting.
Tight clothing will feel uncomfortable and may even lead to the formation of an irritating rash known as
“prickly heat” on your skin. The answer, if it does develop, is to try and stay cool as much as possible. Do this
by either keeping in the shade, or washing the affected area with cold water, but without soap. But prevention
is by far the best approach, so keep your clothing light.
(8) ………….. It’s understandable to want to remove any extraneous clothing when it’s extremely hot,
but it doesn’t really make much sense to take off T- shirts. The sun‘s ray can be quite strong, and the shoulders
are very sensitive to sunburn. This is the worst place to be red and sore when you are wearing a heavy
rushsack on your back. Wearing shorts can also create problems for walkers, as the backs of the legs can catch
the sun easily.
(9) ………….. In fact, those days when an apparently harmless breeze is blowing can be the most
deceptive. It might not feel so hot, so you probably won’t notice the damage being done so soon. As on every
other day then, a good strong sun cream should therefore be applied to any skin which is exposed. (10) ……
A. Take enough water and drink it sensibly
B. There are several alternatives to just water.
C. One part of your body which can suffer is your feet
D. When the heat is on, walkers need to be on their guard.
E. Getting wet is one way of keeping temperatures down
F. The wrong clothing can cause problems.
G. Always protect those parts of your body which you are not covered by clothing
H. The heat can create serious health problems for walkers.
K. Your clothing acts as an important defense against the sun.
51

Downloaded by Nhung Nguy?n (nhungnth2004@gmail.com)


lOMoARcPSD|20277325

L. Make the most of the summer, but treat the sun with the respect it deserves.

V. GlUDED CLOSE TEST:


1. Read ‘be test below and decide which answer (A, B, C, or D) best fits each Spice.
CHILD LABOUR
Child workers, some as young as 10, have been found working in a textile (1)………….in conditions
described as close to slavery to produce clothes that appear destined for one of the major high street retailers.
Speaking to a British newspaper, the children described long hours of (2) ………….work and threats and
beatings. The company said it was unaware that clothing intended for its (3) ………….had been improperly
(4) …………. to a(n) (5) …………. that used child labor. It further announced it had withdrawn the garments
involved until it had investigated the alleged (6) ………….of the (7) ………….code it composed on
manufacturers three years ago.
The discovery of these children working in appalling conditions in the Shahpur Jat area of Delhi has
renewed concerns about the outsourcing by some large retail chains of their (8) ………….production to India,
recognized by the United Nations as one of the world’s hotspots for child labor. According to one estimate,
over 20 per cent of India’s economy is (9) ………….on the children, which comes to a total of 55 million
youngsters under 14 working.
Consumers in the West should not only be demanding answers from retailers about how their (10)
…………. are produced but also should be looking into their consciences at how they spend their money and
whether cheap prices in the West Ị are worth the suffering caused to so many children.
1. A. office B. facility C. factory D. bureau
2. A. impaid B. unpaid C. without pay D. wihout payment
3. A. stores B. branch C. outlet D. warehouses
4. A. outsource B. outsourcing C. outsources D. outsourced
5. A. association B. closed shop C. retailer D. sweatshop
6. A. errors B. breaches C. mistakes D. wrongdoings
7. A. ethical B. ethnic C. ethnical D. ethic
8. A. garments B. garment C. raiment D. raiments
9. A. depends B. dependent C. dependant D. dependence
10. A. goods B. ware C. stuff D. garment
2. Read the test below and decide which answer (A, B, C or D) best fits each space.
MYSTERY OF THE DEEP
Every year half a million visitors make their way to Loch Ness in Scotland. The Loch Ness is a dark
and mysterious expanse of water, 300 metres deep in places, but most visitors come in the hope of (1)………
the glimpse of the famous monster, Nessie. Those who believe in the monster’s existence (2) ………….
that it is a type of marine dinosaur otherwise presumed to have become extinct seventy million years ago.
More special observers, however, regard the story as nothing more than a clever ploy to (3) ………….the
tourists.
Indeed, it was a local hotel owner who made the first modern (4) …………. of the monster in 1933,
The local newspaper ran the story, which was then (5) ………….by the national and international press. A
photograph of the monster, taken the following year by Robert Wilson, a local doctor, created an overnight
(6) ………….worldwide. This photograph (7) …………. the most convincing evidence available of the
monster’s existence for almost sixty years.
Dr Wilson had, however, taken part in an elaborate hoax. A fact only (8) ………….following the
deathbed confession of one of the other people involved in 1993. The photograph, which featured a serpent-
like head and neck rearing up from the waters of the loch had been achieved using a (9) ………….disguised
toy submarine. For the monster’s millions of fans, this was something of a setback, but little more than that.
Only time will tell whether the other photographs that exist are (10) ………….or not, but meanwhile Nessie
continues to intrigue people and the tourist keep coming.
52

Downloaded by Nhung Nguy?n (nhungnth2004@gmail.com)


lOMoARcPSD|20277325

1. A. taking B. gaining C. catching D. finding


2. A. claim B. pretend C. demand D. require
3. A. appeal B. admit C. announce D. attract
4. A. meeting B. viewing C. seeking D. sighting
5. A. picked up B. swept on C. given out D. put through
6. A. recognition B. impression C. sensation D. perception
7. A. persisted B. remained C. insisted D. maintained
8. A. displayed B. revealed C. exhibited D. unclocked
9. A. strongly B. severely C. heavily D. powerfully
10. A. truthful B. honest C. genuine D. sincere
PART B: WRITTEN TEST
I. Open close test:
Test 1: Fill in each blank with ONE word
Although speech is the most advanced form of communication, there are many ways of
communicating without using speech. Signals, signs, symbols, and gestures may be found in every (1) ………
culture. The basic function of a signal is to impinge (2) …………. the environment in (3) …………. a way
that it attracts attention, as, for example, the dots and dashes of a telegraph circuit. Coded to refer to speech,
the potential for communication is very great. Less adaptable (4) ………….the codification of a word, signs
also contain meaning in and of (5) ………….. A stop sign or a barber pole conveys meaning quickly and
conveniently. Symbols are more _______________ difficult to describe than either signals or signs (6)
………….of their intricate relationship with the receiver’s cultural perception. In some cultures, applauding in
a theatre provides performers with an auditory symbol of approval. Gestures such as waving and handshaking
also communicate (7) …………. cultural messages.
(8) …………. signals, signs, symbols, and gestures are very useful, they do have a major
disadvantage. They usually do not allow ideas to be shared (9) ………….the sender being directly adjacent to
the receiver. As a result, means of communication intended to be used for long distances and extended periods
(10) ………….based upon speech. Radio, television, and the telephone are only a few.
Test 2: Fill in each blank with ONE word
STUDYING ENGLISH ABROAD
Just as more English is taught in primary schools worldwide, so there are a (1)………….number of
language schools in English-speaking countries that are increasing courses for younger learners. In Britain,
most young learners’ courses are (2) ………….to 11-years-olds and upwards, with a few that cater for
children as young as seven. But what is a sensible (3) ………….to start sending abroad to study? After all, (4)
………….adults learners find the experience of travelling to a strange country and encountering people (5)
………….speak a barely comprehensible language challenging.
A great (6) …………. depends on how a child has been brought up. For example, (7)………….they
have been opposed to new things and have already started to learn independence, then they are likely to enjoy
the experience. In (8) ………….children who have previously travelled abroad with their parents will be well
prepared to undertake a study trip in an English-speaking country on their (9) ………….. Getting used to the
idea that other countries have different customs and cultures (10)………….an important stage in the
process of learning a foreign language.
II. WORD FORMS
A. Using the right form of the word given in brackets to complete each sentence.
1. Eating fish and lots of vegetables greatly increases your life…………….(EXPECT)
2. The British regarded the colonies as a source of revenue and began to impose……………..taxes upon them.
(EQUITY)
3. It was a voice I had not heard before and it ……………..me. (ORIENTATE)
4. It …………….. my heart to see them all enjoying themselves last night. (GLAD)
5. She has grey hair and a……………..looking face. (YOUNG)
53

Downloaded by Nhung Nguy?n (nhungnth2004@gmail.com)


lOMoARcPSD|20277325

6. The world is becoming increasingly……………..(DEPEND)


7. I did everything I could to make their stay enjoyable, but they weren’t at all……………..(APPRECIATE)
8. After years of……………..himself in drinking, he now suffers from nervous breakdown. (INDULGE)
9. Cenozoic saw the marked ……………..of mammals that would ultimately become man’s ancestors.
(DOMINATE)
10. Gentle massage will……………..your skin. (VITAL)
B. Using the correct word form of the word in brackets to fit each gap.
CREDIT CARDS
The first exprimental step towards creating a credit card was taken by a US company in 1914. They
issued a…………….(1. SOPHISTICATED) metal token to their customers, allowing them to defer their bills.
But ……………… (2. NEAR) half a century was to pass before the……………..(3. POSSIBLE) of
establishing a……………....(4. UNIVESE) credit card would be taken……………..establishing a……………
(4. UNIVERSE) credit card would be taken……………(5. SERIOUS). France was the …………. (6. LEAD)
creating the “Carte Bleue”. Barclays bank in Britain…………. (7. QUICK) followed. When the Visa system
was developed in 1977, credit rose, with most credit holders joining this …………. (8. GLOBE) credit
network. By 1944 the Visa system had a credit total of $ 207.4 billion in 2.8 million…………. (9. LOCATE)
in Europe. The credit card has…………. (10. TRUE) become intematiọnaỉ.
III. ERROR IDENTIFICATION
The passage below contains 10 errors. Identify 10 errors and correct them.
MOUNTAIN CLIMBING
The most frightening moment in my life happened to me a few years ago while I was been at
university. I was a member of the rock climbing club, and every weekend, we would go for climbing in the
Welsh mountains. One Saturday we set up in fine spirits - the sun was shining and it was not a cloud in the
sky. We began to climb the mountain, and by lunchtime we had managed to get at last halfway up. While we
were eating our sandwiches and mint cake we noticed some dark clouds to form in the distance. We weren’t
worried, however, so we continued in our way. This turned out to be a big mistake - the cloud got closer and
closer, then it began to snow. We tried to get away the mountain, but the snowstorm became very much bad
that we got stuck. Eventually we were rescued, but not before we had spent half the night up here, cold,
miserable and sure about we were all going to die. I hope it will never happen again.
1…………….. 6……………..
2…………….. 7……………..
3…………….. 8……………..
4…………….. 9……………..
5…………….. 10……………..

IV. SENTENCE TRANSFORMATION


A. Complete the second sentence so that it has similar meaning to the one printed before it
1. Absolute secrecy was crucial to the success of the mission.
→ Without……………………………………………………………………………………………….
2. Further progress was impossible because of the floods.
→ The floods ……………………………………………………………………………………………
3. The hurricane blew thè roof off the house.
→ The house ……………………………………………………………………………………………..
4. You must never mention this to him.
→ Under………………………………………………………………………………………………….
5. I found his account of the accident was confusing.
→ I………………………………………………………………………………………………………..
B. Complete each sentence so that it contains the word in capitals on brackets, and so that the meaning
stays the same
54

Downloaded by Nhung Nguy?n (nhungnth2004@gmail.com)


lOMoARcPSD|20277325

1. That’s the second time this year that Susan refused to marry him.(DOWN)
→…………………………………………………………………………………………………………..
2 Your examination results will determine whether or not you get into university.(ON)
→…………………………………………………………………………………………………………..
3. She has become a famous novelist.(NAME)
→…………………………………………………………………………………………………………..
4. He often gets ill at this time of the year.(PRONE)
→…………………………………………………………………………………………………………..
5. I am not normally keen on cycling at the weekend. +(IN)
→…………………………………………………………………………………………………………..

TRƯỜNG THPT CHUYÊN PHAN NGỌC HIỂN - CÀ MAU


A. MULTIPLE CHOICE
I. PHONOLOGY
Choose the word whose underlined part is pronounced differently from the other three.
1. A. surgical B. surround C. surrender D. survival
2. A. opportunity B. menu C. attitude D. muscular
3. A. physical B. mythology C. rhythm D. psychology
4. A. apartheid B. thorough C. cathedral D. ethnic
5. A. exhausted B. cheetah C. rhinoceros D. alcoholic
Choose the word whose stress pattern is different from that of the other three.
6. A. advisable B. admirable C. reliable D. desirable
7. A. commerce B. sacrifice C. maintenance D. stability
8. A. equivalent B. architectural C. amphibian D. majority
9. A. argumentative B. suspicious C. compulsory D. divisible
10. A. antibiotic B. unsatisfactory C. democratic D. experimental
II. WORD CHOICE
Circle the correct option (A ,B, C or D) to finish each of the following sentences.
11. Unfortunately our local cinema is on the…………of closing down.
A. verge B. hint C. edge D. threat
12. Your argument…………that Britain is still a great power, but it is no longer the case.
A. outlines B. presupposes C. concem D. presents
13. Although he didn’t actually say he wanted it he did…………pretty strongly.
A. imply B. suggest C. mention D. hint
14. Because of cutbacks in council spending, plan for the new stadium had to be…………
A. stockpiled B. overthrown C. shelved D. discrupted
l5. Mark is very set in his ways, but John has a more…………attitude to life.
A. changeable B. flexible C. moveable D. fluid
16. The scheme is too expensive to be…………at the present moment.
A. liable B. compatible C. feasible D. acessible
17. Please…………from smoking until the plane is airborne.
A. exclude B. refrain C. resist D. restrain
18. No matter how angry he was, he would never…………to violence.
A. resolve B. recourse C. exert D.resort
19. The travel agency sent us a detailed…………for our journey to India.
A. docket B. agenda C. itinerary D. documentation
20. Since our train leaves at 10.30, it is…………that everyone is at the station no later than 10.15.
A. imperative B. urgent C. desired D. inescapable
III. STRUCTURES AND GRAMMAR
55

Downloaded by Nhung Nguy?n (nhungnth2004@gmail.com)


lOMoARcPSD|20277325

Choose the word or phrase that best completes each sentence.


21. The early railroads were…………the existing arteries of transportation: roads, turnpikes, canals and other
waterways.
A. those short lines connected B. short lines that connected
C. connected by short lines D. short connecting lines
22.………… as a masterpiece, a work of art must transcend the ideas of the period in which it was created.
A. Ranking B. To be ranked
C. Being ranked D. In order to be ranking
23. The discovery of the halftone process in photography in 1881 made it…………photographs in books and
newspapers.
A. the possible reproduction B. possible to reproduce
C. the possibility of reproducing D. possibly reproduced
24. When wood, natural gas, oil or any other fuel burns,…………with oxygen in the air to produce heat.
A. combining substances in the fuel B. substances in the fuel that combine
C. substances in the fuel combine D. a combination of substances in the fue
25. In bacteria and in other organisms, …………is the nucleic acid DNA that provides the genetic
information.
A. it B. which C. there D. what
26. She…………fainted when she heard that her child died.
A. rather than B. nothing but C. all but D. near
27. …………the U.S. superiority at that time, it was probable that any threatened U.S response would have
deterred the Soviet Union.
A. If B. Given C. Although D. Since
28. In the 1850’s Harriet Beecher Stowe’s “Uncle Tom’s Cabin” became the best seller of the generation,
…………a host of imitators.
A. inspiring B. inspired C. inspired by D. to inspire
29. - “Who wdn the election for mayor?”
- “A man…………to represent every minority group in the city.”
A. claim B. claimed C. is claimed D. claiming
30. “Although they met with many difficulties, I heard that they succeeded…………”
A. over all B. above all C. after all D. all in all
IV. PREPOSITION AND PHRASAL VERBS
Choose correct alternative to complete each sentence
31. I must go on a diet. I…………a lot of weight while I was on holiday.
A. held up B. put on C. settled down D. weighed up
32. As they’re identical twins, it’s difficult to………….
A. mix them up B. take them apart C. tell them apart D. set them up
33. It's terrible. Pete’s…………pneumonia. He is in hospital now.
A. come round B. gone in for C. got over D. come down with
34. They were very kind. They…………until I found a place of my own.
A. back me up B. bring me round C. put me up D.ypick me up
35. My parents are not interested in modern music. They are…………the times.
A. beyond B. behind C. below D. beneath
36. He gained acendancy…………all his main rivals.
A. against B. over C. through D. for
37. We have to go ………… our work right now, or we won’t finish it on time.
A. through B. after C. about D. over
38. The two trains came…………ten metres of collision.
A. to 3. into C. on D. within
56

Downloaded by Nhung Nguy?n (nhungnth2004@gmail.com)


lOMoARcPSD|20277325

39. The skyscraper stands out ………… the blue sky.


A. against B. of C. off D. through
40. I got…………the Arts Faculty at the University of London to study history.
A. in B. into C. to D. within
V. READING COMPREHENSION
READING 1
Read the following passage and circle A, B, C or D for each of the questions.
Carnegie Hall, the famous concert hall in -New York, has again undergone a restoration. While this is
not the first, it is certainly the most extensive in the buiding’s history. As a result of this new restoration,
Carnegie Hall once again has the quality of sound that it had when it was first built.
Carnegie Hall owes its existence to Andrew Carnegie, the wealthy owner of a steel company in the late
1800s. The hall was finishedcin 1891 and quickly gained a reputation as an excellent performing art hall
where acomplished musicians gained fame. Despite, its. reputation, however, the concert hall suffered from
several detrimental renovations over the years. During the Great Depression, when fewer people could afford
to attend performaces, the directors sold part of the building to Commercial businesses. As a. result, a coffee
shop was opened in one corner of the building, for which the builders replaced the brick and terra cotta walls
with windowpanes. A renovation in 1946 seriously damaged the acoustical quality of the hall when the makers
of the film Carnegie Hall cut a gaping hole in the dome of the ceiling to allow for lights and air vents. The
hole was later covered with short curtains and a fake ceiling, but the hall never sounded the same afterwards.
In 1960, the violinist Issac Stern became involved in restoring the hall after a group of real estate
developers unveiled plans to demolish Carnegie Hall and build a high- rise office building on the site. This
threat spurred Stern to rally public support for Carnegie Hall and encouraged the city of New York to buy the
property. The movement was successful, and the concert hall is now owned by the city. In the current
restoration, builders tested each new material for its sound qualities, and they replaced the hole in the ceiling
with a dome. The builders also restored the outer walls to their original apperance and closed the coffee shop.
Carnegie Hall has never sounded better, and its prospect for the future have never looked more promising.
41. This passage is mainly about…………..
A. changes to Carnegie Hall
B. the appearanc of Carnegie Hall
C. Carnegie Hall’s history during the Great Depression
D. damages to the ceiling in Carnegie Hall
42. The word extensive in the first paragraph could be best replaced by ,which of the following?
A. fabulous B. thorough C. devoted D. continuous
43. What is the meaning of the word detrimental in the second paragraph?
A. dangerous B. significant C. extreme D. harniful
44. What major change happened to the hall in 1946?
A. The acoustic dome was damaged.
B. Space in the building was sold to commercial businesses.
C. The walls were damaged in an earthquake.
D. The stage was renovated.
45. What was Issac Stem’s relationship to Carnegie Hall?
A. He made the movie Carnegie Hall in 1946.
B. He perfomed on opening night in 1891.
C. He tried to save the hall, beginning in 1960.
D. He opened a coffee shop in Carnegie Hall during the Depression.
46. What was probably the most important aspect of the recent renovation?
A. restoring the outer wall B. expanding the lobby
C. restoring the plaster trim D. repairing the ceiling
47. Which of the following is closest in meaning to the word unveiled in the third paragraph?
57

Downloaded by Nhung Nguy?n (nhungnth2004@gmail.com)


lOMoARcPSD|20277325

A. announced B. restricted ; C. overshadowed D. located


48. The author uses the word spurred in the third paragraph to show that Stern…………..
A. predicted the result B. probed the plan
C. was told in advance D. was stimulated to act
49. How does the author seem to feel about thè future of Carnegie Hall?
A. ambiguous B. guarded C. optimistic D. negative
50. Which of the following would most likely be the topic of the next paragraph?
A. a scientific explanation of acoustics and the nature of sound.
B. a description of people’s reactions to the newly renovated hall.
C. a discussion of the coffee shop that once was located in the building
D. furrther discussion about the activities of Issac Stern in 1960.
READING 2
Read the following passage and circle A, B, C or D for each of the questions.
Situated in the central mountains of Alaska, a peak named Denali rises 20,320 feet above sea level. It
is the highest peak in North America and the center of Denali National Park. One of America’s greatest
wilderness areas, the park had limited access to visitors, but in spite of this tourism rose from under 6,000
visitors in 1950 to over 546,000 visitors in 1990. The increasing popularity of this park is prompting serious
dicussions about the future use of Denali as well as how to preserve wilderness areas in general.
One important issue of land use arises when parts of the National Parks are owned by individuals. In
Denali, though most of the land in this vast tract of more than a million acres is owned by the National Park
Service, several thousand acres are still privately owned as mining tracts. These mining tracts in Denali were
once abundant sources of gold, but they were sources of heavy metals such as aesenic and lead that polluted
rivers and streams.
Environmentalists were successful in getting the government to require mining companies to submit
statements showing the potential impact of a mining project before they are allowed to begin mining.
Because of this requirement, many individuals closed their mines and some sold their land to the National
Park Service. Some land owners, however, are wondering if it is better to sell their land to the government or
keep it for possible future use. Tourism in this previously remote area is bound to rise, as more roads are built
to provide easier access to the park. This increase in the number of visitors creates a demand for hotels and
real estate development. The economic implications of this are of interests to the land owner, but are
dismaying to those interested in preserving the wilderness.
5l. What is the primary focus of this passage?
A. controversies over land use in Denali.
B. miners selling their property in Denali.
C. Alaska building more roads to Denali.
D. limiting tourist access to Denali.
52. The word prompting in the first paragraph could best be replaced by which of the following?
A. promising B. sanctioning C. initiating D. trapping
153. The word arises in the second paragraph could best be replaced by……………
A. surrenders B. occurs C. volunteers D. prospers
54. The word tract in the second paragraph refers to……………
A. trail B. resort C. frontier D. expanse
55. Which of the following is most similar to the word abundant in the second paragraph?
A. plentiful B. sparse C. hopeful D. absolute
56. According to the passage, which of the following are pollutants in the Denali area?
A. gold B. pesticides C. human waste D. arsenic
57. Which of the following is closest in the meaning to the phrase potential impact in the third paragraph?
A. appropriate cost B.expected value
C. proposed size D. possible effects
58

Downloaded by Nhung Nguy?n (nhungnth2004@gmail.com)


lOMoARcPSD|20277325

58. The author infers that some mine owners might hesitate to sell their land to the Park Service for which of
the following reasons?
A. There may be increasing demand for the one in the mines.
B. They might want to build a house on their property.
C. They might receive more money selling their lands to developers.
D. They might want to move to the towns.
59. What is the author’s purpose in writing this passage?
A. to demonstrate the changes in Denali National Park.
B. to use Denali as an example of common park issues.
C. to introduce the wonders of the wilderness area in Denali.
D. to explain the problems occurring in Denali Park
60. Which of the following would most likely to be the topic of the next paragraph in this passage?
A. conflict between land owners and environmentalists.
B. the role of the National Park Service in development.
C. tourist needs in Denali Park.
D. wildlife in the park.
VI. CLOZE TEST
CLOZE TEST I
Read the following passage and choose the option that best complete the blanks
WHERE DID THE DINOSAURS GO?
No account of the dinosaurs’ time on Earth is complete without an explanation of their extinction. (61)
…………..the years many theories have been(62) ………….., but no single theory can explain all the facts as
we know them. Nowadays, the most likely theory that scientists have come up with is that dinosaurs were
killed off by a combination of several factors. The (63) …………..effect of these factors was the
extinction of the dinosaurs, together with many other animal groups. It is known that there was a lot of
volcanic activity at the (64) ………….. of the dinosaurs’ extinction. Their environment would have already
been (65) …………..considerable stress as well as being poisoned. Added to this it is believed that a very (66)
…………..meteor or comet, some 10 kilometers wide, (67) ………….. the Earth and caused massive
destruction. (68)……………of the impact can be found in the Gulf of Mexico, where there is a large crater
and a (69) …………..of rock containing the rare element iridium. As the meteor entered the Earth’s
atmosphere, the dinosaurs would have seen a bright light in the sky followed by an explosion. The shock
waves from the explosion flattened everything. (70) …………...after, a rain of dust and rocks fell and covered
everything, the temperature rose dramatically and that was the end of dinosaurs.
61. A. For B. During C. As D. Over
62. A. put forward B. brought up C. handed out D. turned up
63. A. complete B. entire C. overall D. sum
64. A. hour B. era C. time D. period
65. A. in B. below C. into D. under
66. A. long B. large C. enormous D. great
67. A. hit B. crashed C. collided D. smashed
68. A. Signals B. Notices C. Signs D. Tracks
69. A. cover B. layer C. level D. coat
70. A. Briefly B. Quickly C. Sharply D. Shortly
CLOZE TEST 2
Read the following passage and choose the option that best complete the blanks
As time passes, the power of newspaper seems to be on the (71) …………... This is odd because in the
relatively recent past people were predicting that the influence of the written word would diminish in (72)
…………..proportion to the rate of increase of the spoken word and moving image through TV and video.
The Internet, cable and satellite television, Teletext and multi-media computers in (73)…………..other
59

Downloaded by Nhung Nguy?n (nhungnth2004@gmail.com)


lOMoARcPSD|20277325

home should surely have (74) …………..for newspapers by now, particularly alongside a perceptible
resurgence in the audiences for news-carrying radio stations. How have these organs survive, let alone (75)
………….. , particularly on a Sunday? Why do people who have seen a football or tennis match live or on the
small screen rush the next day to read a (76) ………….. version of it in four or five columns which surely
cannot mean more to the reader than that self-same viewer of the previous afternoon or evening? Why would
anyone who has seen a film and formed a (77) ………….. impression of it ihl the following day read a review,
of the (78) …………..film in a newspaper? To see if she is right? Isn’t that what friends are for? Don’t we
have colleague for just that purpose - to see if our ideas on any (79) …………..song, film or program tally
with (80) …………..?
71. A. increase B. rise C. expansion D. build
72. A. exact B. direct C. precise D. equal
73. A. all B. any C. every D. one
74. A. done B. gone C. stood D. set
75. A. flourished B. bloomed C. flowered D. rooted
76. A. curtailed B. cut C. reduced D. potted
77. A. vivid B. coloured C. bright D. direct
78. A. above-mentioned B. aforesaid C. latter D. previous
79. A. given B. taken C. subjected D. written
80. other B. others C. other’s D. others’
B. WRITTEN TEST
I. OPEN CLOZE TEST
CLOZE TEST 1
Fill in each blank with one suitable word.
Some famous places are disappointing: dirty, cramped and a bit of a cliché. But there are others (1)
…………., even though you’ve seen every television programme ever made about it, are every bit as
wonderful as you’d imaged. The Grand Canyon is one of these and so, (2) ………….being next door to a
main road, is Stonehenge. Another is Venice which, in its entirety, (3) ………….a great work of art, each
decaying aspect revealing an (4) ………….glimpse of water or startling achitecture, each individual buidling
or piazza (5) ………….an exquisite sense of proportion. I return to Venice every so (6) ………….in the
course of my work and on each of these occasions I have found something to marvel at. Alarm cries about
how (7) ………….this can last are sounded every now and then each time the water levels rise. But the fact
that this city is (8) …………. into the sea seems to add its romantic atmosphere. Far more serious is the
depopulation, for it seems that just about every week another family leaves. Since 1945 more than half of the
population of Venice has moved to the mainland. The rich (9) ………….the great palazzos along the Grand
Canal and visit every once in a while, but leave the windows dark for the rest of the time. Mass tourism
threatens (10) ………….very structure of the city. It is a sad victim of its own success.
CLOZE TEST 2
Fill in each blank with one suitable word
After living (11) ………….the threat of extinction for more than 30 years, the national bird of the
United States has been granted an official reprieve, as the bald eagle and twenty-eight other animal and plant
species have been earmarked (12) ………….removal from America’s list of endangered species. The bald
eagle, also known as the white-headed sea eagle, took pride of place at the top of a list of species likely to (13)
………….taken off the endangered register in the coming years. The proposed “delistings” are being
promoted by the US interior secretary to counter a growing feeling among Republicians that endangered-
species laws do not work. Charges of ineffectiveness have been (14) …………. against these laws before,
but more recently it has even been suggested that the situation may actually have been (15) ………….
worse by them. The recovery of the bald eagle follows thirty-one years on the critical list. Its numbers had
been (16)………….to fewer than five hundred through the use of pesticides that reacted adversely (17)
………….its productive system. The number of nesting pairs is now estimated at five thousand. The interior
60

Downloaded by Nhung Nguy?n (nhungnth2004@gmail.com)


lOMoARcPSD|20277325

secretary claims that the new list was à vindication of the legislation under which the eagle, a national symbol
(18) ………….originally from the Indians, and more than a thousand other species were protected. (A
spokeperson denied that it was simply a publicity stunt timed to coincide with the swearing (19) ………….of
the president for his second term.) Until now, few species have (20) ………….been removed from the list.
When they were, it was usually because they had become extinct.
II. WORD FORMS
PART 1; Complete the sentences below with the correct form of the words in BOLD at the end of each
sentence.
21. Our teacher told us not to include…………..information in our essay. RELEVANT
22. Some children are very rude. They just don’t speak to adults…………...RESPECT
23. Keeping large animals in cages is ………….. HUMANE
24. Eating that meat could be dangerous. It looks…………..to me. COOK
25. It costs a…………..amount of money to keep up-to-date with technology CONSIDER
26. The record of 47 hours for watching TV without stopping was set last year and is still…………..
BREAK
27. Flooding is…………..rare in this part of the world COMPARE
28. It is a/an…………..fact that children watch too much TV. DENY
29. Tulips are a familiar…………..in British gardens in springtime.SEE
30. In the UK there is no legal…………..for children to go to school OBLIGE
PART 2: Suppy each gap with the correct form of the word given in the box.
know prefer notice great research
recognize vary respond rely contain

GENETIC GENIUS
According to a recent study the best musicians are born, not made. (31) ………….at St Thomas’s
Hospital in London claim that genes are responsible for up to 80 per cent of our ability to recognize pitch, the
key to musical (32) ………….. The discovery by the hospital’s Twin Research Unit, the largest of its kind in
the world, account for the prevalence of musical families from the Bachs to the Corrs and the Strausses to the
Jacksona. In a “distorted tunes test” over 500 twins were played a (33) ………….of popular songs, each (34)
………….a number of errors. A comparison of the (35) ………….of identical twins with those of non-
identical twins revealed that the former were (36) …………. better at spotting the mistakes. The results of the
study suggest that for some children, music lesson may only go so far in improving musical abilities such as
pitch (37) ………….. However, parents hoping to save money on lessons cannot use the test as an early
indicator of musical potential: It is (38) …………. , for children under 12, who do not have sufficient (39)
………….of the tunes played. For its next project, the Twin Research Unit will test whether identical twins
can tell us if genes have a role to play in a (40)………..for classical, jazz or pop music.
III. ERROR IDENTIFICATION
There are 10 errors in 10 lines in the following passage. Identify and correct them.
The first has been done for you.
Most children with (0) health appetites are ready to eat almost anything that is offering (0) healthy
them and a child rarely dislikes food unless it is badly cooked. The way the meal is 41…………..
cooked and served is most important and an attractive served meal will often improve a 42.…………..
child’s appetite. Never ask a child whether he likes and dislikes a food and never discuss 43.…………..
likes and dislikes in front of him or allow nobody else to do so. If the father says he 44.…………..
hates fat meat or the mother refuses vegetables under the child’s hearing he is likely to 45.…………..
copy this procedure. Take it for grant that he likes everything and he probably will. 46.…………..
Nothing healthful should be omitted out the meal because of a supposing dislike. At 47.…………..
meal times it is a good idea to give a child a small portion and let him come back for a 48.…………..
second helping other than give him as much as he is likely to eat all at once. Do not talk 49…………..
61

Downloaded by Nhung Nguy?n (nhungnth2004@gmail.com)


lOMoARcPSD|20277325

too much to the child during meal times, but let him get on with his food, and do not 50.…………..
allow him to leave the table immediately after a meal or he will soon learn to swallow
his food so that he can hurry back to his toys. Under no circumstance must a child be
coaxed or forced to eat.

IV. SENTENCE TRANSFORMATION


51. Buying new clothes is something that I almost never make the effort to do.
→ I hardly………………………………………………………………………………………..
52. There is nobody in this office that will offer a different opinion to the one I gave you.
→ Whoever………………………………………………………………………………………
53. “Without the rapid growth in poluation there would be fewer food shortages.”
→ He blames……………………………………………………………………………………..
54. I didn’t realize how much he was influenced by his brother.
→ I didn’t realize the extent……………………………………………………………………..
55. Not aword came out of her mouth. LOST
→…………………………………………………………………………………………………
56. Your attitude will have to change if you want to succeed. LEAF
→…………………………………………………………………………………………………
57. That hotel is a bit too expensive for us, I am afraid. REACH
→…………………………………………………………………………………………………
58. We had to return home because we had no money left. RUN
→…………………………………………………………………………………………………
59.I don’t .think it wassreasonable of you to complain so much about the service. MAKE
→…………………………………………………………………………………………………
60. Robert was offended when he was left out of the team. EXCEPTION
→…………………………………………………………………………………………………

TRƯỜNG THPT CHUYÊN TRẦN HƯNG ĐẠO - BÌNH THUẬN


A. MULTIPLE CHOICE QUESTIONS
PHONOLOGY
Part 1. Pick out the word whose underlined part is pronounced differently from the rest.
A. beloved B. helped C. naked D. wicked
A. devotion B. congestion C. suggestion D. question
3. A. draught B. naughty C. slaughter D. plaudit
4. A. architect B. parachute C. archaic D. psychology
5. A. placid B. implacable C. explanatory D. plagiarize
Part 2. Pick out the word whose stress pattern is different from that of the words.
1. A. accelerate B. digestive C. disorder D. infantry
2. A. redundancy B. comparison C. maintenance D. capacity
3. A. technological B. available C. alternative D. compulsory
4. A. potentially B. correspondance C. argumentative D. simultaneous
5. A. picturesquest B. pioneer C. eliminate D. introduce
II. VOCABULARY
Choose the word or the phrase which best completes each sentence.
1. Although he didn’t actually say he wanted it he did…………..pretty strongly.
A. imply B. suggest C. mention D. hint
62

Downloaded by Nhung Nguy?n (nhungnth2004@gmail.com)


lOMoARcPSD|20277325

2.The new sports centre…………..for all kinds of leisure activities.


A. deals B. caters C. furnishes D. supplies
3. If you’re at a(an) .…………..end, you could help me in the garden.
A. open B. free C. empty D. loose
4. I intend to…………..an official complaint to the Director.
A. write B. lodge C. place D. take
5. She expressed her…………..for certain kinds of cheaply produced movies.
A. disapproval. B. distaste C. dissatisfaction D. disloyalty
6. He was…………..a mile of the hotel when he ran out of petrol.
A. within B. inside C. only D. hardly
7. Until a…………..agreement has been reached, I am not committed to accepting the offer.
A. hard B. stable C. firm D. settled
8. He agreed to accept the position …………..that he would be given a share of the company’s profits.
A. in the agreement B. with the aim
C. with the purpose D. on the understanding
9. Few people can do creative work unless they are in the right…………..of mind.
A. frame B. trend C. attitude D. tendency
10.I must get to bed early tonight, I sat up till the…………..hours to finish that report.
A. small B. late C. deep D. last
III. STRUCTURES AND GRAMMAR
Choose the word or the phrase which best completes each sentence.
1…………..effects to combat it, drug abuse is on the increase.
A. Instead of B. In the event of C. Throughout D. Despite
2. Could you possibly…………..me at the next committee meeting?
A. stand in for B. make up for C. fall back on D. keep in
with
3. The front door is warped from the humidity. We have a difficult time…………..it.
A. for opening B. opening C. having opened D. to pen
4. He was given a medal in…………..of his service to his country.
A. knowledge B. gratitude C. recognition D. response
5. Don’t let anything…………..you from your training programme.
A. defer B. defect C. distract D. disturb
6. The building work must be finished by the end of the month…………..of cost.
A. ignorant B. thoughtless C. uncaring D. regardless
7. Whenever he had an important decision to make, he…………..a cigar, supposedly to calm his nerves!
A. had lit B. would have lit C. would light D. would be lighting
8………….., the people who come to this club are in their twenties and thirties.
A. By and large B. All together C. To a degree D. Virtually
9. It…………..rains whenever I go out without my umbrella.
A. typically B. continually C. invariably D. infallible
10. He said that the plane had already left and that I…………..arrived an hour earlier.
A. must have B. had to C. should have D. was supposed to
IV. PREPOSITIONS AND PHRASAL VERBS
Choose an appropriate particle or preposition to complete each sentence.
1. The new law on dropping litter comes ………….. force next month.
A. to B. into C. out D. off
2. People who eat an unhealthy diet are susceptible …………..all kinds of illnesses and diseases.
A. from B. in C. to D. about
3. Damage to the building resulted………….. an unsually high wind.
63

Downloaded by Nhung Nguy?n (nhungnth2004@gmail.com)


lOMoARcPSD|20277325

A. from B. in C. of D. into
4. My cousin talked…………..length about his recent holiday and bored everyone to death!
A. in B. for C. by D. at
5. I applied for a part-time job at the supermarket. They’re going to take me…………..
A. up B. on C. in D. out
6. When I was younger I wanted to be an air pilot but I soon went…………..the idea when I realised I hated
flying.
A. off B. away C. by D. out
7. Anna: This problem’s driving me crazy!
Joe: Well, hạye you tried standing …………..from the situation and evaluating it objectively?
A. in B. up C. back D. out
8. The government’s plans to reduce crime came ............... for a lot of criticism fom freedom groups.
A. out B. from C.in D.up
9. I was…………..the impression that you liked Indian food.
A. on B. with C. over D. under
10. This is one of the exceptions…………..the rule.
A. of B. about C. for D. to
V. READING COMPREHENSION
Part 1: Read the following passage and choose the correct answer to each question.
All mammals feed their young. Beluga whale mothers, for example, nurse their calves for some twenty
months, until they are about to give birth again and their young are able to find their own food. The
behavior of feeding of the young is built into the reproductive system. It is a nonelective part of
parental care and the defining feature of a mammal, the
(5) most important thing that mammals - whether marsupials, platypuses, spiny anteaters, or placental
mammals - have in common.
But not all animal parents, even those that tend their offspring to the point of hatching or birth, feed
their young. Most egg-guarding fish do not, for the simple reason that their young are so much smaller
than the parents and eat food that is also much smaller than
(10) the food eaten by adults. In reptiles, the crocodile mother protects her young after they have hatched
and takes them down to the water, where they will find food, but she does not actually feed them. Few
insects feed their young after hatching, but some make other arrangement, provisioning their cells and
nests with caterpillars and spiders that they have paralyzed with their venom and stored in a state of
suspended animation so that their
(15) larvae might have a supply of fresh food when they hatch.
For animals other than mammals, then, feeding is not intrinsic to parental care. Animals add it to their
reproductive strategies to give them an edge in their lifelong quest for descendants. The most
vulnerable moment in any animal's life is when it first finds itself completely on its own, when it must
forage and fend for itself. Feeding postpones that
(20) moment until a young animal has grown to such a size that it is better able to cope. Young that are fed
by their parents become nutritionally independent at a much greater fraction of theft full adult size.
And in the meantime those young are shielded against the vagaries of fluctuating of difficult-to-find
supplies. Once a species does take the step of feeding its young, the young become totally dependent
on the extra effort. If both parents are
(25) removed, the young generally do no survive.
1. What does the passage mainly discuss?
(A) The care that various animals give to their offspring.
(B) The difficulties young animals face in obtaining food.
(C) The methods that mammals use to nurse their young.
(D) The importance among young mammals of becoming independent.
64

Downloaded by Nhung Nguy?n (nhungnth2004@gmail.com)


lOMoARcPSD|20277325

2. The author lists various animals in line 5 to


(A) contrast the feeding habits of different types of mạmmaỉs
(B) describe the process by which mammals came to be defined
(C) emphasize the point that every type of mammal feeds its own young
(D) explain w y a particular feature of mammals is nonelective
3. The word “tend" in line 7 is closest in meaning to
(A) sit on (B) move (C) notice (D) care for
4. What can be inferred from the passage about the practice of animal parents feeding their young?
(A) It is unknown among fish. (B) It is unrelated to the size of the young.
(C) It is dangerous for the parents. (D) It is most common among mammals.
5. The word "provisioning" in line 13 is closest in meaning to
(A) supplying (B) preparing (C) building (D) expanding
6. According to the passage, how do some insects make sure their young have food?
(A) By storing food near their young.
(B) By locating their nests or cells near spiders and caterpillars.
(C) By searching for food some distance from their nest.
(D) By gathering food from a nearby water source.
7. The word "edge" in line 17 is closest in meaning,to
(A) opportunity (B) advantage (C) purpose (D) rest
8. The word "it" in line 20 refers to
(A) feeding (B) moment (C) young animal (D) size ,
9. According to the passage, animal young are most defenseless when
(A) their parents are away searching for food
(B) their parents have many young to feed
(C) they are only a few days old
(D) they first become independent
10. The word "shielded" in line 22 is closest in meaning to
(A) raised (B) protected (C) hatched (D) valued
Part 2:
A. Read the following passage and choose the correct answer to each question.
You are going to read an extract from a book on networking and public speaking skills. Six paragraphs
have been removed from the extract. Choose from the paragraphs A-G the one which fits each gap (1-6).
There is one extra paragraph which you do not need to use.
A. The one thing I try to avoid is approaching two people who are in discussion. If you see two people
talking together, they may be building a rapport and interruption may break that. Alternatively, they
may be discussing business.
B. The easiest way to approach a group is to catch the eye of one of the participants and smile. Usually
they should invite you to join them at the appropriate juncture.
C. The other advantage of this is that your companion, in introducing you, may well talk about how
you’ve helped them, how great you are at what you do or praise you in another way that you would not
have been able to do. This will awaken a greater interest in you from the new contact than may
otherwise have been possible.
D. You can often find these people around a bar ór buffet table (they’ve probably read the advice above!)
or by the walls; Nervous people on their own seldom stand in the middle of a room unless they are
milling around trying to pluck up the courage to approach someone. Often they will be admiring the
art on the walls or the flora in the room, which gives you a nice topic with which to start a
conversation.
E. When you do approach them, take care not to dive in aggressively but be empathetic to their nervous
state. Ask them if they mind if you join them before introducing yourself, rather than running up
65

Downloaded by Nhung Nguy?n (nhungnth2004@gmail.com)


lOMoARcPSD|20277325

asking “So, what do you do then?”


F. If you see a group of people talking, approach the group, but don’t butt in. Remember, as Susan Roane
says in How to Work a Room, “There is a difference between including yourself in other people’s
conversations and intruding on them.”
G. If their body language is ‘closed’, and they are facing each other, you should avoid interrupting them.
If they are more ‘open’ and they are standing at an angle that leaves room for another party in the
conversation, you are likely to be more welcome.
TALKING TO PLANTS AND APPROACHING GROUPS
In this book extract from “... and death came third!” Andy Lopata and Peter Roper show
nervous business people how to network with panache.
At networking events, I will often look to start a conversation with people who are on their own. It is
much easier than breaking into a group conversation and the chances are they won’t tell you to leave them
alone and go away. Very few people go to networking events for solitude. 1……………..
When approaching these people you are already at an advantage because they will both respect your
courage (which they have probably lacked) and be grateful that you’ve taken the time and effort to relieve
them from their anxiety. They are probably just as nervous as everyone else, and they’ll be delighted to get
into a conversation with you. You’ve rescued them from walking around, avoiding interrupting other people
for fear of rejection. 2……………….
Having spoken to them, try not to leave them on their own again because you’ll just return them to the
same state as you found them. Move on with them and introduce them to someone else. 3........................
If someone is talking and you interrupt, or ask if you can join them, people will stop listening to the
person who’s talking, and invite you into their group. That’s great for I you but not so nice for the person who
is talking. Stand just on the edge of the group and wait for the appropriate time. 4………………
Alternatively, it may be that they’re talking about something in which you have an interest, in which
case, when there’s an appropriate pause, you can just say, “Excuse me, I heard you mention so-and-so. Can I
ask you a question? Are you involved in that?” And you’re in the conversation. Or it may just be that you have
a pause, and you ask “May I join you?” But it’s always best to wait for the, right pause in the conversation.
5………………
While the guidelines above are important, you need to be aware of the body language of people talking
to each other and networking events. Whether in couples or groups, people will always send very clear signals
about approachability by the way they are standing. 6............................
Reading this body language may mean that you are better advised approaching two people rather than
a group.
Part 2:
B. Read the following passage and answer the questions that follow. Write A, B, C or D to indicate your
answers on the answer sheet.
Every drop of water in the ocean, even in the deepest parts, responds to the forces that create the tides.
No other force that affects the sea is so strong. Compared with the tides, the waves created by the wind are
surface movements felt no more than a hundred fathoms below the surface. The currents also seldom involve
more than the upper several hundred fathoms despite their impressive sweep.
The tides are a response of the waters of the ocean to the pull of the Moon and the more distant Sun. In
theory, there is a gravitational attraction between the water and even the outermost star of the universe. In
reality, however, the pull of remote stars is so slight as to be obliterated by the control of the Moon and, to a
lesser extent, the Sun. Just as the Moon rises later each day by fifty minutes, on the average, so, in most
places, the time of high tide is correspondingly later each day. And as the Moon waxes and wanes in its
monthly cycle, so the height of the tide varies. The tidal movements are strongest when the Moon is a sliver in
the sky, and when, it is full. These are the highest flood tides and the lowest ebb tides of the lunar month and
are called the spring tides. At these times the Sun, Moon, and Earth are nearly in line and the pull of the two
heavenly bodies is added together to bring the water high on the bedches, to send its surf upward against the
66

Downloaded by Nhung Nguy?n (nhungnth2004@gmail.com)


lOMoARcPSD|20277325

sea cliffs, and to draw a high tide into the harbors. Twice each month, at the quarters of the Moon, when the
Sun, Moon and Earth lie at the apexes of a triangular configuration and the pull of the Sun and Moon are
opposed, the moderate tidal movements called neap tides occur. Then the difference between high and low
water is less than at any other time during the month.
1. What is the main point of the first paragraph?
A. The waves created by ocean currents are very large.
B. Despite the strength of the wind, it only moves surface water.
C. Deep ocean water is seldom affected by forces that move water.
D. The tides are the most powerful force to affect the movement of ocean water.
2. The words "In reality" in. the passage is closest in meaning to
A. surprisingly B. actually C. characteristically D. similarly
3. It can be inferred from the passage that the most important factor in determining how much gravitational
effect one object in space has on the tides is
A. size B. distance C. temperature D. density
4. The word "configuration" in the passage is closest in meaning to
A. unit B. center C. surface D. arrangement
VI. GUIDED CLOZE TEST
Part 1: Read the extract below and circle the answer A, B, C or D that best fits each space.
HIGH STAKES
Few people in the world of high finance had heard of Marc Colombo. There was no reason why they
should have done. He was a mere foreign-exchange (1) …………… at the Lloyds Bank in Lugano,
Switzerland. But in 1974, Colombo (2) …………… the headlines around the world leaving (3) ……………
money experts open-mouthed in aazement. Lloyds announced that ‘irregularities’ had cost the bank a (4)
…………….£32 million. What had the 28-year-old Colombo been up to? And how had he got (5)
……………with it?
Colombo had been watching the world's leading currencies change their values on the foreign
exchange markets. He decided to buy 34 million US dollars with Swiss francs in three months' time. If, as he
(6) ……………, it turned out that the dollar was (7) ……………less when the time came to settle, he would
make a handsome profit. But the dollar's value did not (8) ……………It went up. And Colombo lost £1
million.
Consequently he increased his stake, and went for (9) …………… or - nothing. Without Lloyds (10)
……………a thing, he set up transactions totaling £4,580 million in just nine months. At first, he was betting
that the dollar would lose value. It did not. So he switched to gambling that it would go on rising. It did not.
1. A. dealer B. salesman C. merchant D. retailer
2. A. knocked B. struck C. hit D. beat
3. A. hard-hearted B. hard-headed C. hard-pressed D. hard-hitting
4. A. swaying B. shaking C. staggering D. wobbling
5. A. away B. on C. through D. by
6. A. expected B. contemplated C. wondered D. considered
7. A. value B. cost C. worth D. charge
8. A. tumble B. trip C. spill D. topple
9. A. twice B. pair C. twofold D. double
10. A. considering B. speculating C. suspecting D. believing
Part 2:
Read the passage and use ONLY ONE suitable word to fill in each gap.
Some time ago, scientists began experiments to find out (1) ……………it would be possible to set up
a ‘‘village” under the sea. A special room was built and lowered (2) …………… the Water of Port Sudan in
the Red Sea. For 29 days’, five men lived (3) ……………a depth of 40 feet: At a (4) ……………lower level,
another two divers stayed for a week in a smaller “house”- On returning to the surface, the men said that they
67

Downloaded by Nhung Nguy?n (nhungnth2004@gmail.com)


lOMoARcPSD|20277325

had experienced no difficulty in breathing and had (5) ……………many interesting scientific observations.
The captain of the party, Commander Cousteau, spoke of the possibility of (6) ……………the seabed. He said
that some permanent stations were to be set up under the sea, and some undersea farms would provide food
for the growing population of the world. The divers in both “houses” spent most of their time (7) ……………
the bottom of the sea. On four occasions, they went down to 360 feet and observed many extraordinary (8)
……………of the marine life, some of which had never been seen before. During their stay, Commander
Cousteau and his divers reached a depth of 1,000 feet and witnessed a gathering of an immense (9) ……… of
crabs which numbered, perhaps, hundreds of millions. They also found out that it was (10) …………… to
move rapidly in the water in a special vessel known as a “diving saucer”.
1. A. how B. which C. what D. whether
2. A underneath B. down C. below D.into
3. A. at B. in C. from D. on
4. A. more B. any C. much D. some
5 A.caught B. done C. made D. exercised
6. A. implanting B. transplanting C. growing D. cultivating
7 A. enquiring B. imploring C. exploring D. inquiring
8. A. breeds B. forms C. systems D. castes
9. A. herd B. flock C. school D. pack
10. A. hardly B. able C. possible D. capable

B. WRITTEN TEST
I. OPEN CLOZE TEST
Part 1: Fill each blank with one suitable word.
For questions 1-10, read the text below and type the word which best fits in each space.
Use only one word in each space. There is an example below:
(0) as
GLOBAL ENGLISH
Global English exists (0) ……………a political and cultural reality. Many misguided theories attempt
to explain why the English language should have succeeded internationally, whilst (1) ……………have not. Is
it because there is (2) ……………inherently logical or beautiful about the structure of English? Does
its simple grammar make it easy to learn? Such ideas are misconceived. Latin was once a major international
language, (3) …………… having a complicated grammatical structure, and English also presents learners
with all manner of real difficulties, (4) ……………least its spelling system. Ease of learning, therefore,
has little to do with it. (5) ……………all, children learn to Speak their mother tongue in approximately the
same period of time, (6) …………… of their language. English has spread not (7) ……………much for
linguistic reasons, but rather because it has often found (8) ……………in the right place, at the right
time. Since the 1960s, two major developments have contributed to strengthening this global status. Firstly, in
a number of countries, English is now used in addition to national or regional languages. As well as this, an
electronic revolution has taken (9) …………… It is estimated that (10) …………… the region of 80% of
worldwide electronic communication is now in English.
Part 2: Fill in each numbered blank with ONE appropriate word.
The computer has brought nothing (1) ……………problems to the world of the twentieth century.
Chief among the problems has (2) ……………the dehumanization of society. People are no (3) ……………
human. Each of us is a series of numbers, numbers to be fed into computers. There are our credit card
numbers, our bank account (4) ……………our social security numbers, our telephone and electricity numbers
- the computer number game is endless.
What happens to these computerized numbers ? They are distributed to a network of government
agencies and business (5) …………… can use them to invade our privacy. The Internal Revenue Service
stores millions of facts about every citizen. Credit agencies exchange (6) ……………on the spending and
68

Downloaded by Nhung Nguy?n (nhungnth2004@gmail.com)


lOMoARcPSD|20277325

saving practices of nearly every American adult. Mailing lists are (7) ……………available by computers to
dozens of organizations, public and private, who bombard us with unwanted mail.
Just let the computer which stores (8)……………concerning our accounts, let us say with a credit card
company, make an error and it is almost impossible to correct it. The result is an avalanche of bills, threads,
and loss of credit standing.
The computer has thrown thousands of people out of (9) ……………. The gamut of computer-
generated unemployed runs from highly skilled technicians to typists.
These are some of the reasons why I feel that the (10) ……………of the computer has been
detrimental to the quality of our life in the twentieth century.
II. WORD FORMS
Part 1: Give the correct form of the words in brackets.
A MUSICAL GENIUS
For many people Ludwig van Beethoven (1770-1827) is the most (0) influential INFLUENCE
figure in the history of western classical music. His (1) ……………talent was ORDINARY
already clearly evident as a young man, (2) ……………surviving a somewhat MERCY
unconventional (3) ……………during which his eccentric father BRING
would often force him to take music lessons in the middle of the night.
The young Beethoven’s ability won him the admiration of the leading contemporary
musical figures. Throughout the 1790s he worked hard to secure the interest of
wealthy patrons.
Such patronage (4) …………… himto concentrate on becoming a successful ABLE
composer.
Whatever his awe-inspiring musical (5) …………… however, ACHIEVE
his personal life was something of a disaster. His day-to-day RELATE
(6) ……………with people invariably turned out to be rather
turbulent. Although he apparently fell in love with a number of society
women, the identity of the girl who lay closest to
his heart remains (7) ……………to this day. ELUDE
However, just at the point when Beethoven was beginning
to reap the rewards of his early endeavours, he had to come
to terms with the (8) ……………realisation that his increasing CRUSH
deafness was (9) ……………From that point on, his music CURE
displayed a (10) ……………change in style, becoming both STRIKE
heavier in tone and larger in scale.
Part 2: Complete each of the following sentences with the correct form of the word in CAPITALS using
hyphens where necessary.
1. After many years as a solo researcher, Marion decided to…………her next article with a colleague in the
same field. (AUTHOR)
2. The results of the medical tests showed that the patient had several ………… of the brain that required
urgent treatment. (NORMAL)
3. The living room looked cold and…………when the fire wasn’t lit. (INVITE)
4. John muttered…………under his breath about the volume of work, he’d been given but he didn’t dare
complain to the boss. (CONTENT)
5. Many charities are working to support the needs of ………… children in poorer countries. (PRIVILEGE)
6. The company installed a water cooler in the main office which would …………the drinking water.(PURE)
7. The old man felt…………by the dreadful living conditions he had experienced when he was growing up.
(BITTER)
8. Our marketing strategy seems to be failing - this demands a …………at our next board meeting. (THINK)
9. The school that Peter goes to has a number of………… activities that take place at the end of the school
69

Downloaded by Nhung Nguy?n (nhungnth2004@gmail.com)


lOMoARcPSD|20277325

day. (CURRICULUM)
10. The romantic novel started with a young woman staring…………into the fire. What a cliche! (DREAM)
III. ERROR IDENTIFICATION
There are 10 errors (grammar or word usage) in the following passage. Identify, then underline and
correct them in the space provided in the column on the right. (0) has been done as an example .
Oxford is a city with (0) so a mind-blowing reputation that many who 0. so → such
come here find them intimidated by the place and can’t wait to leave, 1…………….
while others talking with it like a duck to water, find themselves returning 2…………….
again and again. The college lawns provide a gorgeous backdrop to seriously 3…………….
study, and in the right light, on a sunny winter’s morning saying, one 4…………….
feels as if one is floated on air, such is the sense of unreality. Oxford 5…………….
may like to pretend that it is at the intellectual hub of thing, but in 6…………….
many ways it is no less than a sleepy backwater where, to mix 7…………….
metaphors, transitory students, the cream of their generation, wait for 8…………….
the wings, allowing their talents to flourish before moving off into the 9…………….
industrial or political fast-lane. Much of it is the myth, of course. 10…………….
Hardship and hard work are very much part and parcel of student life. 11…………….
The level-headed get through the three years’ hard grind by simple 12…………….
putting their shoulders to the wheel before going on to fairly average 13…………….
jobs. Only for the tiny minor is Oxford the first step on the ladder to 14…………….
fame and fortune.
IV. WRITING
Part 1. For each of the sentences below, write a new sentence as similar as po in meaning to the original
sentence, but using the word given in block letters word must not be altered in any way.
1. She began to suffer from irrational fears. (PREY)
…………………………………………………………………………………………………………
2. Ours is the only company allowed to import these chemicals. (NIONOPOLY)
…………………………………………………………………………………………………………
3. The final version of the plan was quite different from the initial draft. (RESEMBLANCE)
…………………………………………………………………………………………………………
4. Don’t tell your colleagues anything about this. (BREATHE)
…………………………………………………………………………………………………………
5. There was nothing we could do to stop him. (POWERLESS)
…………………………………………………………………………………………………………
Part 2. Complete each of the following sentences in such a way that it means exactly the same as the
sentence printed before it.
1. Experts say that the fall in the birth rate between 1964 and 1977 caused the drop in the number of school
leavers.
→The drop………………………………………………………………………………………………….
2. The boy became so confused and worried that he left home.
→So…………………………………………………………………………………………………………
3. I’m on the tenth page of the report I am writing.
→So far I…………………………………………………………………………………………………….
4. On receipt of your cheque, we shall send the goods to you.
→As soon ………………………………………………………………………………………………….
5. These new machines have put an end to queuing.
→Before these………………………………………………………………………………………………

70

Downloaded by Nhung Nguy?n (nhungnth2004@gmail.com)


lOMoARcPSD|20277325

TRƯỜNG THPT CHUYÊN QUANG TRUNG - BÌNH PHƯỚC


A. MULTIPLE CHOICE
Câu hỏi 1: PHONOLOGY
Choose the word whose underlined part is pronounced differently from the other three.
1. A. underweight B. understand C. underpass D. undertake
2. A. decent B. argue C. infamous D. terrific
3. A. democracy B. predecessor C. indestructible D. competition
4. A. satisfactory B. alternative C. miscellaneous D. sentimental
5. A. conservative B. contributory C. magnificent D. necessarily
Choose the word whose stress pattern is different from that of the other three.
6. A. leopard B. drunkard C. standard D. bombard
7. A. fever B. gene C. enhance D. pretend
8. A. solitude B. gigantic C. wilderness D. ignorant
9. A. occasion B. supervision C. dimension D. invasion
110. A. anxious B. luxurious C. exemplify D. exhibit
Câu hỏi 2: VOCABULARY
Choose the best answer to complete each of the following sentencese
1. He seemed rather........................ Was he upset about something?
A. unsocial B. apathetic C. passive D. subdued
2. I wouldn’t say he was brilliant at his job but he is quite.......................
A. cautious B. quary C. effective D. competent
3. You should at least show your mother some.......................for all the things.
A. kindness B. consideration C. sympathy D. gratitude
4. Sally has an.......................command of the Chinese language.
A. extreme B. utter C. outstanding D. intensive
5. The consultant called in by the firm brought a.......................of experience to bear on the problem.
A. wealth B. realm C. bank D. hoard
6. He has no friends because he's the new kid on the.......................
A. block B. bus C. house D. corner
7. He knew that the beautiful sports car he was looking at must have cost a.......................penny.
A. pretty B. big C. bent D. shiny
8. When you take ice out of the freezer, it.......................
A. melts B. dissolves C. softens D. disappears
9. The transport strike a real.......................because it will make it very difficult for me to get to work.
A. pain in the back B. nuisance C. last straw D. frustration
10. I know it’s not a big problem but it.......................me and I want to sort it out.
A. strains B. stresses C. presses D. bothers
Câu hỏi 3: STRUCTURES AND GRAMMAR
Choose the best answer for each question.
1. That small car,......................., is advertised in many current magazines.
A. with a small price and comfortable
B. cheap and has a lot of comfort
C. low in price but large in comfort
D. inexpensive but some comfortable
2. In spite of the fact that there is much written about Charlemagne,.......................
A. his character and personality are hard to visualize
B. is difficult to discover his character and personality
C. picturing his personality and character are hard things
71

Downloaded by Nhung Nguy?n (nhungnth2004@gmail.com)


lOMoARcPSD|20277325

D. you can't figure out his personality and character


3. Your ideas,......................., seem unusual to me.
A. as hers B. like hers
C. similar as hers D. different than hers
4. Martha almost never studied,.......................?
A. did she B. would she C. had she D. didn't she
5. .......................turn in his assignment now?
A. Has he to B. Does he needs to C. Does he have to D. Should he have
6. Mrs. Lincon has........................ that she is unable to get a job.
A. such small education B. so little education
C. a such little education D. a so small education
7. By 1988, scientists surely.......................a cure for the common cold.
A. have discovered B. must discover
C. are discovering D. had discovered
8. The lawyer insisted that his client.....................never have been arrested in the first place.
A. should B. must C. ought D. would
9. Do you ever see any canal? - Yes, I’ve been to ...........Canal.
A. the Panama’s B. Panama C. the Panama D. a Panama
10. Within ....................... few years,.......................private space travel has gone from concept to reality.
A. no article / an B. the / the C. the /a D. a / no article
Câu hỏi 4: PREPOSITIONS AND PHRASAL VERBS
Choose the best answer for each question.
1. Tom crept..........on Daisy and put his hands over her eyes.
A. out B. off C. round D. up
2. It is incumbent....................students to study so as to prepare well for the future.
A. upon B. for C. to D. about
3. It’s all the same....................me whether we go to Nha Trang or Vung Tau.
A. for B. on C. to D. as
4. The conman was convicted and sentenced .............2 years in prison.
A. for B. on C. with D. to
5. We should keep our valuables ...........lock and key at home.
A. under B. in C. on D. within
6. They are planning to...........down their operation in China and concentrate on ASEAN countries.
A. wind B. tie C. roll D. stretch
7. I don’t know if it’s right to do so, but I’ll try to..............ahead with it anyway.
A. drive B. bang C. touch D. press
8. After months of testing, the Russian space scientists...............a space suit that works better than any other in
history.
A. came to B. came up with C. came up to D. came out with
9. Mr. Hung is a rich man, inasmuch as he has bought..........all the land in this area.
A. up B. out C. for D. into
10. Something’s ..............up, so I don’t think I will be able to finish it this afternoon.
A. shown B. pulled C. cropped D. cut
Câu hỏi 5: READING COMPREHENSION
Reading passage 1
Coincident with concerns about the accelerating loss of species and habitats has been a growing
appreciation, of the importance of biological diversity, the number of species in a particular ecosystem, to the
health of the Earth and human well-being. Much has been written about the diversity of terrestrial organisms,
particularly the exceptionally rich life associated with tropical rain-forest habitats. Relatively little has
72

Downloaded by Nhung Nguy?n (nhungnth2004@gmail.com)


lOMoARcPSD|20277325

been said, however, about diversity of life in the sea even though coral reef systems are comparable to rain
forests in terms of richness of life.
An alien exploring Earth would probably give priority to the planet’s dominant most-distinctive
feature-the ocean. Humans have a bias toward land that sometimes gets in the way of truly examining global
issues. Seen from far away, it is easy to realize that landmasses occupy only one-third of the Earth’s surface.
Given that two- thirds of the Earth’s surface is water and that marine life lives at all levels of the ocean, the
total three- dimensional living space of the ocean is perhaps 100 times greater than that of land and contains
more than 90 percent of all life on Earth even though the ocean has fewer distinct species.
The fact that half of the known species are thought to inhabit the world’s rain forests does not seem
surprising, considering the huge numbers of insects that comprise the bulk of the species. One scientist found
many different species of ants in just one tree from a rain forest. While every species is different from every
other species, their genetic makeup constrains them to be insects and to share similar characteristics with
750,000 species of insects. If basic, broad categories such as phyla and classes are given more emphasis than
differentiating between species, then the greatest diversity of life is unquestionably the sea. Nearly every
major type of plant and animal has some representation there.
To appreciate fully the diversity and abundance .of life in the sea, it helps to think small. Every
spoopful of ocean water contains life, on the order of 100 to 100,000 bacterial cells plus assorted microscopic
plants and animals, including larvae of organisms ranging from sponges and corals to starfish and clams and
much more.
1. What is the main point of the passage?
A. Humans are destroying thousands of species.
B. There are thousands of insect species.
C. The sea is even richer in life than the rain forests.
D. Coral reefs are.similar to rain forests.
2. The word “appreciation” in line 2 is closest in meaning to
A. ignorance B. recognition C. tolerance D. forgiveness
3. Why does the author compare rain forests and coral reefs (lines 3-6)?
A. They are approximately the same size.
B. They share many similar species.
C. Most of their inhabitants require water.
D. Both have many different forms of life.
4. The word “bias” in line 9 is closest in meaning……………..
A. concern B. disadvantage C. attitude D. prejudice
5. The passage suggests that most rain forest species are
A. insects B. bacteria C. mammals D. birds
6. The word “there” in line 24 refers to……………..
A. the sea B. the rain forests
C. a tree D. the Earth’s surface
7. The author argues that there is more diversity of life in the sea than in the rain forests because…………
A. more phyla and classes of life are represented in the sea
B. there are too many insects to make meaningful distinctions
C. many insect species are too small to divide into categories
D. marine life-forms reproduce at a faster rate
8. Which of the following is NOT mentioned as an example of microscopic sea life?
A. Sponges. B. Coral. C. Starfish. D. Shrimp.
9. The word “comparable” in the passage is closest in meaning to “……………..”
A. similar B. identical C. relevant D. related
10. Which of the following conclusions is supported by the passage?
A. Ocean life is highly adaptive.
73

Downloaded by Nhung Nguy?n (nhungnth2004@gmail.com)


lOMoARcPSD|20277325

B. More attention needs to be paid to preserving ocean species and habitats.


C. Ocean life is primarily composed of plants.
D. The sea is highly resistant to the damage done by pollutants.
Reading 2
Read the following passage and choose the best option to complete the blank or answer the questions
“RISING SEA LEVELS”
Perhaps the most pervasive climatic effect of global wanning is rapid escalation of ice melt. Mount
Kilimanjaro in Africa, portions of the South American Andes, and the Himalayas will very likely lose most of
their glacial ice within the next two decades, effecting local water resources. Glacial ice continues its retreat in
Alaska. NASA scientists determined that Greenland’s ice sheet is thinning by about 1 m per year. The
additional meltwater, especially from continental ice masses and glaciers, is adding to a rise in sea level
worldwide. Satellite remote sensing is monitoring global sea level, sea ice, and continental ice. Worldwide
measurements confirm that sea level rose during the last century.
Surrounding the margins of Antarctica, and constituting about 11% of its surface area, are numerous
ice shelves, especially where sheltering inlets or bays exist. Covering many thousands of square kilometers,
these ice shelves extend over the sea while still attached to continental ice. The loss of these ice shelves does
not significantly raise sea level, for they already displace seawater. The concern is for the possible surge of
grounded continental ice that the ice shelves hold back from the sea.
Although ice shelves constantly break up to produce icebergs, some large sections have recently
broken free. In 1998 an iceberg (150km by 35km) broke off the Ross Ice Shelf, southeast of the Antarctic
Peninsula. In March 2000 an iceberg tagged B-15 broke off the Ross Ice Shelf (some 90 longitude west of the
Antarctic Peninsula), measuring 300km by 40km. Since 1993, six ice shelves have disintegrated in Antarctica.
About 8000km of ice shelf are gone, changing maps, freeing up islands to circumnavigation, and creating
thousands of icebergs. The Larsen Ice Shelf, along the east coast of the Antarctic Peninsula, has been
retreating slowly for years. Larsen-A suddenly disingrated in 1995. In only 35 days in early 2002, Larsen-B
collapsed into icebergs. This ice loss is likely a result of the 2.5°C temperature increase in the region in the
last 50 years. In response to the increasing warmth, the Antarctic Peninsula is sporting new vegetation growth,
previously not seen there.
A loss of polar ice mass, augmented by melting of alpine and mountain glaciers (which experienced
more than a 30% decrease in overall ice mass during the last century) will affect sea-level rise. The IPCC
assessment states that “between one-third to one-half of the existing mountain glacier mass could disappear
over the next hundred years”. Also ‘there is conclusive evidence for a worldwide recession of mountain
glaciers . . . This is among the clearest and best evidence for a change in energy balance at the Earth’s surface
since the end of the 19th century.”
A Sea-level rise must be expressed as a range of values that are under constant reassessment. B The
2001 IPCC forecast for global mean sea-level rise this century, given regional variations, is from 0.11 - 0.88m.
C The median value of 0.48m is two to four times the rate of previous increase. These increases would
continue beyond 2100 even if greenhouse gas concentrations are stabilized. D
The Scripps Institute of Oceanography in La Jolla, California, has kept ocean temperature records
since 1916. Significant temperature increases are being recorded to depths of more than 300 m as ocean
temperature records are set. Even the warming of the ocean itself will contribute about 25% of sea-level rise,
simply" because of thermal expansion of the water. In addition, any change in ocean temperature has a
profound effect on weather and, indirectly, on agriculture and soil moisture. In fact the ocean system
appears to have delayed some surface global warming during the past century through absorption of
excess atmospheric heat.
A quick survey of world coastlines shows that even a moderate rise could bring changes of
unparalleled proportions. At stake are the river deltas, lowland coastal farming valleys, and low-lying
mainland areas, ail contending with high water, high tides, and higher storm surges. Particularly tragic social
and economic consequences will affect small island states-being able to adjust within their .present country
74

Downloaded by Nhung Nguy?n (nhungnth2004@gmail.com)


lOMoARcPSD|20277325

boundaries, disruption of biological systems, loss of biodiversity, reduction in water resources, among the
impacts. There could be both internal and international migration of affected human populations, spread over
decades, as people move away from coastal flooding from the sea-level rise.
1. There is more new plant life in Antarctica recently because
A. the mountain glaciers have melted
B. the land masses have split into islands
C. the icebergs have broken into smaller pieces
D. the temperature has risen by a few degrees
2. It may be inferred from this passage that icebergs are formed
A. by a drop in ocean temperatures B. when an ice shelf breaks free
C. from intensely cold islands D. if mountain glaciers melt
3. The word there in the passage refers to
A. polar ice mass in the last 50 years B. the temperature increase
C. new vegetation growth D. in the Antarctic Peninsula
4. In paragraph 4, the author explains the loss of polar and glacial ice by
A. stating an educated opinion B. referring to data in a study
C. comparing sea levels worldwide D. presenting his research
5. The word conclusive in the passage is closest in meaning to
A. definite B. independent C. unique D. valuable
6. Why does the author mention the Scripps Institute of Oceanography in paragraph 6?
A. The location near the coast endangers the Scripps facility.
B. Research at Scripps indicates that the ocean is getting warmer.
C. One quarter of the rising sea levels has been recorded at Scripps.
D Records at Scripps have been kept for nearly one hundred years.
7. Which of the sentences below best expresses the information in the highlighted statement in the passage?
The other choices change the meaning or . leave out important information.
A. Global warming on the surface of the planet may have been retarded during the last hundred years because
heat in the atmosphere was absorbed by the oceans.
B. Global warming on the surface of the ocean was greater than it was on the rest of the planet during the past
century because of heat in the atmosphere.
C. Too much heat in the atmosphere has caused global warming on the surface of the planet for the past
hundred years in spite of the moderation caused by the oceans.
D. There is less heat being absorbed by the oceans now than there was a hundred years ago before the
atmosphere began to experience global warming.
8. According to paragraph 7, why will people move away from the coastlines in the future?
A. It will be too warm for them to live there.
B. The coastlines will have too much vegetation.
C. Flooding will destroy the coastal areas.
D. No agricultural crops will be grown on the coasts.
9. Which of the following statements most accurately reflects the author’s opinion about rising sea levels?
A. Sea levels would rise without global warming.
B. Rising sea levels can be reversed.
C. The results of rising sea levels will be serious.
D. Sea levels are rising because of new glaciers
10. Look at the four squares [■] that show where the following sentence could be ’inserted in the passage.
During the last century, sea level rose 10 - 20cm, a rate 10 times higher than the average rate during the
last 3000 years.
Where, could the sentence best be added?
Câu hỏi 6: GUIDED CLOZE TEST
75

Downloaded by Nhung Nguy?n (nhungnth2004@gmail.com)


lOMoARcPSD|20277325

Passage 1
Choose the answer (A, B, C or D) that best fits each space.
Last year, there were millions of international tourist arrivals across the globe. Whether they were
students on their (0) C years looking for overseas adventure, stressed-out workers hoping to (1) ..................
away for a weekend, or retirees wishing to relax in an exotic location - clearly none of them thought they
could find the same experience at home. The question is (2) .................. foreign travel brings more advantages
or disadvantages.
An obvious positive point is that going abroad sometimes helps people to expand their knowledge of
the world. If they are (3) ..................minded, they can learn about new cultures and hopefully gain a more
accurate understanding about their (4) ..................of life. In addition, there is the economic benefit of tourism
to countries which have few other resources. It can provide an income to many people within the industiy.
Having said this, some people simply go to a foreign (5) ..................surrounded by high walls and
therefore learn little about their holiday (6) ..................Another issue is that (7) ..................of tourists often
spoil the ‘unspoilt’ places they visit. The most recent example of this is Antarctica, where last year cruise ships
took thousands of visitors to view the disappearing icebergs and wildlife. Vegetation, nesting penguins and
resting walrus are vulnerable when humans (8) ..................
Certainly a trip to a foreign country is attractive, but it may be worth (9) ..................one’s own country
first. By doing so, (10) ..................travelers can support their own economy, cut down on fuel use and find
out about their own national heritage.
0. A. break B.space C.gap D. pause
1. A. get B. deal C. organize D. esqape
2. A. which B. how C. although D. whether
3. A. open B. wide C. large D. free
4. A. form B. means C. way D. belief
5. A. reserve B. sanctuary C. resort D. shelter
6. A. departure B. direction C. destination D. situation
7. A. hordes B. sets C. series . D. crews
8. A. influence B. impact C. enter D. intrude
9. A. trekking B. tracking C. exploring D. locating
10. A. domestic B. internal C. border D. insular
Passage 2
The wildlife of the Juan Fernandez Archipelago remained undisturbed until the arrival of European
settlers, who (0) B their animals to the islands. Amongst these were goats and cows to be (1)……….for meat
and milk, but also rats and mice which had jumped ship. Cats later brought over io kill these pests also quickly
escaped into the wild. Little did the settlers understand the impact this would have on local species. The
rabbits they’d brdught over too, could not be held in (2)……….for long, and their numbers rapidly expanded.
But it was not just fauna that arrived but also flora. Plants such as the blackberry bramble flourished in the
tropical climate and spread throughout the (3)……….forest.
In the past, the islands’ birds had no natural (4)………. Many therefore evolved to lay their eggs in
ground nests. This habit now makes them incredibly (5)……….as rats and. mice destroy their eggs and cats
devour the newly hatched chicks. Meanwhile grazing goats, rabbits and and cows have turned once fertile
valleys into wasteland. One now rare tree species is known as the Luma, in which firecrowp hummingbirds
(6).......... Today the Luma is being covered over by spiky blackberry brambles, the hummingbird’s natural
habitat is disappearing and the fear is that they will soon (7)……….Conservationists from Chile and around
the world are uniting to show the urgent need fof (8)……….of these islands’ original and unique species.
They know the best way to do this is to (9)……….the plants and animals that are (10)……….species.
0. A entered B. introduced C. accessed D. provided
1. A. bred B. populated C. multiplied D. cultivated
2. A. custody B. captivity C. territory D. vicinity
76

Downloaded by Nhung Nguy?n (nhungnth2004@gmail.com)


lOMoARcPSD|20277325

3. A. native B. domestic C. endemic D. authentic


4. A. opponents B. predators C. prey D. challengers
5. A. susceptible B. risky C. incapable D. vulnerable
6. A. nest B. construct C. collect D. house
7. A. wipe out B. extinguish C. expire D. die out
8. A. continuation B. maintenance C. duration D. preservation
9. A. eradicate B. abolish C. ban D. demolish
10. A. strange B. abnormal C. alien D. extra
B. WRITTEN TEST
Câu hỏi 1: Open cloze test
OPEN CLOZE TEST 1
Fill in each gap in the following passages with an appropriate word.
THE BLOOD MOON
A lunar eclipse occurs (1) ..................the Earth’s shadow falls directly onto the moon. (2) ..................
happens as a result is that the moon stops (3) ..................a silvery white colour and turns coppery red instead;
the sort of colour usually only seen in the sky (4) ..................dawn or sunset. An astronaut on the moon,
looking towards Earth (5) ..................a lunar eclipse, would see a black disc, surrounded (6) ..................
a bright red ring. It’s the light from this red ring (7) ..................is reflected back to the dark Earth from the
moon’s surface. In ancient times, long (8)..................any of this was understood, the lunar eclipse was known
(9) ..................a blood moon and was thought to be an omen of (10) ..................
Open cloze test 2
A substance produced by disease-transmitting insects may provide the key to fighting these same
diseases. Mosquitoes and blackflies transmit malaria and river blindness respectively, both of (1) ..................
cause a large number of deaths every year. However, scientists (2) .................. now discovered, in some of
these insects, a (3) ..................called a peptide which kills these viruses and parasites (4) ..................carry.
(5) .................. infroduced techniques have allowed researchers to study these minute insects in
(6) ..................to find out more about the peptides.
Professor John Wells said yesterday that if his team could identify the genes responsible for (7)............
the peptides, they could introduce into the world genetically altered mosquitoes which were incapable of
(8) ..................the disease. In the (9) ..................of these exciting developments, it is hope that science will
be
able to eradicate some major tropical diseases (10) ..................rather than later.
Câu hỏi 2: WORD FORMS
1. Complete each sentence with the correct form of the word given in parentheses.
1. With the help of the computer, checking information has become less…………. (labour)
2 . A few jokes can………….up a lecture. (life)
3. Ancient………….techniques for the changing metal or physiological state prescribed alternating one’s
breathing between the two nostrils. (yoga)
4. A renewable resource is one that may be replayed overtime by natural process or is…………. (exhaust)
5. King Henry VIII had his wife………….so he could marry again. (head)
6. The world champion was………….by a younger Russian challenger. (throne)
7. The family managed to get out of the burning house, but they lost nearly all their ………….(possess)
8. He was a………….adventurous person, there is no question about it. (decide)
9. ………….is desfroying large areas of tropical rain forest. (forest)
10. It is………….of you to arrive late when all people are in the party. (courtesy)
2. Read the text below. Use the word given in capitals at the end of some of the lines to form a word that
fits in the space in the same line.
A WORRYING DISEASE
Rubella, also called German measles, is an epidemic (0) viral (VIRUS) disease of mild course (1.
77

Downloaded by Nhung Nguy?n (nhungnth2004@gmail.com)


lOMoARcPSD|20277325

INTENSE) ..................study of epidemics in Germany in the 19th century gave rise to the popular name of the
disease. Although rubella may occur in young children,(2. SUSCEPTIBLE) ..................to the disease is more
commonly seen in older children and young adults.
Usually the (3. COMFORT) ..................rash is the first sign noted (4 LARGE) ..................of the lymph
glands in the neck, behind the ears, and perhaps elsewhere in the body is (5. CHARACTER) ..................
Although it is certainly not pleasant to suffer from rubella,(6. COMPLICATE) ..................are rare. A day or so
of bed rest and a light diet with plenty of fluids is the only (7. TREAT) ..................required in most cases. In
1941 it was discovered that rubella early in pregnancy maybe (8.THREAT) ..................to the health of the
foetus, especially the eyes and heart. Years later it was demonstrated that infants may be bom with active
rubella and may manifest many additional (9 NORMAL) ..................In fact, it has been found capable of
causing extensive damage to almost any organ of the infant's body. Methods of (10. IMMUNE) ..................
have been recommended in the hope of stamping out the virus from the environment.
Câu hỏi 3. ERROR IDENTIFICATION
The following paragraph contains ten vocabulary errors. Underline the errors and write the correct
words in the space provided.
The big majority of students who make well in the Cambridge Proficiency 1....................
Examination have learnt to use a good monolingual dictionary effectively. Such 2....................
dictionaries provide informations, not just about the meanings for words but about 3…................
their pronunciation and grammar as well. A student who studies how to use a 4. ..................
dictionary effectively will be able to work independently for much of the time, and 5....................
will gain considerable insight to the workings of the English language. He or she 6....................
will be able to confirm to the meanings of words in a text where contextual clues 7…................
are insufficient, pronounce words accurately by studying the phonological 8....................
transcriptions, and use words accurately both when speaking and writing. Make 9....................
sure that you make the room for at least one good monolingual dictionary on your 10…...............
bookshelf, and then make sure that you use it at a regular basis.
Câu hỏi 4. SENTENCE TRANSFORMATION
1. Complete the second sentences so that it has a similar meaning to the first sentence, using the word
given. Do not change the word given. You must use between three and eight words, including the word
given.
1. I had to clear the spare room before I could start decorating. CLEARED
Only when.................................................................start decorating.
2. Buying a plane ticket at the last minute isn’t often possible. TO
Rarely......................................................a plane ticket at the last minute.
3. He didn’t ask her anything about her study. QUESTION
Not...................................................................................about her study.
4. Einstein proved that energy and mass are basically the same thing. .
What......................................................energy and mass are basically the same thing.
5. You can do jogging any time whatever the weather is like.
You ...........................................................................................or shine.
2. Complete the second sentence so that it has a similar meaning to the first sentence, using the word
given. Do not change the word given.
6. Mary was born in a small village very far away from the city. NOWHERE
....................................................................................................................................................
7. Jimmy played truant this morning, so his father has punished him severely. BOOK
....................................................................................................................................................
8. The construction workers recklessly disregarded the safety at work. SHOWED
....................................................................................................................................................
9. Coming to class late is inexcusable. EXCUSE
78

Downloaded by Nhung Nguy?n (nhungnth2004@gmail.com)


lOMoARcPSD|20277325

....................................................................................................................................................
10. They have known each other since they were at infant school. INFANTS
....................................................................................................................................................

TRƯỜNG THPT HUỲNH THÚC KHÁNG - QUẢNG NAM


PART ONE: PHONOLOGY
A. Find the word that has its underlined part pronounced differently from the other three in each
question.
1. A. publish B. reputation C. until D. fun
2. A. psychology B. happen C. protect D. stopped
3. A. foul B. enough C. come D. touch
4. A. industrial B. under C. sunlight D. influence
5. A. strike B. conflict C. chill D. single
B. Find the word with the stress pattern different from that of the other three words in each question.
1. A. champagne B. linguistics C. experience D. Intermediate
2. A. Hamburger B. Exchange C. Institude D. Dormitory
3. A. Aviation B. Average C. alcohol D. ancient
4. A. effective B. expensive C. malaria D. engineer
5. A. discussion B. company C. division D. imagine
PART TWO: VOCABULARY AND GRAMMAR
A. Choose the best answer.
1. In the hotel lobby the detective caught…………of the man he had been hired to follow.
A. glance B.view C. vision D. sight
2. ‘Go on, finish the food. It needs…………up because it wontt stay fresh until tomorrow’.
A. eat B. eating C. to eat D. eaten
3. Before he left on his trip to America, the young man promised his parents he…………them regularly.
A. is writing B. will write C. should write D. would write
4. Of course I didn’t break it…………purpose-it was an accident!
A. by B. with C. from D. on
5. Andrew couldn’t…………himself laughing at the expression on Maggie’s face.
A. help B. stop C. escape D. avoid
6.…………young, chimpanzees are easily trained.
A. When are B. When C. They are D. When they
7. “What’s your proposal?” “I propose that the meeting…………
A. is postponing B. be postponed C. to be postponed D. postpones
8. Most of the courses at the banquet were completely consumed, but there…………food still remaining.
A. were few B. was little C. were a few D. was a little
9. The four-storey house…………on that hill is still new.
A. be built B. building C. built D. being building
10. Nam Cao devoted most of his time…………
A. to having written B. to write C. to writing D. to have written
11. I don’t think he will join us, …………?
A. doesn’t he B. won’t he C. will he D. don’t I
12. The young generation must contribute to …………the country.
A. building of B. the building C. the building of D. build
13. I cannot help feeling anxious ………… the exam results.
A. of B. with C. about D. for
14. In some vocational schools, the training quality is worse than…………used to be.
79

Downloaded by Nhung Nguy?n (nhungnth2004@gmail.com)


lOMoARcPSD|20277325

A. they B. it C. them D. that


15. When Elvis Presley died, her daughter came ………… a fortune.
A. into B. up with C. by D. across
16. Not until he came into light, …………him.
A. I only recognized B. did I recognized
C. I didn’t recognize D. did I recognize
17. I was very tired;…………, I determined to walk on to the next village
A. therefore B. however C. and D. for
18. I remember…………you. I distinctively gave you $50 last week.
A. pay B. to pay C. paid D. paying
19…………more carefully, he wouldn’t have been in the hospital now.
A. Had he driven B. If he drove
C. If he drives D. If he hadn’t driven
20. More than 50 films…………in Hanoi since June.
A. show B. were shown C. have been shown D. has been shown
21. You must lend me the money for the trip…………, I won’t be able to go.
A. Consequently B. Nevertheless C. Otherwise D. Although
22. The youths nowadays have many things to do in their ………… time.
A. leisure B. entertainment C. fun D. amusement
23. He always …………the crossword in the newspaper before breakfast.
A. writes B. makes C. works D. does
24. Although the exam was difficult, the students passed it.
A. most of B. none of C. a few D. a lot
25. She has a part-time job; she works…………day.
A. each other B. every other C. one the other D. the other every
26. She …………the money so that no one would be able to find it.
A. disclosed B. conveyed C. let on D. hid
27. John was made ………… outside because he was making a noise.
A. go B. going C. to go D. gone
28. My watch had stopped so I had no way of knowing the right
A. hour B. time C. moment D. o’clock
29. How often do you go swimming, Linda? - …………
A. I guess I’m OK B. Once a week C. About one hour D. Last night
30. “ ………… eating out tonight? ” - “ That’s a good idea.”
A. Would you B. How are you C. Are they D. How about
B. Supply the correct form of the word provided in brackets in each sentence.
1. We were told to stay seated and………….. our seat belts. (fast)
2. At first, there was a lot of opposition from the parents of the…………..children. (able)
3. Computer is a personal…………..which helps you to interact with other computers. (communicate)
4. After the earthquake, the city was badly destroyed and many people became …………... (home)
5. Books in the home are a wonderful source of…………..and pleasure. (know)
6. He is much loved and highly…………..teacher. (respect)
7. A lot of plants and animals could be used as medicines against cancer, AIDS, heart disease and many other
…………..(sick).
8. You may be surprised at the large ………….. of animals that live in Nairobi National Park. (various)
9. There are two things that make humans…………..from all other animals. (differ)
10. Our life is…………..improved. (gradual)
C. Choose the right verbs provided in the box, then use the most suitable forms of the verbs to fill in the
numbered blanks.
80

Downloaded by Nhung Nguy?n (nhungnth2004@gmail.com)


lOMoARcPSD|20277325

take read recognize do say finish


make come rise sell
1. Alice…………..to the dentist many times.
2. He…………..nothing before he saw me.
3. Tom is reading a book. He started two hours ago and he is on page 44. He………….. for two hours.
4. Their old house…………..soon.
5. …………..reading the book, I went to bed.
6. The lamps…………..in China are very cheap.
7. He says as if he …………..back from London.
8. My lawyer advised me …………..anything further about the accident.
9. He wore dark glasses to avoid …………..
10. Over the past few years the cost of living…………..considerably
D. The following passage contains ten errors. Find and correct them.
Things started to go wrongly as soon as we got to the hotel. We were all completely exhausting after
our long journey and looking forward to a shower and a rest. However, we found that our room was not ready,
which was very annoyed, although the manager was extremely apologising. While we were waiting, we asked
about the excursions to places of an interest which we have read,about in the brochure. Imagine how we felt
when we were told they had all been cancelled! Apparently, the person responsible of organizing them had left
suddenly and had not been replaced. Then Sally saw a notice pinning to the door of the restaurant, said it was
closed for redecoration, and Peter discovered that the swimming pool was empty. When we eventually got to
our room we were horrified to find that it was at the back of the hotel, and we had a view of a car park, that
seemed to be used as a rubbish dump. We seriously began to wonder whether or not to stay.
1…………………………………………………………………………………………………….
2…………………………………………………………………………………………………….
3…………………………………………………………………………………………………….
4…………………………………………………………………………………………………….
5…………………………………………………………………………………………………….
6…………………………………………………………………………………………………….
7…………………………………………………………………………………………………….
8…………………………………………………………………………………………………….
9…………………………………………………………………………………………………….
10…………………………………………………………………………………………………….
E. Finish the second sentence in such a way that it means the same meaning as the sentence printed
before it
1. They regret ever doing business with him.
→ They wish ……………………………………………………………………………………………
2. We didn’t finish the work because of his carelessness.
→ If………………………………………………………………………………………………………
3. if you followed my advice, you’d be successful.
→ Were…………………………………………………………………………………………………..
4. “Why didn’t you go to the club last night?”, the girl said to Jim.
→ The girl asked Jim, ……………………………………………………………………………………
5. The film was so interesting that I couldn’t go to bed.
→ It was………………………………………………………………………………………………….
6. Mary is so young that she can’t join our club. (TOO)
→ ..………………………………………………………………………………………………………
7. Jim does not intend to give up his hobby.(INTENTION)
→ ..………………………………………………………………………………………………………
8. The owners of the restaurants are usually very rich. (WHO)
81

Downloaded by Nhung Nguy?n (nhungnth2004@gmail.com)


lOMoARcPSD|20277325

→…………………………………………………………………………………………………………
9. It was easy for us to get tickets for the concert. (NO)
→ ..………………………………………………………………………………………………………
10.They think the burglar got in through the bathroom. (THOUGHT)
→ ..………………………………………………………………………………………………………
PART THREE: READING
A. Read the passages and then decide which word (A, B, C, or D) is the best answer.
Reading 1:
Bill Jarvis took oyer our village news agency at a time of life when most of us only want to relax. He
just thought he would like something but not too much to do, and the news agency was ready-made. The
business produced little enough for him, but then Bill was a chap who only wanted the simplicity and order
and regularity of the job. He had been a long-serving sailor, and all his life had done everything by the clock.
Every day he opened his shop at six a.m to catch the early trade; the papers arrived on his door-step
before that. Many of Bill's customers were city workers, and the shop was convenient for the station. Business
was tailing off by ten o'clock, so at eleven sharp Bill closed for lunch. It was hard luck on anybody who
wanted a paper or magazine in the afternoon, for most likely Bill would be down on the river bank, fishing,
and his nearest competitor was five kilometres away. Some time in the afternoon, the evening papers landed
on the door-mat, and at four o'clock Bill reopened. The evening rush lasted till seven, and it was worthwhile.
He lived in a flat above the shop, alone. Except in very bad weather, you always knew where to find
him in the afternoons, as I have said. Once, on a sunny afternoon, I walked home along the river bank from a
shopping trip to the village. By my watch it was three minutes past four, so I was astonished to see Bill sitting
there on his little chair with a line in the water. He had no luck, I could see, but he was making no effort to
move.
"What's wrong, Bill?" I called out-from the path. For answer, he put a hand inside his jacket and took
out a big, golden object. For a moment I had no idea what it could be, and then it suddenly went off with a
noise like a fire engine. Stopping the bell, Bill held the thing up and called back: "Ten to four, you see, and
this is dead right.”. He stood up then and began to wind in his line. I had never known anyone carrying a brass
alarm clock round with him or her before.
1. Bill Jarvis became a newsagent when……………
A. He needed the money. B. He was quite an old man.
C. He decided to take things easy. D. He gave up clock repairing.
2. What does the passage tell us about the nevys agency?
A. It was an easy job with fixed hours.
B. It was a very profitable business
C. It was opened specially for Bill Jarvis
D. It belonged to the railway and was part of the station.
3. Why did Bill open the shop so early in the day?
A. He liked to do as much as possible before he went to work.
B. Bill was never sure of the time.
C. The shop had to be open when the morning papers came
D. It was then that he did a lot of business.
4. We understand from the passage that the shop closed for lunch…………..
A. At eleven o'clock more or less. B. At exactly eleven o'clock.
C. Before eleven o'clock. D. Always after eleven o'clock.
5. You might say "hard luck" to someone who …………..
A. has just heard'some very good news.
B. puts great effort into whatever he or she tries.
C. is less fortunate than he or she ought to be.
D. fails through his or h’er own fault entirely.
82

Downloaded by Nhung Nguy?n (nhungnth2004@gmail.com)


lOMoARcPSD|20277325

6. Why was the writer on the riverbank that afternoon?


A. He was to d° some shopping in the village.
B. He was fishing.
C. He was going to get the evening paper.
D. He was on his way home from the village
7. Why was the writer surprised when he saw Bill Jarvis?
A. He thought it was late for Bill to be still fishing.
B. Bill had not caught anything, and that seemed strange.
C. He thought Bill was ill, because he was not moving at all
D. He was surprised because Bill stayed in his flat in the afternoons.
8. From the information given in the passage, who - or what - do you think was wrong?
A. The bell was; it must have gone off at the wrong time.
B. The writer's watch was fast.
C. Bill was; he had dropped off to sleep
D. Bill's clock was wrong; it was very old.
9. All of the following are true about Bill Jarvis EXCEPT…………..
A. He lived alone B. He had ever worked as a sailor
C. He was a newspaperman D. Fishing was his past time
10. What did Bill Jarvis often bring with him when he went fishing?
A. a clock B. a gold C. a newspaper D. a fire engine
Reading 2:
While most desert animals will drink water if confronted with it, for many of them the opportunity
never comes. Yet all living things must have water, or they will expire. The herbivores find it in desert plants.
The carnivores slake their thirst with the flesh and blood of living prey. One of the most remarkable
adjustments, however, has been made by the tiny kangaroo rat, who not only lives without drinking but
subsists on a diet of dry seeds containing about 5% free water. Like other animals, he has the ability to
manufacture water in his body by a metabolic conversion of carbohydrates. But he is notable for the
parsimony with which he conserves his small supply by every possible means, expending only minuscule
amounts in his excreta and through evaporation from his respiratory tract.
Investigation into how the kangaroo rat can live without drinking water has involved various
experiments with these small animals. Could kangaroo rats somehow store water in their bodies and slowly
utilize these resources in the long periods when no free water is available from dew or rain? The simplest way
to settle this question was to determine the total water content in the animals to see if it decreases as they are
kept for long periods on a dry diet. If they slowly use up their water, the body should become increasingly
dehydrated, and if they begin with a store of water, this should be evident from an initial high water content.
Results of such experiments with kangaroo rats on dry diets for more than 7 weeks showed that the rats
maintained their body weight. There was no trend toward a decrease in water content during the long period of
water deprivation. When the kangaroo rats were given free access to water, they did not drink water. They did
nibble on small pieces of watermelon, but this did not change appreciably the water content in their bodies,
which remained at 66.3 to 67.2 during this period.
This is very close to the water content of dry-fed animals (66.5), and the availability of free water,
therefore, did not lead to any "storage" that could be meaningful as a water reserve. This makes it reasonable
to conclude that physiological storage of water is not a factor in the kangaroo rat’s ability to live on dry food.
1. What is the topic of this passage?
A. Kangaroo rats B. Water in the desert
C. Desert life D. Physiological experiments
2. The word "expire" in line 2 is closest in meaning to
A. become ill B. die C. shrink D. dehydrate
3. Which of the following is NOT a source of water for the desert animals?
83

Downloaded by Nhung Nguy?n (nhungnth2004@gmail.com)


lOMoARcPSD|20277325

A. Desert plants
B. Metabolic conversion of carbohydrates in the body
C. The blood of other animals
D. Streams
4. The word "it" in line 2 refers to
A. a living thing B. the desert C. the opportunity D. water
5. The author states that the kangaroo rat is known for all of the following EXCEPT
A. the economy with which it uses available water.
B. living without drinking water.,
C. breathing slowly and infrequently.
D. manufacturing water internally.
6. The word "parsimony" in line 7 is closest in meaning to
A. intelligence. B. desire. C. frugality. D. skill.
7. It is implied by the author that desert animals can exist with little or no water because of
A. less need for water than other animals.,
B. many opportunities for them to. find water.
C. their ability to eat plants.
D. their ability to adjust to the desert environment.
8. The word "deprivation" inline 20 is closest in meaning to
A. preservation. B. renewal'. C. examination. D. lossi
9. According to the passage, the results of the experiments with kangaroo rats showed that
A. kangaroo rats store water for use during dry periods
B. kangaroo rats took advantage of free access to water.
C. there was no significant change in body weight due to lack of water or accessibility to water.
D. a dry diet seems detrimental to the kangaroo rat’s health.
10. The tone of the passage is
A. critical B. scientific C. humorous D. negative
B. Choose the most suitable word or phrase for each blank to complete the following passage by circling
A, B, C or D.
Cloze test 1:
(1)……………pollution is one of the most serious problem (2) ……………mankind today. Air, water
and soil are necessary to the (3) …………… of all living things. Man operates machines and motor vehicles
that fill the air (4) ……………disturbing noise. They also dirty the air with gases and smoke. (5) ……………
can cause illness and even death. Man ruins natural (6) …………… by scattering junk and litter on the land
and in the water. They (7)……………the water with chemicals and other substances. Polluted water kills fish
and other marine life. They damage the soil with too many fertilizers and pesticides. Pollution of soil reduces
the amount of land that is available for growing food. Environmental pollution also brings ugliness to man’s
naturally beautiful world.
It’s time people and governments all over the world (8) …………… something about the pollution problem.
People should stop using many things that are harmful (9) ……………the environment. Governments would
have to pass and enforce laws that require individuals and businesses to stop or cut down on certain (10) ……
activities.
1. A. Environmentally B. Environment C. Environmentalist D. Environmental
2. A. face B. faces C. facing D. faced
3. A. survival B. survive C. survivor D. survivally
4. A. with B. in C. out D. on
5. A. Bad polluted air B. Badly polluting air
C. Bad polluting air D. Badly polluted air
6. A. beautiful B. beauty C. beautifully D. beautify
84

Downloaded by Nhung Nguy?n (nhungnth2004@gmail.com)


lOMoARcPSD|20277325

7. A. poisoned B. poisons C. poison D. will poison


8. A. do B. does C. have done D. did
9. A. to B. of C. out D. in
10. A. polluted B. polluting C. pollute D. unpolluted
Cloze test 2:
Everyone wants to reduce pollution. But the pollution problem is (1)……………complicated as it is
serious. It is complicated (2) ……………much pollution is caused by things that benefit people. (3) ………,
exhaust from automobiles causes a large percentage of air pollution. But the automobile (4) ……………
transportation for millions of people. Factories discharge much (5) ……………the material that pollutes the
air and water, but factories give employment to a large number of people.
Thus, to end (6) ……………greatly reduce pollution immediately, people would have to (7) ……….
using many things that benefit them. Most of the people do not want to do that, of course. But pollution can be
(8) ……………reduced in several ways. Scientists and engineers can work to find ways to lessen the (9) ……
of pollution that such things as automobiles and factories cause. Governments can pass and enforce laws that
(10) ……………businesses and traffic to stop, or to cut down on certain polluting activities.
1. A. as B. more C. less D. like
2. A. so B. while C. though D. because
3. A. Specific B. For example C. Such as D. Like
4. A. takes B. affords C. carries D. provides
5. A. about B. for C. of D. with
6. A. or B. and C. as well D. then
7. A. start B. continue C. stop D. go on
8. A. carefully B. unexpectedly C. gradually D. little
9. A. way B. figure C. number D. amount
10. A. forbid B. prevent C. request D. require
C. Fill in each blank with a suitable word.
Open cloze test 1:
Dolphins are not fish. They are mammals (1) …………… live in water. Dolphins are (2) …………
the most intelligent animals on earth. Although they can be found in (3) ……………oceans in the world,
dolphins prefer coastal waters and bays. The size of dolphins vary greatly. The smallest dolphin is just about
50 kg in (4) ……………and 1.2 metres in length while (5) ……………largest one can weigh up to 8,200kg
and is 100 metres (6) ……………. Dolphins eat (7) ……………fish. A female dolphin gives birth (8) ………
one calf every two years after a (9) ……………period of eleven or twelve months. A dolphin can normally
live from twenty five to sixty five years and some species of dolphins can even live longer. Dolphin
populations are at risk (10) ……………to the pollution of their habitat and accidental entrapment in fishing
nets.
Open cloze test 2:
TOO GOOD TO BE TRUE
Some say you can do it in seven days, others promise success in 24 hours, or you may prefer to take
your time and do it in 5 weeks. What are we talking about? Believe it (1) ……………not, these periods (2)
……………time refer to language learning courses that promise excellent results in less time (3) ……………
it takes to say 'Bonjour'! However, the advertisements of these companies are not necessarily (4) ……………
best guide and if you don't know what to look for in a good course you could be left with little (5) ………….
than a large bill. A complaint was recently made to a consumer rights group (6) ……………WhizzLearn
Systems, a language school chain, and the company has been forced to remove its claim that its technique is
ten times better than any (7) ……………method. The Managing Director of WhizLearn Systems said “(8)
………..we still believe our claim is true, we are willing to change our advert. But the fact remains, if you
spend 3, hours (9) ……………day for 5 weeks on our language course you will soon be speaking the
language. What we (10) ……………have done was to make it clearer in the advert. Then we wouldn't have
85

Downloaded by Nhung Nguy?n (nhungnth2004@gmail.com)


lOMoARcPSD|20277325

had these annoying complaints".

TRƯỜNG THPT CHUYÊN NGUYỄN THỊ MINH KHAI - SÓC TRĂNG


PART A: MULTIPLE CHOICE
I. PHONOLOGY
A. Choose the word whose underlined part is pronounced differently from that of the others in each
group
1. A. chaos B. character C. charity D. chord
2. A. sergeant B. servant C. service . D. servile
3. A. zebra B. zipper C. seizure D. hazy
4. A. marriage B. courage C. massage D. hostage
5. A. cyanide B. cycle C. cyclone D. cylinder
B. Choose the word whose main stress syllable is placed differently from that of the others in the list
6. A. turbulent B. whimsical C. genealogy D. harmony
7. A. propaganda B. influential C. estimation D. euphemism
8. A. oblivious B. ferocious C. scandalous D. victorious
9. A. consent B. obstinacy C. condolence D. equality
10. A. presidency B. popularity C. indispensable D. geographical
II. VOCABULARY
Choose the best answer to complete each of the following sentences
1. Is it far to Stambord? - Not at all. It’s only a………….from here.
A. stone’s throw B. bird’s eye view
C. short cut D. step in the right direction
2. Charles says the young artist’s paintings are …………..
A. out of this world B. on top of the world C. worlds apart D. for the world
3. Although the patient received intensive treatment, there was no………….improvement in her condition.
A. decipherable B. legible C. discernible D. intelligible
4. The police have been ordered not to………….if the students attack them.
A. combat B. rebuff C. retaliate D. challenge
5. The unscrupulous salesman………….the old couple out of their life savings.
A. deprived B. swindled C. robbed D. extracted
6. As a matter of……………, I bought my fridge at the same shop.
A. concern B. fact C. truth D. coincidence
7. It was difficult to guess what her……………to the news would be.
A. feelings B. reaction C. capital D. opinion
8. John refused to put his career in …………… by opposing his boss.
A. jeopardy B. hazard C. risk D. stake
9…………….the phone rang later that night did Ann remember the appointment.
A. No sooner B. Only C. Not until D. Just before
10. My English is progressing……………
A. leaps and bounds B. bounds and leaps
C. odds and ends D. ends and odds
III. STRUCTURE AND GRAMMAR
Choose the best answer to complete each of the following sentences
1. Having been selected to represent the Association of American Engineers at the international convention,
…………..
A. the members applauded him B. he gave a short acceptance speech
C. a speech had to be given by him D. the members congratulated him
2…………..his advice, I would never have got the job.
86

Downloaded by Nhung Nguy?n (nhungnth2004@gmail.com)


lOMoARcPSD|20277325

A. Except B. Apart from C. But for D. As for


3. The Olympic Games are held every four years in a selected country, and …………..to athletes of all
nations.
A. are opened B. it is open C. they are open D. they are opened
4. The workmen went to ………….church to repair the roof.
A. X B. the C. a D. an
5.………….at the age of eighty, the Florentines gave him a state funeral.
A. On dying B. Having died C. Dead D. On his death
6. She demanded that she………….allowed to meet her son in prison.
A. was B. might be C. be D. could be
7. From her early teens,…………..
A. Blanche Willis Howard’s determination to be an author
B. was determined to be an author, Blanche Willis Howard
C. Blanche Willis Howard was determined to be an author
D. An author, Blanche Willis Howard was determined to be
8. In the Osage tribe of Oklahoma, beaded belts………….who held a high social status.
A. only by women were fashioned B. by women were fashioned only
C. were fashioned only by women D. were by women fashioned only
9. The teacher proposed that her students………….experiences with ESP.
A. write a composition on their
B. to write composition about the
C. wrote some compositions of his or her
D. had written any compositions for his
10. Astronomer Maria Mitchell was the first woman ………….to the American Academy of Arts and
Sciences.
A. to be elected B. she was elected
C. was elected D. for her to be elected
IV. PREPOSITION AND PHRASAL VERB
Choose the best answer to complete each of the following sentences
1. Although they had only been invited for lunch, they………….until supper time.
A. stayed on B. stayed at C. stayed up D. stayed in
2. I didn’t set………….to start an argument; it just happened.
A. off B. out C. about D. up
3. This situation is analogous………….the one we had faced last year.
A. for B. in C. from D. to
4. Demand for the products is expected to peak 5 years from now and then to…………..
A. taper off B. fall down C. set back D. drift away
5. Lack of sleep over the last few months is finally…………. Jane.
A. catching up with B. getting on with C. coming over D. putting on
6. When John was arrested for drunken driving, he expected to lose his driving license, but he was ………….
with a fine.
A. let through B. let off C. let out D. let down.
7. Could you………….a list of all the things that need to be done?
A. go beyond B. put out C. make up D. come through
8. It never occurred………….me to ask him………….proof………….his identity.
A. for..for....of B. to....for....of C. to for.....for D. for....for...for
9. He said he was ………….debt and asked me………….a loan………….$60.
A. on....for....of B. in for....with C. on by....of D. in....for.:..of
10. Once again poor Colin has been………….for promotion.
87

Downloaded by Nhung Nguy?n (nhungnth2004@gmail.com)


lOMoARcPSD|20277325

A. stood by B. passed over C. locked out D. struck off


V. READING
Reading 1:
Read the following passage and circle the best answer to each of the following questions.
Coincident with concerns about the accelerating loss of species and habitats has been a growing
appreciation of the importance of biological diversity, the number of species in a particular ecosystem, to the
health of the Earth and human well-being. Much has been written about the diversity of terrestrial organisms,
particularly the exceptionally rich life associated with tropical rain-forest habitats. Relatively little has been
said, however, about diversity of life in the sea even though coral reef systems are comparable to rain forests
in terms of richness of life.
An alien exploring Earth would probably give priority to the planet's dominant, most-distinctive
feature - the ocean. Humans have a bias toward land that sometimes gets in the way of truly examining global
issues. Seen from far away, it is easy to realize that landmasses occupy only one-third of the Earth's surface.
Given that two- thirds of the Earth's surface is water and that marine life lives at all levels of the ocean, the
total three-dimensional living space of the ocean is perhaps 100 times greater than that of land and contains
more than 90 percent of all life on Earth even though the ocean has fewer distinct species.
The fact that half of the known species are thought to inhabit the world's rain forests does not seem
surprising, considering the huge numbers of insects that comprise the bulk of the species. One scientist found
many different species of ants in just one tree from a rain forest. While every species is different from every
other species, their genetic makeup constrains them to be insects and to share similar characteristics with
750,000 species of insects. If basic, broad categories such as phyla and classes are given more emphasis than
differentiating between species, then the greatest diversity of life is unquestionably the sea. Nearly every
major type of plant and animal has some representation there.
To appreciate fully the diversity and abundance of life in the sea, it helps to think small. Every
spoonful of ocean water contains life, on the order of 100 to 100,000 bacterial cells plus assorted microscopic
plants and animals, including larvae of organisms ranging from sponges and corals to starfish and clams and
much more.
Questions:
1. What is the main point of the passage?
A. Humans are destroying thousands of species.
B. There are thousands of insect species.
C. The sea is even richer in life than the rain forests.
D. Coral reefs are similar to rain forests.
2. The word "appreciation" in paragraph 1 is closest in meaning to
A. ignorance B. recognition C. tolerance D. forgiveness
3. Why does the author compare rain forests and coral reefs?
A. They are approximately the same size.
B. They share many similar species.
C. Most of their inhabitants require water.
D. Both have many different forms of life.
4. The word "bias" in paragraph 2 is closest in meaning to
A. concern B. disadvantage C. attitude D. prejudice
5. The passage suggests that most rain forest species are
A. insects B. bacteria C. mammals D. birds
6. The word "there" in paragraph 3 refers to
A. the sea B. the rain forests
C. a tree D. the Earth's surface
7. The author argues that there is more diversity of life in the sea than in the rain forests because
A. more phyla and classes of life are represented in the sea
88

Downloaded by Nhung Nguy?n (nhungnth2004@gmail.com)


lOMoARcPSD|20277325

B. there are too many insects to make meaningful distinctions


C. many insect species are too small to divide into categories
D. marine life-forms reproduce at a faster rate
8. Which of the following is NOT mentioned as an example of microscopic sea life?
A. Sponges B. Coral C. Starfish D. Shrimp
9. Which of the following conclusions is supported by the passage?
A. Ocean life is highly adaptive.
B. More attention needs to be paid to preserving ocean species and habitats.
C. Ocean life is primarily composed of plants.
D. The sea is highly resistant to the damage done by pollutants.
10. The word “abundance” in the last paragraph is closest in meaning to
A. a large quantity B. a considerable quality
C. a small number D. an average number
Reading 2:
Jazz has been called “the art of expression set to music”, and “America’s great contribution to music”.
It has functioned as popular art and enjoyed periods of fairly widespread public response, in the “jazz age” of
the 1920s, in the “swing era” of the late 1930s and in the peak popularity of modern jazz in the late 1950s. The
standard legend about Jazz is that it originated around the end of the 19th century in New Orleans and moved
up the Mississippi River to Memphis. St. Louis, and finally to Chicago. It welded together the elements of
Ragtime, marching band music, and the Blues. However, the influences of what led to those early sounds goes
back to tribal African drum beats and European musical structures. Buddy Bolden, a New Orleans barber and
cornet player, is generally considered to have been the first real Jazz musician, around 1891.
What made Jazz significantly different from the other earlier forms of music was the use of
improvisation. Jazz displayed a break from traditional music where a composer wrote an entire piece of music
on paper, leaving the musicians to break their backs playing exactly what was written on thế score. In a Jazz
piece, however, the song is simply a starting point, or sort of skeletal guide for the Jazz musicians to
improvise around. Actually, many of the early Jazz musicians were bad sight readers and some couldn’t even
read music at all. Generally speaking, these early musicians couldn’t make very much money and were stuck
working menial jobs to make a living. The second wave of New Orleans Jazz musicians included such
memorable players as Joe Oliver, Kid Ory, and Jelly Roll Morton. These men formed small bands and took the
music of earlier musicians, improved its complexity, and gained greater success. This music is known as “hot
Jazz” due to the enormously fast speeds and rhythmic drive.
A young cornet player by the name of Louis Armstrong was discovered by Joe Oliver in New Orleans.
He soon grew up to become one of the greatest and most successful musicians of all time, and later one of the
biggest stars in the world. The impact of Armstrong and other talented early Jazz musicians changed the way
we look at music
Questions:
1. The passage answers which of the following questions?
A. Why did Ragtime, marching band music, and the Blues lose popularity after about 1900?
B. What were the origins of Jazz and how did it differ from other forms of music?
C. What has been the greatest contribution of comet players to music in the twentieth century?
D. Which early Jazz musicians most influenced the development of Blues music?
2. According to the passage, Jazz originated in
A. Chicago B. St. Louis
C. along the Mississippi river D. New Orleans
3. The word “welded” in paragraph 1 is closest in meaning to
A. squeezed B. bound C. added D. stirred
4. Which of the following distinguished Jazz as a new form of musical expression?
A. the use of comets B. “hot Jazz” C. improvisation D. New Orleans
89

Downloaded by Nhung Nguy?n (nhungnth2004@gmail.com)


lOMoARcPSD|20277325

5. The word “skeletal” in paragraph 2 is closest in meaning to


A. framework B. musical C. basic D. essential
6. Which of the following can be inferred from the passage?
A. many early Jazz musicians had poor sight
B. there is no slow music in Jazz
C. many early Jazz musicians had little formal musical training
D. the cornet is the most common musical instrument used in Jazz
7. The word “menial” in paragraph 2 is closest in meaning to
A. sociable B. attractive C. degrading D. skilled
8. According to the passage, which of the following belonged to the second wave of New Orleans Jazz
musicians?
A. Louis Armstrong B. Buddy Bolden C. St. Louis D. Joe Oliver
9. All of the following are true EXCEPT
A. the late 1930s was called the “swing era”
B. “hot Jazz” is rhythmic
C. Jazz has been said to be America’s greatest contribution to music
D. Joe Oliver is generally considered to be the first real Jazz musician
10. The word “its” in paragraph 2 refers to
A. small bands B. earlier music C. men D. earlier musicians
VI. GUIDED CLOZE TEST
CLOZE TEST 1:
Read the following passage and choose the options that best complete the blanks
GIRLS AND TECHNOLOGY
If you want your daughter to succeed, buy her a toy construction set. That’s the advice from Britain’s
(1) ……………female engineers and scientists. Marie-Noelle Barton, who heads Engineering Council
campaign to encourage girls into science and engineering, maintains that some of Britain’s most successful
women have had their career shaped by the toys they played with as children. Even girls who end (2)
……………nowhere near a microchip or microscope could benefit from a better grasp of science and
technology.
“It’s a (3) ……………of giving them experience and confidence with technology so that when they
are (4) ……………with a situation requiring some technical know-how, they feel they can handle it and don’t
just (5) …………… defeat immediately,” says Mrs. Barton. “I believe that a lot of girls feel unsure of them
when it comes (6) ……………technology and therefore they might be losing out on jobs because they are
reluctant even to apply for them.”
Research recently carried (7) ……………suggests that scientific and constructional toys should be (8)
……………to girls from an early age, otherwise the result is “socialization” into stereotypically female (9)
……………, which may explain why relatively few girls study science and engineering at university in
Britain. Only 14% of those who have gone for engineering (10) ……………at university this year are
womens although this figure does represent an improvement on the 7% recorded some years ago.
1. A. foremost B. uppermost C. predominant D. surpassing
2. A. in B. by C. on D. up
3. A. matter B. situation C. state D. cause
4. A. approached B. encountered C. presented D. offered
5 A. admit B. allow C. receive D. permit
6. A. for B. to C. from D. with
7 A. off B. through C. forward D. out
8. A. accessible B. feasible C. reachable D. obtainable
9. A. characters B. parts C. states D. roles
10. A. options B. alternatives C. selections D. preferences
90

Downloaded by Nhung Nguy?n (nhungnth2004@gmail.com)


lOMoARcPSD|20277325

CLOZE TEST 2
Read the following passage and choose the options that best complete the blanks
SOUND ADVICE FOR LANGUAGE LEARNERS
A recent issue of a language learning magazine has consulted a number of experts in the (1) ………of
second language acquisition. Their advice may prove invaluable for those (2) ……………a language course.
One suggestion is that you (3) ……………whether you are likely to be successful at learning a language. Did
you enjoy studying languages at school, for example? Do you have enough time to learn a language? The
major (4) …………… will be your own time and effort.
If proof of your level of proficiency is important you must make sure that the course on offer leads to a
(5) ……………qualification. Also, be realistic in your (6) ……………. If you don't set achievable aims you
are more likely to give up. Do not be deceived into thinking that the most expensive courses are the best. (7)
……………around to get the best possible value for money. You should also bear in mind that the quicker you
learn a language the more quickly you forget it. Sandra Miller, a French teacher, tried to teach herself German
by enrolling on a crash course. Already fluent in four languages and with a sound knowledge of teaching
methodology her chances of (8) ……………progress were high. Three years (9) …………… she remembers
very little. She feels her biggest mistake was not to follow (10) ……………her first experience. "I should
have consolidated what I'd learn by continuing to study, even if it were by myself."
1. A. domain B. branch C. field D. area
2. A. wondering B. thinking C. looking D. considering
3. A. assess B. review C. balance D. survey
4. A. change B. cost C. price D. evaluation
5. A. recognised B. understood C. valued D. regarded
6. A. sights B.ends C. objects D. goals
7. A. Nose B. Push C. Run D. Shop
8. A. achieving B. doing C. gaining D. making
9. A. on B. forward C. from D. onward
10. A. up B. on C. through D. out
PART B: WRITTEN TEST
I. OPEN CLOZE TEST
Cloze Test 1:
Fill in each blank with ONE suitable word
Of all the accounts of premonitions, one of the most dramatic and most easily verifiable concerns the
sinking of the Titanic in 1912. In 1898, author Morgan Robertson wrote a novel (1) ……………Futility which
bore many striking similarities to the loss of the Titanic 14 years later. Robertson’s ship, the Titan, was the
largest ship afloat and had the most modern equipment and the most highly qualified (2) ……………
members. The only thing that she lacked was a sufficient (3) ……………of lifeboats to accommodate
everyone (4) …………… board. However, this did not seem to matter as the Titan was believed to be
unsinkable.
It was April when the voyage in question took (5) ……………, and the Titan was steaming at great
speed. On each of her two masts the crew had hoisted great triangular sails to help the ship make a record
crossing. So intent on breaking this record (6) ……………the crew that when they rammed a windjammer
they did not stop to (7) …………… up anyone who might have survived. A curse was shouted by one of the
sailors in the water, calling down (8) ……………wrath of God on the doomed vessel.
Later on a foggy but moonlit night, Titan encountered an iceberg. She didn’t strike (9) ……………
squarely but slid up a gradual slope of ice until she was almost completely out of the water; the severely
damaged ship then slid backwards into the water, after alio having her starboard lifeboats smashed in the
process. (10) ……………of 3,000 people on board, only 13 survived when Titan sank.
Cloze Test 2:
Fill in each blank with ONE suitable word
91

Downloaded by Nhung Nguy?n (nhungnth2004@gmail.com)


lOMoARcPSD|20277325

Diseases of the teeth and oral cavity occur very frequently. They owed their development primarily (1)
……………poor oral hygiene, but the general state of health is also (2) ……………some importance.
Depending on the state of the whole organism, diseases of the teeth may run a more acute course and may
quickly involve a number of teeth. Moreover, through a defective tooth, an infection may gain entrance into
the organism.
Dental caries is a wide-spread disease. In addition to oral hygiene, the character of nutrition and the
temperature of the food, occupation and physiological state of the organism also (3) ……………an important
part in the disease. The molars are most frequently affected.
The disease begins (4) …………… dissolution of the enamel and ends in destruction of the teeth.
Microorganism can gain entrance into such teeth and into the pulp (5) ……………they cause inflammation
(pulpitis).
During pulpitis teeth are sensitive (6) …………… chemical, mechanical and thermal stimuli. In
pulpitis spontaneous pains arise and sometimes spread over the jaw or the ear, temple and so on. The pain may
be very severe and may (7) ……………for a long time.
The treatment of caries must consist of cleansing the cavity and filling the (8) ……………
tooth. In (9) ……………to alleviate the pains, doctors use tooth drops. They introduce them into the cavity of
the carious tooth on a small cotton ball after first removing the remains of food (10) ……………the tooth.
II. WORD FORMS
Word form 1:
Give the correct forms of the words in the brackets
1. I regret to say you have been……………about that. (INFORM)
2. The……………of the castle makes us gasp in wonder. (GRAND)
3. There was a heavy……………yesterday afternoon which completely ruined the church Garden Party.
(POUR)
4. The……………staff consists of ten experienced journalists. (EDIT)
5. The situation is so……………in some cities now that it is difficult to see any solution. (CHAOS)
6. I think it’s sheer……………to get married in church if you don’t believe in God. (HYPOCRITE)
7. Travelling in big cities is becoming more and more……………today. (TROUBLE)
8. She’s so……………that she won’t let anything stand in the way of her ambition. (MIND)
9. You must be careful about what you say about his poems. He’s a ……………person. (SENSE)
10. The control center is deep underground and completely ……………except by a direct hit from a nuclear
missile. (DESTROY)
Word form 2:
Complete the paragraph, using the correct forms of the given words in the box.
precise intend practice clear designer
depend engine manufactory specify implicit
Designers do not (1) ……………things. All good designers ask questions of their client and spend
time helping the client to (2) ……………what he or she really wants. If the product is to be made to the
designer’s (3) ……………, then the designer must ensure that the factory has the tools and the intelligence
and that each element specified is (4) ……………. On complex jobs several product (5) …………… will be
involved, today with computer-aided software packages, to help to realize a design (6) ……………. The
greatest difference between the designer and the single (7) …………… craftsperson is that the craftsperson
does not have the problem of communicating his or her (8) ……………to others for translation into objects.
The designer; however, must make his or her intentions (9) …………… communication is at the heart of (10)
……………
II. ERROR IDENTIFICATION
Identify 10 errors in the following passage and correct them. One has been done as an example for you.
(1) Music can bring us to tears or to our feet, drive us into battle or lull us sleeping. Music is
(2) indeed remarkable in it power over all humankind, and perhaps for that very reason, no
92

Downloaded by Nhung Nguy?n (nhungnth2004@gmail.com)


lOMoARcPSD|20277325

(3) human culture on earth has never lived without it. From discoveries made in France and
(4) Slovenia even Neanderthal man, as long as 53,000 years ago, had developed surprisingly
(5) sophisticated, sweet-sounding flutes carving from animal bones. It is perhaps then, no
(6) accident that music should strike so a chord with the limbic system - an ancient part of
(7) our brain, evolutionarily speaking, and one that we share with much of the animal kingdom.
(8) Some researchers even said that music came into this world long before the human race
(9) ever did. For example, the fact that whale and human music have so much in common even
(10) then our evolutionary paths have not intersected for nearly 60 million years suggests
(11) that music may predate humans. They assert that rather than being the inventors of music,
(12) we are latecomers to the musical scene.
(13) Humpback whale composers employ many of the same tricks that human songwriters do.
(14) In addition to being used similar rhythms, humpbacks keep musical phrases to a
(15) few seconds, creating themes out of several phrases before singing the next one. Whale
(16) songs in general are no longer symphony movements, perhaps because they have a similar
(17) attention span. Although they can sing over a range of seven octaves, the whales typically
(18) sing in key, spreading adjacent notes no farther apart than a scale. They mix percussive
(19) and pure tones in pretty much the same ratios as human composers - and follow their
(20) ABA form, which a theme is presented, elaborated on and then revisited in a slightly
(21) modified, form. Humpback whale songs include repeating refrains that rhyme.
(22) It has been suggested that whales might use rhymes of exactly the same reasons
(23) that we do: as devices to help them remember. Whale songs can also be rather catchy.
(24) When a few humpback from the Indian Ocean strayed into the Pacific, some of the whales
(25) they met there quick changed their tunes - singing the new whales’ songs within three
(26) short years. Some scientists are even tempted to speculate that a universal music
(27) awaits discovery.
Line Error Correction
(1) Sleeping To sleep
……………………. ……………………. …………………….
……………………. ……………………. …………………….
……………………. ……………………. …………………….

IV. SENTENCE TRANSFORMATION


Rewrite each of the following sentences in such a way that it means exactly the same as the one given.
1. I was surprised to discover that the hotel was a long way from the center. (TURNED)
→ To my………………………………………………………………………………………
2. He easily loses his temper. (FLIES)
→ He …………………………………………………………………………………………
3. I find Tom’s behaviors quite incomprehensible. (LOSS)
→ I find………………………………………………………………………………………..
4. They’ve been arguing about the motorway extension for years. (BONE)
→ The proposed……………………………………………………………………………….
5. I’m afraid our problems are just beginning. (ICEBERG)
→ The problems………………………………………………………………………………..
6. You don’t have to buy one if you don’t like it.
→ You are under………………………………………………………………………………..
7. Even though I admire his courage, I think he is foolish.
→ Much…………………………………………………………………………………………
8. The chances are that the whole thing will have been forgoten by next term.
→ In all…………………………………………………………………………………………
93

Downloaded by Nhung Nguy?n (nhungnth2004@gmail.com)


lOMoARcPSD|20277325

9. There aren’t many other books which explained this problem so well.
→ In few other books…………………………………………………………………………..
10. The only thing that prevented the passing of the bill was the death of the Prune Minister.
→ Had it………………………………………………………………………………………..

TRƯỜNG THPT CHUYÊN NGUYỄN DU - ĐẮK LẮK


MULTIPLE CHOICE TEST
PART ONE: PHONOLOGY
A. Pick out the word whose underlined part is pronounced differently from that of the others in each
group.
1. A. pessimistic B. business C. dissolve D. miss
2. A. unity B. suite C. studious D. volume
3. A. hasty B. tasty C. wastage D. nasty
4. A. merchant B.sergeant C. commercial D. term
5. A. learned B. wicked C. sacred D. looked
B. Pick out the word whose main stress is placed differently from the others in each group.
6. A. recommend B. hurricane C. photograph D. separate
b. A. fertilize B. implement C. prevention D. enterprise
h. A. dismal B. presented C. contents D. decent
19. A. deteriorate B. provincial C. conscientious D. conspiracy
110. A. immerse B. submerge C. commerce D. reverse
PART TWO: VOCABULARY AND STRUCTURE
Choose the correct answers to complete the sentences
1. “Can you come away with me for the weekend?”
“I can’t as I’m decorating the kitchen……………, my mother is ill.”
A. Even so B. After all C. All the same D. On top of that
2. We will accept your cheque; although it is not our normal……………
A. practice B. occasion C. intention D. state
3. ……………of commodities by air began in the 1920s at the same time as airmail service.
A. The shipping B. A ship C. The shipped D. To ship
4. Despite claims that filters and low-tar tobacco make smoking somewhat safer, in fact, they only marginally
reduce, ……………eliminate, the hazards.
A. none B. no C. not D. nor
5. Mozart composed music when he was still a child. He had a great……………for it.
A. gift B. force C. ability D. skill
6. The light gradually……………and shapes and colors grew fainter.
A. died B. disappeared C. faded D. melted
7.The children gazed at the magician……………as he performed his tricks.
A. opened-eyed B. lit-eyed C. hawked-eyed D. wide-eyed
8. The speaker ……………the audience’s attention to the numbers on the screen.
A. drew B. paid C. pulled D. made
9. When he saw her with Tom, Jack……………anger.
A. showed up B. bristled with C. pricked with D. brushed with
10. Will was made…………… three months ago and is still looking for a job.
A. sacked B. retired C. fired D. redundant
11. Christine felt that she was among friends so she…………… a little.
A. looked up B. melted away C. opened up D. screwed up
12. I didn’t like this album at first, but it’ s really starting to……………….. on me now.
A. grow B. raise C. develop D. increase
94

Downloaded by Nhung Nguy?n (nhungnth2004@gmail.com)


lOMoARcPSD|20277325

13. I’m afraid I have to……………….to a dictionary in order to complete the translation.
A. apply B. avail C. resort D. recourse
14. Fred bought his new car on …………… quickly and without thinking.
A. impulse B. expectation C. prejudice D. reaction
15. I hate Joan Clifford! One day I’ll……………for all the pain she’s caused.
A. sound her out B. show her off C. tell her apart D. pay her back
16. Deserts are arid land areas where …………… through evaporation than is gained through precipitation.
A. the loss of more water B. loss more water
C. is more water lost D. more water is lost
17. She was overwhelmed with honor……………recognition……………her bravery.
A. with/of B. on/in C. by/about D. in/of
18. If you don’t stop smoking, you……………this risk of developing chronic bronchitis.
A. bear B. suffer C. make D. run
19. Psychologists define anxiety as a feeling of dread, apprehension, or……………
A. afraid B. be afraid C. having fear D. fear
20. Automatic fire alarms, ……………smoke detector, are installed in almost all public buildings.
A. such as the ubiquitous B. so ubiquitous
C. so as the ubiquitous D. such is the ubiquitous
21. The news of his sister’s death came as a (n)………….shock to him.
A. utter B. entire . C. extreme D. great
22. Educational toys and games give children an opportunity to enjoy themselves………….
A. and they are learning B. while learning
C. are they learning D. while their learning
23. I got very nervous during the exam. When the examiner asked my name, my mind went
completely………….
A. empty B. blank C. white D. void
24. You are not allowed to drive………….the influence …………. alcohol.
A. under/of B. in/of C. under/by D. by/in
25. The staff can’t take leavès at the ẩame time. They have to take holidays ………….rotation.
A. on B. under C. by D. in
26. The interviewees are supposed to give their answers to the job offers ………….
A. on the spot B. all in all C. beyond the joke D. within reach
27. When Helen and Andrew saw how ridiculous they looked they………….laughing.
A. came round B. burst out C. broke in D. flowed over
28. Sonja and Shirley haven’t spoken to each other since they…………. 2 years ago.
A. fell out B. stepped back C. mixed up D. died down
29. I think we can safely say now that we have got our money back, we are home and dry.
A. have not got wet B. have got no water
C. have been successful D. have got home dry
30. ………….exception………….the little baby, everybody in my family has to jog every morning.
A. With/to B. With/of C. In/of D. By/on
PART THREE: READING COMPREHENSION READING!:
Read the following passage and choose the best answer
Lead poisoning in children is a major health concern. [A]. Both low and high doses of paint can have
serious effects. Children exposed to high doses of lead often suffer permanent nerve damage, mental
retardation, blindness, and even death. Low doses of lead can lead to mild menial retardation, short attention
spans, distractibility, poor academic performance, and behavioral problems. [B]
This is not a new concern. [C]. As early as 1904, lead poisoning in children was linked to lead-based
paint. Microscopic lead particles from paint are absorbed into the bloodstream when children ingest flakes of
95

Downloaded by Nhung Nguy?n (nhungnth2004@gmail.com)


lOMoARcPSD|20277325

chipped paint, plaster, or paint dust from sanding. Lead can also enter the body through household dust, nail
biting, thumb sucking, or chewing on toys and other objects painted with lead-based paint. Although
American paint companies today must comply with strict regulations regarding the amount of lead used in
their paint, this source of lead poisoning is still the most common and most dangerous. Children living in
older, dilapidated houses are particularly at risk. [D]
1. What is the main topic of the passage?
A. Problems with household paint B. Major health concerns for children
C. Lead poisoning in children D. Lead paint in older homes
2. Which part of the passage discusses symptoms and consequences of lead poisoning?
A. Lines 2-5 B. Lines 6-8 C. Lines 10-12 D. lines 12-13
3. The phrase “exposed to” in line 2 could best be replaced by which of the following?
A. Familiar with B. In contact with C. Displaying D. Conducting
4. As used in line 2, which of the following is closest in meaning to the word “suffer”?
A. experience B. reveal C. feel pain from D. grieve with
5. Which of the following does the passage infer?
A. Paint companies can no longer use lead in their paint.
B. Paint companies aren’t required to limit the amount of lead used in their paint.
C. Paint companies must limit the amount of lead used in their paint.
D. Paint companies have always followed restrictions regarding the amount of lead used in their paint.
6. Which of the following is closest in meaning to the phrase “linked to” in line 6?
A. endorsed by B. threatened C. combined D. associated with
7. The word “absorbed” in line 7 could best be replaced by………………
A. fixed B. assimilated C. soaked D. accepted
8. Where is the best place in the passage to add the following sentence “Older, run down houses commonly
have chipped, flaking paint and are more likely to have been painted with lead-based paint”? [A], [B], [C], or
[D]
9. Which of the following is closest in meaning to the word “chipped” as used in line 8?
A. fragmented B. canned C. sprayed D. unhealthy
10. In line 12, the word “dilapidated” is closest in meaning to which of the following?
A. poorly painted B. unpainted C. fallen down D. broken down
11. According to the passage, what is the most common source of lead poisoning in children?
A. household dust B. lead-based paint
C. painted toys D. dilapidated houses
12. What does the authors imply in the final sentence of the passage?
A. Lead-based paint chips off more easily than the newer paints.
B. Poor people did not comply with the regulations.
C. Old homes were painted with lead-based paint.
D. Old homes need to be rebuilt in order to be safe for children
READING 2:
Read the text and then choose from the list A -1 given below the best sentence to fit each gap (1-7).
Write one letter in the correct box. (14 points)
IN HOT WATER
Rachel Mills is a scientist who spends as much time as she can at the bottom of the Atlantic Ocean.
Rachel Mills teaches and does research into marine geochemistry, (1)…………... She is a lecturer at the
Oceanography Centre at Southampton University. When she isn’t teaching, she lowers herself into a steel
vehicle, a vessel for underwater exploration the size of a small car, and dives three kilometres down into the
Atlantic Ocean to study underwater volcanoes.'
“Inside,” she says, “space is so limited that I can reach out and touch the two pilots”. (2) …………...
A dive can last for 16 hours - three hours to reach the ocean floor, ten hours gathering samples of rock and
96

Downloaded by Nhung Nguy?n (nhungnth2004@gmail.com)


lOMoARcPSD|20277325

water and then three hours to get back up to the surface again.
“If anything happens, and you have a problem and have to get to the top quickly, you can hit a panic
button”. The outside drops away leaving a small circular escape vessel that gets released, and it’s like letting
go of a ping-pong ball in the bath - it goes rapidly to the surface. (3) …………..
“I didn’t know how I was going to react the first time I climbed into the vehicle. It was on the deck of
a ship and I got in with an instructor. (4) …………... They were testing me to see how I would react to being
in such a small place.’’. Now Rachel has made six dives. Last year she dived with a Russian crew. “We went
to a site which was a five-day sail west of the Canary Islands in the Atlantic. (5) …………... It is where
the Atlantic Ocean comes alive. The Russian team were dropping off some scientific equipment there to
discover the effect of a multi-national programme that would make a hole 150 metres through a volcano.”
When she isn’t at sea, Rachel is in her office at the Oceanography. Centre, Southampton. “Two thirds
of my salary comes from teaching which I love, but I do it so I can get on with my research into the “black
smokers”. This is'just another name for underwater volcanoes - water comes out of the rock and turns into
what looks like black smoke. (6) …………...
“The only time I’ve been frightened is when I first went down with the Americans. We were towing
equipment on a 50-metre rope when suddenly there was an explosion. There was this immense bang as the
shock waves hit our vehicle and I thought, “I’m going to die?’ We stared at each other in silence, waiting. (7)
…………... The relief was incredible - we were still alive!”
“It’s such an adventure diving down;to the deepest part of the ocean. Every time I look out of the
porthole and see those chimneys, there is such a sense of wonder. (8) …………... I had studied the black
smokers for three years for my Ph.D. When I got down there and saw them for real, it was such an amazing
feeling.”
A. Here, on the ocean floor, is a huge area of underwater volcanoes, their chimneys all blowing out black,
smoke.
B. Here I am on the bottom of the sea, and no one else on this planet has ever before seen them.
C. “No one’s tested it yet, but I don't think it would be a very pleasant journey”
D. He then talked me through the emergency procedures, including what to do if the pilot has a heart attack!
E. They are used to these conditions, which mean they can’t stand up or move, and they must stay inside until
someone opens the door from the outside.
F. When it didn t happen, we couldn’t believe it.
G. This pours out at a rate of one metre per second and at a temperature of 350 degrees.
H. After that, as you get really deep, it's near freezing point so you need a sweater, thick socks, gloves and a
woolly hat.
I. which means she studies the chemical processes happening in the sea
PART FOUR: GUIDED CLOZE TEST
Read the passages below and decide which answer (A, B, C or D) best fits each space.
PASSAGE 1
One rather unlikely word that has recently entered the language is “blog”, a shortened form of “web
log”. A blog is a diary (1) ………….. on the Internet by the person writing it - the blogger - who presumably
(2) ………….. other people to read it. It is ironical that modern technology is being used to (3) …………..
new life into such an old-fashioned form as the personal journal. And now, as the technology about video
cameras is making them easier to use, we have the video log, or “vlog”. Vlogging does not require (4)
………….. sophisticated equipment: a digital video camera, a high-speed Internet connection and a host are
all that is needed. Vloggers can put anything that (5) ………….. their fancy onto their personal website. Some
vloggers have no ambitions rather than to show films they have (6) …………..while on holiday in exotic
places. However, vlogs can also (7) …………..more ambitious purposes. For instance, amateur film-makers
who want to make a (8) …………..for themselves might publish their work on the Internet. And increasingly,
vlogs are being used to (9) ………….. political and social issues that are not newsworthy enough to get
coverage by the mass media. It is still too early to predict whether vlogging will ever (10) …………..off in a
97

Downloaded by Nhung Nguy?n (nhungnth2004@gmail.com)


lOMoARcPSD|20277325

major way or if it is just a passing fade, but its potential is only now becoming apparent.
l. A. released B.sent C. posted D. mounted
2. A. believes B. expects C. assumes D.supposed
3. A. add B. inhale C. insert D. breathe.
A A. absolutely B. largely C. utterly D. highly
5. A. grasps B. appeals C. takes D. gives
6. A. shot B. photographed C. snapped D. captured
7. A. serve B. employ C. function D. play
8. A. publicity B. fame C. name D. circulate
9. A. emphasize B. publicize C. distribute D. bit
10. A. fly B. show C. take D. make
PASSAGE 2
Lately, there has been a dramatic rise in the number of burglaries in the area. John Amos came home to
find (1) …………..in his flat. He frightened them off when his car pulled up on the drive but by then the T.V
and video were gone. Despite their hurry they had the (2) ………….. to empty the drawers and cupboards into
a heap on the floor and had obviously taken anything of any value.
John (3) …………..local kids who had been pestering him, and hurling insults at him on his way to
work. Sometimes they came (4) …………..on his door and yelled at him through the letter box. His neighbors
said it was because John was a teacher and the kids were probably playing truant. They complained to John
that kids like that should be put behind (5) …………..and if they were too young for that then they should be
sent to secure units as far away as possible.
But John did not bother to call the police. Instead he decided to (6) …………..an alarm system. The
advertising blurb claimed that this was no mere burglar alarm but a highly sophisticated (7) ………..system.
The equipment included a detector, which would not only warn you that someone was approaching the (8)
…………..but would video record them as well. It was expensive but John was keen to outdo even the most
determined of burglars.
Somehow word got round that his flat was a fortress and this seemed to incite the kids to (9) …………
him more than ever. Their activities escalated and the attacks became more frequent. John became convinced
that the only (10) …………..way of dealing with the situation was to move. The following spring he
emigrated to Canada.
1. A. interlopers B. intruders C. invaders D. infiltrators
2. A. audacity B. rudeness C. bravery D. enterprise
3 A. accused B. presumed C. blamed D. assumed
4. A. hitting B. crashing C.banging D. smashing
5. A. barriers B. barricades C. rods D. bars
6. A. input B. install C. institute D. instigate
7. A. surveillance B. inspection C. supervision D. vigilance
8. A. establish B. building site C. premises D. environment
9. A. torment B. tackle C. alarm D. treat
10. A. adequate B. satisfactory C. capable D. passable.
WRITTEN TEST
PART ONE: OPEN CLOZE TEST
Fill in each gap in the passages below with ONE suitable word
PASSAGE 1:
Going to parties can be fun and enjoyable. If you are invited to a party, do call your host up early to (1)
…………..him of whether you are going. If you want to bring someone who has not been invited along with
you, you should ask for (2) …………..first. Remember to (3) …………..appropriately for the party. You will
stick out like a sore thumb if you are dressed formally while everyone else is in T-shirt and jeans. If you are
not sure what to wear, do ask your host.
98

Downloaded by Nhung Nguy?n (nhungnth2004@gmail.com)


lOMoARcPSD|20277325

(4) …………..the party, you may perhaps like to help your host by (5) ………….. to serve drinks or
wash the dishes. Your host would certainly appreciate these efforts. If you happen to be ỉn a party (6) ……..
you do not know anyone, do not try to monopolize the host’s attention. This is inconsiderable since your host
has many people to attend to and cannot spend all his time with you. Instead, learn to mingle (7) …………..
the others at the party. You could try breaking the (8) …………..by introducing yourself to someone who is
friendly-looking.
Before you leave the party, remember to (9) …………..your host first. If you have the time, you could
offer to help your host (10) …………..up the place.
PASSAGE 2:
A new report from the World Wildlife Fund (WWF) reveals that fish populations are under increasing
(1) …………..from global warming. The report “Are we putting our fish in hot water?” describes how climate
change is causing temperatures to (2) …………..in rivers, lakes and seas. This means less food and oxygen
for (3) ………….. life, stunted fish growth and (4) ………….. offspring. The report says that temperate fish
such as salmon, catfish and sturgeon cannot spawn if winter temperatures do not drop below a certain (5)
…………..Warmer water also means fish could mass migrate to cooler areas, (6) …………..the temperature
resembles their normal habitat. This could impact on many species’ ability to survive. Some species will
become extinct if the water temperature rises by a degree or two.
WWF director Andrew Lee said: “Climate change increases the (7) …………..on fish populations that
are already strained to the limit by over-fishing in the marine environment. We must act urgently to (8)
…………..both carbon dioxide emissions and fishing pressures to protect fish populations as they are one of
the world’s most valuable biological, nutritional and economic assets.” Forty percent of the world’s people are
reliant on fish for basic sustenance and a main (9) …………..of protein. Dr Richard Dixon, director of WWF
Scotland spoke ahead of next week’s UN Climate Change Conference in Montreal. He said: “If we fail to
secure deeper reductions in greenhouse gas (10) …………..we will increase the pressures on fish and the
billions of people that depend on them.”
PART TWO: WORD FORMS
A Supply the correct form of the words in the CAPITAL LETTER
1. EXIST Frank told everyone that he worked for a large company, but the company is………..
2. ERR The president ………… drew his conclusions from that ill-founded evidence.
3. FACT “What if” questions involving…………….. are familiar in historical speculations.
4. LIVE So ……………. was her passion for coin collecting that it wouldn’t run in her blood.
5. DISTANCE David and Jane are basically …………….from here, so it’ll take about the same time
to get to either of them.
6.COMMUNICATE The prisoners have been held……………. for eighteen years.
7.BRAIN Joining the saving plan is a……………… Just do it.
8. DRAMATIC Don’t be so ……………..
9. VACUUM If you want to store your beef to last longer, it should be …………….
10. GLOW Now I can still feel the …………….of our victory.

B. Supply the correct forms of the words in the CAPITAL LETTER


One of the greatest problems with holidays, apart from the usual travel(1)………..and 1.COMPLICATE
accommodation difficulties, is the (2)…………….people have of them. When we go 2.EXPECT
on holiday we expect to leave all stresses and strains of our daily lives behind us. We
imagine we will be able to escape to such ạ degree that we even tend to believe,
(3.) …………..or not, that we can change our own 3. CONSCIENCE
(4.) ………….. and become completely different people. The average business- 4. PERSON
person, tense, (5.) ………….., short-tempered, unable to relax, envisages 5. OCCUPY
herself/himself as, from the moment of locking the office door, a radically different
kind of person: carefree, good-humored, ready to relax and enjoy whatever
99

Downloaded by Nhung Nguy?n (nhungnth2004@gmail.com)


lOMoARcPSD|20277325

adventures present themselves. In practice, we take ourselves with us wherever we


go, and the personality is (6.) ………….. over years of stress and tension is almost 6. SHAPE
(7.) ………….. to shake off at the moment’s notice. 7. POSSIBILITY
It is no (8.) ………….. so many holidays are a disappointment, no matter how 8. WONDERFUL
(9.) …………... they go or how lovely the weather is. In fact, the frequent problems 9. SMOOTH
that crop up during the average holiday are probably a welcome distraction from the
(10.) ………….. feeling that we are not enjoying ourselves as much as we should 10. NAG

PART THREE: ERROR IDENTIFICATION


There are ten mistakes in the following paragraph. Find them and correct them
Carefully conducting studies that have followed the children of Line 1:…………
working mothers have not been able to show any long-term Line 2:…………
problems, compared with children their mothers stayed at Line 3:…………
home. My personal view is that mothers should allow to work Line 4:…………
if they wish. Whether we like it or not, there are a number of Line 5:…………
mothers who just have to work. There are those who have Line 6:…………
invested such a big part of their lives in establishing a career Line 7:…………
that they cannot expect to see them lost. Then there are many Line 8:…………
who must work out of pure economical necessity. Many Line 9:…………
mothers are not born to be full-time parents. After a few Line 10:…………
months at home with a much love infant, they feel trapped and Line 11:…………
isolated. Line 12:…………
There are a number of options when it comes to choose Line 13:…………
childcare. This range from child minders and nannies through Line 14:…………
to Granny or the kind lady across the street. In reality, however, Line 15:…………
many parents do not have any choice; they have to accept Line 16:…………
anything they can get. Be prepared! No matter how good the Line 17:…………
childcare may be, some children are going to protest wild if Line 18:…………
they are left. This is a perfectly normal stage of child Line 19:…………
development. Babies separate well in the first six months, but Line 20:…………
soon later that they start to get a crush on Mum and close Line 21:…………
family members. Make sure that in the first week, you allow Line 22:…………
plenty time to help your child settle in. Line 23:…………

PART FOUR: SENTENCE TRANSFORMATION


A. Rewrite the following sentences in such a way that the second sentence has the atne meaning as the
first one.
1. An announcement of their engagement appeared in the local paper.
→Their engagement……………………………………………………………………………..
2. Everyone was exhausted apart from Sally.
→ With…………………………………………………………………………………………..
3. His efforts to find a solution didn’t deserve such savage criticism.
→ He shouldn’t ………………………………………………………………………………….
4. You won’t reach the station in less than twenty minutes.
→ It will take ……………………………………………………………………………………
5. She took extra night classes, so as not to have to repeat the course.
→ She avoided …………………………………………………………………………………..
B. Complete the second sentence so that it has a similar meaning to the first sentence, using the word
given. Do not change the word given. You must use between three and eight words, including the word
100

Downloaded by Nhung Nguy?n (nhungnth2004@gmail.com)


lOMoARcPSD|20277325

given.
1. He is becoming quite famous as an interviewer. (NAME)
→ He is……………………………………………………………………………..as an interviewer.
2. We must include buying new furniture in our household budget this year. (ACCOUNTED)
→ Buying new furniture …………………………………………… in our household budget this year.
3. Julie always listens to my complaints about work. (EAR)
→ Julie always ……………………………………………………………my complaints about work.
4. The novel didn’t come up to my expectations.: (SHORT)
→ The novel……………………………………………………………………………..expectations.
5. It’s just possible that we may have to introduce pay cuts. (RULED)
→ We…………………………………………………………………………………………pay cuts.

TRƯỜNG THPT CHUYÊN LÊ QUÝ ĐÔN - ĐÀ NẴNG


PART A: MULTIPLE CHOICE
I. PHONOLOGY
A. Pronunciation:
Choose the word whose underlined part is pronounced differently from that of the other.
1. A. duvet B. ballet C. amulet D. gilet
2. A. thorough B. enough C. rough D. tough
3. A. island B. aisle C. conserve D. debris
4. A. absinthe B. soothe C. loathe D. wreathe
5. A. sizzle B. wily C. quits D. pidgin
B. Stress:
Choose the word whose stress pattern is different from that of the others.
1. A. ingenuity B. bacterium C. phenomenon D. pneumonia
2. A. misinterpret B. advantageous C. adolescent D. contributory
3. A. marmalade B. infamous C. coincide D. diagram
4. A. compromise B. correspond C. interview D. innocent
5. A. antecedent B. magnificent C. benevolent D. sobriety
II. VOCABULARY AND GRAMMAR
Choose the best answer for each question.
1. The bus arrived………………on time.
A. slim B. clean C. dead D. live
2. I don’t want to burden my daughter with my problems; she’s got too much………………
A. up her sleeve B. in her mind C. in effect D. on her plate
3. Keng has cried……………… too often and now nobody believes a word of what he is saying.
A. wolf B. sheep C. donkey D. lion
4. Tom is normally very efficient but he’s been making a lot of mistakes……………….
A. for now B. of late C. in a while D. shortly
5. From time to time, he………………himself to a weekend in a five-star hotel.
A. indulges B. treats C. craves D. benefits
6. We may win, we may lose - it’s just the luck of the………………
A. chance B. odds C. fate D. draw
7. Do you know what the main characteristics of your ... ................ sign are?
A. zodiac B. astrology C. star D. horoscope
8. A traveler looks down on anyone who seems to be a(n) ………………tourist.
A. mere B. sole C. only D. lone
9. We weren’t hit as the truck swerved at us but it was a………………shave.
A. nearby B. close C. narrow D. tiny
101

Downloaded by Nhung Nguy?n (nhungnth2004@gmail.com)


lOMoARcPSD|20277325

10. I love to do things for children because I get a………………out of it.


A. move B. shoot C. pass D. kick
11. The candle flame began ………………a little in the wind.
A. sparkle B. twinkle C. dazzle D. flicker
12. The noise got………………as the car disappeared into the distance.
A. smaller B. fainter C. weaker D. slighter
13. If you want a good flat in London, you have to pay through the………………for it.
A. mouth B. ear C. nose D. eye
14. The project was terminated………………April 1st.
A. as of B. by far C. until D. somewhat
15. It is imperative ………………what to do when there is a fire.
A. that he knew B. that everyone know
C. we knew D. he must know about
16. I was astonished that he turned down the job -I………………it would have been ideal for him.
A. have thought B. had been thinking
C. am thinking D. would have thought
17. Margaret was slow at school but she went on ..................... the Director.
A. to become B. becoming C. became D. having become
18. This is the first time Sue has met John. They……………..have met before.
A. needn’t B. mustn’t C. can’t D. shouldn’t
19. It pays……………..some professional advice before you make a decision.
A. get B. getting C. to getting D. to get
20. In twenty-four hours’ time, I……………..on my yacht.
A. will relax B. will be relaxing
C. will have relaxed D. am going to relax
III. PREPOSITIONS AND PHRASAL VERBS
Choose the best answer for each question.
1. The bedroom is infested ..................... bugs and beetles.
A. in B. against C. for D. with
2. That man is so arrogant that he completely impervious ..................... all criticism.
A.to B. against C. with D. for
3. The small boat drifted helplessly ............... the mercy of the wind and waves.
A. to B. with C. at D. in
4. Parents should never dote……………..their children if they do not want to spoil them.
A. to B. for C. on D. in
5. It stands ..................... reason that you can’t simply say sorry and hope everything will be normal again.
A. to B. for C. on D. in
6. Everyone approved of the scheme but when we asked for volunteers, they all……………..
A. put back B. turned back C. fell back D. hung back
7. If you would like to wait a moment, sir, I will just ..................... your file on the computer screen.
A. pull down B. call up C. bring in D. pick up
8. Mr. Jones gave his sons some money to……………..them up in business.
A. get B. make C. put D. set
9. Many pensioners find it hard to ..................... by on their small pensions after their retirement.
A. turn B. hold C. stand D. get
10. Constant arguing doesn’t..................... for a happy marriage.
A. make B. call C. set D. put
IV. READING COMPREHENSION Passage
1. Read the passage and choose the best answer to each question.
102

Downloaded by Nhung Nguy?n (nhungnth2004@gmail.com)


lOMoARcPSD|20277325

Simply being bilingual doesn’t qualify someone to interpret. Interpreting is not only a mechanical
process of converting one sentence in language A into the same sentence in language B Rather it’s a complex
art in which thoughts and idioms that have no obvious counterparts from tongue to tongue - or words that have
several meanings must be quickly transformed in such a way that the message is clearly and accurately
expressed to the listener.
At one international conference, an American speaker said, “You can’t make a silk purse out of a sow's
ear”, which meant nothing to the Spanish audience. The interpretation was, “A monkey in a silk dress is still a
monkey” - an idiom the Spanish understood and that expressed the same idea.
There are 2 kinds of interpreters, simultaneous and consecutive. The former, sitting in a separated
booth, usually at a large multilingual conference, speaks to listeners wearing headphones, interpreting what a
foreign language speaker says - actually a sentence behind. Consecutive interpreters are the ones most
international negotiations use. They are employed for smaller meetings without sound booths and headphones.
Consecutive interpretation also requires two-person teams. A foreign speaker says his piece while the
interpreter, using a special shorthand, takes notes and during a pause, tells the client what was said.
1. What is the purpose of the passage?
A. To differentiate between simultaneous and consecutive interpreters.
B. To state the qualifications of an interpreter.
C. To point out the importance of an interpreter. ..
D. To explain the scope of interpreting.
2. The author implies that most people have the opinion that the skill of interpreting is……………
A. simpler than it really is B. very complex and demanding
C. highly valued and admired D. based on principles of business
3. The example “You can’t make a silk purse out of ạ sow's ear” is used to……………
A. show the differences in language A and language B
B. stress the importance of word for word translation
C. emphasize the need for translation of the meaning of what is said
D. point out the difference in attributes of animals in English and Spanish
4. A precondition of being a translator is……………
A. being a linguist B. being bilingual
C. being able to use high-tech equipment D. working well with people
5. Which of the following would a consecutive interpreter be used for?
A. A business transaction between 2 foreign speakers.
B. A large meeting of many nations.
C. A translation of a foreign book.
D. An interpretation of a major literary work.
6. What would a simultaneous interpreter be most in need of?
A. A dictionary or phrase book. B. Advanced technical style ỉn writing.
C. Headphones and a booth. D. Shorthand skills and a notepad.
7. What is, a difference mentioned between a simultaneous interpreter and a consecutive interpreter?
A. The money they are paid.
B. The size of group with whom they work.
C. Their proficiency in the language.
D. The type of dictionary they use.
8. The word "converting" is closest in meaning to……………
A. understanding B. changing C. reading D. concluding
9.The phrase "the former" refers to……………
A. simultaneous interpreters B. the booth
C. consecutive interpreters D. the conference
10. The word "rather " is closest in meaning to……………
103

Downloaded by Nhung Nguy?n (nhungnth2004@gmail.com)


lOMoARcPSD|20277325

A. in general B. as a result C. in brief D. on the contrary


Passage 2 Read the passage and choose the best answer to each question.
Animation traditionally is done by hand-drawing or painting successive frames of an object, each
slightly different than the preceding frame. In computer animation, although the computer may be the one to
draw the different frames, in most cases the artist will draw the beginning and ending frames and the computer
will produce the drawings between the first and the last drawing. This is generally referred to as computer-
assisted animation, because the computer is more of a helper than an originator.
In full computer animation, complex mathematical formulas are used to produce the final sequence of
pictures. These formulas operate on extensive databases of numbers that define the objects in the pictures as
they exist in mathematical space. The database consists of endpoints, and color and intensity information.
Highly trained professionals are needed to produce such effects because animation that obtains high degrees
of realism involves computer techniques for three-dimensional transformation, shading, and curvatures.
High-tech computer animation for film involves very expensive computer systems along with special color
terminals or frame buffers. The frame buffer is nothing more than a giant image memory for viewing a single
frame. It temporarily holds the image for display on the screen.
A camera can be used to film directly from the computer’s display screen, but for the highest quality
images possible, expensive film recorders are used. The computer computes the positions and colors for the
figures in the picture, and sends this information to the recorder, which captures it on film. Sometimes,
however, the images are stored on a large magnetic disk before being sent to the recorder. Once this process is
completed, it is repeated for the next frame. When the entire sequence has been recorded on the film, the film
must be developed before the animation can be viewed. If the entire sequence does not seem right, the
motions must be corrected, recomputed, redisplayed, and rerecorded. This approach can be very expensive
and time consuming. Often, computer-animation companies first do motion tests with simple computer-
generated line drawings before setting their computers to the task of calculating the high-resolution, realistic-
looking images.
1. What aspect of computer animation does the passage mainly discuss?
A. The production process B. The equipment needed
C. The high cost D. The role of the artist
2. According to the passage, in computer-assisted animation the role of the computer is to draw the .............
A. first frame B. middle frames
C. last frame D. entire sequence of frames
3. The word "they” in the second paragraph refers to……………
A. formulas B. databases C. numbers D. objects
4. According to the passage, the frame buffers mentioned in the third paragraph are used to……………
A. add color to the images B. expose several frames at the same time
C. store individual images D. create hew frames
5. According to the passage, the positions and colours of the figures in high-tech animation are determined
by……………
A. drawing several versions
B. enlarging one frame at a lime
C. analyzing the sequence from different angles
D. using computer calculations
6. The word "captures" in the fourth paragraph is closest in meaning to……………
A. separates B. registers C. describes D. numbers
7. The word "Once" in the fourth paragraph is closest in meaning to…………….
A. before B. since C. after D. while
8. According to the passage, how do computer-animation companies often test motion?
A. They experiment with computer-generated line drawings
B. They hand-draw successive frames.
104

Downloaded by Nhung Nguy?n (nhungnth2004@gmail.com)


lOMoARcPSD|20277325

C. They calculate high-resolution images.


D. They develop extensive mathematical formulas.
9. The word "task" in the 4th paragraph is closest in meaning to ……………
A. possibility B. position C. time D. job
10. Which of the following statements is supported by the passage?,.
A. Computers have reduced the costs of animation.
B. In the future, traditional artists will no longer be needed. :
C. Artists are unable to produce drawings as high in quality as computer drawings.
D. Animation involves a wide range of technical and artistic skills.
V. GUIDED CLOZE TEST
Passage 1. Read the following passage and decide which answer (A, B, C or D) best fits each gap.
Last year Gladys Kalema became the Uganda Wildlife Service’s (1)……………(and only) vet after
graduating from the Royal Veterinary College in London. She was the first person to fill the (2)……………...
for 30 years and, at the age of 26, easily the youngest.
If Gladys did nothing else, caring for the world’s (3)……………population of 650 mountain gorillas
would (4) ……………justify her wages. Since the 1970s, gorillas have suffered severely from war and
poaching. Now for $150 each, tourists can be led through the fofest and corrie within five metres of a gorilla -
no closer, for (5) ……………of transmitting diseases such as measles and flu.
The gorillas here make a small but viable population. (6) ……………in the national parks, the usual
animals, elephants, rhinos, giraffes are either not there or present in (7) ……………numbers which are
dangerously out of (8) ……………with the creatures around them. If Uganda stays calm, wildlife may, in
time, return by itself. But Gladys believes the country cannot wait. Animals must be brought in to (9) ………
tourism and provide funds to expand her work.
Despite her difficulties, Gladys feels more useful and fulfilled than she would be aywhere else. “At
this moment, my friends from vet school are reading the best way to treat a cal or dog, and here am I planning
to translocate elephants. In my small (10) …………… I am part of the reconstruction and rehabilitation of my
country.
1. A. top B. main C. first D. chief
2. A. post B. occupation C. appointment D. career
3. A living B. surviving C. continuing D. lasting
4. A only B. alone C. just D. merely
5. A. risk B. danger C. fright D. fear
6. A. Somewhere B. Anywhere C. Nowhere D. Elsewhere
7. A. slight B. little C. insignificant D. minor
8. A. balance B. relation C. comparison D. equality
9. A. raise B. swell C. boost D. multiply
10. A. manner B. way C. method D. means
Passage 2. Read the following passage and decide which answer (A, B, C or D) best fits each gap.
A team of experts has arrived in Venice to save it from increasing incidences of flooding. A
controversial plan to construct a barrier with 79 gates, each weighing 300 tones, has been given permission to
(1) ……………ahead. Once constructed, this will be raised whenever a high tide (2) ……………to cover the
city.
Everyone has known for centuries that Venice is sinking further into the mud but floods are becoming
a regular nuisance. Rising sea levels have gradually (3) …………… the salt marches and mud banks that (4)
…………….. between the city and the Adriatic. Winter storms cause higher waves, which are (5) ……… the
walls of the old palace.
But there are fears about how the building of such a barrier might affect the Venice lagoon,particularly
the possibility that it could further (6) ……………the flushing of the city’s waterways by the tide, making the
famous foul-smelling canals even more stagnant.
105

Downloaded by Nhung Nguy?n (nhungnth2004@gmail.com)


lOMoARcPSD|20277325

To avoid making a bad situation worse, the experts have been (7) ……………to analyse tidal flows,
marine plants and then suggest ways to prevent the city becoming the first high-profile (8) ……………of
global warming and rising sea levels.
But with global warming expected to add at least another half meter to the sea level this century, the situation
is bound to (9) ……………A spokesman for the team said, “We cannot hope to stop Venice submerging
eventually but we can slow the whole (10) ............... down and so enjoy the city for a while longer.”
1. A. be B. proceed C. go D. advance
2. A. threatens B. endangers C. risks D. jeopardizes
3. A. eroded B. worn C. corrupted D. broken
4. A. faced B. occurred C. featured D. stood
5. A. offending B. assaulting C. crashing D. opposing
6. A. delay B. direct C. restrict D. impose
7. A. asked over B. called up C. taken on D. brought in
8. A. victim B. target C. sufferer D. subject
9. A. denote B. deny C. deteriorate D. distract
10. A. damage B. process C. water D. event
PART B: WRITTEN TEST
I. OPEN CLOZE TEST
Passage 1. Read the passage and fill in the blank with a suitable word.
If you put a group of people who don’t know (1) …………… other in a room together and asked them
to pair up, they will naturally gravitate towards others of similar family (2) ……………social class and
upbringing. We are all looking for something familiar (3) ……………we may not be aware of exactly what
it is. Facial attractiveness has a big (4) ……………on our choice of partners, too. People tend to seek out and
form long-lasting relationships with others of a similar level of attractiveness. Several studies have confirmed
this. Researchers (5) ……………a selection of wedding photos and cut them up to separate the bride and
groom. They then asked people to rate how attractive each person’s face was. When the researchers put the
photos back into their (6) …………… pairs, they found that most of the couples had been rated at similar
levels. Not only (7) ……………we rate others, but each of us carries a rough estimate in our heads of how
facially attractive we might be. We realized subconsciously that if we approach someone who is significantly
higher up the scale than we are, we run the (8) ……………of being rejected.
But (9) ……………the explanation for how and why we fall in love, one thing is clear. Nature has
made the whole process as blissful and addictive as possible (10) ……………the purpose of bringing and
keeping couples together.
Passage 2. Read the passage and find one suitable word for each gap.
A hundred years ago, the principle (1) ……………of communication for individuals and for
businesses was by letter and telegram, but nowadays telephones are used daily in private houses, offices and
factories. This is certainly the (2) ……………widespread and convenient way for people to communicate
with one another.
With the telephone, business deals can be arranged and competed rapidly: In addition, financial centers
and stock exchanges, by constant (3) ……………of the telephone system, can be assured of getting the very
latest, up-to-the-minute information about the ever changing currency and share values. This type of
information is essential (4) ……………economic stability worldwide.
The telephone network also (5) ……………a direct line into the homes of friends and relatives. For
old people and for those (6) …………… people living on their own, the telephone is a friend. At present,
families are often scattered, not only throughout the country but also throughout the world. Therefore, a few
minutes’ (7) ……………with a loved one who may be thousands of miles away is a source of great comfort
to everyone.
Of course very often the telephone can be a (8) ……………Some will say that it intrudes too much
(9) ……………people’s privacy, always ringing at the most inconvenient times. Moreover, obscene callers
106

Downloaded by Nhung Nguy?n (nhungnth2004@gmail.com)


lOMoARcPSD|20277325

can make life a misery for some of the wbenble members of society, particularly for elderly women living
alone.
On the whole, I think that the changes (10) ……………about by the introduction of the telephone have
been of great benefit to everyone. The telephone has helped to bring people closer together in a difficult world
and has made life, for many, much easier much more enjoyable.
II. WORD FORM
A. Complete the sentences with the correct form of the words given.
1. The world of cyberspace is a……………..one where people from around the globe can communicate
almost instantaneously with one another. BORDER
2. Suffering from visual…………….. since young, Jane sees double images of ewything. IMPAIR
3. …………….. is useful when companies want to know the profiles of the employees they are going to
employ. GRAPH
4. The candidate delivered a……………..speech before the election. NATION
5. How……………..the book is! It is so interesting that I can hardly stop reading it. PUT
6. Thanks to the……………..policy, more and more forests have been formed on locations that used to be
treeless. FOREST
7. She is so ……………..that she won’t let anything stand in the way of her ambition. MIND
8. Heavy rain and excessive use have the soil…………….. POVERTY
9. This charitable organization has been sending …………….. aid to areas affected by droughts. HUMAN
10. The electricity was cut off during the storm and we had a ……………..for several hours. BLACK
B. Read the passage and fill in each blank with the correct form of one of the words given in the box.
define reason confuse think doubt
conceive explain logic sense assume

Students learning English as a second language are sometimes given a word by their teacher and asked
to give an (1) …………… as to what that word means; in other words, to provide a (2) ……………The (3)
…………… isthat if you know a word, you can define it. (4) .................., that might make sense, but in reality
it is not always (5) .................. to assume that. They are words and phrases that even native speakers use in
conversation without much (6) .................. which can lead to (7) …………… when you ask a native speaker
to define them. Take the (8) .................. of “zeitgeist”, for example, which has entered English from German,
It’s (9) ……………much easier to use thán it is to define. With a word like “zeitgeist”, it may be more (10)
……………… to test the student’s understanding in ways other than asking thèm to define it.
III. ERROR IDENTIFICATION
The passage below contains 10 errors. Underline the errors and correct them.
The market for tourism in remote areas is booming as ever before. Countries all over the world are
active promoting their “wilderness” regions - such as mountains, Arctic lands, deserts, small islands and
wetlands - to highly spending tourists. The attraction of these areas is obvious: by definition, wilderness
tourism requires few or no initial investment. But that does not mean that there is no cost. As the 1992 UN
Conference on Environment and Development recognized, these regions are fragile (i.e. highly vulnerable of
abnormal pressures) not just in terms of the culture of their inhabitant. The three most significant types of
fragile environment in these respects are deserts, mountains and Arctic areas. An important character is their
marked seasonality. Consequently, most human actions, including tourism, are limited to clearly defined parts
of the year.
Tourists are draw to these regions by their national beauty and the unique culture of their people. And
poor governments in these areas have welcomed the “adventure tourists”, grateful for the currency they bring
to them. For years, tourism is the prime source of foreign exchange in Nepal. It is also a key element in the
economics of Arctic zones.
IV. SENTENCE TRANSFORMATION
Rewrite the following sentences, beginning with the words/ phrases given or using the given words in
107

Downloaded by Nhung Nguy?n (nhungnth2004@gmail.com)


lOMoARcPSD|20277325

brackets.
1. The trip was so amazing that we will never forget.
→It’s too …………………………………………………………………………………………….
2. If anyone succeeds in solving the problem, it will probably be him.
→He is the most ……………………………………………………………………………………..
3. He threatened the officers with violence.
→He made ……………………………………………………………………………………………
4. I really think she should stop ignoring me in meetings. SHOULDER
→It’s about time ……………………………………………………………………………in meetings.
5. In my opinion, it was an absolute miracle that they survived the accident. SHORT
→The fact that……………………………………………………………………………,in my opinion.
6. She disapproved of my suggestion. DIM
→……………………………………………………………………………………………………….
7. Payment will be made when the order is received. RECEIPT
→……………………………………………………………………………………………………….
8. The villagers prepared themselves to withstand the coming storm. BRACED
→……………………………………………………………………………………………………….
9. I hardly think she will agree to giving you a pay rise. LIKELIHOOD
→……………………………………………………………………………………………………….
10. Organic vegetables are said to be very healthy. WONDERS
→……………………………………………………………………………………………………….

TRƯỜNG THPT CHUYÊN BẾN TRE - BẾN TRE


MULTIPLE CHOICE QUESTIONS:
Câu hỏi 1
PHONOLOGY
A. Pick out the word whose underlined part is pronounced differently from that of the rest, and then
circle the letter (A, B, C or D) which corresponds to your answer.
1. A. cheap B. chicken C. chef D. chief
2. A. program B. process C. promise D. progress
3 A. replace B. purchase C. surface D. palace
4. A. chorus B. cherish C. chaos D. character
5. A. measure B. dreadful C. treasure D. breathe
B. Pick out the word which is stressed differently from the rest.
6. A. necessity B. dignity C. quantity D. poverty
7. A. development B. contentment C. investment D. instrument
8. A. museum B. cathedral C. skyscraper D. apartment
9. A. astronaut B. gardener C. policeman D. officer
10. A. element B. elephant C. elevator D. elector
Câu hỏi 2:
l. We’d better insure the house…………..fire.
A. in case of B. but for C. as a result of D. in addition
2. …………..wants to enter the building should apply for a permit.
A. Who B. Those who C. One D. Whoever
3. Next month while I…………..my report you…………..yourself at Pattaya.
A. will arrive/ will enjoy B. am writing/ enjoy
C. am writing/ will be enjoying D. have writen/ will have enjoyed
4. The reason for the traffic accident was…………..one of the drivers had lost control of his car.
A.why B.that C.when D. how
108

Downloaded by Nhung Nguy?n (nhungnth2004@gmail.com)


lOMoARcPSD|20277325

5. Professor Lock wood reccom ended that Michael…………..in chemistry.


A. not major B. wouldn’t major
C. not to major D. isn’t majoring
6. “Why did you go back upstairs?” “I thought I had left the windows…………...”
A. opened B. open C. being open D. to be open
7. ………….., one tin will last for at least six weeks.
A. Used economical B. Using economical
C. Used economically D. Using economically
8. I’ll be kind to her…………..she decide to leave me.
A. in case B. whereas C. so as not D. lest
9. When Susan was told the whole story, she…………..in the film.
A. ceased interest B. ceased to interest
C. ceased being interested D. ceased interested
10. Being a Puritan, John denies himself all the luxuries of life.
This sentence tells us that John…………...
A. denies that he loves the luxuries of life because he was a Puritan .
B. denies that he is a Puritan because he lives a life of luxury.
C. lives a life of luxury despite being a Puritan.
D. rejects all the luxuries of life because he is a Puritan.
Câu hỏi 3:
PREPOSITIONS AND PHRASAL VERBS
Circle the best option for each of the following sentence:
1. British Leyland is aiming to push…………its share of UK car sales to 25 % over the next two years.
A. up B. out C. on D. through
2. The factory paid ………… nearly a million pounds to their employees who were injured in the explosion.
A. off B. out C. back D. in
3. Before they opened the new factory, a lot of the young people round here were ………… the dole.
A. in B. up C. at D. on
4. Mr Horrid was a terrible teacher and obviously not…………for teaching.
A. cut in B. cut on C. cut up D. cut out
5. I have always ………… my older brother for his courage and honesty.
A. taken up B. taken after C. looked up to D. looked after
6. I can't cancel my arrangements ………… such short notice.
A. on B. at C. with D. by
7. All the police's efforts to find him were ………… vain.
A. in B. for C. on D. at
8. We were …………tenterhooks all morning waiting for the telephone to ring.
A. about B. at C. in D. on
9. The farmhouse we stayed in was completely ………… the beaten track.
A. off B. for C. without D. on
10. She's gone and this time it's…………good.
A. at B. for C. on D. with
Câu hỏi 4:
Read the following passage and choose the correct answer to each of the questions that follow by
circling A, B, C, or D.
Music can bring us to tears or to our feet, drive us into battle or lull us to sleep. Music is indeed
remarkable in its power over all humankind, and perhaps for that very reason, no human culture on
earth has ever lived without it. From discoveries made in France and
Line Slovenia even Neanderthal man, as long as 53,000 years ago, had
109

Downloaded by Nhung Nguy?n (nhungnth2004@gmail.com)


lOMoARcPSD|20277325

(5) developed surprisingly sophisticated, sweet-sounding flutes carved from animal bones. It is perhaps
then, no accident that music should strike such a chord with the limbic system - an ancient part of our
brain, evolutionarily speaking, and one that we share with much of
(10) the animal kingdom. Some researchers even propose that music came into this world long before the
human race ever did. For example, the fact that whale and human music have so much in common
even though our evolutionary paths have not intersected for nearly 60 million years suggests that
(15) music may predate humans. They assert that rather than being the inventors of music, we are
latecomers to the musical scene.
Humpback whale composers employ many of the same tricks that human songwriters do. In
addition to using similar rhythms, humpbacks keep musical phrases to a few seconds, creating themes
(20) out of several phrases before singing the next one. Whale songs in general are no longer than
symphony movements, perhaps because they have a similar attention span. Even though they can sing
over a range of seven Octaves, the whales typically sing in key, spreading adjacent notes no farther
apart than a scale. They mix percussive and
(25) pure tones in pretty much the same ratios as human composers - and follow their ABA form, in which
ia theme is, presented, elaborated on and then revisited in a slightly modified form. Perhaps most
amazing, humpback whale songs include repeating refrains that rhyme. It has been suggested that
whales might use rhymes for exactly the same reasons that we do: as devices to help them remember.
Whale songs can also be rather catchy. When a few humpbacks from the Indian Ocean strayed into the
Pacific, some of the whales they met there quickly changed their tunes - singing the new whales’ songs
within three short years. Some scientists are even tempted to speculate that a universal music awaits
discovery.
1. Why did the author write the passage?
A. To describe the music for some animals, including humans
B. To illustrate the importance of music to whales
C. To show that music is not a human or even modern invention
D. To suggest that music is independent of life forms that use it
2. The word “sophisticated” in line 6 is closest in meaning to
A. complex B. intricate C. well-developed D. entangled
3. The word “one” in line 9 can be replaced by
A. the chord B. the left brain C. the right brain D. the limbic system
4. According to the passage, which of the following is true of humpback whales?
A. their tunes are distinctively different from human tunes
B. they can sing over a range of seven octaves
C. they do not use rhyme, unlike humans
D. whale songs of ạ particular group cannot be learned by other whales?
5. The word “they” in line 22 refers to
A. human composers B. whale songs C. octaves D. whales
6. Which of the following is NOT true about humpback whale music?
A. It uses similar patterns to human songs
B. It’s comparative in length to symphony movements
C. It’s easy to learn by other whales
D. It’s in a form of creating a theme, elaborating and revisiting in rhyming refrains
7. The word “refrains” in line 28 is closest in meaning to
A. tunes B. notes C. musical phrases D. sounds
8. Which of the following can be inferred from the passage?
A. The earliest human beings came from France and Slovenia
B. Music helped to shape the whale brain
C. Humpback whales imitate the way human composers do in creating their own music
110

Downloaded by Nhung Nguy?n (nhungnth2004@gmail.com)


lOMoARcPSD|20277325

D. The research of musical brain will lead to a discovery of a universal music


9. Where in the passage does the author first mention whales?
A. Lines 5-9 B. Lines 10-14
C. Lines 15-19D. Lines 20-24
10. The word ‘their’ in line 33 refers to
A. Indian Ocean humpbacks B. Pacific Ocean humpbacks
C. ‘all whales D. whale songs
Câu hỏi 5:
Read the passage below carefully, and complete it with the appropriate words provided:
VIETNAM TO HOLD TRADITIONAL KITCHEN GOD CEREMONY IN HANOI
Last updated: Friday, February 01, 2013 04:00:00
Vietnam will hold its first-ever large-scale Ong Tao (the (1 )…………….ceremony in the capital city
of Hanoi on Febfoary 3, the 23rd day of the last lunar month, one week before Tet, or Lunar New Year.
The festival has been organized by the Ministry of Culture, Sports and Tourism and Hanoi (2)
……………., The Thao & Van Hoa newspaper reported February 1.
The ceremony will begin early in the morning in the famous (3) ……………. Village of Bat Trang
with a parade including a 1.2-mreter tall, ceramic Kitchen God and a 3.5-meter-long carp crafted by, local
artists, along with 12 trays of fruits and traditional cakes, through historic areas gf Hanoi, which will (4)
…………….,up at the Centre for Art and Cultural Exhibition on Hoa Lu Street.
There, a live carp will be placed in a pond, symbolizing the ascension of the Kitchen God to the
Heavenly (5) …………….
Then the procession will move on to the Lý Thai To statue, where the 12 trays of food will be placed
on its (6)……………. , before visiting the Ngoc Son temple, and Thang Long imperial citadel
In traditional Vietnamese culture, oh every 23rd day of the last month in the Lunar Calendar, people
from every household bid farewell to Ong Tao, thanking him for watching over their kitchen for another year.
Then, the Kitchen God is believed to visit Ngoc Hoang (the (7) …………….in heaven.
The release of the strong-swimming carp into local ponds, rivers and lakes, is believed to transport the
Kitchen God to heaven. (8) …………….delivering the God, the carp transforms into a dragon.
It is said that the God will then report to the Jade Emperor on the life of homeowners and pray for
luck, happiness and (9) …………….for all members of the house in the coming year.
The Kitchen God returns to Earth on New Year’s Eve (February 9 this year) and (10) ………his duties
as the caretaker of Vietnamese kitchens.
1. A. Kitchen God B. God Kitchen C. Mr. Apple D. Sir Apple
2. A. governments B. governors C. authorities D. organizers
3. A. glass B. brick C. mud D. pottery
4. A. go B. come C. wind D. sit
5 A. House B. Palace C. Villa D. Building
6. A. table B. desk C. altar D. statue
7. A. God King B. King God C. Jade Emperor D. Heaven Emperor
8. A. Before B. After C. While D. During
9. A. longevity B. money C. health D. prosperity
10. A. resume B.take C. consider D. make
Câu hỏi 6:
Read the passage below carefully, and complete it with the appropriate words provided:
INFLATION HOLDS STEADY DESPITE TÉT DEMAND
Last updated at 13h45, day 01 March, 2013
The Hanoitimes - The consumer price index (CPI) in February rose by only 1.32% against the previous
month, lower than (1)…………. thanks to abundant supplies of essential commodities, the General Statistics
Office (GSO) reported.
111

Downloaded by Nhung Nguy?n (nhungnth2004@gmail.com)


lOMoARcPSD|20277325

Compared with the same period last year, CPI was up 7.02%, the office said.
Director of the GjSO’s Price Department Nguyen Due Thang said that a sufficient (2) ………… of
essential commodities, especially in big cities, during the weeklong Lunar New Year holiday in February
halted the increasing price trend that usually occurred during the holidays.
Monthly CPI often increases by more than 2% at Tet.
In addition, price stabilization programs also helped to (3) …………prices.
The slow place of CPI was also attributed to many businesses cutting Tet bonuses by 10% compared to
2012. The national economy’s sluggish recovery also had an unpact on spending during Tet.
Ten out of the 11 groups of commodities saw price hikes (4) …………from 0,03% to 2.28%. The
highest rise was seen in the restaurants and catering services group, while education experienced the. lowest
increase.
Food prices this month witnessed a modest increase of 0.37%, mainly because of a 5.85% fail in the
price of rice in the Mekong Delta
Two groups of commodities thát 'also saw price increases were beverage and transportation due to (5)
…………demand during the Lunar New Year.
During February, gold prices dropped by 0.33% while the US dollar price saw a light increase of
0.03%.
The CPI in March and the following months are expected to (6) …………following an increase in oil
and gas prices.
If retail petrol prices were adjusted upwards by 1,000 VND per litre in late February or early March as
proposed by petrol businesses, March’s CPI would rise an additional 0.1%, the office estimated.
Though there has been no (7) …………information about a price hike, many petrol stations have used
a variety of excuses to stop services in the hopes of making higher profits in the near future.
One of the most popular excuses used in Hanoi’s suburbs and neighbouring provinces was “power (8)
…………” and “out of fuel”. Many of those still in operation only have one employee on duty in order to
limit the amount of fuel sold.
Previously, petroleum companies, including Vietnam National Petroleum Group (Petrolimex), Dong
Thap Petroleum Trading Company (Petimex), Sai Gon Petro Company and PetroVietnam Oil Corporation
(PVOIL) asked the Ministry of Finance for a price adjustment.
Even though their request did not ask specifically for an increase or decrease in price, Tran Ngoc Nam,
deputy head of Petrolimex, said they were incurring (9) …………
Nguyen Tien Thoa, head of the Price Management Department at the Ministry of Finance, said
enterprises were under pressure because global demand had surged, leading to a 2 - 3% increase in the world
price. The supply has also decreased recently because many plants have temporarily closed for maintenance.
A number of international organizations have also (10) ………… that fuel prices
will continue to increase.
1. A. suggested B. expected C. regarded D. considered
2. A. stock B. control C. supply D. ability
3. A. warm B. stop C. lift D/cóól' 1
4. A. ranging B. ranged C. marking D. marked
5. A. low B. high C. small D. big
6. A. raise B. rise C. drop D. fall
7. A. major B. main C. official D. chief
8. A. shortage B. errors C. mistakes, D. failure
9. A. losses B. debts C. punishment D. sadness
10. A. said B. thought C. believed D. forecast ...

H. PHẦN TỰ LUẬN
Câu hỏi 1: Read the passage below carefully, and complete it with the missing words:
112

Downloaded by Nhung Nguy?n (nhungnth2004@gmail.com)


lOMoARcPSD|20277325

VTV VOWS TO PREVENT K+ FROM HOLDING EXCLUSIVE EPL RIGHTS


Last updated; Thu, February 28, 2013,2:25 PM (GMT+0700)
Vietnam Television (VTV) yesterday agreed with other local TV stations to find a way to prevent K+
from holding exclusive rights to the English Premier League for the 2013-16 seasons.
The rights to air live (1)…………..in Vietnam now belong to, France-based Canal+, who is belịèved to
transfer them to K+. K+ is a joint venture in Vietnam with Canal+ holding a 49 percent stake, while VTV
owns the remaining 51 percent.
At a meeting yesterday in Hanoi, VTV and local TV stations came to an agreement that VTV will
suggest that Canal+ share the rights with other stations, and VTV will resort to its control power at K+ to stop
it from airing (2) ………….. matches.
Canal+ bought the rights on its own
Nguyen Thanh Luong, deputy general director of VTV, said at the meeting that VTV knew nothing
about the purchase of the (3) …………..by Canal+, and the foreign firm told VTV about the deal after it was
completed.
Earlier, VTV had asked Canal+ to buy the rights in Vietnam directly from the EPL organization board,
but Canal+ had lost the deal to IMG Media, Luong added. After IMG media got the rights from EPL
organizers, Canal+ (4) ………….. them back from IMG.
So far, Canal+ has not officially confirmed if it will transfer the exclusive rights to K+. Without K+,
other TV (5)………….. in Vietnam would be the targets of Canal+ sothat it would be able to air the matches
live.
At the meeting Vietnam’s TV stations agreed that VTV and the Association of Pay- TVs (VNPayTV)
will stand on behalf of them to officially send two dispatches to Canal+ and IMG. One will ask Canal+ to
share the rights, and the other for IMG will (6) ………….. that the negotiation board is the unique legal
partner in discussing the purchase of the EPL rights in Vietnam.
Reactions from Vietnam
Local TV stations yesterday expressed satisfaction over the outcome of the meeting and the
willingness of VTV to solve the issue.
VTC even demanded that VTV to refuse the transfer of the EPL rights from Canal+ to K+ based on the
(7) ………….. that VTV, a majority stake holder, was not informed of the deal.
VTC also urged VTV to stop K+ from airing live EPL matches based on the Law of Journalism in
Vietnam.
“VTV is the unique partner ip controlling the content of (8) …………..on K+,” said the VTC
representative at the meeting.
Le Dinh Cuong, deputy chairman of VNPayTV, said, “Consumers hold the ultimate decision in using
or boycotting K+.”
“In case Canal+ refuses to share the rights with other TV stations, VNPayTV will stand up to call on
all other TV stations in Vietnam to refuse totally the EPL rights (9) …………..of the price,” Cuong stressed.
Jacques-Aymar de Roquefeuil, deputy general director of K+, said that the sharing of the EPL rights depends
on the (10) …………..signed between Canal+ and IMG. Not all matches are sold exclusively, as there are still
250 non-exclusive matches available for all TV stations, he added.
Câu hỏi 2:
Read the passage below carefully, and complete it with the missing words:
PARTY LEADER CONGRATULATE VIETNAMESE DOCTORS
Last updated at 13h4ỉ, day 01 March, 2013
The Hanoitimes - Party General Secretary Nguyen Phu Trong visited and congratulated staff at K
hospital and the National (1) …………..of Pediatrics on February 26, on the occasion of Vietnamese Doctors’
Day (February).
During the visits, General Secretary Trong (2) …………..that the Vietnamese Party and State regard
public healthcare as an important strategic task. Doctors’ Day is a chance for all people to (3) …………..and
113

Downloaded by Nhung Nguy?n (nhungnth2004@gmail.com)


lOMoARcPSD|20277325

express their gratitude to doctors.


He lauded generations of medical staff for overcoming difficulties, providing health check-ups and (4)
…………..to people, and engaging in scientific research and training.
General Secretar Trong said he hopes Vietnamese (5) …………..will follow President Ho Chi Minh’s
teachings and strive to build a modern medical profession, meeting the aspirations of the people. He also (6)
………….. gifts to patients at both hospitals.
As a national leading cancer hospital, the 1,000-bed K hospital is always filled to capacity as the
number of (7) …………..rises by 5-10% per annum.
Bui Dieu, Director of K hospital, said with a population of 88 million, (8) …………..sees 170,000 -
200,000 cancer sufferers, and over 1 million tumour patients every year, posing challenges for K hospital and
the (9) …………... sector.
The National Hospital of Pediatrics boasts 1,250 beds and over 1,600 medical staff. Last year, it
offered health check-ups to 700,000 children and treatment to 60,000 using advanced (10) …………...
Câu hỏi 3:
Write the correct FORM of each bracketed word in the numbered space.
A live broadcast of any public event, such as a space flight or sporting occasion, is almost (1) ………
(VARIABLE) accompanied by the thoughts of a (2) ………….. (COMMENT). This may be on television,
along with the relevant pictures, alternatively on radio. The technique involved (3) ………….. (DIFFERENT)
between two media, with radio broadcasters needing to be more explicit and (4) ………….. (DESCRIBE)
because of the absence of visual information. TV commentators do not need to paint a picture for their
audience; instead, their various (5) …………..(OBSERVE) should add to the images that are already there.
There will sometimes be silences and pauses in a TV commentary, although these are becoming (6)………….
(INCREASE) rare. Both types of commentators should try to be informative, but should advoid sounding (7)
………….. (OPINION). In sports commentaries, fairness and (8) ………….. (IMPART) to both types is vital,
but spontaneity and enthusiasm are valued by those watching or listening. Sport commentators usually
broadcast live in an essentially unscripted way, although they may refer to previously prepared materials such
as sports statistics. Because of the (9) ………….. (PREDICT) nature of live events, thorough preparation in
advance is vital. The internet has helped enormously with this aspect of the job. Anyone interested in
becoming a commentator should have excellent (10) ………….. (ORGANISE) skills, the willingness to work
irregular hours, and a strong voice.
Câu hỏi 4:
Fill each blank with the correct form of the word given in capital letters.
1. It is very rude to interrupt someone in ……………..(SENTENCE).
2. The new film is …………….. (EXCEPTION) good.
3. There's been yet another …………….. (BREAK) of cholera in Dehli.
4. It may be……………..(PRODUCE) to force them into making a decision, and if you upset them they’re
quite likely to ……………..(ACT).
5. Heavy rain and excessive use have …………….. (POOR) the soil.
6. To his great loss his wife died in …………….. (BIRTH) when he was only twenty-four years old.
7. I don’t think Tom’s getting too much sleep lately. His eyes are terribly …………….. (BLOOD)
8. Leisure habits won’t change much in the……………..(SEE) future.
9. Although the splitting of the atom was one of the greatest scientific……………..(BREAK) of this century,
there are many people who wish it had never happened.
Câu hỏi 5:
ERROR IDENTIFICATION
The passage below contains 10 mistakes. Underline the mistakes and write their correct forms in the
space provided in the column on the right. (0) has been done as an example .
The development and widespread use of computer technology and an internet have 0. an → the
transformed how we communicate, how business is conducting, how information is 1:………….
114

Downloaded by Nhung Nguy?n (nhungnth2004@gmail.com)


lOMoARcPSD|20277325

dispersed, and how society is organized. Before to approximately 1980, in-deep 2:………….
information about any one subject matter was attained through laborious research 3:………….
involved countless visits to libraries and via repeated interviews with persons of known 4:………….
reputation and reputable expertise. Now, a great plenty of information is available at the 5:………….
click of a mouse button, all attainable from within the confines of one’s own home or 6:………….
from the use of a computer in an office. Previous labor-intensive support jobs, such as 7:………….
loading and unpacking of trucks, luggage handling at airports, and food manufacturing, 8:………….
once performed by a large middle-class workforce, are now performed routine by robots 9:………….
which are monitored by computercontrolling systems. 10:………….
Our lives have been simplified by the advent of computer and internet technologies, 11:………….
but likewise these benefits which have been ushered in by the technology revolution have 12:………….
had an averse effect on the core of our interpersonal-relationships. Merely communication 13:………….
is no longer via postal mail or face-to-face contact, but rather via electronic email, 14:………….
personal internet message boards, and by virtue of hand-held personal electronic 15:………….
assistance. Although computer technology has brought us to within a mouse-click of any 16:………….
sought-after piece of information, this technology boom has sequestered us to the 17:………….
confines of our computer desks and homes and have removed us away from those 18:………….
ưaditional settings . 19:………….
20:………….
21:………….
22:………….

Câu hỏi 6:
Rewrite the sentences with the given words or beginning in such a way that their meanings remain
unchanged.
1. It is certain that he will compensate you for the damage he has done.
→He is bound……………………………………………………………………………………
2. I don’t mind staying in on a Saturday night if I have good company.
→I’m not averse……………………………………………………………………………………
3. Helen won’t be happy till she gets a full refund.
→Nothing………………………………………………………………………………………….
4. It wasn’t very polite of you not to notify them about the change of plansi
→You could……………………………………………………………………………………….
5. We were elated by the bird of our first grandchild.
→We……………………………………………………………………………………………….
6. Being her only niece, Ann is very precious to her. APPLE
→…………………………………………………………………………………………………..
7. A holiday in the UK is very expensive these days. ARM
→…………………………………………………………………………………………………..
8. Could you come on Saturday? SUIT
→How ……………………………………………………………………………………………..
9. There’s a possibility that you may be able to get a grant. ELIGIBLE
→We are pleased…………………………………………………………………………………..
10. If you wait for a good opportunity until the market improves, you’ll get a bettei return on your investment.
BIDE
→If you ……………………………………………………………………………………………
ĐÁP ÁN

115

Downloaded by Nhung Nguy?n (nhungnth2004@gmail.com)


lOMoARcPSD|20277325

ĐỀ THI OLYMPIC TRUYỀN THỐNG NĂM 2013


ĐỀ THI CHÍNH THỨC
A. MULTIPLE CHOICE
1.D 2. D 3. C 4. D 5. D 6. B 7. D 8. C 9. C 10.B
11.D 12. C 13.D 14.A 15.C 16.C 17.B 18.B 19.B 20.D
21.A 22.D 23.B 24.C 25.A 26.C 27.D 28.B 29.A 30.B
31.C 32.A 33.C 34.A 35.A 36.A 37.D 38.C 39.B 40.C
41.A 42.C 43.C 44.B 45.D 46.A 47.D 48.B 49.A 50.A
51.A 42.C 53.B 54.B 55.C 56.D 57.A 58.C 59.C 60.A
61.D 62.C 63.B 64.A 65.B 66.D 67.A 68.B 69.D 70.A
71.B 72.A 73.B 74.B 75.A 76.A 77.B 78.D 79.C 80.B
B. WRITTEN TEST
I. CLOZE TEST
Open cloze 1
l. name 2. little 3. that 4. open/deep 5. that
6. height 7. heavy 8. well 9. occurs 10. warning
Open cloze 2
11. Natural 12. species 13. speed 14. way 15. make
16. prey 17. presence 18. slower 19. immobile 20. particular
II. WORD FORMATION
part 1:
1. underhand/underhanded 2. multicolored 3.miniature 4. allegations
5. dehumanizing 6.intermarriage 7. bloodshot 8. paranormal
9. Nationalism 10. dethroned
part 2:
11. countless 12. misadventures 13. sidekick 14. left 15. unattended
16. Remarkably 17. unlikely 18. Related 19. ecstatically 20. nutritious
III. ERROR CORRECTION
Line Error Correction Line Error Correction
3 for to 8 color-code color - coded
3 other others/ another 11 of by (triggering)
4 pollination pollinator 14 response responsiveness
6 (picked) out (picked) up 14 it’s it was
7 remaining maintaining 16 with to

IV. SENTENCE TRANSFORMATION


1. This liver condition is common among those who drink a lot.
→ This liver condition is common among heavy drinkers.
2. “I think the whole idea is ridiculous,” he said.
→ He dismissed the whole idea as being ridiculous.
3. He threatened the officers with violence.
→ He made threats of violence against the officers.
4. It shouldn’t have surprised me that my children didn’t like the new, cheaper ice-cream.
→ I might have known my children would not like the new, cheaper ice-cream.
5. Her lateness made the boss angry. (BANANAS)
→ It was because she was late that the boss went bananas.
6. She wore headphones in order not to disturb anyone.
→ She wore headphones lest she (should) disturb anyone.
7. She is not very good at arranging flowers. (FLAIR)
116

Downloaded by Nhung Nguy?n (nhungnth2004@gmail.com)


lOMoARcPSD|20277325

→ She doesn 9t have much (of a) flair for arranging flowers.


8- Bill changed his ways when he came out of prison. (LEAF)
→ Bill has turned over a new leaf since he came out ofprison.
9.You must not, I repeat not, open this box until Christmas Day. (TO)
→ Under no circumstances is this box to be opened before Christmas Day.
10. Her early success has made her very conceited. (HEAD)
→ Her early success has gone to her head.

TRƯỜNG THPT MẠC ĐĨNH CHI - TP. HỒ CHÍ MINH


A. MULTIPLE CHOICE QUESTIONS
I. PHONOLOGY
l.B 2.D 3. A 4.B 5.C 6.B 7.C 8. A 9.D 10. B
II. WORD CHOICE
l.A 2.C 3.C 4.C 5.D 6.B 7. B 8.D 9. A 10. B
III. STRUCTURES AND GRAMMAR
l.A 2. D 3. D 4. B 5.D 6. D 7. B 8.C 9. A 10. D
IV. PREPOSITIONS AND PHRASAL VERBS
l.B 2. A 3.B 4. D 5.B 6. A 7.D 8. A 9. C 10. D
V. READING COMPREHENSION
l.D 2. D 3. A 4. A 5. A 6. C 7.C 8.D 9. A 10. B
11.B 12. D 13. B 14. C 15. C 16. C 17. D 18. B 19. A 20. A
VI. CLOZE TESTS
l.D 2. D 3. B 4.C 5.C 6.D 7.B 8.C 9.B 10. C
11.C 12. B 13. B 14. C 15. D 16. D 17. B 18. B 19. C 20. C
B. WRITTEN TEST I. OPEN CLOZE TESTS
1. fully 2. result 3. birds 4. have/seen/appear 5. based
6. determine 7. use 8. makes 9. bodies 10. able
11. one 12. apart 13. find 14. where 15. relation
16. carried 17. nearer/closer 18. doubt 19. mistake 20. solution
II. WORD FORMS
1 dedication 2 Alarmingly 3 heartwarming 4 destructiveness
5 disqualification 6 answerable . 7 animated 8 effortlessly
9 coolheaded 10 sportsmanlike 11 infertile 12 swollen
13 sleepless 14 Initially 15 unhelpful 6 surgeon
17 retrieval 18 specialist 19 treatment 20 successfully
III. ERROR IDENTIFICATION
1. time enough → enough time 2. seem slipping away → seem to slip away
3. how to do → what to do 4. of trivial → by trivial
5. being accomplished → to be accomplished 6. checked regularly → are checked regularly
7. Not only this is → not only is this 8. it serves giving → it serves to give
9. People not keep lists → People who do not keep lists
10. contribute with → contribute to
IV. SENTENCE TRANSFORMATION
1. Mrs. Tipper greatly regretted not being able to celebrate the New Year with her husband.
2. It was only when they arrived at the venue that the journalists heard about the changes to the
wedding plans.
3. The more qualifications you are able to amass, the more success you will have in the academic field.
4. Hardly had we finished dinner when Mrs Jones arrived on the doorstep.
5. Can you cast/shed/throw any light on how Jack made enough money to buy that new sports car?
117

Downloaded by Nhung Nguy?n (nhungnth2004@gmail.com)


lOMoARcPSD|20277325

6. Kale has finally come to terms with the fact that their friendship is over.
7. Under no circumstances should this door be locked when the building is open to the public.
8. As far as Colin is concerned, he hasn’t done anything he should apologise for.
9. But for the ingenuity of his brother’s plan, the stranded climber would never have been rescued.
10. I didn't have any clue (about) how to send a fax when I started work.

TRƯỜNG THPT NGUYỄN THƯỢNG HIỀN -TP. HỒ CHÍ MINH


A MULTIPLE CHOICE QUESTIONS
I. PHONOLOGY
Part 1:1. A 2.C 3.C 4.C 5.A
Part 2:1. D 2.A 3.B 4.B 5.D
II. VOCABULARY AND STRUCTURE
1.B 2. A 3.C 4.D 5.A 6.D 7.C 8.A 9.C 10.A
11.A 12.D 13.B 14.D 15.D 16.D 17.C 18.B 19.C 20.A
III. PREPOSITION
1.A 2. A 3.C 4.A 5.B 6.A 7.C 8.A 9.A 10.C
IV. READING
Passage 1
1.B 2.C 3.B 4.B 5.D 6.B 7.D 8.C 9.D 10.C
Passage 2
1. Paragraph B III 2. Paragraph C VII 3. Paragraph. D V
4. Paragraph E I 5. Paragraph F IX 6. Paragraph G II
7. Paragraph H X 8 . YES 9. NOT GIVEN 10. NO
V. GUIDED CLOZE TEST
Passage 1:
1.A 2.C 3.A 4.B 5. B 6.D 7.B 8. B 9.A 10.D
Passage 2:
1.D 2.B 3.B 4.A 5. D 6.B 7.D 8. A 9.D 10.C
B. WRITTEN TEST
I. OPEN CLOZE
Passage 1 1. go 2. on 3. far 4. without 5. alone
6. somehow 7. aware 8. themselves 9. which 10. more
Passage 2 1.into 2. may / might / can 3. terms 4. Regardless/Irrespective
5. for 6. so 7. result/ consequence 8. let
9. What 10. however
II. WORD FORM
Part 1
1. possession 2. sight 3. derivation 4. habitually 5. lyricists
6. indebted 7. misgivings 8. bitterness 9. addictions 10. disqualified
Part 2
1. descendants 2. outmoded 3. exceedingly 4. necessity 5. upgrading
6. visually 7. signify 8. persuasive 9. advantageous 10. reliably
III. ERROR IDENTIFICATION
Adolescence always has been and always will be a very difficult time in life. You are lost where
between childhood and adulthood, but still, this is the time in life when you have to break freely from the
conformity of your peers to find yourself. Some people argue that it’s more difficult to be young today than it
used to be. Is this true, and in that case, why?
In modern society teenagers pressure to mature much more quickly than one or two hundred years
118

Downloaded by Nhung Nguy?n (nhungnth2004@gmail.com)


lOMoARcPSD|20277325

ago. Today, minors on a very early stage have to make decisions regard education, often bearing upon their
future careers. In the past children were expected to follow in the footsteps of their parents, that is to say, the
son was supposed to take in the profession of his father, while the daughter was expected to stay at home to
take care of domestic duties such as cooking and cleaning.
Furthermore, today it’s much mare difficult to find your place in society. As cities grow, crime
increases, and the anonymity people experiment grow as well. It becomes more difficult to find and cultivate
your own ideals and values.
On the other hand, the adolescent of today have great opportunities than ever before. In the past, if
your father was a blacksmith or a farmer, in ten years, so you would be. Today, teenagers have the possibility
to fulfil in all their dreams and ambitions.
1. Where → somewhere 2. Freely → free 3. Pressure → are pressured
4. Regard → regarding 5. Take in → take up 6. Experiment → experience
7. Grow → grows 8. Great → greater
9. So you would → so would you 10. Fulfil in all → fulfil all
IV. SENTENCE TRANSFORMATION
1. It’s unknown what caused the collapse of the building.
2. Far from being willing to support the strike, we don’t approve of it.
3. Scarcely had he drunk a glass of wine when his mood began to improve.
4. But for Mike’s affluence Ann wouldn’t be dating him.
5. By and large, you did a good job.
6. The book fell short of my expectations even though it had been written by such a good novelist.
7. She was so wrapped up in her work that she didn’t notice when I came in.
8. She doesn’t have much (of a) flair for arranging flowers.
9. Diane finds that being creative takes her mind off her work.
10. One particular player was singled out for praise by the manager.

THPT CHUYÊN TRẦN ĐẠI NGHĨA – TP. HỒ CHÍ MINH


A. MULTIPLE CHOICE QUESTIONS
I. PHONOLOGY
1.A 2.B 3.C 4. A 5.D 6. A 7.D 8.A 9.C 10.A
II. USE OF ENGLISH: Word choice/ Structure and Grammar
1.C 2. A 3. A 4. B 5.B 6. A 7. A 8.C 9.D 10.A
11.A 12. A 13. B 14. D 15.D 16.B 17.C 18. C 19. B 20. D
Prepositions and Phrasal Verbs:
l.C 2.C 3. A 4.C 5.D 6. A 7.B 8.B 9. D 10. D
III. Reading Comprehension
Reading 1:
l. B 2. D 3.A 4.D 5.A 6.B 7.D 8.A 9.C 10.B
Reading 2:
l.D 2. A 3.C 4.B 5.A 6.B 7.D 8.A 9.C 10.C
IV. Cloze test:
Cloze Test 1:
1.A 2. B 3.D 4.D 5.C 6.D 7.C 8.A 9.C 10.B
Cloze Test 2:
1.A 2. C 3.D 4.C 5.D 6.A 7.B 8.D 9.D 10.B
B. WRITTEN TEST
1. OPEN CLOZE TEST:
Open Cloze Test 1:
1. burned 2. with 3. contain 4. estimated 5. currently
119

Downloaded by Nhung Nguy?n (nhungnth2004@gmail.com)


lOMoARcPSD|20277325

6. lost 7. of 8. left 9. frame 10. save


Open Cloze Test 2:
1. value 2. contribute 3. considered 4. However 5. derived
6. endangered 7. estimates 8. to 9. off 10. causing
II. WORD FORMS
A.
1. ill-gotten 2. acculturate 3. enumerated 4. underhand(ed) 5. radioactive
6. domineering 7. incriminate 8. benefactor 9. Moneywise 10. ready-made
B.
1. Speculations 2. puzzling 3. inactivity 4. plausible 5. fundamental
6. punctuated 7. woken 8. disturbed 9. unaffected 10. implications
III. ERROR IDENTIFICATION
Journeys are the midwives of thought. Few places are more conducive to internal conversations than a
moving plane, ship or train. There is an almost quaint correlation between what is in front of our eyes and the
thoughts that we are able to have in our heads: large thoughts at times requiring large views, new thoughts,
new places. Introspective reflections that are liable to stall are helped along by the flow of the landscape. The
mind may be reluctant to think properly when thinking is all it is opposed to do. The task can be as paralyzing
as having to tell a joke or mimic an accent on demand. Thinking improves when parts of the mind are given
other tasks, are charged with listening to music or following a line of trees.
Of all modes of transport, the train is perhaps the best aid to thought: the views have none of the
potential monotony of those on a ship or plane, they move fast enough for us not to get exasperated but slowly
enough to allow us to identify objects. They offer us brief, inspiring glimpses into private domains, letting us
see a woman at the precise moment when she takes a cup from a shelf in her kitchep, then carrying us on to a
patio where a man is sleeping and then to a park where a child is catching a ball thrown by a figure we cannot
see.
IV. Sentence Transformation
1. He has a reputation for being a good pianist.
2. He dismissed the whole idea as being ridiculous.
3. I have serious doubts about his ability to do the job.
4. My understanding when I agreed to do the work was that I would be paid for it.
5. Rather than disturb the meeting, I left without saying goodbye.
6. Most of the play was inaudible to those of us at the back.
7. Sean wasn’t given any credit for doing all the illustrations in the book.
8. You must decide upon your future now that you have finished your studies.
9. We didn’t phone you so as not to wake/ so that we wouldn’t wake you up.
10. Martha was the sole survivor of the crash.
TRƯỜNG THPT CHUYÊN LƯƠNG THẾ VINH – ĐỒNG NAI
A. PHONETICS
I.
l.B 2.A 3. B 4.D 5.D
II.
l.C 2.C 3.A 4.B 5. A
B. VOCABULARY
I
1.B 2.C 3. B 4.D 5.C 6.D 7. A 8.B 9.C 10.B
II.
l.D 2.D 3. A 4. C 5.B 6.C 7. B 8. D 9.A 10. B
III.
1. an all-rounder 2. apple of his eye 3. take up arms
120

Downloaded by Nhung Nguy?n (nhungnth2004@gmail.com)


lOMoARcPSD|20277325

4. keep the ball rolling 5. a bear with a sore head 6. made a bẹe-Ịine for,
7. in cold blood 8. bolt from the blue 9. bombshell 10. nipped in a bud
IV.
1. adorable 2. accelerator 3. acquaintance 4. admitted 5. advisory
6. cannery 7. Centrepiece 8. chairman 9. descriptive 10. desirous
C. GRAMMAR AND STRUCTURES
I.
1d 2b 3a 4c 5b
6d 7a 8c 9b 10d
II.
1. work → works 2. by → to 3. dating →dated
4. Believing → Believed 5. in → on
D.READING COMPREHENSION
I.
1. do 2. is 3. as 4. less 5. get
6. with 7. its 8. present 9. consist 10. which / that
II.
1.C 2.B 3.A 4. A 5. B 6.D 7. B 8. A 9. A 10.D
11.C 12.C 13.D 14.C 15. A 16. C 17. B 18. B 19. C 20.A
E. WRITING
1. parents’ consent to marry nothing stands in the
2. has had a passion for puzzle-solving / solving puzzles since
3. was hailed as acomplete success by
4. did she know (that) her husband was about
5. permits herself more than

TRƯỜNG CHUYÊN NGUYỄN TẤT THÀNH - KON TUM


MULTIPLE CHOICE QUESTIONS
I. Phonology:
1.C 2.B 3.C 4.C 5.A 6.B 7.C 8.B 9.A 10.B
II. Word choice:
11.C 12.C 13. B 14. B 15.C 16.B 17.A 18.C 19.D 20.C
III. Structures and grammar:
21.C 22.D 23.A 24.D 25.C 26.B 27.B 28.A 29.B 30.C
IV. Prepositions and phrasal verbs:
31.C 32.D 33.B 34.D 35.B 36.B 37.A 38.D 39.A 40.C 41.B 42.D
IV. Reading Comprehension:
A. 43. D 44. C 45.B 46.D 47.A 48.C 49.D 50.A
B. 51.D 52.D 53.A 54.C 55.A 56.C 57.C 58.B 59.D 60.B
VI. Guided cloze test
A. 61.B 62.D 63.B 64.A 65.B 66.D 67.B 68.B 69.C 70.A
B. 71. A 72.B 73.C 74.D 75.A 76.C 77.B 78.B 79.A 80.A
WRITTEN TEST
I. open cloze test
A. Open cloze test 1:
1 .of 2. recognize/ recognise 3.for 4. Should 5.run/face 6. Given
7 . together 8. will 9. anything/anyone/anybody 10. in/ despite
B. Open Cloze test 2:
1. No 2. has 3. but 4. by 5.(al) though/while/whilst/whereas
121

Downloaded by Nhung Nguy?n (nhungnth2004@gmail.com)


lOMoARcPSD|20277325

6. as 7. goes 8.that 9.having 10.what


II. Word forms:
A. Supply the correct form of the word to fill in the blank.
1. fulfillment 2.substantial (substantive) 3. underestimated 4. emphasis 5. adherence
6. anatomical 7. suspicion 8. noticeably 9. seductive 10. significance
B. Supply the correct form of the word provided in brackets.
1. outward 2. ill-advised 3. opinionated 4. torrential 5. embankment
6. spin-offs 7. flawless 8. counter productive 9. imperceptibly 10. enlivened
III. Error identification:
1. particular → particularly 2. consideration → considerations
3. date → dating 4. almost → most
5. parts → part 6. disapproval for → disapproval of
7. marriages of → marriages between 8. mobile → mobility
9. as → than 10. one → once
IV. SENTENCE TRANSFORMATION
1. The police had to let the suspect go because new evidence was brought to light.
2. Don’t let yourself be (get) taken in by her relaxed manner; she is an extremely shrewd woman!
3. The orphanage left no stone unturned in their attempt to find the boy’s parents.
4. I feel like a fish out of water in the new office.
5. Fierce argument broke out between the two children about who had damaged the bicycle.
6. No matter how hard they tried, they could not get the old car to start.
7. A mistake in an interview may cost you a job so it is important to be well prepared.
8. She flatly refused to sleep in that haunted house.
9. Everyone was taken aback by the news of the Prime Minister’s resignation.
10. A computer breakdown was given as the reason for the delay in dispatching the brochures.

TRƯỜNG THPT CHUYÊN NGUYỄN ĐÌNH CHIỂU - ĐỒNG THÁP


PART A: MULTIPLE CHOICE QUESTIONS
I. PHONOLOGY
A 1.D 2. C 3. A 4.B 5.B
B 1.D 2. B 3.C 4. A 5. A
II. VOCABULARY AND STRUCTURE:
l.C 2. B 3. A 4. A 5. C 6.D 7. D 8. D 9. A 10. D
11.A 12.C 13.B 14 D 15. C 16. A 17. B 18. C 19. D 20. D
III. PREPOSITIONS AND PARTICLES
l.C 2. A 3.C 4.D 5.D 6.C 7. A 8.B 9.D 10. B
IV. READING COMPREHENSION
Reading 1:
1.B 2.D 3.B 4.C 5.D 6. A 7.D 8. B 9. A 10. C
Reading 2:
1D 2.H 3 .A 4.B 5.E 6.C 7.F 8.K 9.G 10. L
V. GIUDED CLOSE TEST:
1. l.C 2.B 3. A 4.D 5.D 6.B 7. A 8.B 9.B 10. A
2. 1.C 2.A 3.D 4.D 5. A 6.C 7.B 8.B 9.C 10 C
FARTB: WRITTEN TEST
I. Open close test:
Test 1:
1. known 2. upon 3. such 4. to 5. themselves
6. because 7. certain 8. Although 9. without 10. are
122

Downloaded by Nhung Nguy?n (nhungnth2004@gmail.com)


lOMoARcPSD|20277325

Test 2:
1. growing 2. open 3. age 4. even 5. who/that
6. deal 7. if 8. addition 9. own 10. is
II. WORD FORMS
A.
l.expectancy 2. inequitable 3. disorientated 4. gladdened 5. youngish
6. interdependent 7. appreciative 8. overindulging 9. pre-dominance 10. revitalize
B.
1. unsophisticated 2. nearly 3. possibility 4. universal 5. seriously
6. leader 7. quickly 8. global 9. locations 10. truly
III. ERROR IDENTIFICATION
1. was been → was 2. go for → go 3. up → off 4. it → there
5. at last → at least 6. to form → forming 7. away → down 8. very much → very
9. here → there 10. about → that
IV. SENTENCE TRANSFORMATION
A.
1. Without absolute secrecy, the mission wouldn’t have succeeded
2. The floods made further progress impossible
3. The house had its roof blown off by the hurricane
4. Under no circumstance must you mention this to him.
5. I was confused by his account of the accident.
B.
1. Susan has turned down his proposal of marriage twice this year
2. Your entry to university will depend/depends on your examination results
3. She made a name for herself as a novelist.
4. He is prone to illness/getting to ill at this time of the year.
5. I don’t normally go in for cycling at the weekend.
TRƯỜNG THPT CHUYÊN PHAN NGỌC HIỂN - CÀ MAU
A. MULTIPLE CHOICE
I. PHONOLOGY
l.A 2. D 3. D 4. A 5. D 6. B 7. D 8- B 9. A 10. C
II. WORD CHOICE
11. A 12. D 13. D 14. C 15. B 16. C 17.B 18. D 19. C 20 A
III. STRUCTURES AND GRAMMAR
21.B 22. B 23.B 24 C 25.A 26.C 27. B 28. A 29. D 30. C
IV. PREPOSITION AND PHRASAL VERBS
31. B 32. C 33. D 34. C 35. B 36. B 37. C 38. D 39. A 40. B
V. READING COMPREHENSION
Reading 1
41. A 42. B 43. D 44. A 45. C 46. D 47. A 48. D 49. C 50. B
Reading 2
51. A 52. C 53. B 54. D 55. A 56. D 57. D 58. C 59. D 60. A
VI. CLOZE TEST
Cloze test 1
61. D 62. A 63. C 64. C 65. D 66. B 67. A 68. C 69. B 70.D
Cloze test 2
71. B 72. B 73. C 74. A 75. A 76. D 77. A 78. B 79. A 80. D
B. WRITTEN TEST
I. OPEN CLOZE TEST
123

Downloaded by Nhung Nguy?n (nhungnth2004@gmail.com)


lOMoARcPSD|20277325

Cloze test 1
1. which 2. despite 3. remains 4. unexpected 5. displaying
6. often 7. long 8. sinking 9. own; 10. the
Cloze test 2
11. under 12. for 13.be 14. leveled 15. made
16. reduced 17. with 18. taken/borro\yed 19. in 20. ever
II. WORD FORMS
Parti:
21. irrelevant 22. respectfully 23. inhumane 24. undercooked 25. considerable
26. unbroken 27. comparatively 28. undeniable 29. sight 30. obligation
Part 2:
31. Researcher32. greatness 33. variety 34. containing 35. responses
36. noticeably 37. recognition 38. unreliable 39. knowledge 40. preference.
III. ERROR IDENTIFICATION
41. offering → offered 42. attractive → attractively
43. and → or 44. nobody → anybody
45. under → in/within 46. grant → granted
47. out → from 48. supposing → supposed
49. other than → rather than 50. circumstance → circumstances
IV. SENTENCE TRANSFORMATION
51. I hardly ever bother purchasing new clothes.
52. Whoever you ask in this office will say the'same as I did.
53. He blamed the food shortages on the rapid growth in population
54. I didn’t realize the extent to which he was influenced by his brother.
55. She was lost for words.
56. You’ll have to turn over a new leaf if you want to succeed.
57. That hotel is a bit beyond our reach, I am afraid
58. We had to return home because we had run out of money.
59. I don’t think it was reasonable of you to make such a fuss.
60. Robert took exception to being left out of the team

TRƯỜNG THPT CHUYÊN TRAN


A. MULTIPLE CHOICE QUESTIONS
I. PHONOLOGY
Part 1. 1.B 2. A 3. A 4. B 5.D
Part 2. l.D 2. B 3. A 4. A 5.C
II. VOCABULARY
1.D 2. B 3. D 4. B 5. B 6. A 7. C 8. D 9. A 10.A
III. STRUCTURES AND GRAMMAR
1. D 2. A 3. B 4. C 5. C 6. D 7.C 8. A 9. C 10. C
IV. PREPOSITIONS AND PHRASAL VERBS
1. B into 2. C to 3. A from 4. D at 5. B on
6. A off 7. C back 8. C in 9. D under 10.D to
V. READING COMPREHENSION
Part 1:
l.A 2.C 3.D 4.D 5. A 6. A 7.B 8.C 9.D 10. B
Part 2:
124

Downloaded by Nhung Nguy?n (nhungnth2004@gmail.com)


lOMoARcPSD|20277325

A. l.D 2. E 3. F 4. B 5. A 6.G
B. l.D 2. B 3.B 4.D
VI. GUIDED CLOZE TEST
Part 1:
l.A 2. C 3. B 4.C 5. A 6. A 7.C 8. A 9.D 10. C
Part 2:
1.D 2. D 3. A 4.C 5.C 6.D 7.C 8.B 9.C 10. C
B. WRITTEN TEST
I. OPEN CLOZE TEST
Part 1:
1. others 2. something 3. despite 4. not 5. after
6. regardless/irrespective 7. so 8. itself 9. place 10. in
Part 2:
1. but 2. been 3. longer 4. numbers 5. which
6. information 7. made 8. data 9. work 10.advent
II. WORD FORMS
Part 1:
1. extraordinary 2. mercifully 3. upbringing 4. enabled 5. achievement(s)
6. relationships 7. elusive 8.crushing 9. incurable 10. striking
Part 2:
1. co-author 2. abnormalities 3. uninviting 4. discontentedly 5. underprivileged
6. purify 7. Embittered 8. rethink 9. extrA.curricular 10. dreamily
III. ERROR IDENTIFICATION
Line 1: them → themselves Line 7: less → more
Line 2: with → to Line 8: for → in
Line 3: seriously serious Line 9: the → a
Line 4: saying say Line 12: simple → simply
Line 5: floated→ floating Line 14: minor → minority
IV. WRITING
Part 1.
1. She fell prey to irrational fears.
2. Our company has/holds the/a monopoly of/on/over the import/importing/ importation of these chemicals.
3. The final version of the plan bore/had no/little/not much resemblance to the initial draft.
(There was no/little/not much resemblance between the final version and the initial draft.)
4. Don’t breathe a word of this to your colleagues.
5. We were powerless to stop him
Part 2.
1. The drop in the number of school leavers is said to have been caused by the fall in the birth rate between
1964 and 1977.
2. So confused ànd worried did the boy become that he left home.
3. So far I have written ten pages of the report.
4. As soon as we receive your cheque, we shall’send the goods to you.
5. Before these new machines were invented, people had to queue.

TRƯỜNG THPT CHUYÊN QUANG TRUNG - BÌNH PHƯỚC


A. MULTIPLE CHOICE
Câu hỏi 1: PHONOLOGY
l.C 2.D 3. A 4.B 5.D 6.D 7.D 8.B 9.C 10. A
Câu hỏi 2: VOCABULARY
125

Downloaded by Nhung Nguy?n (nhungnth2004@gmail.com)


lOMoARcPSD|20277325

l.D 2.D 3.D 4.C 5. A 6. A 7. A 8. A 9. B 10. B


Câu hỏi 3: STRUCTURES AND GRAMMAR
l.B 2. A 3. B 4. A 5.C 6.B 7.D 8. A 9.C 10. D
Câu hỏi 4: PREPOSITIONS AND PHRASAL VERBS
l.D 2. A 3. C 4. D 5. A 6. A 7.D 8.B 9. A 10. C
Câu hỏi 5: READING COMPREHENSION
Reading 1
l.C 2. B 3. D 4. D 5. A . 6. A 7.A 8. D 9. A 10. B
Reading 2
l.D 2. B 3. D 4.B 5. A 6.B 7. A 8. C 9. C 10. B
Câu hỏi 6: GUIDED CLOZE TEST
Passage 1
l.A 2.D 3. A 4.C 5.C 6. C 7.A 8. D 9. C 10. A
Passage 2
l.A 2. B 3. A 4. B 5.D 6. A 7. D 8. D 9. A 10. C
B. WRITTEN TEST
Câu hỏi 1:
Open cloze test
1. when 2. What 3. being 4. at 5. during
6. by 7. which/that 8. before 9. as 10. disaster
Open cloze test 2
1. which 2. have 3. substance 4. they 5. Recently / Newly
6. order / retail 7. producing 8. spreading / transmitting 9. light / wake 10. sooner
Câu hỏi 2: WORD FORMS
1.
1. laborious 2. liven 3. yogic 4. inexhaustible 5. bebead
6. dethroned 7. Possessions 8. decidedly 9. deforestation 10. discourteous
2.
1.Intensive 2. susceptibility 3. uncomfortable 4. Enlargement 5. characteristic
6. complications 7. treatment 8. threatening 9. abnormalities 10. immunization
Câu hỏi 3. ERROR IDENTIFICATION
1. big → large 2. make well → do well 3. informations → information
4. meanings for words → meanings of words 5. studies → learns
6. insight to → insight into 7. confirm to → confirm
8. phonological → phonetic 9. make the room for → make room for
10. at a regular basis → on a regular basis
Câu hỏi 4. SENTENCE TRANSFORMATION
1. Complete the second sentences so that it has a similar meaning to the first sentence, using the word
given. Do not change the word given. You must use between three and eight words, including the word
given.
1. Only when I had cleared the spare room could I start decorating.
2. Rarely is it possible tơ buy a plane ticket at the last minute.
3. Not a/one (single) question did he ask her about her study.
4. What Einstein proved is /was that energy and mass are basically the same thing.
5. You can do jogging any time, rain or shine.
2. Complete the second sentence so that it has a similar meaning to the first sentence, using the word
given. Do not change the word given.
6. Mary was born in a small village in the middle of nowhere.
7. Jimmy played truant this morning, so his father has thrown the book at him.
126

Downloaded by Nhung Nguy?n (nhungnth2004@gmail.com)


lOMoARcPSD|20277325

8. The construction workers showed a reckless disregard for the safety at work.
9. There is no excuse for coming to class late.
10. They have known each other since they were at the infants.

TRƯỜNG THPT HUỲNH THÚC KHÁNG – QUẢNG NAM


PART ONE : PHONOLOGY
A 1.B 2. A 3. A 4. P 5. A
B. 1.D 2. B 3. A 4. D 5. B
PART TWO: VOCABULARY AND GRAMMAR
A.
1.D 2. B 3.D f.D 5B 6. A 7. B 8. D 9. C 10. C
11.C 12. C 13. C 14. B 15. A 16. D 17. B 18. D 19. A 20. C
21. C 22. A 23. D 24. A 251B 26 D 27. C 28. B 29. B 30. D
B.
1. We were told to stay seated and fasten our seat belts. (fast)
2. At first, there was a lot of opposition from the parents of the -disabled-children. (able)
3. Computer is a personal communicator which helps you to interact with other computers.
(communicate)
4. After the earthquake, the city was badly destroyed and many people became homeless . (home)
5. Books in the home are a wonderful source of knowledge and pleasure. (know)
6. He is much loved and highly respected teacher. (respect)
7. A lot of plants and animals could be used as medicines against cancer, AIDS, heart disease and many other
sicknesses (sick).
8. You may be surprised at the large variety of animals that live in Nairobi National Park. (various)
9. There are two things that make humans different from all other animals. (differ)
10. Our life is gradually improved. (gradual)
C.
1. Alice has been taken to the dentist many times.
2. He had done nothing before he saw me.
3. Tom is reading a book. He started two hours ago and he is on page 44. He has been reading for two
hours.
4. Their old house will be sold..............soon.
5. Having finished reading the book , I went to bed.
6. The lamps made in China are very cheap.
7. He says as if he came back from London.
8. My lawyer advised me not to say anything further about the accident.
9. He wore dark glasses to avoid being recognized.
10. Over the past few years the cost of living...has risen...considerably.
D.
Things started to go wrongly as soon as we got to the hotel. We were all completely exhausting after
our long journey and looking forward to ạ shower and a rest. However, we found that our room was not ready,
which was very annoyed, although the manager was extremely apologising. While we were waiting, we
asked about the excursions to places of an interest which we have read about in the brochure. Imagine how
we felt when we were told they had all been cancelled! Apparently, the person responsible of organizing them
had left suddenly and had not been replaced. Then Sally saw a notice pinning to the door of the restaurant,
said it was closed for redecoration, and Peter discovered that the swimming pool was empty. When we
eventually got to our room we were horrified to find that it was at the back of the hotel, and we had a view of
a car park, that seemed to be used as a rubbish dump. We seriously began to wonder whether or not to stay.
1. wrongly → wrong 6. have read → had read
127

Downloaded by Nhung Nguy?n (nhungnth2004@gmail.com)


lOMoARcPSD|20277325

2. exhausting → exhausted 7. of → for


3. annoyed → annoying 8. pinning → pinned
4. apologising → apologetic 9. said → saying
5. places of an interest → places of interest 10. that → which
E.
1. They regret ever doing business with him.
→They wish they had never done business with him.
2. We didn’t finish the work because of his carelessness.
→ If he hadn’t been careless/ had been (more) careful, we would have finished the work.
3. If you followed my advice, you’d be successful.
→ Were you to follow my advice, you’d be successful.
4. “Why didn’t you go to the club last night?”, the girl said to Jim.
→ The girl asked Jim why he hadn’t gone to the club the night before/the previous night.
5. The film was so interesting that I couldn’t go to bed.
→ It was such an interesting film that I couldn’t go to bed.
6. Mary is so young that she can’t join our club. (TOO)
→ Mary is too young to join our club.
7. Jim does not intend to give up his hobby.(INTENTION)
→ Jim has no intention of giving up his, hobby.
8. The owners of the restaurants are usually very rich.(WHO)
→ The people who own the restaurants are usually very rich.
9. It was easy for us to get tickets for the concert. (NO)
→ We had no difficulty (in) getting tickets for the concert.
10. They think the burglar got in through the bathroom. (THOUGHT)
→ The burglar is thought to have got in through the bathroom.
PART THREE: READING
A.
Reading 1:
l.C 2. A 3. D 4. B 5. B 6. D 7. A 8. B 9. C 10. A
Reading 2:
l.A 2. B 3. D 4. D 5. C 6. C 7. D. 8. D 9. C 10. C
B.
Cloze test 1:
l.D. 2. C. 3. A. 4. A. 5. D. 6. B. 7. C. 8. D. 9. A. 10. B.
Cloze test 2:
l.A 2. D 3.B 4. D 5. C 6. A 7. C 8. C 9. D 10. D
C. Fill in each blank with a suitable word.(iop)
Open cloze test 1:,
Dolphins are not fish. They are mammals that/ which live in water. Dolphins are among the
most intelligent animals on earth. Although they can be found in all oceans in the world, dolphins prefer
coastal waters and bays. The size of dolphins vary greatly. The smallest dolphin is just about 50 kg in weight
and 1.2 metres in length while the largest one can weigh up to 8,200 kg and is 100 metres long .Dolphins
eat mainly fish. A female dolphin gives birth to one calf every two years after a gestation period of
eleven or twelve months. A dolphin can normally live from twenty five to sixty five years and some species of
dolphins can even live longer. Dolphin populations are at risk due to the pollution of their habitat and
accidental entrapment in fishing nets.
Open cloze test 2:
1.or 2. of 3. than 4. the 5. more
6. against/ about/ concerning 7. other/ similar 8. Although/ While
128

Downloaded by Nhung Nguy?n (nhungnth2004@gmail.com)


lOMoARcPSD|20277325

9. a/ pet/ every10. should/ could

TRƯỜNG THPT CHUYÊN NGUYỄN THỊ MINH KHAI - SÓC TRĂNG


PART A: MULTIPLE CHOICE
I. PHONOLOGY
A. l.C 2. A 3.C 4.C 5.D
B. 6.C 7. D 8. C 9. B 10. A
II. VOCABULARY
1. A 2. C 3. C 4.C 5.B 6. B 7.B 8. A 9. C 10. A
III.
l.B 2.C 3.C 4B 5.D 6.C 7.C 8.C 9. A 10. A
IV. PREPOSITION AND PHRASAL VERB
l.A 2.B 3.D 4. A 5. A 6.B 7.C 8. B 9.D 10. B
V. READING
Reading 1
l.C 2. B 3.D 4.D 5. A 6. A 7. A 8.D 9. B 10. A
Reading 2
1.B 2.D 3.B 4.C 5. A 6.C 7.C 8.D 9.D 10. B
VI. GUIDED CLOZE TEST
Cloze test 1
l.A 2.D 3. A 4.C 5. A 6.B 7.D 8. A 9. D 10. A
Cloze test 2
l.C 2.D 3. A 4. B 5. A 6.D 7.D 8.D 9. A 10. A
PART B: WRITTEN TEST
I. OPEN CLOZE TEST
Cloze Test 1:
1. called 2. crew 3. number 4. on 5. place
6. were 7. pick 8. the 9. it 10. Out
Cloze Test 2:
l.to 2. of 3. play 4. with 5. where
6. to 7. last 8. decayed 9. order 10. from
II. WORD FORMS
Word form 1:
1. misinformed 2. grandeur 3. downpour 4. editorial 5. chaotic
6. hypocrisy 7. troublesome 8. single-minded 9. hypersensitive 10. indestructible
Word form 2:
1. manufacture 2. clarify 3. specification 4. practicable 5. engineers
6. precisely 7. independent 8. intentions 9. explicit. 10. design
III. ERROR IDENTIFICATION
Line Error Correction
(2) it its
(3) never ever
(5) carving carved
(6) so such
(10) then though
(14) being used using
(20) which in which
(22) of for
(24) humpback humpbacks
129

Downloaded by Nhung Nguy?n (nhungnth2004@gmail.com)


lOMoARcPSD|20277325

(25) quick quickly


IV. SENTENCE TRANSFORMATION
1. To my surprise, it turned out that the hotel was a long way from the center.
2. He easily flies off the handle.
3. I find myself at a loss to comprehend Tom’s behaviors.
4. The proposed motorway extension has been a bone of contention for years.
5. The problems of ours are just the tip of the iceberg.
6. You are under no obligation to buy one if you don’t like it.
7. Much as I admire his courage, I think he is foolish.
8. In all probabilities, the whole thing will have been forgot by next term.
9. In few other books is this problem so well-explained.
10. Had it not been for the death of the Prime Minister, the passing of the bill would have been carried out.

TRƯỜNG THPT CHUYÊN NGUYỄN DU – ĐẮK LẮK


MULTIPLE CHOICE TEST
PART ONE: PHONOLOGY
l.C 2. B 3. p 4. B 5. D 6. A 7. C 8. B 9. C 10. C
PART TWO: VOCA BULARY AND STRUCTURE
1.D 2. A 3. A 4. C 5. A 6. C 7. D 8. A 9. B 10. D
11. C 12. A 13. C 14. A 15. D 16. D 17. D 18. D 19. D 20. A
21. A 22. B 23. B 24. A 25. D 26. A 27. B 28. A 29. C 30. B
PART THREE: READING COMPREHENSION
Reading 1:
l.C 2. A 3.B 4. A 5. C 6. D 7. B 8. D 9. A 10. D
11.B 12. C
Reading 2:
I.I 2E 3.C 4. D 5. A 6. G 7. F 8. B
PART FOUR: GUIDED CLOZE TEST
Passage 1
l.C 2.B 3.D 4. D 5.C 6. A 7. A 8.C 9. B 10. C
Passage 2
l.B 2. A 3.C 4.C 5.D 6.B 7. A 8.C 9. A 10. B
WRITTEN TEST
PART ONE: OPEN CLOZE TEST Passage 1:
1. inform 2. permission 3. dress niisd 4. During 5. offering
6. where 7. with , 8. ice 9. thank 10. clear
Passage 2:
1.threat 2.-rise 3. marine .J 4. fewer / 5. level
6. where 7. pressure 8. reduce 1 9. source 10. emissions
PART TWO: WORD FORMS A
1. non-existence 2; erredly 3. counterfactuals
4. short-lived 5. equidistant 6. incommunicado
7. no-brainer B. 8. melodramatic 9. vacuum-packed 10. afterglow
1. complication 2. expectations - 3. consciously
4. personalities ?5. preoccupied 6. shaped
7. impossible 8. wonder 9. smoothly To. nagging
PART THREE: ERROR IDENTIFICATION
L1. conducting → conducted L13: choose → choosing
L4. allow → be allowed L14. this →these
130

Downloaded by Nhung Nguy?n (nhungnth2004@gmail.com)


lOMoARcPSD|20277325

L8. them → it L18. wild → wildly


L9. economical → economic L21. later → after
L11. love → loved L23. plenty → plenty of
PART FOUR: SENTENCE TRANSFORMATION
A.
1. Their engagement was announced in the local newspaper.
2. With the exception of Sally, everyone was exhausted.
3. He shouldn’t have been savagely criticized for his efforts to find a solution.
4. It will take you at least 20 minutes to reach'the station.
5. She avoided having to repeat the course by taking extra night classes.
B.
1. He is making quite a name for himself as an interviewer.
2. Buying new furniture must be accounted for in our household budget this year.
3. Julie always lends an ear to my complaints about work.
4. The novel fell short of my expectations.
5. We have not ruled out the possibility of introducing pay cuts.

TRƯỜNG THPT CHUYÊN LÊ QUÝ ĐÔN - ĐÀ NẴNG


PART A: MULTIPLE CHOICE
I. PHONOLOGY
A. l.C 2. A 3.C 4. A 5.B
B.1. A 2. D 3.C 4.B 5. A
II. VOCABULARY AND GRAMMAR
1.C 2. D 3. A 4. B 5. B 6. D 7. C 8.A 9.B 10. D
11.D 12. B 13. C 14. A 15. B 16. D 17. A 18. C 19. D 20. B
III. PREPOSITIONS AND PHRASAL VERBS
l.D 2. A 3.B 4.C 5. A 6. D 7. B 8.D 9.D 10.A
IV. READING COMPREHENSION
Passage 1
1.D 2. A 3. C 4.B 5. A 6. C 7. B 8.B 9. A 10. D
Passage 2
1.A 2.B 3.D 4. C 5. D 6. B 7. C 8. A 9.D 10. D
V. GUIDED CLOZE TEST
Passage 1
1.D 2. A 3.B 4. B 5.D 6. D 7. C 8. A 9.C 10. B
Passage 2
l.C 2. A 3. A 4. D 5. B 6. C 7. D 8. A 9. C 10. B
PART B: WRITTEN TEST
1. OPEN CLOZE TEST
Passage 1
1. each 2. background 3. although 4. influence 5. took
6. original 7. do 8. risk 9. whatever 10. for/ with
Passage 2
1.1. means 2. most 3. use 4. to/ for 5. provides
6. lonely 7. conversation 8. nuisance 9. on/ upon/ into 10. brought
II.WORD FORM
A.
1. borderless 2. impairment 3. Graphology 4. nationalistic 5. unputdownable
6. afforestation 7. single-minded 8. impoverished 9. humanitarian 10. blackout
131

Downloaded by Nhung Nguy?n (nhungnth2004@gmail.com)


lOMoARcPSD|20277325

B.
1.1. explanation 2. definition 3. assumption 4. Logically 5. reasonable
6. thought 7. confusion 8. concept 9. undoubtedly 10. sensible
III. ERROR IDENTIFICATION
1. ever → never 5. vulnerable of → vulnerable to
2. active → actively 6. inhabitant → inhabitants
3. highly → high 7. character → characteristic
4. few → little 8. national → natural
9. draw → drawn 10. economics → economy
IV. SENTENCE TRANSFORMATION
1. It’s too amazing a trip for us to forget.
2. He is the most likely person to succeed in solving the problem.
3. He made threats of violence against the officers.
4. It’s about time she stopped giving me a cold shoulder in meetings.
5. The fact that they survived the accident was nothing short of a miracle, in my opinion..
6. She took a dim view of my suggestion.
7. Payment will be made on receipt of the order.
8. The villagers braced themselves for the coming storm.
9. There’s little likelihood of her giving/ that she will agree to giving/ give you a pay rise.
10. Organic vegetables are said to do wonders for one’s/ your health.

TRƯỜNG THPT CHUYÊN BẾN TRE – BẾN TRE


I. MULTIPLE CHOICE QUESTIONS
Câu hỏi 1:
PHONOLOGY
l.C. chef 2. C. promise 3. A. replace 4. B. cherish
5. D. breathe 6. A. necessity 7. D. instrument 8. C. skyscraper
9. C. policeman 10. D. elector
Câu hỏi 2:
1. A 2.D 3.C 4.B 5.A 6.B 7.C 8.D 9.C 10.D
Câu hỏi 3:
1. A 2.B 3.D 4.D 5. C 6. B 7. A 8. D 9. A 10. B
Câu hỏi 4:
1.A 2. C 3.D 4.B 5. D 6. D 7. A 8.B 9.B 10. B
Câu hỏi 5:
1. A. Kitchen God 2. C. authorities 3. D. pottery 4. C. wind
5. B. Palace 6. C. altar 7. C. Jade Emperor 8. B. After
9. D. prosperity 10. A. resume
Câu hỏi 6:
1. B. expected 2. C. supply 3. D. cool 4. A. ranging 5. B. high
6. B. rise 7. C. official 8. D. failure 9.losses 10. D. forecased
II. PHẦN TỰ LUẬN
Câu hỏi 1:
1. matches 2. exclusive 3. rights 4. bought 5. stations
6. confirm 7. fact 8. programs 9. regardless 10. contract
Câu hỏi 2:
1. Hospital 2. affirmed 3. honor 4. treatment 5. doctors
6. offered 7. patients 8. Vietnam 9. medical 10. technologies
Câu hỏi 3:
132

Downloaded by Nhung Nguy?n (nhungnth2004@gmail.com)


lOMoARcPSD|20277325

1. invariably 2. commentators 3. differs 4. descriptive 5. observations


6. increasingly 7. opininately 8. impartiality 9. unpredictable 10. organizational
Câu hỏi 4:
1. midsentence 2. exceptionally 3.outbreak
4. counter-productive - overact 5. impoverished 6. childbirth
7. blood-shot 8. foreseeable 9. breakthroughs
Câu hỏi 5:
line 1: conducting → conducted line 2: Before → Prior
line 3: in-deep → in depth line 4: involved → involving
line 6: plenty → deal line 12: routine → routinely
line 12: computer-controlling → computer-controlled
line 17: Merely → Mere
line 20: assistance → assistants line 23 : have → has
Câu hỏi 6:
1. He is bound to make it up to you for the damage he has done.
2. I’m not averse to staying in on a Saturday night if I have good company.
3. Nothing other than a full refund will satisfy Helen.
4. You could have notified them about the change of plans,.
5. We were over the moon whẹn our first grandchild was bom.
6. Being her only niece, Ann is the apple of her eye.
7. A holiday in the UK costs an arm and a leg these days.
8. How does coming on Saturday suit you?
9. We are pleased to say that you may be eligible for a grant.
10. If you bide your time until the market improves, you’ll get a better return on your investment.

ĐỀ THI OLYMPIC TRUYỀN THỐNG 30/4


NĂM 2014
ĐỀ THI CHÍNH THỨC
A. MULTIPLE CHOICE
I. PHONOLOGY
Choose the word whose underlined part is pronounced differently from the others.
1. A. wretchedly B. ruggedly C. confusedly D. determinedly
2. A. slumber B. doubter C. dumbly D. debtor
3. A. expansion B. conversion C. precision D. explosion
4. A. athlete B. ethnic C. asthma D. breath
5. A. finale B. machete C. facsimile D. denote
Choose the word which is stressed differently from the other three.
6. A. magnifier B. preferable C. interpolate D. jeopardy
7. A. horizontal B. adolescent C. panorama D. infrastructure
8. A. photography B. advantageous C. proverbial D. magnificent
133

Downloaded by Nhung Nguy?n (nhungnth2004@gmail.com)


lOMoARcPSD|20277325

9. A. lunatic B.nomadic C. aroma D. heroic


10. A. constituency B. constitution C. presentiment D. subsidiary
II. WORD CHOICE
Choose the best options to complete the following sentences.
11. The news of his death was like a bolt…………..
A. from the red B. from the blue C. from the black D. from the white
12. Environmental groups are locked in…………..with the council over the proposed new bypass through
parts of Charmy Wood.
A. argument B. battle C. debate D. controversy
13. He was…………..with an extraordinary ability.
A. entrusted B. ensured C. endowed D. entreated
14. “There is no further treatment we can give”, said Dr. John. “We must let the disease take its…………..”.
A. end B. term C. way D. course
15. What on earth made you risk your life and…………..by driving that fast?
A. death B. health C. limb D. liberty
16. During the evening football match, the stadium was illuminated by…………...
A. flashlights B. floodlights C. highlights D. spotlights
17. Tom admitted that he couldn’t concentrate on his work…………..due to this irritating noise.
A. in all B. the bit least C. in the least D. at the least
18. You can buy fake designer clothes much more cheaply than the …………..articles.
A. exact B. real C. true D. genuine
19. Could I pick your…………..on the subject before the meeting?
A. brains B. mind C. head D. intellect
20. Four people drpwned when the yacht…………..in a sudden storm.
A. inverted B. overflowed C. upset D. capsized
III. GRAMMAR AND STRUCTURES:
Choose the best options to complete the following sentences.
21. Everything looks very positive for the company, …………..the current investors do not default on their
agreements.
A. assuming that B. whether C. whereas D. as if
22.…………..came ………….. as the injured cat was so weak.
A. Afterwards/dying B. Death/X C. Soon/death D. Then/to die
23. Probably no man had more effect on the daily lives of most people in the United States…………..
A. as Henry Ford, a pioneer in automobile production.
B. rather than Henry Ford, a pioneer in automobile production.
C. than did Henry Ford, a pioneer in automobile production.
D. more than Henry Ford, a pioneer in automobile production.
24. The picking of the fruit,………….., takes about a week.
A. whose work they receive no money
B. as they receive no money for that work
C. for which work they receive no money
D. they receive no money for it
25. It was not until they sent me the invitation…………..how meaningful I was to them.
A. that I realized B. I realized C. did I realize D. had I realized
26. I would rather ………….. my holiday in Egypt than in Turkey. I really couldn’t stand the food.
A. take B had taken C. to have taken D. have taken
27. …………..that we decided to cancel the polo match.
A. Such was a weather B. Such terrible weather was it
C. So terrible a weather was it D. The weather was, such
134

Downloaded by Nhung Nguy?n (nhungnth2004@gmail.com)


lOMoARcPSD|20277325

28. Why didn’t you tell me you needed help? I…………..you.


A. must have helped B. could have helped C. could help D. was able to help
29.………….., he remained optimistic.
A. Though badly wounded he was B. Badly wounded as he was
C. As he was badly wounded D. As badly wounded he was
30. Mark Twain began his career on a newspaper and…………..to be a journalist.
A. long considering himself B. himself long considered
C. long considered himself D. was long himself considered
IV. PHRASAL VERBS AND PREPOSITIONS
31. I can’t come to your birthday party because something has…………..
A. done away with B. come off C. dropped off D. cropped up
32. Many students couldn’t…………..but the teacher…………..the problem by going into a detailed
explanation.
A. catch up/cleared away B. catch up with/cleared off
C. catch on/cleared up D. catch at/clear out
33. The robbers packed diamonds into a suitcase and…………..in a van that waited for them in the street.
A. put through B. rolled over C. carried away D. made off
34. It is not surprising that he became a writer because he always longed to see his name…………..
A. in type B. in print C. in letters D. in edition
35. At first they refused but I managed to…………..them round to my way of thinking.
A. put B. force C. push D. bring
36. We put his rude manner …………..ignorance of our British customs.
A. off at B. up with C. upto D. down to
37. Bert is a bit dotvn in the …………... He was sacked yesterday.
A. dumps B. sad C. low D. bad
38. Thanks to her regular workouts and sensible diet, she certainly strikes me as in the…………...
A. blue B. green C. pink D. red
39. What’s all this crying ………….. ?
A. without the aid of B. with the aid of C. in aid of D. within the aid of
40. Could you reach me …………..that vase from the top shelf?
A. down B. for C. forward D. up for
V. GUIDED CLOZE 1:
Read the text below and decide which answer best fits each space.
Red Rock Canyon, part of the Red Rock Recreation Lands in Nevada, is an escarpment of crimson
Aztec sandstone cliffs and canyon walls that (41)…………..the geologic history of the area. Bands of
sediment layers tell of a deep-sea bed that 400 million years ago (42)…………..eastward to shoreline in
present - day western Utah. As the ancient sea grew (43)…………..more shallow, about 225 million years
ago, marine limestone and shales were overlaid by sediments washed in from emerging land areas. As the
water in the shallow inland sea evaporated, salts and minerals were deposited in thick beds and fluctuating
shorelines created intermixed beds of limestone, shales and minerals. Sediments from this period gave the
canyon its name. Their red color was created from the weathering of iron (44)…………..within. About 180
million years ago the area became arid and was covered in sand dunes more than 2.000 feet deep, which
became cemented into the Aztec sandstone that is (45) …………..in the canyon today. Its alternating hues of
red, yellow, and white are believed to have resulted from ground water percolating through the sand and
leaching (46)…………..the oxidized iron.
The most significant geologic (47)………….., of the area is the Keystone Thrust, Fault, a fracture in
the earth’s crust. Sixty- five million years ago, (48) …………..pressure thrust one rock plate over another,
a(n) (49)…………..that can clearly be seen in the contrasting bands of gray limestone and red sandstone
beneath it. The Keystone is one of the most easily identifiable thrust faults to be (50)…………..anywhere.
135

Downloaded by Nhung Nguy?n (nhungnth2004@gmail.com)


lOMoARcPSD|20277325

41. A. enclose B. give C. mention D. reveal


42. A. grew B. evolved C. rose D. stood
43 A. progressively B. painstakingly C. constantly D. precedingly
44. A. complexities B. compounds C. combinations D. composites
45. A. dominating B. sufficient C. prominent D. principal
46. A. out B. into C. against D. with
47. A. character B. feature C. trait D. property
48. A. tense B. intensive C. intense D. extreme
49. A. event B. incident C. phenomenon D. occurrence
50. A. found B. located C. noted D. realized
GUIDED CLOZE 2:
Read the text below and decide which answer best fits each space.
(51)…………..popular belief, one does not have to be a trained programmer to work online. Of
course, there are plenty of jobs available for people with high-tech computer skills, but the growth of new
media has (52)…………..up a wide range of Internet career opportunities requiring only a minimal level of
technical (53) …………... Probably one of the most well-known online job opportunities is the job of
webmaster. However, it is hard to define one basic job description for this position. The qualifications and
responsibilities depend on what tasks a particular organization needs a webmaster to (54)…………...
To specify the job description of a webmaster, one needs to identify the hardware and software that the
website will manage to run (55)…………...Different types of hardware and software require different skill
sets to manage them. Another key factor is whether the website will be running internally or externally.
Finally, the responsibilities of a webmaster also depend on whether he or she will be working independently,
or whether the firm will provide people to help. All of these factors need to be considered before one can
create requiring (56) …………..knowledge of the latest computer applications. (57)…………..,there are also
online jobs available for which traditional skills remain in high (58) …………... Content jobs require
excellent writing skills and a good sense of the web as a “new media”.
The- term “new media” is difficult to define because it encompasses a (59) …………..growing set of
new technologies and skills. Specifically, it includes websites, email, Internet technology, CD-ROM, DVD,
sưeaming audio and video, interactive multimedia presentations, e-books, digital music, computer illustration,
video games, (60)…………..reality, and computer artistry.
51. A. Apart from B. Contrary to C. Prior to D. In contrast to
52. A. taken B. sped C. set D. opened
53. A. expertise B. master C. efficiency D. excellency
54. A. conduct B. perform C. undergone D. overtake
55. A. on B. over C. in D. with
56. A. built-in B. up-market C. in-service D. in-depth
57. A. However B. Therefore C. Moreover D. Then
58. A. content B. demand C. reference D. requirement
59. A. constantly B. continually C. increasingly D. invariably
60. A. fancy B. imaginative C. illusive D. virtual
VI. READING PASSAGE 1:
Read the text below and choose the best answer to each question.
THE BALANCE IN THE OCEANS
The oceans’ predators come in all shapes and sizes. For example, one of the less infamous ones is the
colorful starfish, which feeds on plant life, coral, or other shellfish such as mussels for sustenance. A more
bloodcurdling example, especially to human beings and most other species of fish, is the shark, though most
scientists agree that only ten per cent of the 450 plus species of sharks have been documented as actually
attacking a human. Still, there is another predator lurking invisibly in the bodies of water of the world, one
which poses one of the greatest threats to all species of ocean life - bacteria. Though many types of fish are
136

Downloaded by Nhung Nguy?n (nhungnth2004@gmail.com)


lOMoARcPSD|20277325

continually stalking and evading one another for survival, they all band together in an attempt to keep
bacteria levels at bay in order to allow their own existence to continue.
Bacteria play a dual role in the ecotystems of the oceans. On the one hand, they are beneficial as they
stimulate plant life through food decomposition, which releases the necessary chemicals for the growth of
plant life. This is called nutrient recycling and helps keep the oceans alive. But, on the other hand, bacteria
are a major predator for all fish because they attack fragile, weaker individuals. If they are allowed to run
rampant and not kept in check, they could virtually suffocate the oceans. In water, bacteria prove to be an even
greater threat than on land because, as they proliferate, they reduced the oxygen levels necessary for
organisms in the oceans to live. Further, when fish populations become depleted due to factors like
overfishing, microbes such as algae expand and threaten the fragile ecosystems of the ocean. Therefore, ocean
predators play a critical role by thwarting bacteria growth and maintaining the oceans’ equilibrium by
reducing vulnerable links in the food chain.
In many ways, the balance within the oceans’ ecosystems mirrors the human body. That is, all of their
components must work in harmony for them to stay healthy, efficient, and alive. If one of them is missing or
deficient, an entire system can be placed in jeopardy. In both the human body and the ocean, bacteria play a
vital role because, at manageable levels, they aid in protecting and cleaning each system of foreign agents that
can be of harm. On the other hand, if bacteria levels increase and become out of control, they can take hold of
a system, overrun, it, and become debilitating. Therefore, both oceans and the human body have a kind of
custodian that maintains bacteria levels. In the human body, it is called a phagocyte. Phagocytes eat up sick,
old, or dying cells, which are more prone to bacterial invasion, and thus keep the body healthy. Like in the
human body, bacteria can prove fatal to the living organisms in the ocean.
Like phagocytes in the human body ocean predators work as antibacterial custodians of the seas. In
essence, they are the immune system and a vital link in the food chain because they remove small, injured, or
sickly fish from the ocean environment before bacteria can become too comfortable and multiply. By ridding
the ocean of weaker fish, predators allow the stronger ones to multiply, making their species stronger and
more resilient. Without their services and with their declining numbers, bacteria will blossom to levels that
will eventually overpower and kill even the strongest species of fish, because of the depletion of their number
one source of life, all important oxygen.
While the greatest battle in the ocean may seem on the surface to be the survival of the fittest fish, a
closer look reveals something completely different: fish versus microorganisms. Clearly, most living
organisms in the oceans are hunters by nature, but this way of life does not merely provide a food source for a
dominant species. It also maintains a heạlthy level of bacteria in an ocean’s ecosystem, thus ensuring the
continuation of all species of life within. Major predators are necessary, like the antibacterial cells of the
human body, to keep this delicate balance in synch. If their numbers continue to decline and humans ignore
their vital role in the ocean, dire consequences will definitely result.
61. The word lurking in the passage is closest in meaning to …………...
A. attacking B. increasing C. waiting D. approaching
52. According to paragraph 1, which of the following is true of ocean predators?
A. The shark is the deadliest one for all other kinds of life in the oceans.
B. One of the most threatening to all fish populations is bacteria.
C. Starfish do little damage to the population of mussels and shellfish.
D. Most of the killers that hide in the oceans are unknown to humans.
63. Which of the following can be inferred from paragraph 1 about bacteria?
A. They can be extremely detrimental to fish if their numbers increase.
B. They are able to feed off themselves when other food sources are limited.
C. They stimulate plant life, which in turn releases oxygen into the water.
D. They present themselves in numerous shapes and forms as well as colors.
64. Which of the sentences below best expresses the essential information in the sentence in bold (“Though...
continue”)?
137

Downloaded by Nhung Nguy?n (nhungnth2004@gmail.com)


lOMoARcPSD|20277325

A. Evasion tactics help fish escape from the threats posed by an increasing number of bacteria.
B. Various species of fish prey upon one another in order to lower bacteria levels in the ocean.
C. High bacteria levels in the ocean help most species of fish tq survive by providing them with food.
D. Rivals or not, all fish help one another survive by preventing bacteria from proliferating.
65. The author discusses nutrient recycling in paragraph 2 in order to…………...
A. show how bacteria act similarly in the ocean and the human body
B. explain the different roles of nutrients and oxygen for species of fish
C. indicate that bacteria do have a positive impact in the oceans
D. note how chemicals from bacteria are able to stimulate plant growth
66. The word thwarting in the passage is closest in meaning to…………....
A. encouraging B. preventing C. slowing D. sustaining
67. According to paragraph 2, bacteria are dangerous to ocean life because
A. they have the capability to attack both strong and weaker fish
B. they could monopolize the critical breathable gas in the ocean
C. they get rid of vulnerable links, like dying fish, in the ịíood chain
D. they blossom out of control when overfishing becomes dominant
68. The word debilitating in the passage is closest in meaning to…………....
A. stimulating B. hindering C. elevating D. weakening
69. The author’s description of phagocytes mentions all. of the following EXCEPT:
A. They rid the human body of potentially dangerous organisms.
B. They act in a similar manner as the predatqrs of the ocean.
C. They dispose of bacteria to make weakened cells revive.
D. They are cleaning agents in humans to maintain bacteria levels.
70. According to paragraph 4, the elimination of weaker fish by ocean predators…………....
A. can often have an adverse effect on the population of the certain prey species
B. inadvertently helps stronger species of fish to proliferate more easily
C. reduces oxygen levels, thereby causing bacteria to multiply in their prey
D. allows bacteria to grow and multiply in the stronger individuals of a species
READING PASSAGE 2 :
Read the text below and choose the best answer to each question.
UFFINGTON WHITE HORSE: ANCIENT HILLSIDE CHALK ART
When I was a teenager, I once carved my initials and thọse of my girlfriend into a tree, something I
thought of at the time as being a permanent statement of our eternal devotion to each other. When we broke up
a year later, I felt obliged to return to the tree, put an X through our initials, and add the words ‘Null and
Void’. The next time I went to find the tree, a number of years after that, it was gone. My guess is that the tree
was so ashamed at having been defaced with self-contradictory graffiti that it simply fell over in act of suicidal
protest.
The urge to leave one’s mark on the landscape - whether in a tree, a newly poured sidewalk, or the wall
of a cave - goes way, way back. One rather unusual form of ancient markings is found in the picturesque,
pastoral setting of rural England. About a 30-minute drive from the city of Oxford is a large area covered
with the rolling green hills and herds of grazing sheep that have found their way into countless works of
literature and film. Beneath the veneer of grass and soil, some of these hills are made of chalk. And over the
millennia, the landscape has become dotted with at least fifty large images made by carving through the top
layers of earth to expose the chalk beneath. Of these, about a dozen are pictures of horses, and of the horse
carvings, the oldest and best known is the Uffington White Horse.
Although less famous than, say, Stonehenge, the Uffington White Horse ranks right up there among
ancient and inexplicable English monuments. It is a highly stylized outline of a horse - recognizable, but not
as well-defined as the other, more solid horse images. The carving is about 374 feet (113m) long, with the
lines forming it ranging in width from about 5 to 10 feet (two to three meters). This particular carving doesn’t
138

Downloaded by Nhung Nguy?n (nhungnth2004@gmail.com)


lOMoARcPSD|20277325

actually go all the way through the crust to the chalk beneath; instead; a relatively shallow trench was dug
and filled in with chalk to make it almost flush with the surface.
The Uffington White horse has the distinction of being the largest of Britain’s horse carvings
(measured from head to tail).It’s also one of only four such horses facing to the right, though no one knows for
sure the significance of the horse’s direction, if any. And it’s the oldest horse carving, meaning it may have
served as a prototype for the others.
Scientists have determined that the carving is about 3000 years old (give or take a few centuries), and
though it is mentioned in literature dating back to the eleventh century, its original purpose - along with the
identity of its creators - is uncertain. Conjecture ranges from a tribal emblem, the equivalent of a modern
flag, to a commemorative symbol of King Alfred’s triumphs over the Danes. Or perhaps it was a Celtic
symbol of the goddess Epona, whose job was to protect horses. It may have been a territory marker, or simply
(perish the thought) a giant piece of abstract art. Although it has been referred to as a “horse” for at least 1,000
years, there are some who believe that it was intended to represent a dragon. If so, then dragons must have
been much more horse-shaped in those days. In any case, the carving has been well tended over the centuries.
Every seven years, weeds are removed and the outline smoothed to maintain its original size and shape.
One of the most interesting things about the Uffington White Horse is that the only place to get a good
view of the whole thing is from the sky above. There are a few spots several miles away that provide a fair
view of most of the outline, but the local topography is such that there is just no vantage point from which you
can get a good view of the whole horse. This has, predictably, led some people to speculate that it was created
as a signal to UFOs, although what exactly it would signify is a bit unclear (“Horses for sale - next exit?”). Be
that as it may, this peculiarity of perspective must have made it a challenge to carve, and it makes the horse’s
original purpose all the more mysterious.
71. The word “pastoral” is closest in meaning to…………..
A. peaceful B. panoramic C. delirious D. unpolluted
72. Why does the writer tell us about the time he carved initials into a tree?
A. To prove that hẹ loved his girlfriend.
B. To tell us about the damage we can do to the landscape.
C. To illustrate our need to preserve the present into the future.
D. To show that nothing lasts forever.
73. The writer mentions the area around Oxford to illustrate that it…………..
A. has featured in too many books and films.
B. lends itself to the creation of hill carvings.
C. is covered in chalk and grass.
D. is covered in carvings of horses.
74. The horse of Uffington differs in that…………..
A. it’s not as well known as many of Britain’s monuments.
B. nobody knows why it was originally built.
C. it wasn’t carved into the chalk like the others.
D. it’s the only horse that does not face to the left.
75. The word “trench” is closest in meaning to…………..
A. a shallow pond B. a deep track
C. a long deep hole D. a short bending tunnel
16. The White Horse of Uffington…………...
A. is the largest carved chalk figure in Britain.
B. may well have inspired similar designs.
C. was first mentioned 3000 years ago.
D. is a few centuries older than originally thought.
77. The true function of the white horse is thought to be…………...
A. a religious symbol. B. a monument to a victory in battle.
139

Downloaded by Nhung Nguy?n (nhungnth2004@gmail.com)


lOMoARcPSD|20277325

C. a decorative design. D. a mystery.


78. The writer believes…………...
A. the carving looks more like a horse than a dragon.
B. the horse was originally meant to be a dragon.
C. the carving has changed shape over the years.
D. the original design is restored every seven years.
79. The writer believes that…………...
A. the horse may have been built to attract;extra-terrestrial.
B. there’s no logical reason to assume the carving was intended for aliens.
C. the whole horse can be seen well from several places.
D. it’s unclear how the original designers used perspective.
80. The word “Conjecture” is closest in meaning to…………...
A. Belief B. Vision C. Guess D. Saying
B. WRITTEN TEST
I. CLOZE TEST:
Read the text below and think of the word which best fits each space. Use only ONE WORD for each
space.
OPEN CLOZE 1
he first two decades of this century were dominated by the microbe hunters. These hunters had tracked
(1)…………… one after another of the microbes responsible for the most dreaded scourges of many
centuries: tuberculosis, cholera, diphtheria. But (2)…………… remained some terrible diseases for which no
microbe could be incriminated: scurvy, pellagra, rickets, beriberi. Then it was discovered that these diseases
were (3)……………, by the lack of vitamins, a trace substance in the diet. The disease could be prevented or
cured by consuming foods that contained the vitamins. And so in the (4)……………of the 1920’s and 1950’s,
nutrition became a science and the vitamin hunters replaced the microbe hunters.
In the 1940’s and 1950’s, biochemists strived to learn (5)……………each of the vitamins was
essential for health. They discovered that key enzymes in metabolism depend on one or (6)……………of the
vitamins as coenzymes for growth and function. Now, these enzymes hunters occupied center stage.
You are aware that the enzyme hunters have been replaced by a new breed of hunters who are tracking
genes - the blueprints for each of the enzymes and are discovering the defective genes that caused (7) ………
diseases - diabetes, cystic fibrosis. These gene hunters, or genetic engineers, use recombinant DNA
technology to identity and clone genes and (8)……………them into bacterial cells and plants to create
factories for the massive production of hormones and (9)……………for medicine and for better crops for
agriculture. Biotechnology has become a multibillion dollar (10)…………….
OPEN CLOZE 2
Situated in the central mountains of Alaska, a peak named Denali rises 20,320 feet above sea level. It
is the (11)……………peak in North America and the center of Denali National park. One of the America’s
greatest wilderness areas, the park has had (12)……………access to visitors, but in spite of this, tourism
rose from 6000 visitors in 1950 to over 546,000 visitors in 1990. The increasing (13)……………of this park
is prompting serious discussions about the future use of Denali as well as how to preserve wilderness areas in
(14)…………….
One important issue of land use arises when parts of the National Park are owned by individuals. In
Denali, (15)……………most of the land in this vast tract of more than a million acres is owned by the
National Park Service, several thousand acres are still (16)……………owned as mining tracts. These mining
tracts in Denali were once abundant sources of gold, but they also were sources of (17)……………metals
such as arsenic and lead that polluted rivers and streams.
Environmentalists were successful in getting the government to require mining companies to submit
statements showing the potential impact of a mining project before they are (18)……………to begin mining.
Because of this requirement, many individuals closed their mines and some sold their land to the National
140

Downloaded by Nhung Nguy?n (nhungnth2004@gmail.com)


lOMoARcPSD|20277325

Park Service. Some land owners, (19)……………, are wondering if it is better to sell their land to the
government or keep it for possible future use. Tourism in this previously remote area is bound to rise, as more
roads are built to provide easier access to the park. This increase in the number of visitors creates a demand
for hotels and other real estate development. The economic implications of this, are of (20)……………to the
land owners, but are dismaying to those interested in, preserving the wilderness.
II. WORD FORMATION:
PART 1: Complete each sentence, using the correct form of the wordin parentheses
1. The…………..sports stadium is the pride of the city. (FUTURE)
2. She is one of those…………..people who never get angry or upset. (PERTURB)
3. Nadal has defeated his…………..Ferrer in the quarter final. (PATRIOTISM)
4. Salt water has more…………..than fresh water. (BUOY)
5. His…………..injury put him out of contention for the championship. (CAPACITY)
6. She …………..the things we need to buy^sugar, tea, and sandwiches. (NUMBER)
7.………….., we should stay in a hostel instead of a hotel. (BUDGET)
8. The new BMW has a more powerful engine than its………….. (PRECEDE)
9. She …………..herself, left the farm and moved to London. (ROOT)
10. Many fish…………..protect their eggs from predators. (GENIUS)
PART 2: Complete the passage with appropriate forms from the words given in the box
south lie exploit ultimacy ornament
hill migrate front conserve originate
The whooping crane is a grand, (11)………….. waterfowl native only to North America. Its features
are striking, as a mature whooping crane is all white and stands five feet tall with a wingspan of about eight
feet. Previously, wild flocks spent the warmer parts of the year in their native habitat of northern Canada, and,
around August, they made the 2,500-mile (12)…………..journey to the gulf coast of Texas in the United
States. Today, only one flock remains in the wild, and it consists of about one hundred and fifty to two
hundred whooping cranes. Continuing (13)…………..efforts are attempting to increase its populations, which
face, as they traditionally have, a/an (14)…………..battle, by introducing new habitats and educational
methods of migration.
There are two major factors which (15) ………….. the decline of the whooping crane in the early
twentieth century. First, they were (16)………….. hunted by people for food as well as beautiful feathers,
which were used for (17) …………..purposes. Second, their natural habitat wetlands were beginning to dry up
due to agricultural (18)…………..and development. Thirdly, which perhaps had the most dramatic effect, was
the fact that the eggs of the whooping crane were prized by collectors. Once the eggs were pillaged from
nests, future generations became placed even more in jeopardy. Fortunately, the whooping crane, like other
endangered species such as the bald eagle, (19)…………..became a protected species by federal law. The
issue of how to increase their numbers back to comfortable levels, therefore, has moved to the (20) ………….
of attention.
III. ERROR CORRECTION:
The following passage contains 10 errors. Identify and correct them.
Since water is the basis of life, composing the greater part of the tissues of all living things, the crucial
problem of desert animals is to survive in a world where sources of flowing water are rare. And since man’s
inexorably necessity is to absorb large quantities of water in frequent intervals, he can scarcely comprehend
what many creatures of the desert pass their entire lives without a single drop.
Uncompromising as it is, the desert has not eliminated life but only those forms unable to withstand its
desiccating effects. No moist-skinned, water-loving animals can exist there. Few large animals are found. The
giants of the North American desert are the deer, the coyote, and the bobcat. Since desert country is open, it
holds more swift-footed running and leaping creations than the tangled forest. Its population is hugely
nocturnal, silent, filled with reticence, and ruled stealth. Yet they are not emaciated.
Having adapted to their austere environment, they are as healthful as animals anywhere else in the
141

Downloaded by Nhung Nguy?n (nhungnth2004@gmail.com)


lOMoARcPSD|20277325

world. The secret of their adjustment lies in the combination of behavior and physiology. Nobody could
survive if, like mad dogs and Englishmen, they go out in the midday sun; many would die in a while of
minutes. So most of them pass the burning hours asleep in cool, humid burrows underneath the ground,
emerging to hunt only by night. The coverage of the sun-baked desert averages around 150 degrees, but 18
inches down the air is only 60 degrees.
IV. SENTENCE TRANSFORMATION:
Rewrite the following sentences using the words given.
1. Richard only took over the family business because his father decided to retire early.
→ Had…………………………………………………………………………………………………
2. We would always take great care when flying at night. (WITS)
→ We always used……………………………………………………………………………………..
3. There haven’t been such long queues at the cinema since the release of the last blockbuster.
→ Not …………………………………………………………………………………………………
4. I will only be satisfied if the manager apologizes fully.
→ Nothing short ……………………………………………………………………………………..
5. She was concentrating so hard on her work that she didn’t notice when I came in. (WRAPPED)
→ She was so…………………………………………………………………………………………
6. The price of the house has been reduced as much as possible because the owner needs some money.
(BONE)
→ The house………………………………………………………………………………………….
7. The permit expires at the end of the month.
→ The permit is not…………………………………………………………………………………..
8. The first sign of the disease is blurred vision. (ONSET)
→……………………………………………………………………………………………………..
9. Don’t say anything negative about her hair because she’s very sensitive and might be offended by your
remarks. (OFFENCE)
→ …………………………………………………………………………………………………….
10. The new teacher was so nervous that the class reacted mischievously. (REACTION)
→ The new teacher’s…………………………………………………………………………………

TRƯỜNG THPT CHUYÊN LÊ HỒNG PHONG - TP. HCM


A. MULTIPLE CHOICE
I. PHONOLOGY:
Choose the word whose underlined part is pronounced differently from the other three.
1. a. residue b. hesitancy c. capsicum d. herpes
2. a. facsimile b. hyperbole c. apostrophe d. Anglophobe
3. a. legionary b. pesto c. sesame d. menace
4. a. debris b. dermis c. islet d. ascendency
5. a. seizure b. putz c. pizza d. blitz
Choose the word whose stress pattern is different from that of the other three.
6. a. acropolis b. puppeteer c. antenna d. contributory
7. a. mischievous b. algorithm c. calligraphy d. heliograph
8. a. magnifier b. preferable c. jeopardy d. interpolate
9. a. elite b. consul c. obscure d. baboon
10. a. triathlon b. maidservant c. pragmatic d. precarious
II. VOCABULARY AND STRUCTURE.
142

Downloaded by Nhung Nguy?n (nhungnth2004@gmail.com)


lOMoARcPSD|20277325

Choose the best answer to each questions.


11. Everyone was shocked to hear that a politician of his…………would stoop so low.
A. credence B. stature C. guile D. affinity
12. She opened a window to allow the smoke from the burnt meal to…………
A. debilitate B. extricate C. dissipate D. evaporate
13. Her young daughter is really…………one would think she’s twice her age.
A. incongruous B. extraneous C. superfluous D. precocious
14. The firemen who died while battling the forest fire were honored………….
A. posthumously B. ostensibly C. laboriously D. expressly
15. The abandoned warehouse was used by smugglers to hide .…………
A. merchandise B. spoils C. fodder D. contraband
16. The Baileys have made…………plans for a vacation in August; nothing is certain yet.
A. tentative B. culpable C. sagacious D. exemplary
17. The new butcher is a most individual; he never stops talking!
A. equivocal B. magnanimous C.garrulous D. meticulous
18. e decided to withdraw from the powerboat race as he had a(n)…………of danger.
A. interim B. foreboding C. dearth D. prediction
19. The…………with which she approached the routine test was quite uncalled for.
A. trepidation B. composure C. terrain D. overture
20. Mother Nature has provided the island with a…………supply of natural resources.
A. vociferous B. viable C. gregarious D. bountiful
21. You can put an advert in some magazines, newspapers,………….
A. or whatever else B. or suchlike C. and the same D. or similarly
22. For the first few months the babies looked so alike I couldn't tell………….
A. who is who B. which is which
C. which from which D. whom with whom
23.…………, the meeting began.
A. After we have sat down B. All of us having taken the seats
C. Our having seated D. Once we had seated
24. They must have gone away,…………?
A. shouldn’t they B. haven’t they C. mustn’t they D. didn’t they
25. “Why did Tom’s boss ask to see him?” - “He told him it was necessary…………”
A. that Tom obtain a driving license B. Tom to obtain a driving license
C. Tom must obtain a driving license D. Tom obtaining a driving license
26. The king was delighted…………....a little surprised by the arrival of his uninvited guests.
A. let alone B. beside C. except D. if
27. It was strange that you…………….staying in the same hotel last year.
A. could have been B. should have been
C. might have been D. must have been
28. …………, he will have to be punished.
A. Should he have cheated B. Were he cheated C. Did he cheat D. If cheated
29. I’d…………be a doctor than be a teacher.
A. prefer B. sooner C. as well D. more
30. When he started that company, he really went…………. It might have been a disaster.
A. out on the limb B. on and off C. over the odds D. once too often
31. He’s doing…………again, this time for armed robbery.
A. gauntlet B. powder C. skeleton D. porridge
32. She ………… scorn on his plans to get rich quickly.
A. threw B. splashed C. poured D. spread
143

Downloaded by Nhung Nguy?n (nhungnth2004@gmail.com)


lOMoARcPSD|20277325

33. Everyone was there save…………Linda. She was too sick to come.
A. on B. with C. to D. on
34. I am too busy now. You’d better…………it…………with your mother.
A. come-off B. make - off C. bring-over D. take - up
35. As soon as their policies became popular, all the other parties started to…………on the bandwagon.
A. jump B. heap C.join D. walk
36. There are $25,000 worth of prizes up for…………in our competition.
A. grasps B. grabs C. cracks D. crabs
37. The fighting has stopped, so to…………, the war is over.
A. prim and proper B. pins and needles
C. intents and purposes D. chop and change
38. I’d like to take…………with you about what you have just said.
A. words B. discussion C. issue D. matter
39. I promise you he’s…………. He’s never been involved in anything criminal.
A. spoilt for choice B. below par C. off the peg D. on the level
40. Even though this is a film about politics, the writer manages to …………many references to football
which I thought very strange.
A. make out B. work in C. put up D. bring on
III. READING COMPREHENSION:
Read the following passage and choose the best option to complete the blank or answer the question.
PASSAGE!
THE HISTORY OF THE ENGLISH LANGUAGE
The English language is part of the Indo-European family of languages. Most modern European
languages (but not, for example, Hungarian) and some Asian languages, like Hindi and Sanskirt, are members
of this family as well. Two European branches of Indo-European are particularly important to consider
when looking at the history of how English developed: Germanic languages and the Romance
languages, such as French, Italian, and Spanish, which have their roots in Latin.
English is in the Germanic branch of Indo-European languages. This branch of languages came into
existence about three thousand years ago in an area around the Elbe River in what is now Germany. Around
the second century BC, what had begun as one Germanic language split off into three disparate groups. One of
these groups eventually evolved to become the German language spoken today* Another group was known as
West Germanic.
In the fifth and sixth centuries AD, West Germanic invaders called the Angles and the Saxons came to
the British Isles. They spoke two dialects of West Germanic that were similar to one another, and these
dialects merged into what we today call Old English, which was very similar to West Germanic. These
invaders pushed the original Celtic-speaking inhabitants out of what is now England. There are still a few
Celtics words in English, though. The English language of that period was also influenced by the Vikings.
Old English lasted until 1100, about the time of Norman Conquest. When the Normans, residents of
what is now a region of France, invaded and conquered England, the English language was forever changed.
The Normans spoke a dialect of Old French known as Anglo-Norman, whiqh had both French and Germanic
influences. This became the chosen language of the upper class of England, the Normans, for more than one
hundred years, while the majority of people continued to speak Old English. However, the loss of Normandy
to France in 1204 changed this situation. The Norman nobles started to sever their connections with France
and speak a modified version of Anglo-Norman. This new language, which was a combination of Anglo-
Norman and Old English, is now known as Middle English. While modern speakers of English would not be
able to read or understand Old English, they could, with some difficulty, read Middle English. By 1362,
English had become the official language of England.
The alterations that marked these early periods of English were not the end of its transformation,
however. During the Renaissance, English changed again. One reason for this was that many Latin and Greek
144

Downloaded by Nhung Nguy?n (nhungnth2004@gmail.com)


lOMoARcPSD|20277325

words were introduced into English as scholars rediscovered important ancient works written in those
languages. In addition, many other completely original words were introduced into English, and the famous
playwright William Shakespeare is credited with having either created or recorded over two thousand of these
words.
There are two other major influences in the transformation of Middle English into Modern English.
One is called the ‘Great Vowel Shift’. This term refers to a change in pronunciation that began around 1400.
Though a Modern English reader could make sense of something written in Middle English, it would sound
completely foreign. One example of the change in pronunciation is that the letter ‘e’ at the end of many
English words became silent. Middle English speakers would have pronounced it. The other influence in the
development of Modem English was the invention of the printing press and the subsequent proliferation of
books. Prior to the printing press, spelling, grammar, and even usage were very flexible. However, with the
advent of printed texts, these elements of the language became standardized, so there has been far less
variability in Modern English than there was in earlier forms.
Changes since the time of Shakespeare have been mainly in the category of vocabulary. Technological
advances have made necessary the creation of words to describe new objects, processes, and ideas. Industry,
computers, and space travel all required new descriptive terms. Words can also fall out of usage from one
generation to the text. Another reason for the evolving vocabularies has been the increasing communication
between and the migration of people all over the world. Words from foreign places have been absorbed into
the language. For example, the common ‘shampoo’ comes from the Hindi language. Overall, it can be seen
that English has undergone great transformations since its origin, over three thousand years ago, on the shores
of the Elbe River.
1. Which of the following sentences below best expresses the essential information in the highlighted sentence
in the passage? Incorrect choices change the meaning in important ways or leave out essential information.
A. Romance languages are derived from Latin and include French, Italian, and Spanish.
B. Germanic and Romance languages significantly influenced the development of English.
C. Two European language families broke into many languages that are currently spoken throughout Europe.
D. English evolved from a mixture of many languages, such as French, Italian, and Spanish.
2. According to paragraph 1 and paragraph 2, which of the following is TRUE about the English language?
A. It is more closely related to Sanskirt than Hungarian.
B. It is considered a Romance language, like Spanish.
C. It is more closely related to French than German. .
D. It is not considered an Indo-European language.
3. According to paragraphs, which of the following is TRUE about Old English?
A. It was most influenced by the Celtic and Viking languages.
B. It was widely used prior to the fifth and sixth centuries AD.
C. It was similar to the language spoken by the original inhabitants of England.
D. It was a combination of two groups of the West Germanic language family.
4. The word SEVER in the passage is closest in meaning to…………..
A. break B. define C. change D. support
5. In paragraph 4, the author states that…………...
A. immediately following the Norman Conquest, the residents of England spoke Middle English,
B. the nobility spoke a different language than the common people for more than a century in England.
C. the loss of Normandy to France began a period of cultural interchange between England and France.
D. Middle English texts would be impossible for modern English speakers to read and understand.
6. Why does the author discuss current English speakers in paragraph 4?
A. To compare the modern British population with that of 1204.
B. To clarify the relationship between the Normans and the Anglo-Saxons.
C. To emphasize how long English has been the official language of England.
D. To highlight a difference between Old English and Middle English.
145

Downloaded by Nhung Nguy?n (nhungnth2004@gmail.com)


lOMoARcPSD|20277325

7. The word PROLIFERATION in the passage is closest in meaning to…………...


A. writing style B. rapid creation
C. modern design D. unintended effect
8. In paragraph 6, the author describes THE ‘GREAT VOWEL SHIFT’ by .…………...
A. discussing the standardization of spelling, grammar, and usage.
B. citing a specific example of a pronunciation divergence.
C. explaining the results of the innovation of the printing press.
D. defining the difference between spoken and written Middle English.
9. Based on the information in paragraph 5 and paragraph 6, what can be inferred about how Middle English
changed into Modern English
A. The transition was a smooth one because Middle English sounded very similar to Modern English?
B. The transformation occurred as a result of developments in English society
C. Modern English created a precedent by pronouncing vowels that had been silent in Middle English.
D. Playwrights played a larger role in developing the language than most people realize.
10. In paragraph 7, why does the author mention COMPUTER?
A. To provide an example of a word that is derived from Middle English
B. To show how English has not changed much since the Renaissance
C. To identify a foreign-language word that has become part of English
D. To illustrate a modern creation that has resulted in new English words
PASSAGE 2
People appear to be born to compute. The numerical skills of children develop so early and so
inexorably that it is easy to imagine an internal clock of mathematical maturity guiding their growth. Not long
after learning to walk and talk, they can set the table with impressive accuracy - one plate, one knife, one
spoon, one fork, for each of the five chairs. Soon they are capable of noting that they have placed five knives,
spoons, and forks on the table and, a bit later, that this amounts to fifteen pieces of silverware. Having thus
mastered addition, they move on to subtraction. It seems almost reasonable to expect that if a child were
secluded on a desert island at birth and retrieved seven years later, he or she could enter a second-grade
mathematics class without any serious problems of intellectual adjustment.
Of course, the truth is not so simple. This century, the work of cognitive psychologists has illuminated
the subtle forms of daily learning on which intellectual progress depends. Children were observed as they
slowly grasped - or, as the case might be, bumped into - concepts that adults take for granted, as they refused,
for instance, to concede that quantity is unchanged as water pours from a short stout glass into a tall thin one.
Psychologists have since demonstrated that young children, asked to count the pencils in a pile, readily report
the number of blue or red pencils, but must be coaxed into finding the total. Such studies have suggested that
the rudiments of mathematics are mastered gradually, and with effort. They have also suggested that the very
concept of abstract numbers - the idea of a oneness, a twoness, a threeness that applies to any class of objects
and is a prerequisite for doing anything more mathematically demanding than settings a table - is itself far
from innate.
1. What does the passage mainly discuss?
A. Trends in teaching mathematics to children
B. The use of mathematics in child psychology
C. The development of mathematical ability in children
D. The fundamental concepts of mathematics that children must learn
2. It can be inferred from the passage that children normally learn simple counting……………
A. soon after they learn to talk
B. by looking at the clock
C. when they begin to be mathematically mature
D. after they reach second grade in school
3. The word “illuminated” is closest in meaning to…………….
146

Downloaded by Nhung Nguy?n (nhungnth2004@gmail.com)


lOMoARcPSD|20277325

A. Illustrated B. accepted C. clarified D. lighted


4. The author implies that most small children believe that the quantity of water changes when it is transferred
to a container of a different…………….
A. Color B. quality C. weight. D. shape
5. According to the passage, when small children were asked to count a pile of red and blue pencils
they……………
A. counted the number of pencils of each color
B. guessed at the total number of pencils
C. counted only the pencils of their favorite color
D. subtracted the number of red pencils from the number of blue pencils
6. The word “They” refers to…………….
A. Mathematicians B. children C. pencils. D. studies
7. The word “prerequisite” in line 23 is closest in meaning to…………….
A. Reason B. theory C. requirement D. technique
8. The woijd “itself’ refers to……………
A. the total C. the concept of abstract numbers
B. any class of objects D. settings a table
9. With which of the following statements would the author be LEAST likely to agree?
A. Children naturally and easily learn mathematics.
B. Children learn to add before they learn to subtract.
C. Most people follow the same pattern of mathematical development.
D. Mathematical development if subtle and gradual.
10. Where in the passage does the author give an example of a hypothetical experiment?
A. Lines 3 - 6 B. Linnes 8 - 10 C. Lines 11 - 14 D. Lines 17 - 20
IV. GUIDED CLOZE:
Read the following passages and choose the options that best complete the blanks. GUIDED CLOZE 1:
HORSES FOR COURSES
American psychologists have been looking into how a child’s position in the family can affect his or
her selected career.
It would appear that first bom and only children are more likely to be high achievers,(1)………….
professions such as law, finance, engineering or medicine. According to Professor Frederick Leong, co-author
of the psychologists’ report, the (2)………….is for them to head towards ‘cognitive and analytical’ work.
Professor Leong puts this (3) ………….to the fact that parents are more likely to be more over-(4)
………….towards first born or only children. Examples of famous names demonstrating this principle include
Anthony Scrivener QC, Sir Winston Churchill and Sir Edward Heath.
Younger siblings, on the other hand, are more likely to be (5)………….to take up more creatively
based or outdoor based occupations, such as landscape gardening, fashion, music and art as (6) ………….by
Oscar Wilde, Yehudi Menuhin and Madonna.
Professor Leong’s overall explanation is that, ‘Parents typically (7)………….different demands and
have different (8)…………. of children, depending on their birth order. Parents of only children may
discourage pursuit of physical or outdoor activities because they are more fearful of physical harm to their
child. That, and the fact that they get more time and attention from their parents than children with siblings,
may be why only children are likely to be academic.’
He goes on to point out how the (9)…………., tends to happen in larger families, ‘As they have more
children, parents tend to become more open and relaxed, and that may allow younger children to be more risk-
taking. If the first born or only child wants to be a poet, that may concern parents. But by the fourth child
parents may not (10),………….as much.’
1. A. pursuing B. chasing C. driving D. tracking
2. A. trend B. direction C. bias D. tendency
147

Downloaded by Nhung Nguy?n (nhungnth2004@gmail.com)


lOMoARcPSD|20277325

3. A. on B. up C. down D. over
4. A. guarding B. protective C. covering D. watchful
5. A. destined B. intended C. designed D. fated
6. A. exhibited B. displayed C. depicted D. exemplified
7. A.rest B. plant C. fix D. place
8. A. prospects B. expectations C. outlook D. possibilities
9. A. alternative B. counter C. converse D. contrast
10. A. oppose B. mind C. dispute D. contend
GUIDED CLOZE TEST 2
COCA-COLA
Coca-Cola was invented in 1886 by John Pemberton, a 50-year-old chemist from Atlanta, USA. He
decided to develop a soft drink to sell as a “brain tonic”. Working tirelessly in the back room of his drugstore,
he produced a (1)………….containing, among other things, coca leaves, cola nut oil, sugar and caffeine. (The
exact (2) ………….is still a secret, but the tiny cocaine content was removed in 1903). A few months later, an
assistant (3)…………. a customer Coca-Cola mixed with soda water by mistake. It was this small addition
that turned out to be the vital (4)…………. that made the drink a success.
Coke has always been cleverly marketed. The (5)………….. design of the bottle was introduced in
1915 to prevent imitations and a 1920s advertising campaign even gave the world Father Christmas as we now
know him - with a red and white costume rather than the blue, yellow or green he had often (6)………….
worn. Coca-Cola was a major sponsor of the 1984 Olympic Games in Los Angeles, as well as being the
Games’ “Official Drink”.
The red and white Coca-Cola logo can be (7)………….at once- a powerful symbol of the American
way of life. Coke was the most widely (8)………….mass-produced item in America when World War II broke
out and the war provided the (9)………….to spread the product into Europe and Asia. When
conservative Europeans complained about the invasion of modem American values (10)………….their
ancient cultures, the act of drinking Coke became for the young a form of rebellion against tradition. Even
today, Coke is still closely linked with the image of youth.
1. A. composition B. mixture C. chemical D. drug
2. A. receipt B. menu C. recipe D. prescription
3. A. provided B. supplied C. served D. presented
4. A. material B. dishes C. ingredient D. proportion
5. A. distinctive B. absolute C. perfect D. representative
6. A. already B. previously C. formally D. lastly
7. A. found B. realized C. recognized D. identified
8. A. promoted B. distributed C. located D. dispensed
9. A. possibility B. hope C. occasion D. opportunity
10. A. into B. to C. with D. in
B. WRITTEN TEST
I. CLOZE TEST:
Fill each blank with ONE word.
OPEN CLOZE TEST 1
It’s the curse of online clothes shopping. You come across a shirt you simply must have, (1)
………….to find that what you receive doesn’t fit (2)………….being in your size. How can you order clothes
(3)………….confidence when you can’t try them on?
A new wave of start-ups are finding clever ways to address the problem. Virtual (4)…………. rooms
are one solution. The London-based firm Fits.me, founded in 2010, creates them for brands such as Hugo
Boss and Superdry. The company (5)………….up with researchers at several universities to build robot
mannequins that can adjust their proportions to match just about any set of human measurements.
To set up the room, developers run through most of the size-shape combinations the dummies can
148

Downloaded by Nhung Nguy?n (nhungnth2004@gmail.com)


lOMoARcPSD|20277325

assume, and take several thousand photos of them (6)………….in every available size of each shirt or dress,
from extra small to XXXL. Software then looks at the measurements (7)………….in by shoppers, such as
height, arm length and collar size, and displays the mannequin photo set that (8)………….matches their body
trying on clothes in sizes the user is interested in. Potential problems - where a shirt is too tight, for example
are flagged up. In a trial involving the British clothing brand Henri Lloyd, the return (9)………….for
garments was 4.5 per cent for a group of customers who used the software, compared with 15.3 per cent for a
group that did not.
“There’s no (10)………….that virtual tools will become a standard part of online shopping,” says Fits
me CEO Heikki Haldre.
OPEN CLOZE TEST 2
Eleanor was nearly 50 when she collapsed and died. While African elephants can live up to 70 years,
female life (1) ………….is just 22 in her group in Samburu, Kenya, and Eleanor was the oldest member of
her family - the matriarch. This made her passing particularly significant. For almost a week after her death,
her carcass was visited not just by members of her immediate family, but by a succession of animals from four
unrelated families. Elephants are mysteriously curious about death, a response perhaps (2)………….when a
leader dies.
It has (3) ………….been clear that elephant groups rely on their elder stateswomen, but just how
important these females are is only gradually becoming apparent. Matriarchs are at the hub of a complex,
multilayered social network, and we are now getting insights into the nature of the ties that (4) ………….these
close-knit groups and the key (5)………….that wise old leaders play in enhancing the survival of their
members. Matriarchs carry with them a treasure trove of crucial information. They have a unique influence
over group decision-making. And like our own leaders, the most successful may even possess certain
personality traits.
Much of what we know about elephant social life comes from research done at Amboseli National
Park in Kenya, (6)………….the populations lives in (7)…………. close to a natural, undisturbed state. But
this is unusual. Across Africa elephant numbers are dwindling as (8)………….for ivory has surged in recent
years. On the black market, a pair of tusks can (9)………….the equivalent of 15 years of an unskilled
worker’s salary, so the incentive for poachers is (10)………….. If we want to help elephants we need to
understand the structure and function of leadership within their society.
II. WORD FORMATION:
Part 1: Complete the sentences with the correctforms of the given words.
1. The pilot did manage to get the plane ………….. (AIR)
2. He was still rumbling………….when Pike returned bearing a folder of foolscap sheets. (CONTENT)
3.…………. conditions can be diagnosed from the early stage. (CANCER)
4. I've finally figured out why soap operas are, and logically should be, so popular with generations
of………….people who are too old to go out. (HOUSE)
5. Dozens of police motorcycles have been taken off the road in London after routine inspections found them
not to be………….. (ROAD)
6. The boat had been freshly repainted in bright colours for the occasion, and beside it stood 5 sinewy
………….sailors.(WEATHER)
7. The two-year course in art and design has turned him into a………….young man, interested in fashion and
photography. (GROOM)
8. I had no choice but to cut back to a………….style-getting involved in exceptions only. (HAND)
9. The boats surrounded the whales, drove them into nets, where they became ………….and were rendered
helpless by harpoon thrusts. (MESH)
10. The economy appears to have begun recovering, but businesses don't expect consumers to return
to………….ways anytime soon. (SPEND)
Part 2: Complete the following passage with the correctforms of the given words.
KNOW FRONT FINE PASS RANGE
149

Downloaded by Nhung Nguy?n (nhungnth2004@gmail.com)


lOMoARcPSD|20277325

GROW BACK FOCUS HEAD OPERA

So (1)………….was the first decades of cinema that America and Europe can be forgiven for
assuming that they were the only game in town. In less than twenty years, western cinema has grown out of all
recognition; its (2)………….became the most famous people in the world; it made millions. German directors
used it as an analogue to the human mind and the modernizing city, Soviet emphasized its agitational and
intellectual properties, and the Italians reconfigured it on a/an (3)………….scale. It never occurred to
its financial (4)…………. that another continent might borrow their magic box and make it its own. But film
industries were emerging in Shanghai, Bombay and Tokyo, some of which would (5)………….those in the
west.
Between 1930 and 1935, while China produced more than 500 films, mostly conventionally made in
studios in Shanghai, without soundtracks, India followed a different course. Indian films were stylistically
more (6)………….than the western musical, (7)………….realism and escapist dance witbin individual
consequences, and they were often three hours long rather than Hollywood’s 90 minutes. In Japan, the film
industry did not rival India’s in size but was unusual in other ways. In Tokyo, the director chose the stories and
hired the producer and actors, which led to the production of some of Asia’s finest films in the 1930s and
1940s.
The films of Kenji Mizoguchi were among the greatest of these. His films were usually set in the
nineteenth century and analyzed the way in which the lives of the female characters whom he chose as his (8)
………….point were constrained by the society of the time. He also evolved a sinuous way of moving his
camera in and around a scene, advancing towards significant details but often retreating at moments of (9)
………….or strong feeling. No one had used the camera with such (10) ………….before.
III. ERROR IDENTIFICATION:
Identify 10 errors in the following passage and correct them.
It is hip to be eco-friendly in the business world nowadays, and even the fashion industry has got on
the bandwagon. A network of designers and businesses connected to the industry have collaborated to form
the Ethical Fashion Forum. I'ts aims are, ostensibly, to bring together all the components in the global garment
supplying chain and to promote sustainable practices in the manufacturing of clothing, particularly in
developing countries. The Ethical Fashion Forum encourages manufacturers to improve working conditions
by reducing the usage of dangerous chemicals and dyes in the treatment of fabrics. This is one of the ways, it
is hoped, that the clothing industry can alleviate the impact it has on the environment.
All noble intentions, and admirable indeed. However, the cynicals among us cannot help but to
question the sincerity of entrepreneurs in an industry as famously exploitative as this one. Is the fashion
industry honestly becoming more socially aware? Or is it simply capitalizing in the eco-friendly fad of a
moment in an attempt to combat the growing number of anti-consumerism campaigns by marketing the idea
that the fashion industry is now socially and environmentally aware?
The truth of the matter is that it will be extremely difficult to persuade all the links in the chain to act
responsibly, especially when profits are at stake. Furthermore, we cannot ignore the fickle nature of fashion
itself, which is constantly subject to change tastes.
IV. SENTENCE TRANSFORMATION:
Rewrite the sentences with the given words or beginning in such a way that their meanings remain
unchanged
1. You can be highly intelligent but not have much common sense.
→ Having …………………………………………………………………………………………
2. The audience didn’t enjoy his performance. (DOWN)
→ ……………………………………………………………………………………………………..
3. Although she was upset, there was never any question of revenge. (LAST)
→ Upset.……………………………………………………………………………………………….
4. You must not leave the iron switched on for a long time.
150

Downloaded by Nhung Nguy?n (nhungnth2004@gmail.com)


lOMoARcPSD|20277325

→ On …………………………………………………………………………………………………
5. If you want, you can leave. (LIBERTY)
→ You ……………………………………………………………………………………………. wish.
6. That jacket of yours is worn-out - isn’t it time you bought a new one? (DAYS)
→…………………………………………………………………………………………………….
7. Joe waited for her, pretending to tie his shoelaces. (PRETENCE)
→ ……………………………………………………………………………………………………
8. She didn’t make a noise for fear of waking heirparents. (LEST)
→ ……………………………………………………………………………………………………
9. Because of being injured, he was no longer a candidate for the title. (CONTENTION)
→ Injury …………………………………………………………………………………………….
10. As far as I am concerned, the project is nearly completed. (NEARING)
→ To the…………………………………………………………………………………………….

TRƯỜNG THPT CHUYÊN TRÀ VINH - TRÀ VINH


PART A: MULTIPLE CHOICE.
QUESTION 1: PHONOLOGY
Choose the word whose underlined part is differently from the other three.
1. A. crooked B. wretched C. hooked D. scared
2. A. decay B. perishable C. benzene D. supreme
3. A. contribute B. syndrome C. home D. microphone
4. A. comfortable B. intimacy C. apprehension D. disappointed
5. A. pizza B. lizard C. puzzle D. muzzle
Choose the word that is stressed differently from the others in the list(5 points)
6. A. Aborigine B. appearance C. emergency D. contribute
7. A. gasometer B. agriculture C. nostalgic D. enthronement
8. A. guarantee B. committee C. pioneer D. volunteer
9. A. retrovirus B. indestructible C. advantageous D. abnormality
10. A. mysterious B. preferential C. modernity D. historical
QUESTION 2: VOCABULARY
Choose the best answer to complete each of the following sentences.
1. It was decided that the cost of the project would be………….. and so it was abandoned.
A. repressive B. prohibitive C. restrictive D. exclusive
2. Tessa was determined to become wealthy and to that…………..she started her own company.
A. view B. aim C. end D. object
3. He made a number of…………..remarks about my cooking, which upset us.
A. slashing B. stabbing C. chopping D. cutting
4.………….., Americans eat a light breakfast. They usually don’t eat a lot of food in the morning.
A. By and large B. Fair and square C. Ins and outs D. Odds and ends
5. The truant was…………..from school for unbecoming behavior.
A. dispelled B. repelled C. expelled D. compelled
6. She is afraid she is rather…………..about the existence of ghosts.
A. skeptical B. partial C. adaptable D. incapable
7. His emotional problems…………..from the attitudes he encountered as a child.
A. flourish B. stem C. root D. spout
8. I am sorry to have bothered you- I was under the…………..that you wanted me to call you.
A. mistake B. miscalculation
C. misconception D. misapprehension
9. Many children who get into trouble in their early teens go on to become…………..
151

Downloaded by Nhung Nguy?n (nhungnth2004@gmail.com)


lOMoARcPSD|20277325

A. persistent B. insistent C. inverted D. innate


10. Five readers…………..the correct solution to our recent competition.
A. communicated B. qualified C. submitted D. subscribed
QUESTION III: GRAMMAR AND STRUCTURES
Choose the best answer that grammatically fits each of the blanks in the following sentences
1. I could tell from the …………..look on her face that something terrible…………...
A. frightened/ happened B. frightening / had happened
C. frightened/ would happen D. frightening/ had been happening
2. A: “Monica brought some food for the party.” - B: “She………….., we have plenty.”
A. needn’t have brought B. doesn’t have to
B. didn’t need to D. needn’t
3. The government would be forced to use its emergency power…………..further rioting to occur.
A. should B. did C. were D. had
4- ", he remained optimistic.
A. Though badly wounded he was B. Badly wounded as he was
C. As he was badly wounded D. As badly wounded he was
5. It is imperative…………..what to do when there is a fire.
A. he must know about B. that he knew
C. we knew D. that everyone know
6. Recent evidence makes it possible for investigators to conclude that …………..
A. some paintings by Rembrandt were faked
B. some paintings by Rembrandt must be faked
C. Rembrandt was attributed to some paintings
D. some paintings by Rembrandt gets faked
7. Public transportation in most of the nation is expanding.…………..the use of the subways and buses is
declining in some metropolitan areas.
A. Nevertheless B. Consequently
C. Despite the fact that D. Although
8. Marine reptiles are among the few creatures that are known to have a possible life span greater
than…………..
A. man B. the man’s C. that of man D. the one of man’s
9. Among the astronomers of ancient Greece, two theories…………..concerning the place of the earth in the
universe.
A. developing B. developed
C. in development D. which they developed
10. I’d rather…………..until Jill comes back. Let’s go home.
A. not to wait B. not wait C. have not waited D. I didn’t wait
QUESTION IV: PHRASAL VERBS AND PREPOSITIONS
Choose the best answer to fill in each of the blanks in the following sentences
1. Karen was terribly nervous before the interview but she managed to pull herself…………..and act
confidently.
A. through B. over C. together D. off
2. When Mr. Benson ran out of money, he…………..his mother for help.
A. fell in with B. fell upon C. fell behind D. fell back on
3. I’m not sure whether this new style of shoes will…………...
A. get on B. turn up C. catch on D. come up
4. If you would like to wait for a moment, Sir, I will just…………..your file on the computer screen.
A. call up B. pull down C. bring in D. pick up
5. Within a few weeks all the present trouble will have blown…………..
152

Downloaded by Nhung Nguy?n (nhungnth2004@gmail.com)


lOMoARcPSD|20277325

A. along B. over C. out D. away


6. If you pay the restaurant bill with your credit card, I’ll…………..with you later.
A. settle down B. pay back C. settle up D. pay up
7. Announcing that he was totally done………….., Grandfather retired to bed.
A. out B. with C. in D. down
8. The police…………..a great deal of criticism over their handing of the demonstration.
A. came in for B. brought about C. went down with D. opened up
9. The prospective buyer had decided to look…………..the property before committing himself.
A. about B. over C. into D. up
10. The old ship will be towed into harbor and…………..
A. broken up B. broken down C. broken in D. broken off
QUESTION V. READING COMREHENSION
Passage 1 :Read the following passage and choose the best option to complete the blank or answer the
question
Red Rock Canyon, part of the Red Rock Recreation Lands in Nevada, is an escarpment of crimson
Aztec sandstone cliffs and canyon walls that reveal the geologic history of the area. Bands of sediment layers
tell of a deep-sea bed that 400 millions years ago rose eastward to shoreline in present- day western Utah. As
the ancient sea grew progressively more shallow, about 225 million years ago, marine limestone and shales
were overlaid by sediments washed in from emerging land areas. As the water in the shallow inland sea
evaporated, salts and minerals were deposited in thick beds and fluctuating shorelines created intermixed
beds of limestone, shales and minerals. Sediments from this period gave the canyon its name. Their red color
was created from the weathering of iron compounds within. About 180 million years ago the area becam arid
and was covered in sand dunes more than 2.000 feet deep, which became cemented into the Aztec sandstone
that is prominent in the canyon today. Its alternating hues of red, yellow, and white are believed to have
resulted from ground water percolating through the sand and leaching out the oxidized iron.
The most significant geologic feature of the area is the Keystone Thrust Fault, a fracture in the earth’ crust.
Sixty- five million years ago, intense pressure thrust one rock plate over another, a phenomenon that can
clearly be seen in the contrasting bands of gray limestone and red sandstone, where the gray limestone cap is
actually older than the sandstone beneath it. The Keystone is one of the most easily identifiable thrust faults
to be found anywhere .
1. With what topic is the passage mainly concerned ?
A. The creation of Keystone Thrust Fault.
B. How Red Rock Canyon acquired its name.
C. The formation of the Aztec sandstone.
D. The geologic history of Red RockCanyon.
2. The word “progressively” is closest in meaning to…………..
A. suddenly B. mysteriously C. naturally D. gradually
3. Which of the following can be concluded from this passage?
A. Red Rock Canyon was created in a relatively short time span.
B. The location of rock layer is not always an indication of its age.
C. The explanation of the sea bed played a significant role in the creation of Red Rock Canyon.
D. Emerging land areas eventually caused the sea to evaporate.
4. The word “identifiable” could best be replaced by which of the following?
A. observable B. significant C. discovered D. created
5. According to the passage, the red of the canyon walls is primarily a result of
A. groundwater percolating through the sand.
B. the weathering of iron compounds.
C. the evaporation of the inland sea.
D. intense pressure on rock plates.
153

Downloaded by Nhung Nguy?n (nhungnth2004@gmail.com)


lOMoARcPSD|20277325

6. The word “fluctuating” is closest in meaning to…………..


A. intermixing B. flooding C. changing D. withdrawing
7. The word “arid” is closest in meaning to …………..
A. dead B. uninhabited C. reddened D. dry
8. The word “its” refers to…………..
A. Aztec sandstone B. shorelines C. cement D. hues
9. According to the passage, when did Red Rock Canyon become dry ?
A. 400 million years ago. B. 225 million years ago
C. 180 million years ago. D. 65 million years ago.
10. According to the passage, all of the following remained after the sea evaporated EXCEPT
A. shale B. sandstone C. limestone D. minerals
Passage 2: Read the following passage and choose the best option to complete the blank or answer the
question
Ragtime, developed primarily by black pianists, is a style of composed piano music that was popular
from the 1890s to about 1915. It is a style of jazz characterized by an elaborately syncopated rhythm in the
melody and a steadily accented accompaniment. Ragtime quickly gained popularity after its first appearances,
reaching millions on a national scale through sheet music, player pianos, ragtime songs, and arrangements for
dance and marching bands. The leading ragtime composer was Scott Joplin, known as the “King of Ragtime”,
whose most famous piano piece, “Maple Leaf Rag,” was published in 1899.
Ragtime piano music has a generally standard form, duple meter (2/4, or two beats per measure)
performed at a moderate march tempo. The pianist’s right hand plays a highly syncopated melody, while the
left hand steadily maintains the beat with an “oom-path” accompaniment. A ragtime piece usually consists of
several similar melodies that takes such forms as AA BB A CC DD or Infroduction AA BB cc ĐD EE, where
each letter represents a melodic phrase. “Maple Leaf Rag” is a classic example of ragtime. About three
minutes long, it had the standard AA BB A cc DD form, and each section is 16 bars in length. The opening
melody, in march tempo, features the typical ragtime right-hand syncopations.
The forms of ragtime derive from European marches and dances, but the rhythms are rooted in
African- American folk music, the same rich body of music that became a vital source of jazz. Early jazz
musicians often used ragtime melodies as introductions to their improvisations. With its. syncopations, steady
beat, and piano style, ragtime played an integral part in the jazz legacy.
1. The author’s main purpose in the passage is …………….
A. compare ragtime and jazz
B. criticize the composition of Scott Joplin.
C. show how ragtime is arranged
D. discuss the origin and elements of ragtime.
2. Ragtime reached people nationwide through all of the following EXCEPT
A. composition for orchestra. B. player pianos
C. sheet music D. arrangements for marching bands.
3. According to the passage, the letters A in AA and B in BB each represent
A. march templo B. oom-path accompaniment
C. a melodic phrase . D. syncopated melody.
4. Which of,the following is NOT characteristic of ragtime
A. Fast march tempo B. Sixteen-bars section.
C. Left-hand accompaniment D. Highly syncopate^ melody.
5. This passage would be part of required reading in which of the following courses?
A. Africa- American history B. Music appreciation
C. Art history D. Social studies
6. The word “rooted ” is closest in meaning to…………….
A. stacked to the ground. B. unmoving C. derived from D. never changing
154

Downloaded by Nhung Nguy?n (nhungnth2004@gmail.com)


lOMoARcPSD|20277325

7 The word “it” refers to…………….


A. melodic phrase B. each letter
C. ragtime D. “Maple Leaf Rag”
8.Which of the following conclusions can be made from the passage?
A. Ragtime music is complex and hard to identify.
B. Ragtime was popular only with African Americans.
C. Ragtime has an easily recognizable rhythm.
D. Ragtime is completely different form of music than jazz.
9. The word “elaborately” is closest in meaning to…………….
A. eloquently B. simply C. intricately D. melodically
10. The word “maintain” may best be replaced by…………….
A. withholds B. keeps C. exaggerates D. interrupts
GUIDED CLOZE TEST
Passage 1: Read the following passage and choose the options that best complete the blanks
It stands to (1)……………that a city like Los Angeles, which is home to the rich and famous, is also
where you find the Association of Celebrity Personal Assistants.
Celebrity personal assistants are a unique group among Hollywood professionals. (2)……………the
lawyers and agents who rubs shoulders with the stars and (3)…………… millions, personal assistants (PAs)
are not paid well. They typically earn about $56,000 a year which, (4)……………their round- the-clock
obligations, isn’t much by Hollywood standards. As for the job description, it is also far glamorous.
Responsibilities include doing laundry, (5)……………groceries and paying bills. So what’s the attraction?
One celebrity PA says, “ I don’t (6)……………myself a vain or superficial person, but it would be (7)………
to say that we all don’t like being close to someone’s that’s powerful:’’ But not everyone is qualified for the
job. Rita Tateel teaches would- be assistants to the stars and begins her lessons with some (8)……………
truths: “You must be in good health at all times, because you are (9)……………a celebrity’s life. If you get
sick their life can’t just stop. And you need to be flexible and able to (10)……………in all kinds of hours.
You have to be a can-do person. If there is one word the celebrity don’t want to hear, that word is “no”.”
1. A. truth B. discussion C. argument D. reason
2. A. However B. Unlike C. Despite D. Similarly
3. A. give B. take C. make D. pay
4. A. given B. received C. spend D. being
5. A. shopping B. clutching C. fetching D. bringing
6. A. describe B. know C. consider D. think
7. A. true B. honest C. lie D. wrong
8. A. such B. hard C. heavy D. advice
9. A. working B. running C. dealing D. getting
10. A. take B. adapt C. get D. put
Passage 2: Read the following passage and choose the options that best complete the blanks
From the moment they leave the security of their accustomed environment, travelers are at risk.
Hazards arise not just from strange diseases they meet on their travels but from other factors too: seemingly
uninspiring home (1)……………such as safe water supplies, sanitation and public hygiene controls, legal
safety standards for monitoring vehicles and road (2)……………,are easily taken for granted, but simply do
not exist in many countries. Environmental factors such as arduous conditions, (3)……………climate, and
high altitude man constitute a danger; and so may traveler’s own behavior, free from the (4)……………of the
daily routine, and determined to have a good time with scant regard for consequences.
When illness and injuries occur abroad, travelers are again at the disadvantage from (5)……………to
communicate with a doctor on account of language or cultural difficulties, or being unable to find a doctor
owing to (6)……………of the local medical system. There may be a complete (7)……………of skilled
medical care, or of medical facilities of a standard acceptance to travelers from technologically sophisticated
155

Downloaded by Nhung Nguy?n (nhungnth2004@gmail.com)


lOMoARcPSD|20277325

countries.
When symptoms of an illness (8)……………aboard do not appear until after return home, a final
hazard becomes apparent: the symptoms may be (9)……………,may pass (10)……………and the correct
diagnosis may not be considered until it is too late.
1. A. comforts B. helps C. cares D. aids
2. A. correction B. maintenance C. improvement D. quality
3. A. worsening B. unusual C. sudden D. adverse
4. A. restraints B. assurance C. certainties D. regulations
5. A. inability B. difficulty C. inflexibility D. timidity
6. A. misuse B. doubt C. ignorance D. disbelief
7. A. breakdown B. failure C. disruption D. absence
8. A. received B. formed C. gained D. acquired
9. A. unfamiliar B. unlikely C. unpleasant D. uncovered
10. A. unrecognized B. unknown C. unforeseen D. unearth
PART TWO: WRITTEN TEST
QUESTION 1: WORD FORMATION
Complete each space with a word formedfrom the words in capital
1. Many people donated money to help the people in Africa who are starving as a result of……………of
food .(SCARCE)
2. It may be……………to force them into making a decision, and if you upset them, they’re quite likely to
overact. (PRODUCE)
3. I was surprised by his……………to break the law. (PREPARE)
4. It was very……………of them to call round and ask if we wanted help in moving furniture.(NEIGHBOR)
5. The printed instructions aretquite……………; there’s no need for me to further explain anything.
(EXPLAIN)
6. The control center is deep underground and completely……………except by direct hit from a nuclear
missile. (DESTROY)
7. The committee members promised that appropriate……………measures would be taken against the
offenders. (DISCIPLINE)
8. I have a(n)……………neighbor who plays loud music late at night. (CONSIDER)
9. Politeness is one thing. Real kindness is another. You must learn to…………… between the two
(DIFFERENT).
10.……………fish always live in lakes and rivers. (WATER)
Complete the following passage with the correctforms ofgiven words in the box.
essence critic various efficient logic
centre press exceed retail avoid

FOOD MILES
In Britain, what is described as “food miles”, the distance which food is transported from the place
where it is grown to its point of sale, continues to rise. This has major economic, social and environmental
consequences, given the traffic congestion and pollution which (11)…………..follow.
According to (12)………….. groups, the same amount of food is traveling 50 percent further than
twenty year ago. What’s more, the rise in the demand for road haulage over this period has mostly been due to
the transport of food and drink. The groups assert that the increase in the number of lorry journeys is (13)
…………..and that many of these are far from (14)…………..
In the distribution systems employed by British food (15)………….., fleets of lorries bring all goods
into more (16)…………..located warehouses for redistribution across the country.(17)…………..as this might
appear, the situation whereby some goods get sent back to the same areas from which they came is (18)……
In response to scathing (19)…………..from environmentalists, some food distributors now aim to
156

Downloaded by Nhung Nguy?n (nhungnth2004@gmail.com)


lOMoARcPSD|20277325

minimize the impact of food miles by routing vehicles, wherever possible, on motorways after dark. This
encourages greater energy (20)…………..while also reducing the impact on the residential areas through
which they would otherwise pass.
QUESTION 2: ERROR IDENTIFICATION
Find and correct all the mistakes in the numbered lines of the following passage
Simply be bilingual does not qualify anyone to interpreting. Interpreting does not merely a mechanical
process of converting one sentence in language A Into a same sentence in language B. Rather, it is a complex
art in that thoughts and idioms which have no obvious analogues from tongue to tongue- or words which have
multiple meanings - must quickly be transformed in many a way that the message is clearly and accurately
expressing to the listener.
There are two kinds of interpreter?, simultaneous and consecutive, each requires separate talents. The
former, sitting in an isolated booth, usually at a large multilingual conference, speaks tó listeners wearing
headphones, interpreting that a foreign - language speaker says as he says it- actually a sentence afterwards.
Consecutive interpreters are the one most international negotiatons use. They are mainly employed for smaller
meetings without sound booths, headphones, and another high- tech gear.
QUESTION III: OPEN CLOZE TEST
Passage 1: Fill in each blank with ONE suitable word:
Man is a unique being. He is different from all other (1)…………..because he does not merely form
part of the environment. Man reshapes his environment at will to suit his purposes. He does not have a natural
(2)…………..and is able to adapt himself to living in different environments. Among the many ways in which
Man has (3)…………..his environment are the building of cities and roads, the utilization of (4)…………..
for farming and reclaiming of land from the sea. While some of the changes are (5)…………..some are
extremely harmful to the environment.
An example of a harmful cọnseqụence of Man’s (6)…………..is that of pollution. Through the use of
scientific knowledge and advance technology, Man (7)…………..his wellbeing and life expectancy. In the (8)
…………..however, he has also brought about the growing problem of worldwide population. One of the
main sources of air pollution is motor vehicles. Gas emitted from the car (9)…………..contain many
chemicals, which are harmful to people, animals and plants. Industry also contributes significantly to the
pollution of the (10)…………... There is no majority city in the world today in which we can breathe fresh,
clean air.
It is therefore important for us to be environment - conscious and avoid actions that bring about
harmful effects to our environment.
Passage 2: Fill in each blank with ONE suitable word:
CELL PHONES
Cell phones have been popular in Japan since the early 1990s, but it was not (11)…………..1999 that
their use really took off. The age of cell phones has emerged, but with it come problems.
Cell phones are used on buses and trains, in restaurants, and in all (12)…………..of life. They cause
problems when they (13)………….. during meetings, concerts,wedding, or even funerals. What's more,
people speak loudly in public, and students read and text messages during lesson. More seriously, when a cell
phone is used near a person using a (14)…………..to regulate his heartbeat, its radio waves may (15)………
with the functioning of the machine.
Now, something is being done to (16)…………..these problems. In many places, new technology is
being used to block cell phones. Airline (17)…………..are requested to stop using cell phones while on board.
Concert halls ask their audience to switch their phones to the (18)…………..mode. However, phone users fear
that if they do not answer their phones, they will (19)…………..valuable business opportunities. That's why
many do not (20)…………..off their phones even when they are asked to.
IV. SENTENCE TRANSFORMATION
1. The two sides never looked likely to reach an agreement.
→ At no time ……………………………………………………………………………………
157

Downloaded by Nhung Nguy?n (nhungnth2004@gmail.com)


lOMoARcPSD|20277325

2. For further information, please send a self - addressed envelope to the above address.
→ Further information can……………………………………………………………………….
3. Richard only took over the family business because his father decided to retire early.
→ But for his ……………………………………………………………………………………
4. With six children to look after, she is extremely busy. HANDS
→…………………………………………………………………………………………………
5. Only final- year students are allowed to use the main college car park. RESTRICTED
→…………………………………………………………………………………………………
6. The new computer system has caused quite e few problems for the manager.
AGAINST
→The manager………………………………………quite a few problems with the new computer systems.
7. We agreed that each of us would do this washing - up on alternative days. TURNS
→…………………………………………………………………………………………………
8. You should be able to dress yourself by now!
→It’s high..………………………………………………………………………………………
9. Could I stay with you in Chicago for a few days, John? PUT
→…………………………………………………………………………………………………
10. I didn’t feel like doing anything energetic. MOOD
→…………………………………………………………………………………………………
TRƯỜNG THPT CHUYÊN HUỲNH MẪN ĐẠT - KIÊN GIANG
I. MULTIPLE CHOICE
1. PHONOLOGY
A. Choose the word whose bold part is pronounced differently from the others.
1. A. isthmus B.sheath C.thorn D. sooth
2. A. mnemonics B. condemn C. solemn D. column
3. A. Illinois B. viscount C. bourgeois D. Persian
4. A. Greenwich B. Norwich C. wrist D. awkward
5. A. plague B. argue C. tongue D. dialogue
B. Choose the word that has different stress pattern from the others.
6. A. circumstellar B. circumstance C. circumlocutorily D. circumvent
7. A. outwork B. outdo C. outdoor D. outflow
8. A.event B. lament C. present D. compromise
9. A. dehydrate B. infiltrate C. considerate D. distribute
10 A. post paid B. cool off C. broken down D. brick house
2. WORD CHOICE
11. I…………..that you won’t be inviting that awful woman to the meeting.
A. assume B. instruct C. entrust D. rely
12. The accused was given a short offence sentence as he had committed only a …………..offence
A. subordinate B. minimal C. secondary D. minor
13. The lecture was rather boring, but the…………..discussion proved fruitful.
A. subsequent B. latter C. consecutive D. successive
14. The teacher suspected cheating as soon as he noticed the pupil’s…………..glances at his classmate’s
paper.
A. futile B. furtive C. cold D. inconsequential
15. Although gregarious by nature, Lisa became quiet and…………..after she was unexpectedly laid off from
work.
A. autonomous B. withdrawn C. susceptible D. composed
16. Because of the shortage of water, there is a …………..on the use of hope -pipes.
A. ban B. veto C. taboo D. boycott
158

Downloaded by Nhung Nguy?n (nhungnth2004@gmail.com)


lOMoARcPSD|20277325

17. I am never free on Tuesday evening as I have a…………..arrangement to go to the cinema with you.
A. long - range B. long - live C. long - standing D. long - lasting
18. Students sometimes support themselves by…………..of evening jobs.
A. ways B. efforts C. methods D. means
19. We need both ornament and implement in our society, we need the artist and the…………...
A. writer B. politician C. artisan B. beautician
20. Any business venture contains an element of…………..
A. risk B. chance C. possibility D. luck
3. STRUCTURES AND GRAMMAR
21. Rumors going round, Mr. Long is…………..Head of the Department.
A. in the proximity of B. in line for C. in adjacent to D. in view of
22. I think it is no more delicious than…………..rice.
A. to eat B. eaten C. eating D. when I eat
23. Entertainment is what game …………...
A. is all about B. is C. be D. about
24. She goes to school late, but…………..she won’t be punished.
A. she thinks B. I dare say C. she dare say D. of course
25. He is our idol,………….., the man we all want to be.
A. as it is B. as things stand C. as you can see D. as it were
26. You can see…………..a cable-car in San Francisco.
A. you are riding B. yourself riding C. to ride D. riding
27. I just …………..go out…………..stay at home.
A. as soon/ as B. want to/ and also C. like/ more than D. rather/ and
28. You may put on a brave…………..but…………..you are fearful and anxious.
A. outside/ inside B. bearing/ inner. D. look/ interior D. front/ inside
29. The material world greatly influences …………..young people…………..old people.
A. X/ more than B. X/ greater than C. far more/ than D. more in/ than in
30.…………..came …………..as it was so weak.
A. Then/ to die C. Soon/ death B. Afterwards/ dying D. Death/ X
4. PREPOSITION AND PHRASAL VERB
31. I’m afraid we haven’t got a spare bed. Can you………….. with the mattress on the floor?
A. make do B. make by C. make over D. make up
32. I didn’t want to discuss the matter but he insisted on bringing it…………..
A. up B. out C. about D. over
33. I was very taken…………..by her aggressive attitude.
A. about B. aside C. apart D. aback
34. It was too late to…………..of the contract.
A. back out B. back down C. back up D. back away
35. The little boy was continually…………..the ornament.
A. tripping out B. falling down C. breaking up D. knocking over
36. Let Ella…………..self-centered, neurotic juice, she thought.
A. stew in her own B. be on her own C. stand on her seat of D. be alone
37. He…………..his wife’s death very hard.
A. passed B. took C. got D. received
38. Their warnings about the recent robbery put her…………..the defense.
A. on B. at C. in D. under
39. I must…………..lunch.
A. see about B. look to C. see through D. look on
40. The walls were…………..trophies.
159

Downloaded by Nhung Nguy?n (nhungnth2004@gmail.com)


lOMoARcPSD|20277325

A. hung with B. húng over C. covered with D. covered up


5. READING COMPREHENSION
A. Read the passage and choose the best answer for the following question
The Sun today is a yellow dwarf star. It is fueled by thermonuclear reactions near its center that
convert hydrogen to helium. The Sun has existed in its present state for about four billion six hundred million
years and is thousands of times larger than the Earth.
By studying other stars, astronomers can predict what the rest of the Sun’s life will be like. About five
billion years from now, the core of the Sun will shrink and become hotter. The surface temperature will fall.
The higher temperature of the center will increase the rate of thermonuclear reactions. The outer regions of the
Sun will expand approximately 35 million miles, about the distance to Mercury, which is the closest planet to
the Sun. The Sun will then be a red giant star. Temperatures on the Earth will become too high for life to exist.
Once the Sun has used up its thermonuclear energy as a red giant, it will begin to shrink. After it
shrinks to the size of the Earth, it will become a white dwarf star. The Sun may throw off huge amounts of
gases in violent eruptions called nova explosions as it changes from a red giant to a whitedwarf.
After billions of years as a white dwarf, the Sun will have used up all its fuel and will have lost its
heat. Such a star is called a black dwarf. After the Sun has become a black dwarf, the Earth will be dark and
cold. If any atmosphere remains there, it will have frozen over the Earth’s surface.
41. What is the primary purpose of this passage?
A. To alert people to the dangers posed by the sun.
B. To discuss conditions on Earth in the far future.
C. To present a theory about red giant stars.
D. To describe changes that the sun will go through.
42. The word “fueled” in line 1 is closest meaning to…………...
A. powered B. bombarded C. created D. propelled
43. The word “state” in line 3 is closest meaning to
A.^shape B. condition C. location D. size
44. It can be inferred from the passage that the Sun
A. is approximately halfway through its life as a yellow dwarf.
B. has been in existence for 10 billions year
C. is rapidly changing in size and brightness
D. will continue to be a yellow dwarf for another 10 billion years
45. What will probably be the first stage of change for the sun to become a red giant?
A. Its core will cool off and use less fuel
B. Its central part will grow smaller and hotter.
C. It will throw off huge amounts of gases
D. Its surface will become hotter and shrink
46. When the Sun becomes a red giant, what will the atmosphere be like on the Earth?
A. It will become too hot for life to exist.
B. It will be enveloped in the expanding surface of the sun
C. It will freeze and become solid.
D. It will be almost destroyed by nova explosions.
47. When the sun has used up its energy as a red giant, it will…………....
A. stop to expand B. cease to exist C. become smaller D. get frozen
48. As a white dwarf, the Sun will be…………...
A. a cool and habitable planet.
B. the same size as the planet Mercury
C. thousands of times smaller than it is today.
D. around 35 million miles in diameter.
49. The word “there” in the last sentence of paragraph 4 refers to…………....
160

Downloaded by Nhung Nguy?n (nhungnth2004@gmail.com)


lOMoARcPSD|20277325

A. our own planet B. the core of a blank dwarf


C. the outer surface of the Sun D. the planet Mercury.
50. The passage has probably been taken from…………....
A. a scientific chronicle B. a news report
C. a work of science fiction D. a scientific journal
Rearrange the passages into their correct order
A. The need to win unites the members of one team, but it inevitably creates a division between this team and
the opposing team. In the case of young people playing an informal game of football, this division is often no
more than friendly rivalry. They may show great competitive spirit against the opposition during the game, but
at the end of it, they are all good friends again. Any sense of division would have disappeared.
B. Such violence often erupts at international matches. An interest in a sport should unite the world. Sadly,
violent football fans sometimes ensure that it divides it instead.
C. Football is extremely important in many countries, especially among the males. Far more people watch it,
either in person at the pitch or on television, than play it and there are many very enthusiastic fans. There is a
great sense of unity among the fans of a particular club, indicated by the wearing of scarves of the colours of
their team. Sadly, there is also a great sense of division between the fans of rival teams and this is exacerbated
by the wearing of team colours.
D. There are substantial profits to be made and the top players receive very high financial rewards. All this
makes the need to win far greater than it is in an informal match played by young amateurs. The extent of the
financial rewards means that the feelings towards the opposing team are stronger than just friendly rivalry.
There is a definite sense of division and even, sometimes, of enmity.
E. One of the good things about i football , is that it is accessible to a great .many people, partly because it
does not require expensive equipment. All that is needed, besides a ball, is a stretch of ground with a set of
goalposts. Such football’pitches are usually readily available in school grounds and in leisure areas provided
by local authorities.
F. Sadly, this is not always the case with professional ’football players. They have a great sense of unity with
their teammates, but because professional football is now big business, the sense of division between them
and the players on the other side is oftenquite serious.
G. Because of its accessibility, football is a,game which is played by many young people. Most of them are
male, but increasingly some of the young players are female. Because so many young people play it, the game
often acts as bond among them. They have something in common that unites them.
H. Unfortunately, this sense of division can lead to such a feeling of hatred towards the fans of the opposition
that violence may ensue. Fans of rival team, especially when they have consumed a lot of alcohol, sometimes
begin to fight each other, often .also attacking the police who are trying to stop the fights and causing damage
to nearby property.
I. Young people who decide to play a football match divide themselves into two teams of eleven players. This
act at once unites and divides the players. All the players who are chosen to be on one side instantly feel a
sense of unity. They have a common goal which unites them, and this is to beat the opposition..
J. It is unfortunate that people who engage in a sport should have such feelings towards the opposition.
However, what is worse is the sense of division that exists among the football fans of some teams. Some
football fans regard the fans of other teams, especially those teams which are seen as the closest rivals of their
own team, as hated adversaries.
6. GUIDED CLOZE TEST
A. Choose the words that best complete the blanks
NATURE’S CLOCKS
Our biological docks govern almost every aspect of our lives. Our sensitivity to stimuli (61)…………..
over the course of the day, and our ability to perform certain functions is subject to fluctuations. Consequently,
there is a (n) (62)………….. time for tasks such as making decisions: around the middle of
the day. Anything that (63)…………..physical co-ordination, on the other hand, is best attempted in the early
161

Downloaded by Nhung Nguy?n (nhungnth2004@gmail.com)


lOMoARcPSD|20277325

evening. What is more, there is a dramatic drop in performance if these activities are (64)…………..out at
other times. The risk of accident in a factory, for example, is 20% higher during the night shift.
Primitive humans lived their lives in tune with daily cycle of light and dark. Today we are (65)………
convinced that we can impose schedules on our lives at (66)…………..Sooner or later, however, we pay a
price for ignoring our natural rhythms. A good example is jet lag, caused when we confuse our body’s
biological clocks by (67)…………..several time zones. People suffering from jet lag can take several days to
adjust to new time zones, and have a reduced ability to make decisions, which is a worrying thought, as
serious (68)…………..of judgment can be made. And this may be just the (69)………….. of the iceberg. An
increasing number of people suffer from seasonal affective disorder (SAD), a form of depression that can be
(70) …………..by living in artificial conditions SAD can be serious, and sufferes may even need to take
antidepressant drugs.
61. A. modifies B. ranges C. varies D. wavers
62. A. peak B. summit C. maximum D. optimum
63. A. requests B. demands C. dictates D. stipulates
64. A. made B. done C. carried D. performed
65. A. powerfully B. firmly C. steadily D. highly
66. A. whim B. determination C. will D. desire
67. A. landing B. penetrating C. crossing D. travelling
68. A. errors B. mistakes C. inaccuracies D. fallacies
69. A. peak B. pinnacle C. top D. tip
70. A. triggered B. developed C. created D. launched
B. Choose the words that best complete the blanks
ENVIRONMENT CONCERNS
The Earth is the only place we know of in the universe that can support human life. (71)…………..
human activities are making the planet less fit to live on. As the western world carries on consuming two-
thirds of the world’s resources while half of the world’s population do so (72)…………..to stay alive we are
rapidly destroying the (73)…………..resource we have by which all people can survive and prosper.
Everywhere fertile soil is (74)…………..built on or washed into the sea. Renewable resources are exploited so
much that they will never be able to recover (75)…………... We discharge pollutants into the atmosphere
without any thought of the consequences. As a (76)………….., the planet’s ability to support people is being
reduced at the very time when rising human numbers and consumption are (77)…………..increasingly heavy
demands on it.

The Earth’s (78)………….. resources are there for us to use. We need food, water, air, energy,
medicines, warmth, shelter and minerals to (79)…………..us fed, comfortable, healthy and active. If we are
sensible in how we use the resources they will (80)…………..indefinitely. But if we use them wastefully
and excessively they will soon run out and everyone will suffer.
71. A. Still B. Despite C. Yet D. Although
72. A. for B. just C. already D. entirely
73. A. lone B. individual C. on D. alone
74. A. sooner B. rather C. either D. neither
75. A. completely B. quite C. greatly D. utterly
76. A. result B. product C. development D. reaction
77. A. having B. doing C. taking D. making
78. A. natural B. real C. living D. genuine
79. A. stay B. keep C. maintain D. hold
80. A. last B. stand C. remain D. go
II. WRITTEN TEST
1. OPEN CLOZE TEST
162

Downloaded by Nhung Nguy?n (nhungnth2004@gmail.com)


lOMoARcPSD|20277325

A. Fill each blank with ONE suitable word


WORLD CUP AFTER WORLD WAR II
The 1950 World Cup, held in (1)…………..was the first to include British (2)…………...British teams
withdrew from FIFA in 1920, partly out of (3)…………..to play against, the countries they had been at war
with, and partly as a protest against foreign influence on football, but rejoined in 1946 following FIFA's
invitation. The tournament also saw the return of 1930 champions Uruguay, who had (4)………….. the
previous two World Cups. Uruguay won the tournament again after defeating the host nation Brazil, in the
match called "Maracanazo" ((5)…………..: Maracanaco).
In the tournaments between 1934 and 1978, 16 teams competed in each todrnament, except in 1938,
when Austria was (6)………….. into Germany after qualifying, leaving the tournament with 15 teams, and in
1950, when India, Scotland and Turkey withdrew, leaving the tournament with 13 teams. Most of the
participating nations were from Europe and South America, with a small (7)…………..from North America,
Africa, Asia and (8)…………... These teams were usually defeated easily by the European and South
American teams. Until 1982, the only teams from (9)…………...Europe and South America to advance ' out
of the first round were: USA, (10)…………..in 1930; Cuba, quarter-finalists in 1938; Korea DPR, quarter
finalists in 1966; and Mexico, quarter-finalists in 1970.
B. Fill each blank with ONE suitable word
SHARK
Sharks are a group of fish (1) ………….. by a (2)………….. skeleton, five to seven gill slits on the
sides of the head, and pectoral fins that are not fused to the head. Modern sharks are classified within the clade
Selachimorpha (or Selachii) and are the sister group to the (3)…………... However, the term ’’shark" has also
been used for extinct members of the subclass Elasmobranchii outside the Selachimorpha, such as
Cladoselache and Xenacanthus. Under this broader definition, the earliest known sharks date from more than
420 million years ago.
Since then, sharks have diversified into over 470 species. They range in size from the small dwarf (4)
………….. (Etmopterus perryi), a deep sea species of only 17 centimetres (6.7 in) in length, to the whale
shark (Rhincodon typus), the (5)………….. fish in the world, which reaches approximately 12 metres
(39 ft). Sharks are found in all seas and are common to (6)…………..of 2,000 metres (6,600 ft). They
generally do not live in freshwater although there are a few known exceptions, such as the bull shark and the
river shark, which can survive in both (7)…………..and freshwater. They breathe through five to seven gill
slits. Sharks have a covering of dermal (8)…………..that protects their skin from damage and parasites in
addition to improving their fluid dynamics. They have several sets of (9)…………..teeth.
Well-known species such as the great white shark, tiger shark, blue shark, mako shark, and the
hammerhead shark are apex (10)…………..organisms at the top of their underwater food chain.
Many shark populations are threatened by human activities.
2. WORDFORM
A. Supply the correct word form
1. James is a…………... He eats chocolate as a hobby. (chocolate).
2. In…………..., descent and relationship are determined through the female line. (matri)
3. Impedance …………...is a plethysmography technique of using sensors to detect the properties of the blood
flow in the thorax. (graph)
4. Last year I had a…………...(appendical)
5. My house is …………...from the two pubs in the village. (distance)
6. When gorillas descend, they do so…………..., lowering themselves with their arms. (foot)
7. You can see the display of …………...in the exhibition. (Wellington)
8. The unresponsive audience made the lecturer somewhat…………....What a shame. (heart)
9. They won the case because of the…………...in court of the defendant. (disappear)
10. He accused the local of authority of………….... (adminisưate)
B. Complete the following passage with the correct from of the word given in the box
163

Downloaded by Nhung Nguy?n (nhungnth2004@gmail.com)


lOMoARcPSD|20277325

refer masterfulness thick maths dimension


philosophy long particular geometric similar
t is simply this. That Space, as our (1) …………. have it, is spoken of as having three dimension,
which one may call (2)…………., breath, and (3)………….and is always definable by (4)………….to three
planes, each at right angels to the others. But some (5)………….people have been asking why three
dimensions (6)………….-why not another direction at right angels to the other three? -and have even tried to
construct a Four-Dimension (7)…………. Professor Simon Newcomb was expounding this to the New York
Mathematical Society only a month or so ago. You know how on a flat surface, which has only two
dimensions, we can represent a figure of a three- (8)………….solid, and (9) …………. they think that by
models of three dimension they could represent one or four -if they could (10)………….the perspective of the
thing. See?
3. ERROR IDENTIFICATION
Find out the errors in the passage and then correct
RELAXATION
True relaxation is most certainly not a matter of flopping on in front of the television with a welcome
drink. Also is it about drifting into an exhausted sleep. Useful though these responses to tension and over -
tiredness may be, we should distinguish between them and conscious relaxation in spite of quality and effect.
Regardless of the level of tiredness, real relaxation is a state of alert yet at the same time passive awareness, in
which our bodies are in rest while our minds are awake.
Moreover, it is so natural for a healthy person to be relaxed when moving as resting. Having relaxed in
action means we bring the appropriate energy to everything we do, so as to have a feeling of healthy tiredness
by the end of the day, more than one of exhaustion.
Unfortunately, as a result of living in today competitive world, we are under constant strain and have
difficulty in coping, feeling alone nurturing our body’s abilities. That needs to be rediscovered is conscious
relaxation. With this in mind we must apply ourselves to understanding stress and the nature of its causes,
however deep - seated.
4. SENTENCE TRANSFORMATION
1. If I hadn’t helped you, you would have been in trouble with your girlfriend.
→Were…………………………………………………………………………………………………..
2. He wasn’t present at the party, which made her very happy. However, no one expected to welcome him.
→ Had...………………………………………………………………………………………………...
3. People shouldn’t think that Miss World must have the great appearance.
→ It is not always ………………………………………………………………………………………
4. I think that these talkative girls are chatting about my problem.
→ These talkative girls………………………………………………………………………………….
5. He causes me a lot of trouble. (flesh)
→ ………………………………………………………………………………………………………..
6. I’m sorry, James. But I’m very busy with my work. (up)
→ ………………………………………………………………………………………………………..
7. If you continue to be stuck-up, everyone will ignore you. (ears)
→ You will……………………………………………………………………………………………..
8. Our dependence on overseas trade sets a special value upon drive and resourcefulness in export industries.
(puts)
→ ………………………………………………………………………………………………………..
9. The audience were misled by the idea that economic recovery was close at hand. (away)
→ ………………………………………………………………………………………………………..
10. I decided to go home and make myself clean and smart -to let everyone know that life had not got me
down. (up)
→ ………………………………………………………………………………………………………..
164

Downloaded by Nhung Nguy?n (nhungnth2004@gmail.com)


lOMoARcPSD|20277325

TRƯỜNG THPT CHUYÊN LÝ TỰ TRỌNG - CẦN THƠ


A. MULTIPLE CHOICE
I. PHONOLOGY
Choose the word whose underlinedpart ispronounced differently from the others.
1. A. merchant B. sergeant C. commercial D. term
2. A. adventure B. addict C. adjective D. advent
3. A. exercise B. expertise C. release D. advertise
4. A. bound B. sound C. county D. poultry
5. A. months B. paths C. wreaths D. youths
Choose the word which is stressed differently from the other three.
6. A. magnificent B. photography C. proverbial D. advantageous
7. A. barrier B. dilemma C. imminent D. unity
8. A. majority B. ceremony C. astronomy D. investiture
9. A. amphibian B. appropriate C. cathedral D. creature
10. A. misinterpret B. tuberculosis C. cinematography D.
nevertheless
II. WORD CHOICE
Choose the best option to complete the following sentences.
1. My cousin was nervous about being interviewed on television, but she rose to the………….. wonderfully.
A. event B. performance C. incident D. occasion
2. It is no use…………..over…………..milk.
A. crying / spilt B. to cry / spilling C. crying / spilling D. crying / to spill
3.…………..of the financial crisis, all they could do was hold on and hope that things would improve.
A. In the end B. At the height C. On the top D. At the bottom
4.I was…………..in the book I was reading, and I didn’t hear the phone.
A. distracted B. submerged C. gripped D. engrossed
5. Our country has large areas of………….. beauty.
A. uninjured B. undamaged C. unharmed D. unspoilt
6. Some kinds of birds are on the…………..of extinction.
A. side B. end C. tip D. brink
7. I realized…………..that he was a thief.
A. sooner or later B. all along C. at the beginning D. eventually
8. They seemed to be…………..to the criticism and just carried on as before.
A. disinterested B. sensitive C. uncaring D. indifferent
9. I’ve never really enjoyed going to the ballet or the opera; they’re not really my…………...
A. piece of cake. B. sweets and candy C. biscuit D. cup of tea
10. I know you have been working very hard today. Let’s…………..and go home.
A. pull my leg B. call it a day
C. put your back up D. pull your finger out
III. GRAMMAR AND STRUCTURES
Choose the best option to complete the following sentences.
1. …………..,William Shakespeare is the most widely known.
A. All Writers in English B. All of the writers in English
C. Of all writers in English D. With all, writers in English
2.…………..that she burst into tears.
A. Such her anger was B. she was so anger
C. So angry she was D. Such was her anger
3. Out……………. from its tiny cage.
165

Downloaded by Nhung Nguy?n (nhungnth2004@gmail.com)


lOMoARcPSD|20277325

A. does the bird fly B. fly the bird C. did the bird fly D. flew the bird
4. It is essential that Alice…………..Tom of the meeting tomorrow.
A. remind B. must remind C. reminds D. will remind
5. My supervisor is angry With me. I didn’t do all the work that I…………... last week.
A. must have done B. can have done C. may have done D. should have done
6. ‘Did you enjoy the picnic?’ - ‘It was okay, but I'd rather…………..to a movie.’
A. go B. be going C. have gone D. went
7. Reagan…………..an actor years ago.
A. is said to be B. was said being
C. was said have been D. is said to have been
8…………..70 percent alcohol is more effective than 100 percent alcohol.
A. An antiseptic used B. How an antiseptic is used
C. When used as an antisepticD. An antiseptic when used
9………….., we tried our best to complete it.
A. Difficult as the homework was
B. Thanks to the difficult homework
C. As though the homework was difficult
D. Despite the homework was difficult
10. Tom: Who did you invite to dinner? - Mary: No one…………..than Frank and his family.
A. rather B. other C. except D. besides
IV. PHRASAL VERBS AND PREPOSITIONS
Choose the best option to complete the following sentences
I. Just say you need me and I am…………..hand.
A. on B. for C. at D. by
2. For my part, it doesn’t interest me…………..the least whether you keep this money or give it away.
A. at B. in C. for D. with
3. Are you still…………..an illusion that Mr. Spike will agree to your conditions?
A. at B. on C. in D. under
4. We can safely trust Iris. She is…………..the know about everything that happens on the Stock Exchange.
A. of B. in C. with D. at
5. Things are beginning to look…………..now that we have received the aid.
A. on B. up C. back D. over
6.I’m not going to give your car back until you have paid…………..the whole debt you owe me.
A. through B. on C. off D. out
7. He claims his role in the plot wasn’t important, but I suppose he’s trying to play it…………..intentionally.
A. off B. out C. down D. back
8. Alice is going to…………..the job of a sales assistant in Newcastle.
A. put in for B. set off C. make out D. stand out
9. The conflictr in the production department ………….. the employees’ dissatisfaction with their wages.
There’s nothing more to it.
A. eases off B. boils down to C. rules out D. thinks through
10. How are they going to…………..the time they wasted playing cards in the barracks?
A. make up for B. break out of C. brush up on D. cut out for
V. GUIDED CLOZE 1
Read the text below and decide which answer best fits each space.
A GREAT COMPOSER
The classical composer Ernst Hoffsberger, who passed away earlier this week, truly (1)……………
the world of contemporary classical music and was a great source of inspiration to a whole generation of (2)
……………young artists in various fields. In many ways his three symphonies completely (3) ……………
166

Downloaded by Nhung Nguy?n (nhungnth2004@gmail.com)


lOMoARcPSD|20277325

the achievements of all other composers of the late twentieth century and by (4) ……………the classical
genre with jazz, rock and latterly hip-hop, his work at times bore little (5)……………to what is commonly
considered to be a classical sound.
Born in California just after the Second World War, Hoffsberger had a strict religious (6)……………
during which he was taught classical piano by his father. He first found work as a(n) (7)…………….
journalist, playing and composing music in his free time. During the late sixties, he worked together in
collaboration with a number of other amateur musicians before finally (8)…………… professional with
the first public performance of his inspirational Tenor Sax Concerto in 1971.
From then on, throughout the seventies and eighties, each new work seemed to (9) …………… the
limits, of the orchestral, medium and also helped to bring classical music to a wider audience. What many
people consider Hoffsberger's defining quality that kept his music fresh and original was that he never lost the
human (10) …………… which gave him the ability to site? down and jam with musicians and artists from all
walks of life.
1. A. revolutionized B. restored C. renovated D. refurbished
2. A. branching B. budding C. blooming D. bursting
3. A. overcame B. overshadowed C. overturned D. overwhelmed
4. A. adjoining B. attaching C. co-joining D. fusing
5. A. similarity B. familiarity C. resemblance D. identification
6. A. family B. background C. childhood D. upbringing
7. A. non-contract B. off-the-books C. freelance D. odd-job
8. A. takjng B. getting C. making D. turning
9. A. overpass B. bypass C.surpass D. encompass
10. A. touch B. feeling C. contact D. aspect

VI. GUIDED CLOZE 2


Read the text below and decide which answer best fits each space.
ECOTOURISM
Many people nowadays are keen to get away from it all for a couple of weeks without putting too
much of a (1) …………… on thee environment. In terms of maintaining the moral high ground, the benefits
of an ecotourism holiday seem overwhelming. But just how environmentally-friendly can we make (2)
……………up our tans? And for those who want to go a (n) (3)……………further and drastically
reduce their environmental impact, how much do we really need to (4)……………it?
The answer, it seems, is more complicated than you might at first think.
For those who crave a bit of luxury in a warm climate, there are countless resorts to choose from,
aimed squarely at (5)……………rich, time poor Europeans and North Americans. Resorts .like Australia's
TurtleBeach and Malaysia's Sukai Resort are on (6)…………… with some of the most luxurious non-eco
establishments.
But they don’t come cheap. A one-week (7)……………holiday for two at one of these places can
easily set you back two thousand pounds. Plus there's the awkward fact that you still need to (8)………
considerable distances in some gasguzzling (9)……………of transport to get there in the first place.
Surprisingly, nearer the other end of the scale, a week of luxury at the UK's priciest and most polluting
resort, Gluttonness in Scotland turns out to be more than (10)……………for the eco-tourism rainforest
options when you factor in the huge carbon footprint that your flight would otherwise create.
1. A. strain B. damage C. stress D. harm
2. A. bringing B. filling C. beefing D. topping
3. A. increment B. step C. foot D. degree
4. A. deny B. suffer C. rough D. abstain
5. A. cash B. money C. finance D. wealth
6. A. equality B. comparison C. a level D. a par
167

Downloaded by Nhung Nguy?n (nhungnth2004@gmail.com)


lOMoARcPSD|20277325

7. A. combination B. package C. included D. combo


8. A. go over B. cover C. complete D. fulfill
9. A. choice B. kind C. type D. mode
10. A. an equal B. a match C. a win D. victory
VII. READING PASSAGE 1
Read the text below and choose the best answer to each question.
In the course of history, human inventions have dramatically increased the average amount of energy
available for use per person. Primitive peoples in cold regions burned wood and animal dung to heat their
caves, cook food, and drive off animals by fire. The first step toward the developing of more efficient fuels
was taken when people discovered that they could use vegetable oils and animal fats in lieu of gathered or cut
wood. Charcoal gave off a more intensive heat than wood and was more easily obtainable than organic fats.
The Greeks first began to use coal for metal smelting in the 4th century, but it did not come into extensive use
until the Industrial Revolution.
In the 1700s, at the beginning of the Industrial Revolution, most energy used in the United States and
other nations undergoing industrialization was obtained from perpetual and renewable sources, such as wood,
water streams, domesticated animal labor, and wind. These were predominantly locally available supplies. By
mid-1800s, 91 percent of all commercial energy consumed in the United States and European countries was
obtained from wood. However, at the beginning of the 20th century, coal became a major energy source and
replaced wood in industrializing countries. Although in most regions and climate zones wood was more
readily accessible than coal, the latter represents a more concentrated source of energy. In 1910, natural gas
and oil firmly replaced coal as the main source of fuel because they are lighter and, therefore, cheaper to
transport. They burned more cleanly than coal and polluted less. Unlike coal, oil could be refined to
manufacture liquid fuels for vehicles, a very important consideration in the early 1900s, when the automobile
arrived on the scene.
By 1984, nonrenewable fossil fuels, such as oil, coal, and natural gas, provided over 82 percent of the
commercial and industrial energy used in the world. Small amounts of energy were derived from nuclear
fission, and the remaining 16 percent came from burning direct perpetual and renewable fuels, such as
biomass. Between 1700 and 1986, a large number of countries shifted from the use of energy from local
sources to a centralized generation of hy¬dropower and solar energy converted to electricity. The energy
derived from nonrenewable fossil fuels has been increasingly produced in one location and transported to
another, as is the case with most automobile fuels. In countries with private, rather than public transportation,
the age of nonrenewable fuels has created a dẹpendency on a finite resource that will have to be replaced.
Alternative fuel sources are numerous, and shale oil and hydrocarbons are just two examples. The
extraction of shale oil from large deposits in Asian and European regions has proven to be labor consuming
and costly. The resulting product is sulfur- and nitrogen-rich, and large-scale extractions are presently
prohibitive. Similarly, the extraction of hydrocarbons from tar sands in Alberta and Utah is complex. Semi-
solid hydrocarbons cannot be easily separated from the sandstone and limestone that cany them, and modem
technology is not sufficiently versatile for a large-scale removal of the material. However, both sources of fuel
may eventually be needed as petroleum prices continue to rise and limitations in fossil fuel availability make
alternative deposits more attractive.
1. What is the main topic of the passage?
A. Applications of various fuels B. Natural resources and fossil fuels
C. A history of energy use D. A historical overview of energy rates
2. In line 2, the phrase “per person” is closest in meaning to…………….
A. per capita B. per year C. per family D. per day
3. It can be inferred from the first paragraph that……………..
A. coal mining was essential for primitive peoples
B. the Greeks used coal in industrial production
C. the development of efficient fuels was a gradual process
168

Downloaded by Nhung Nguy?n (nhungnth2004@gmail.com)


lOMoARcPSD|20277325

D. the discovery of efficient fuels was mostly accidental


4. In line 5, the phrase “in lieu” is closest in meaning to…………….
A. in spite B. in place C. in every way D. in charge
5. The author of the passage implies that in the 1700s, sources of energy were……………..
A. used for commercial purposes B. used in various combinations
C. not derived from mineral deposits D. not always easy to locate
6. According to the passage, what was the greatest advantage of oil as fuel?
A. It was a concentrated source of energy.
B. It was lighter and cheaper than coal.
C. It replaced wood and coal and reduced pollution.
D. It could be converted to automobile fuel.
7 According to the passage, the sources of fossil fuels will have to be replaced because…………….
A. they need to be transported B. they are not efficient
C. their use is centralized D. their supply is limited
8. It can be inferred from the passage that in the early 20th century, energy was obtained primarily
from……………..
A. fossil fuels B. nuclear fission
C. hydraulic and solar sources D. burning biomass
9. The author of the passage implies that alternative sources of fuel are currently……………..
A. being explored B. being used for consumption
C. available in few locations D. examined on a large scale
10. In line 38, the word “prohibitive” is closest in meaning to……………..
A. prohibited B. provided C. too expensive D. too expedient
VIII. READING PASSAGE 2
Read the text below and choose the best answer to each question.
When jazz began to lose its reputation as ‘low-down’ music and to gain well-deserved acclaim
among intellectuals, musicians began to feature many instruments previously considered inappropriate
Line for jazz. Whereas before the 1950s, jazz musicians played only eight basic
(5) instruments in strict tempo, in this decade they started to improvise on the flute, electric organ,
piccolo, accordion, cello, and even bagpipes, with the rhythm section composed for strings or piano.
Big bands no longer dominated jazz, and most changes emerged from small combos, such as the Dave
Brubeck Quartet and the Gerry Mulligan Quartet. The
(10) Gerry Mulligan Quartet proved that a small, modem band could sound complete without a piano; the
rhythm section consisted only of a set of drums and a string bass.
Jazz continued to move in new directions during the 1960s. Saxophonist and composer Ornette
Coleman led a quartet playing (15) “free” jazz that was atonal. Pianist Cecil Taylor also conducted
similar experiments with music, and John Gohrane included melodies from India in his compositions.
In the 1970s, musicians blended jazz and rock music into fusion jazz which combined the melodies
and the improvisations of jazz with the rhythmic qualities of rock ’n’ roll, with three or five beats to
the bar and in other meters. The form of jazz music was greatly affected by electric instruments and
electronic implements to intensify, distort, or amplify their sounds. However, the younger musicians of
the time felt compelled to include a steady, swinging rhythm which they saw as a permanent and
essential element in great jazz.
1. Which of the following would be the best title for the passage?
A. Popular Beats in Classical and Modern Jazz
B. Quintessential Moments in Jazz Music
C. The Achievements of Famous Jazz Musicians
D. The Rising Prestige and Diversity of Jazz
2. In line 3, the word “feature” is closest in meaning to……………
169

Downloaded by Nhung Nguy?n (nhungnth2004@gmail.com)


lOMoARcPSD|20277325

A. profess B. prohibit C. protest D. promote


3. The paragraph preceding this passage would most likely describe…………….
A. instruments used in jazz B. instrumental pieces in jazz
C. jazz in the 1940s D. the origins of jazz
4. The author of the passage implies that in the 1950s, jazz musicians…………….
A. strictly adhered to its traditions and compositions
B. probably continued with its tempo and instrumentation
C. experimented with rhythm and instruments
D. increased the tempo to keep up with the changes
5. The author of the passage mentions all of the following EXCEPT…………….
A. bagpipes B. percussion C. string bass D. harpsichord
6. It can be inferred from the passage that small jazz bands ……………
A. were dominated by large orchestras
B. consisted of drums and a string bass
C. were innovative in their music
D. included modem sound systems
7. The author believes that the developments in jazz described in the passage…………….
A. should be seen as precocious B. should be considered influential
C. appear largely suggestive D. may be perceived as discrete
8. The passage implies that representative jazz musicians……………
A. concentrated on melodious combinations of sounds
B. blended improvisations and sheet music together
C. created and modernized sophisticated devices
D. sought novel techniques in form and content
9. According to the passage, the changes in jazz music in the 1970s came from…………….
A. harmonious scale B. another musical trend
C. ambitious aspirations D. sound amplifications
10. Which of the following best describes the organization of the passage?
A. Chronological innovations In jazz music
B. Definitions of diverse jazz styles
C. A classification of prominent jaz2 musicians
D. Descriptions and examples to illustrate jazz rhythms
B. WRITTEN TEST
I. CLOZE TEST:
Read the text below and think of the word which best fits each space. Use only ONE WORD for each
space.
OPEN CLOZE 1
GETTING AHEAD AT WORK
One job is pretty much like another. In fact, having a job is a bit like being in a family. Everybody
assumes a different role and conflict is (1)…………….with from that perspective. The difference in the
workplace is that even if you don’t like the role you have, you need to continue to (2)…………….
a brave face on it. And unlike in a family where you might do little or nothing to help out, in your job, you are
expected to do your (3)…………….share - nobody else wants to do your work for you.
Some people go out of their way to be helpful at work although (4) …………….is not always noticed
or appreciated. Some people are naturally very (5)…………….the ball. Others resent the fact that they have
their boss (6)…………….down their necks all the time and they try to avoid having to (7)…………….their
weight and to leave some of their allotted work to others.
If you are ambitious and wish to further your career and (8)…………….promoted, it's a good idea to
ensure the support of others by (9)…………….thoughtful and doing little things to get yourself noticed. This
170

Downloaded by Nhung Nguy?n (nhungnth2004@gmail.com)


lOMoARcPSD|20277325

often means having to (10) ……………. on your feet; like when you discover it's somebody's birthday but
nothing has been organized, so you create a card on your PC, wrap the bottle of wine you have just bought for
dinner and organize for everybody to sing Happy Birthday out of tune. Do this often enough and you will be
well on the way to becoming the most popular person around.
OPEN CLOZE 2
THE IMPORTANT OF BEING SPORTY
It is a multi-million dollar industry that attracts some of the most talented individuals alive, it generates
further millions (1) …………….advertising revenue and has spawned a whole celebrity culture of its (2)
……………..But what role does sport play in our ordinary lives?
Few people are untouched (3)…………….sport. We all have (4)…………….favorite football team or
tennis player or, at least, support our national side in major sporting events like the Olympics. How can it be,
then, that so (5)…………….people actually play sports and that obesity is becoming a major threat in the
developed world?
Well, in part, the answer is (6)…………….the question. Obesity is not a problem in the developing
world (7)…………….participation in sport is still high. True, there may be few organized leagues but children
still go out to play games like football with other children, which comes the supremacy of countries like Brazil
in the world of football.
So, what is stopping the British or the Americans? After (8)…………….,they still have organized
sports in schools and a myriad of clubs and teams to join in their spare time. What they lack is a public space
for children to play unsupervised outdoors. As a result, they are kept indoors and encouraged to play online
(9) …………….of getting out in the fresh air. They may of course still develop a love of sport as spectators
but this is unlikely to do much to lower levels of obesity (10)…………….the young.
II. WORD FORMS
PART 1: Complete each sentence, using the correct form of the word in parentheses.
1. I want to make sure all my dependants will be financially secure if I’m…………….in any way.
(CAPABLE)
2. I really believe that it would be a major mistake to…………….any drugs that are currently illegal.
(CRIME)
3. There is so much fighting between rival groups that the country has become practically……………
(GOVERN)
4. I was a bit……………by my performance in the first exam, but I decided to make extra effort in the ones
left. (MORAL)
5. James got into trouble for……………a police officer. (PERSON)
6. She was given the……………task of informing the losers. (ENVY)
7. Travelling in Europe was something of a(n)……………after the years he had spent in Africa. (CLIMAX)
8. The rumours are completely……………and I would urge everyone not to believe them. (SUBSTANCE)
9. Don’t you think it Is a bit early to……………blame? (PORTION)
10. Some analysts worry that violence on TV might ……………children to violence in real life. (SENSE)
PART 2: Complete the passage with appropriate forms from the words given in the box.
revolt persist alter electric process
endure continue place modern capable

Born in 1940, UK businessman Sir Clive Sinclair has an (1)……………place in the minds of British
people for two reasons. First, he was the man who (2)……………home computing with the ZX series of
computers, and secondly, he was the man whose (3)……………to the car, the C5, failed spectacularly
to capture the public imagination.
Sinclair’s products, the ZX81 and its successful (4)……………,the ZX Spectrum, were small,
affordable computers that sold in huge numbers in the early 1980s. Despite limited (5) ……………they
allowed people to play computer games in their own home for the first time, and even introduced people to the
171

Downloaded by Nhung Nguy?n (nhungnth2004@gmail.com)


lOMoARcPSD|20277325

word (6)……………
The C5, a one-person vehicle that ran on (7)……………, was produced in 1984 and was Sinclair’s
attempt to (8)……………transport. However, it was (9) ……………criticized in the press for being
unsafe and impractical in the British climate and production of the C5 was (10) ……………in August, 1985.
HI. ERROR CORRECTION
The following passage contains 10 errors. Identify and correct them.
HOW TO BE A BETTER FRIEND Instead of
Instead giving endless advice, learn to listen more. Listening is an underestimating 1……………
skill, and it is easy to forget when you are worrying about other people problems. 2……………
How many times do we say “Oh yeah, that happened to me and..:” before we give the 3……………
other persons a chance to explain what happen to them? Sometimes, a friend may just 4……………
want to talk something about with someone else to sort things out in their mind - so 5……………
listen hardly to what they are saying, and try to offer advices only when you think 6……………
they are asking for it. 7……………
We all feel we’ve been let down by a friend at some points in our lives. Perhaps they 8……………
let out a secret we trusted them to keep under wraps, or sudden sided with the 9……………
opposition during an argument. Nobody’s perfect, so try to have realistic expectation. 10……………
Friendships don’t develop overnight; they deepen over time as you begin to trust one 11……………
other. Don’t place unrealistic demands on your friendship. 12……………
13……………
14……………

IV. SENTENCE TRANSFORMATION


Rewrite the following sentences using the words given.
1. I couldn’t stop thinking about the exam despite playing backgammon with Paul. (MIND)
- Playing backgammon……………………………………………………………………………………
2. Pete said that Josh doesn’t usually tell lies. (LIKE)
- ‘It ……………………………………………………………………………………………………….’
3. I can spend more time with my grandchildren when I retire. (FREE)
- Retirement………………………………………………………………………………………………
4. Oscar is qualified to teach Russian because he has a degree in it. (HIM)
- “Oscar’s Russian degree………………………………………………………………………………..
5. I’m glad you now see sense and agree that your parents are right. (SENSES)
- “ I’m glad……………………………………………………………………………………………..
6. I didn’t tell Angie because I didn’t want to hurt Eddie’s feelings. (CONSIDERATION)
- I didn’t tell ……………………………………………………………………………………………
7. You have to use logic and lateral thinking in equal measure in this job. (STRIKE)
- ………………………………………………………………………………………………………..
8. Ralph would hand in his resignation immediately if he could find a better iob. (HAT)
- Ralph would hand……………………………………………………………………………………
9. The parents of that girl are furious about her expulsion. (ARMS)
- That’s the girl……………………………………………………………………………………………
10. We would always take great care when flying at night. (WITS)
…………………………………………………………………………………………………………….
TRƯỜNG THPT CHUYÊN THOẠI NGỌC HẦU - AN GIANG
PHẦN TRẮC NGHIỆM (Multiple choice questions)
CÂU 1: PHONOLOGY
A. Pick out the word that has the underlined part pronounced differently from that of the others.
172

Downloaded by Nhung Nguy?n (nhungnth2004@gmail.com)


lOMoARcPSD|20277325

1. A. blame B. exchange C. danger D. anger


2. A. debt B. climbing C. timber D. lamb
3. A. expansion B. conversion C. precision D. explosion
4. A. merchant B. sergeant C. commercial D. term
5. A. dome B. comb C. home D. tomb
B. Pick out the word whose stress syllable is placed differently from that ofthe others.
6. A. reindeer B. engineer C. pioneer D. referee
7. A. magnificent B. photography C. proverbial. D. advantageous
8.A. descent B. goodbye C. nobody D. postpone
9. A. overseas B. overcoat C. overcome D. overnight
10. A. conservative B. explanatory C. harmonica D. triangle
CÂU 2: VOCABULARY
Select the best option for each blank.
1. In some cases of ............... the loss may be limited to a single incident, whereas in other cases it may be so
inclusive as to involve everything about an individual's life.
A. paralysis B. aphasia C. deafness D. amnesia
2. They had...............evidence that he had been defrauding his customers.
A. conclusive B. swindling C. twisting D. superficial
3. Attitudes on the two sides in the Revolutionary War ...............the possibility of a peaceful solution.
A. presaged B. prejudiced C. anticipated D. precluded
4. The telephone rang and interrupted my...............of thought.
A.train B. chain C. series D. ranges
5. After World War II, Russia ...............as a world power.
A. surrendered B.emerged C. fought D. dismissed
6. After leaving a particularly ............... concert, some rock music fans complain about ringing in their ears
known as tinnitus.
A. ambiguous B. raucous C. intricate D. lively
7. The tornado caused...............damage to the Florida citrus crop.
A. irresolute B. irresponsible C. irreparable D. irregular
8. Boys' Clubs do not...............poor children of the opportunity to participate in sports.
A. deprive B. retract C. improvise D. dilute
9. One must...............the United States five years in order to apply for citizenship.
A. reside in B. accommodate to C. invade D. abandon
10. The wind blew so strongly that the windows...............in their frames.
A. flapped B. shocked C. rattled D. slapped
CÂU 3: GRAMMAR AND STRUCTURES
Select the best option for each blank.
1. The doctors are examining the dog ...............the child for rabies, which is a dangerous disease...............
immediate treatment.
A. biting/ required B. bitten/ required C. bitten/ requiring D. biting/ requiring
2. This is ...............the most difficult job I’ve ever had to do.
A. by heart B. by chance C. by far D. by myself
3. Psychologists have found that the number of social contacts we have...............only reason for loneliness.
A. are not the B. is not the C. are not an D. is not an
4. Many American have less time than ever before...............preparing food.
A. for spending B. by spending C. that they spend D. to spend
5. “What instructions did your .boss give you?” “He stipulated that everyone...............the meeting.”
A. would attend B. attends C. attended D. attend
6. It’s true that the old road is less direct and so a bit’ longer. We don’t take the new one,...............because we
173

Downloaded by Nhung Nguy?n (nhungnth2004@gmail.com)


lOMoARcPSD|20277325

don’t feel as safe on it.


A. anyway B. therefore C. though D. otherwise
7...............you to be offered that job, would you have to move to another city?
A. Provided that B. Should C. Were D. Had
8...............is the cause of most small-business failures is virtually an economic truism.
A. Undercapitalization B. Undercapitalization that
C. Where undercapitalization D. Thatiundercapitalization
9..............., Renaldo Nehemiah decided to take up professional football in 1982.
A. Nevertheless he was unbeatable as a hurdler
B. As a hurdler he was virtually unbeatable
C. Virtually unbeatable as a hurdler
D. He was virtually unbeatable as a hurdler
10. Last Sunday was............... that we decided to go on a picnic.
A. sobeautiful a day B. such beautiful a day
C. such beautiful day D. so beautiful day
CÂU 4: PREPOSITIONS AND PHRASAL VERBS
Select the best option for each blank.
1. The company was so successful during the 1990s that it...............500 new employees in a period of six
months
A. put on B. took on C. caught on D. laid on
2. The government have tried to...............the price of petro, but they failed.
A. get over B. bring on C. bring down D. put in
3. Although the coach had not thought her a good tennis player at first, she...............to be a champion.
A. came out B. turned out C. turned up D. came round
4. I didn't set...............to start an argument, it just happened.
A. off B. out C. up D. about
5. We can put you ...............for a few days if you have nowhere else to live.
A. on B. out C. up D. off
6. He...............a big fortune when he was young, so he didn’t have to work hard.
A. came into B. came up C. came across D. came round
7. The police are going to look...............the disappearance of the child.
A. around B. into C. through D. after
8. I cannot stay up late at night. I prefer...............early.
A. turning on B. turning up C. turning in D. turning out
9. I couldn’t...............what she was doing. It was so dark down there.
A. make up B. make out C. make for D. make from
10. The smell of the sea...............him...............to his childhood.
A. took / back B. brought / back C. reminded / of D. called / off
CÂU 5: READING COMPREHENSION
Passage A. Read the following passage and choose the best answer for each of the following questions
What we today call American folk art was; ỉndèed, art of/ by, and for ordinary, everyday "folks" who,
with increasing prosperity and leisure, created a market for art of all kinds, and especially for portraits.
Citizens of prosperous, essentially f middle-class republics -whether ancient Romans, seventeenth-century
Dutch burghers, or nineteenth-century Americans - have always shown a marked taste for portraiture.
Starting in the late eighteenth century, the United States contained increasing numbers of such people, and of
the artists who could meet their demands.
The earliest American folk art portraits come, not surprisingly, from New England - especially
Connecticut and Massachusetts - for this was a wealthy and populous region and the center of a strong craft
tradition. Within a few decades after the signing of the Declaration of Independence in 1776, the population
174

Downloaded by Nhung Nguy?n (nhungnth2004@gmail.com)


lOMoARcPSD|20277325

was pushing westward, and portrait painters could be found at work in western New York, Ohio, Kentucky,
Illinois, and Missouri. Midway through its First century as a nation, the United States's population had
increased roughly five times, and eleven new states had been added to the original thirteen. During these years
the demand for portraits grew and grew, eventually to be satisfied by the camera. In 1839 the daguerreotype
was introduced to America, ushering in the age of photography, and within a generation the new invention
put an end to the popularity of painted portraits. Once again an original portrait became a luxury,
commissioned by the wealthy and executed by the professional.
But in the heyday of portrait painting - from the late eighteenth century until the 1850's - anyone with
a modicum of artistic ability could become a limner, as such a portraitist was called. Local craftspeople - sign,
coach, and house painters - began to paint portraits as a profitable sideline; sometimes a talented man or
woman who began by sketching family members gained a local reputation and was besieged with requests for
portraits; artists found it worth their while to pack their paints, canvases, and brushes and to travel the
countryside, often combining house decorating with portrait painting.
1. In lines 5-6 the author mentions seventeenth-century Dutch burghers as an example of a group that……….
A. consisted mainly of self-taught artists B. appreciated portraits
C. influenced American folk art D. had little time for the arts
2. The word marked in line 6 is closest in meaning to...............
A. pronounced B. fortunate C. understandable D. mysterious
3. According to the passage, where were many of the first American folk art portraits painted?
A. In western New York B. In Illinois and Missouri
C. In Connecticut and Massachusetts D. In Ohio
4. The word this in line 10 refers to...............
A. a strong craft tradition B. American folk art
C. New England D. western New York
5. How much did the population of the United Stats increase in the first fifty years following independence?
A. It became three times larges B. It became five times larger
C. It became eleven times larger D. It became thirteen times larger
6. The phrase ushering in in line 18 is closest meaning to...............
A. beginning B. demanding C. publishing D. increasing
7. The relationship between the daguerreotype and the painted portrait is similar to the relationship between
the automobile and the...............
A. highway B. driver C. horse-drawn carriage D. engine
8. According to the passage, which of the following contributed to a decline in the demand for painted
portraits?
A. The lack of a strong craft tradition
B. The westward migration of many painters
C. The growing preference for landscape paintings
D. The invention of the camera
9. The author implies that most limners...............
A. received instruction from traveling teachers
B. were women
C. were from wealthy families
D. had no formal art training
10. The phrase worth their while in line 28 is closest in meaning to...............
A. essential B. educational C. profitable D. pleasurable
Passage B. Read the following passage and mark the letter A, B, C, or D on your answer sheet to
indicate the correct answer to each of the questions.
The first animated film Humurous Phases of Funny Faces,was made in 1906 by newspaper illustrator
James Blackton. He filmed faces that were drawn on blackboards in progressive stages. In New York City,
175

Downloaded by Nhung Nguy?n (nhungnth2004@gmail.com)


lOMoARcPSD|20277325

Winsor McCay exhibited his most famous film, Little Nemo (1910) and Gertie the Dinosaur (1914). His films
featured fluid motion and characters with individual personalities. For the first time, characters drawn of life
seemed to live on the screen. In 1914, John R. Bray streamlined the animation process, using assembly line
techniques to turn out cartoons.
By 1915,1 film studios began producing cartoons series. The Pat Sullivan studio produced the series
featuring Felix the Cat. He became one of the most beloved characters of the silent-film era. The Max
Fleischer studio produced series starring Ko-Ko the Clown and later Betty Boop and Popeye.
The first cartoon with sound was Steamboat Willie (1928), which introduced Mickey Mouse. This,
film was produced by Walt Disney, the most famous of American animators. His early success enabled Disney
to train his animators in anatomy, acting, drawing and motion studies. The results of this are apparent in Snow
White and the Seven Dwarfs (1937), the first full- length animated feature. It became an instant success and
still remains popular. Other important Disney films followed.
Warner Brothers’ Studio challenged Disney for leadership in the field with cartoons starring Bugs
Bunny, Daffy Duck, and other characters. These films were faster-. paced and featured slapstick humor. In the
1950s, a igroup of animators splintered off from Disney and formed United Production of America, which
rejected Disney’s realism and employed a bold, modernistic approach.
In the 1950s, children’s cartoons began to be broadcast on Saturday morning television and, later, in
prime time. Among the most successful were those made by William Hanna and Joseph Barbera, such as those
featuring Yogi Bear and the Flinstones.
The full- length animated film became popular again in the l980s and ‘90s. Producer Steven Spielberg
released his first animated film, American tail (1986), and Disney began a series of remarkable annual hits
with The Little Mermaid (1989). Who Framed Roger Rabbit (1988), a joint production of Spielberg and
Disney, blurred the lines between live action and animation. Animation returned to prime-time television
with the Fox Network’s The Simpsons. Animators had experimented with computer animation as early as the
1950s, but Toy Story (1955) was the first-full length film to be entirely computer animated. These
developments promise to bring about the most exciting era in animation since its heyday.
1. What does the passage mainly discuss?
A. the history of animated film
B. the life of Walt Disney
C. the development of one animated cartoon
D. the use of computer in animation
2. It can be inferred from the passage that the characters in Little Nemo and Gertie the Dinosaur...............\
A. were first drawn on a blackboard
B. were part of cartoon series
C. seemed to have their own personalities
D. did not look as life- like as Bladkton’s characters.
3. The word “streamlined” is closest meaning to...............
A. simplified B. revolutionized C. bypassed D. invented
4. The word “he” refers to...............
A. Pat Sullivan B. Felix the Cat
C. Max Fleischer D. Ko-Ko the Clown
5. What can be inferred from the passage about animated films produced before 1928?
A. They were not very popular B. They were longer than later movies
C. They were not drawn by hand D. They were silent films
6. According to the passage, the film Snow White and the Seven Dwarfs
A. showed the benefits of training the Disney animators.
B. was the first movie produced by Walt Disney.
C. was the last movie Disney made before his death.
D. did not become successful until many years later.
176

Downloaded by Nhung Nguy?n (nhungnth2004@gmail.com)


lOMoARcPSD|20277325

7. The author did not specifically mention characters produced by...............


A. Walt Disney B. Hanna and Barbera
C. United Production of America D. Warner Brothers
8. The phrase “blurred the lines” is closest in meaning to...............
A. eliminated the distinctions B. obscured the issues
C. answered the questions D. emphasized the problems
9. The first experiment with computer animation took place during the................
A. 1950s B. 1960s C. 1980s D. 1990s
10. Which of the following is closest in meaning to the word "heyday”?
A beginning B. decline C. prime D. rebirth
CÂU 6: CLOZE TEST
Passage A. Choose the word that best fits the blank space in the following passage.
If you’re an environmentalist, plastic is a word you tend to say with a sneer or a snarl. It has become a
symbol of our wasteful, throw-away society. But there seems little (1)……………it is here to stay, and the
truth is, of course, that plastic has brought enormous benefits even environmental ones. It’s not really the
plastics themselves that are the environmental (2)……………- it’s the way society choose to use and (3)
……………them.
Almost all the 50 or so different kinds of modem plastic are made from oil, gas or coal - non-
renewable natural resources. We import jvell ovpr three million tones of the stuff in Britain each year and,
sooner or later, most of it is thrown away. A high (4)…………… of our annual consumption it is in the form
of packaging, and this (5)……………about seven per cent by weight of our domestic (6)……………Almost
all of it could be recycled, but very little of it is, though the plastic recycling industry is growing fast.
The plastics themselves are extremely energy-rich - they have a higher calorific (7)……………than
coal and one (8)…………… of ‘recovery’ strongly (9)…………… by the plastic manufacturers is the (10).
…………… of waste plastic into a fuel.
1. A. evidence B. concern C. doubt D. likelihood
2. A. poison B. disaster C. disadvantage D. evil
3. A. dispose B. store C. endanger D.abuse
4. A. portion B. amount C. proportion D. rate
5. A. takes B. makes C. parries D. constitutes
6. A. refuse B. goods C. requirements D. rubble
7. A. degree B. value C. demand D. effect
8. A. measure B. mechanism C. method D. medium
9. A. desired B. argued C. favored D. presented
10. A. conversion B. melting C.change D. replacement
Passage B. Choose the word that best fits the blank space in the following passage.
HELP ALWAYS AT HAND:
A MOBILE IS A GIRL’S BEST FRIEND
If it fits inside a pocket, keeps you safe as well as in touch with yọur office, your mother and your
children,, it is (1)........... worth having. This is the (2)…………… of the dwelling ranks, of female mobile-
phonp users who are beginning to (3),……………the consumer market.
Although Britain has been,(4)……………to be one of the most expensive places in the world to run a
mobile phone, both professional women and (5).……………mothers are undeterred. At first, the mobile
phone was a rich man’s plaything, or a businessman’s (6)……………symbol. Now women own almost as
many telephones as men do - but for very different reasons.
The main (7)……………for most women customers is that it provides a form of communications
back-up, wherever they are, in case of contingency. James Tanner of Tancroft Communications sayis: “The
majority of people buying phones from us this year were women - often young women - or men who were
buying for their mothers, wives and girlfriends. And it always seems to be a question of (8)……………of
177

Downloaded by Nhung Nguy?n (nhungnth2004@gmail.com)


lOMoARcPSD|20277325

mind.”
“Size is also (9)……………for women. They want something that will fit in a handbag,” said Mr Tanner.
“The tiny phones coming in are having a very big (10)……………This year’s models are only half the size of
your hand.”
1. A. totally B. certainly C. absolutely D. completely
2. A. vision B. vista C. view D. conception
3. A. master B. dominate C. overbear D. command
4. A. demonstrated B.seen C. established D. shown
5. A. complete B. total C. absolute D. full-time
6. A. status B. fame C. power D. prestige
7.A. attraction B. enticement C. charm D. lure
8. A. tranquility B. calmness C. peace D. serenity
9. A. necessary B. crucial C. urgent D. essential
10. A. impression B. perception C. impact D. image

PHẦN TỰ LUẬN (Written Questions)


CÂU 1:
Filled each numbered blank with ONE suitable word.
Passage A
Bees, classified ……………(1) over 10,000 species, are insects found in almost every part of the
world except the northernmost and southernmost regions. One commonly known ……………(2) is the
honeybee, the only bee that produces honey and wax. Humans use the wax in making candles, lipsticks, and
other products, and they use the honey ……………(3) a food. While gathering the nectar and pollen with
……………(4) they make honey, bees are simultaneously helping to fertilize the flowers on which they ,
……………(5). Many fruits and vegetables would not survive if bees did not carry the pollen from blossom to
blossom.
Bees live in a structured environment and structure within a ……………(6), which is a nest with
storage space for the honey. The different types of bees each perform a unique function. The worker bee
carries nectar to the hive in a special stomach ……………(7) a honey stomach. Other workers make beeswax
and shape it into a honeycomb, which is a waterproof mass of six-sided compartments, or cells. The queen
lays ……………(8) ip completed cells. As the workers build more cells, the queen lays more eggs.
All workers, like the queen, are female, but the workers are smaller than the queen. The male
honeybees are called drones; they do no work and cannot ……………(9). They are developed from
unfertilized eggs, and their only job is to impregnate a queen. The queen must be fertilized in order to lay
worker eggs. During the season when less honey is available and the drone is of no further use, the workers
block the drones from……………(10) the honey so that they will starve to death.
Passage B
Sometimes you know things about people the first (11)……………you see them, for example, that you
want to be friends with them (12)……………that you don't trust them. But perhaps this kind of intuition isn't
as hard to explain as (13) may seem. For instance, people give out body (14)……………signals
all the time. The wạy you hold your body, head and arms tells people about your (15)……………If you hold
your arms tightly at your sides, or fold them across your (16).……………, people will generally feel that you
are being defensive. Holding your head to one side shows interest in the other, (17)…………… an easy, open
posture indicates that yóu are self-confident. All this affects the way you feel about someone
Also, a stranger may remind you (18)……………a meeting with someone. This may be because of
something as simple as the fact that he or she is physically similar to someone who (19)……………you well
or badly. Your feelings about a stranger could be (20)……………by a smell in the air that brings to
mind a place where you were happy as a child. Since even a single word can bring back a memory such as
that, you may never realize it is happening.
178

Downloaded by Nhung Nguy?n (nhungnth2004@gmail.com)


lOMoARcPSD|20277325

CÂU 2:
A. Use the correct form of the word given in parentheses to fill in the blank in each sentence.
1. The government lSi taking care not to rush...............into another controversy. (head)
2................, she was looking for the father she had never known. (conscious)
3. She added two...............of sugar to the soup. (spoon)
4. All letters will be treated with complete...............(confidential)
5. The V8 engines are all...............with each other. (change)
6. The painting was sold to an ...............American dealer. His name was not given. (identify)
7. They tried to ensure ...............across the different departments. (uniform)
8. The building did not fit to live in. It was totally...............(habitat)
9. This year saw a...............in the upward trend in sales. (continue)
10. Claire has a wide circle of friends and...............(acquaint)
B. Complete the following passage by using the correct forms of the words given in the box. (0) has been
done as an example.
conceive history knowledge obrserve continue
document replace believe planet little discover

(0) Historically, Mars was thought to be the most likely planet to harbour life There is a reflection of
such (11) …………..in popular culture as expressed in literature, radio and film. Public fascination with
Martians began in the late 19th century when, in 1877, astronomer Giovanni Sciaparelli reported (12).
…………..of large channels on Mars.
In 1897, H. G. Wells’ The War of the World was the first major work to explore the (13)………….. of
the “extraterrestrial invader" and exerted a substantial influence on the public psyche. A few years later, even
(14)…………..astronomers such as Percival Lowell seriously advocated the possibility of life forms as
described in his book Mars as the Adobe of Life (1910). Consequently, Mars began to take a special place in
popular culture around the turn of the 20th century, (15)…………..until today. However, this does not (16)
…………..the unique role of Mars in the history of science.
Specifically, the (17)…………..of the movement of Mars, by Johannes Kepler (1571 - 1630), led to
the formation of his three laws of (18)…………..motion which shattered mediaeval anthropocentric notions of
astronomy and laid the foundations for the (19) .………….. of Isaac Newton (1643 - 1727). Like no other
planet, Mars has left (20)…………..marks on human imagination and thought.
Ex: 0. Historically
CÂU 3:
The passage below contains 10 mistakes. Underline the mistakes and correct them in the space provided
in the column on the right (0) has been done as an example.
Reading to one is a modern activity which was almost unknown to the scholars of the classical and
medieval worlds, while during the fifteenth century the term “reading” undoubtedly means reading aloud.
Only during the nineteenth century did silent reading become commonplace.
One should be wary, therefore, of assuming that silent reading came about simply because reading
aloud was a distraction to others. Examinations of factors related to the historical development of silent
reading has revealed that it became the usual mode of reading for most adults mainly because the tasks
themselves changed in character.
The last century was seen a steady gradual increase in literacy and thus in the number of readers. As
the number of readers increased, the number of potentially listeners declined and thus there was some
reduction in the need to read aloud. As reading for the benefit of listeners grew less common, so come the
flourishing of reading as a private activity in such public places as libraries, railway carriages and offices,
where reading aloud would cause distraction to other readers.
Towards the end of the century, there was still considered argument over whether books should be
used for information or treated respectfully and with whether the reading of materials such as newspapers was
179

Downloaded by Nhung Nguy?n (nhungnth2004@gmail.com)


lOMoARcPSD|20277325

in some way mentally weakening. Indeed, this argument remains with us still in education. However,
whatever their virtues, the old shared literacy culture had gone and was replaced by the printed mass media on
the one hand and by books and periodicals for a specialised readership on the other.
By the end of the twentieth century, students were being recommended adopting attitudes to books and
to use reading skills which were inappropriate, if not impossible, for the oral reader. The social, cultural and
technological changes in the century had greatly altered what the term “reading” implied.
0. one → oneself
CÂU 4:
A. Complete the sentences in such a way that each of the sentences means almost the same as the
sentence printed before it.
1. You cannot say anything that would persuade me to live overseas.
→ Nothing…………………………………………………………………………………………………
2. The chairman’s leaving just before you’re due to arrive:
→ By ……………………………………………………………………………………………………..
3. It was difficult to understand her colleagues’ open hostility towards her proposal.
→ That her colleagues ……………………………………………………………………………………..
4. The club owner became a media celebrity, as well as extremely rich.
→ Not only …………………………………………………………………………………………………
5. David played the plain role when the proposal was drafted.
→ David was………………………………………………………………………………………………..
B. Use the wordfs) given in brackets and make any necessary additions to write a new sentence in such
a way that it is as similar as possible in meaning to the original sentence. Do not change the form of the
given word(s).
6. If you hadn’t changed our original agreement, everything would have been fine. (stuck to)
→……………………………………………………………………………………………………….
7. I think you should have some consideration for those who don’t have lives as privilege as yours, (spare)
→………………………………………………………………………………………………………
8. We decided to stay longer because we were so thrilled by the place, (extend)
→………………………………………………………………………………………………………
9. Mai felt entirely comfortable when her boss was around (ease)
→………………………………………………………………………………………………………
10. John found it difficult to get used to the fact that he was fired. (terms)
→ ………………………………………………………………………………………………………

TRƯỜNG THPT CHUYÊN THĂNG LONG - ĐÀ LẠT LÂM ĐỒNG


A. MULTIPLE CHOICE
I. PHONOLOGY
Choose the word which has the underlined part pronounced differently from the rest:
1. A. width B. anthem C. thus D. thumb
2. A. arrive B. steward C. coward D. award
3. A. footage B. fruitage C. stillage D. birdcage
4. A. biopsy B. unify C. polity D. goofy
5. A. thereat B. thermite C. thrice D. theist
Choose the word that is stressed differently from the others in the list:
1. A. manure B. fertile C. creature D. decade
2. A. carpenter B. forefinger C. aesthetic D. boundary
3. A. diplomacy B. synonymous C. elephantine D. petroleum
4. A. register B. orchestra C. dramatist D. deposit
5. A. lunatic B. nomadic C. republic D. heroic
180

Downloaded by Nhung Nguy?n (nhungnth2004@gmail.com)


lOMoARcPSD|20277325

II. WORD CHOICE


Choose the best options to complete the following sentences.
1.I am not by…………… a particularly ambitious man.
A. inclination B. habit C. character D. tendency
2. It is essential to be on the……………for any signs of movement in the undergrowth since there are
poisonous snakes in the area.
A. guard B. care C. alarm D. alert
3. I was …………… by his lack of intelligence. I couldn’t believe anyone could be so stupid.
A. shattered B. staggered C. speechless D. startled
4. After much debate, he reached a……………
A. manifestation B. consensus C. precision D. cohesion
5. I could see the lantern……………in the dark.
A. gleaming B. glowing C. glistening D. glimmering
6. It was a daring robbery, which took place in……………daylight.
A. broad . B. total C. wide D. absolute
7. I’m afraid Tim doesn’t take much care over his homework. He usually does it……………
A. any old how B. any how C. how on earth D. how come
8. When I advised you to change jobs, I had your best ……………at heart.
A. feelings B. interests C. thoughts D. aspects
9. Did Jane pass her exam? Yes, but only just. It was……………the pass mark was forty -five percent and she
got forty-six.
A. a narrow escape B.atightspot C. a clear cut D. a close thing
10. What a mad thing to do! You could all have been killed! It was……………folly.
A. merely B. only C. sheer D. wild
III GRAMMAR AND STRUCTURES
Choose the best options to complete the following sentences.
1. Beneath the streets of a modem city……………of walls, columns, cables, pipes,and tunnels required to
satisfy the needs of its inhabitants.
A. where exists B. the existing network
C. the network’s existence D. exists the network
2. The recommendation that all people affected by the storm……………immediately was proved.
A. had to be evacuated B. be evacuated C. evacuated D. were evacuated
3. It should be easy for Peter to find more time to spend with his children……………he no longer has to work
in the evenings and on weekends.
A. now that B. owing to
C. in the event that D. in consequence
4. Sunshine - at last! Imagine if the English weather was like this all the time……………wonderful?
A. Will it not be B. Had it not been C. Wouldn’t it be D. Would it not be
5. Most of……………archaeologists know about prehistoric cultures is based on studies of material remains.
A. these B. what C. which D. the
6. She……………fainted when she heard that her child died.
A. rather than B. nothing but C. all but D. near
7.- “Who won the election for mayor?” - "A man ……………to represent every minority group in the city.”
A. claim B. claimed C. is claimed D. claiming
8. There is a real possibility that these animals could be frightened……………a sudden loud noise.
A. being there B. should there be C. were there D. there has been
9. Excessive logging of forests in the past century has resulted in……………
A. which it is known as deforestation
B. knowing this as deforestation
181

Downloaded by Nhung Nguy?n (nhungnth2004@gmail.com)


lOMoARcPSD|20277325

C. that is known as deforestation


D. what becomes known as deforestation
10. Valentine’s Day gradually became associated with the feast day of two Roman martyrs,……………St.
Valentine, who lived in the 3rd century.
A. who named B. two of whom named C. were called D. both named
IV. PHRASAL VERBS AND PREPOSITIONS
Choose the best options to complete the following sentences.
1. Despite all the interruptions, he……………with his work.
A. stuck at B. held on C. hung out D. pressed on
2. When the funds finally……………, they had to abandon the scheme.
A. faded away B. clamped down C. petered out D. fobbed off
3. I meant to sound confident at the interview but I'm afraid I……………as dogmatic.
A. came out B. came through C. came off D. came over
4. Having decided to rent a flat, we……………contacting all the accommodation agencies in the city.
A. set about B. set to C. set out D. set off
5. The new regime determined to……………compulsory military service.
A. stop off B. phase out C. end up D. break off
6. After standing in the sun for more than an hour, two of the people in the queue passed……………and they
came……………after some women threw water in their faces.
A. out/ to B. out/ over C. away/ round D. by /, round
7. If you never put oil in your engine, one day it will……………
A. flake out B. shut down C. go off D. seize up
8. It's very late and I have a busy day tomorrow. If you don't mind, I think I’ll……………now.
A. layoff B. turn in C. blackout D. sleep on
9. If you think and behave as if you were better than anyone else, people may describe you as……………up.
A. wound B. pop C. dressed D. stuck
10. No one can function properly if he or she is……………adequate sleep.
A. deprived of B. derived from C. took away D. got rid of
V. READING COMPREHENSION
READING PASSAGE 1
Read the text below and choose the best answer to each question.
Sigmund Freud was . not a literary theorist. However, he did contribute to critical theory through both
his theories and his use of art to show that the application of psychology can extend to the highest forms of
cultures. Freud was always interested in literature, and he drew some of the best illustrations of his theories
from classic poems and plays.
Freud saw the unconscious as the impetus of both cultural and psychic activity. Therefore, the same
principles operated in both, and that the same mechanisms - such as displacement and symbolization -
applied. While Freud was not the first to note the importance of the unconscious mind, he was the first to
attempt a coherent theory of its operation and function. He argued that the unconscious operates according to
universal law, and is crucial to all aspects of mental life that involve fantasy, or diversion from reality. From
this point of view, it is natural to apply Freudian principles to imaginative literature. Writers transform
individual, unconscious fantasy into universal art - a kind of formal fantasy halfway between a reality that
denies wishes and a world of imagination in which every wish is granted.
In focusing on the unconscious origins for literature, Freud was in a sense reviving the traditional idea
of divine inspiration. [1] Philosophers and art theorists have often turned to such a theory of the imagination to
explain multiple meanings, repetition, and any apparent disorder in art. Similarly, psychoanalysis uses the
theory of the unconscious to explain examples of “disorder’ in consciousness, such as dreams.
[2] This analogy allowed Freud to suggest that fantasies called art could be interpreted in the same way
as dreams. Writers, as Freud noted, have always seen great significance in dreams. In his view, portrayals of
182

Downloaded by Nhung Nguy?n (nhungnth2004@gmail.com)


lOMoARcPSD|20277325

dreams in works of literature supported his own theories about their structures, mechanisms, and
interpretation. For example, the mechanisms of displacement and symbolization obviously resemble the
literary devices of metaphor and symbolism. [3] Critics of Freud have objected that the non-logical processes
of the unconscious do not resemble the conscious effort that results in work of literature. Freud would reply
that while conscious thought is necessary to produce works of art, the creative sources of art remain in the
conscious. In this view, conscious activity merely obscures what is truly important in art. What interested
Freud were the deep unconscious structures literature shares with myth and religion, as well as with drfeams.
The apparent individuality of literature was not as significant as its ultimate universality. [4]
1. Which of the following best states the main idea of the reading?
A. The best way to understand the creation of literature is through Freud’s theory of psychoanalysis.
B. Freud argued convincingly that both psychic phenomena and literature may be interpreted with reference to
the unconscious.
C. Creating works of literature is very similar to dreaming.
D. Freud’s theories explain why both dreams and literature contain various forms of disorder
2. According to the passage, which of the following is true of Freud?
A. He was- a literary theorist.
B. He has had an influence on literary theory.
C. He wrote several plays and poems that illustrate his theories.
D. He was the first to discover the unconscious.
3. The word impetus in paragraph 2 could best be replaced by…………..
A. source B. opposite C. form D. reason
4. The word both in paragraph 2 refers to…………..
A. displacement and symbolization
B. repression and the economy of psychic expenditure
C. cultural and psychic phenomena
D. principles and mechanisms
5. The author uses the phrase formal fantasy in paragraph 2 in order to…………..
A. describe the nature of literature
B. describe the nature of the unconscious
C. give an example of diversion from reality
D. give an example of a Freudian principle
6. Which is the best place for the following sentence?
“And like dreams, literary works can have more than one interpretation.”
A. [1] B. [2] C. [3] D. [4]
7. According to the passage, displacement in dreams is similar to…………..
A. symbolization B. metaphor C. symbolism D. repression
8. What possible objection to the passage’s main idea does the author discuss in the last paragraph?
A. Freud emphasized the unconscious, but writing results from conscious thought.
B. Freud claimed that art is created logically, but it really has unconscious origins
C. Writers have never placed much significance on dreams.
D. Freud argued that literature is individual, but it is actually universal.
9. The word their in paragraph 4 refers to…………..
A. writers B. works C. theories D. dreams
10. Why does the author mention multiple meaning and repetition in paragraph 3?
A. To emphasize the non-rational nature of art
B. To give examples of “disorder” in art
C. To show the similarity between art and dreams
D. To give examples of divine inspiration
READING PASSAGE 2
183

Downloaded by Nhung Nguy?n (nhungnth2004@gmail.com)


lOMoARcPSD|20277325

Read the text below and choose the best answer to each question.
ARTS AND CRAFTS MOVEMENT
The Arts and Crafts Movement in the United States was responsible for sweeping changes in attitudes
toward the decorative arts, then considered the minor or household arts. Its focus on decorative arts helped to
induce United States museums and private collectors to begin collecting furniture, glass, ceramics, metalwork,
and textiles in the late nineteenth and early twentieth centuries. The fact that artisans, who were looked on as
mechanics or skilled workers in the eighteenth century, are frequently considered artists today is directly
attributable to the Arts and Crafts Movement of the nineteenth century. The importance now place on
attractive and harmonious home decoration can also be traced to this period, when Victorian interior
arrangements were revised to admit greater light and more freely flowing spaces.
The Arts and Crafts Movement reacted against mechanized processes that threatened handcrafts and
resulted in cheapened, monotonous merchandise. Founded in the late nineteenth century by British social
critics John Ruskin and William Morris, the movement revered craft as a form of art. In a rapidly
industrializing society, most Victorians agreed that art was an essential moral ingredient in the home
environment, and in many middle-and working-class homes craft was the only form of art. Ruskin and his
followers criticized not only the degradation of artisans reduced to machine operators, but also the impending
loss .of daily contact with handcrafted objects, fashioned with pride, integrity, and attention to beauty.
In the United States as well as in Great Britain, reformers extolled the virtues of handcrafted objects:
simple, straightforward design; solid materials of good quality; and sound, enduring construction techniques.
Whether abstract, stylized, or realistically treated, the consistent theme in virtually all Arts and Crafts design
is nature.
The Arts and Crafts Movement was much more than a particular style; it was a philosophy of domestic
life. Proponents believed that if simple design, high- quality materials, and honest construction were realized
in the home and its appointments, then the occupants would enjoy moral and therapeutic effects. For both
artisan and consumer, the Arts and Crafts doctrine was seen as a magical force against the undesirable effects
of industrialization.
1. The passage primarily focuses on nineteenth-century arts and crafts in terms of which of the following?
A. Their naturalistic themes B. Their importance in museum collections
C. Their British origin D. Their role in an industrialized society
2. According to the passage, before the nineteenth century, artisans were thought to be…………...
A. defenders of moral standards B. creators of cheap merchandise
C. skilled workers D. talented artists
3. It can be inferred from the passage that the Arts and Crafts Movement would have considered all of the
following to be artists EXCEPT…………...
A. creators of textile designs
B. people who produce handmade glass objects
C. operators of machines that automatically cut legs, for furniture
D. metalworkers who create unique pieces of jewelry
4. The word "fevered" in the second paragraph is closest in meaning to…………...
A. respected B. described C. avoided D. created
5. According to paragraph 2, the handcrafted objects in the homes of middle- and working-class families
usually were…………...
A. made by members of the family
B. the least expensive objects in theirrhomes
C. regarded as being morally uplifting
D. thought to symbolize progress
6. The word "extolled" in the third paragraph is closest in meaning to…………...
A. exposed B. praised C. believed D. accepted
7. The author mentions all of the following as attributes of handcrafted objects EXCEPT…………..
184

Downloaded by Nhung Nguy?n (nhungnth2004@gmail.com)


lOMoARcPSD|20277325

A. the pride with which they were crafted


B. the complexity of their design
C. the long time that they lasted
D. the quality of their materials
8. The word "consistent" in the third paragraph is closest in meaning to…………...
A. conservative B. considerable C. constant D. concrete
9. According to the passage, which of the following changes occurred at the same time as the Arts and Crafts
Movement?
A. The creation of brighter and more airy spaces inside homes
B. The rejection of art that depicted nature in a realistic manner
C. A decline of interest in art museum collections
D. An increase in the buying of imported art objects
10. Which of the following statements is supported by the passage?
A. Private collectors in the nineteenth century concentrated on acquiring paintings.
B. The Arts and Crafts Movement in the United States, unlike the one in Britain, did not react strongly against
mechanized processes.
C. Handcrafted objects in the United States and Britain in the nineteenth century did not use geometric
designs.
D. The Arts and Crafts Movement believed in the beneficial effect for people from being surrounded by
beautiful objects.
VI. GUIDED CLOZE TEST
GUIDED CLOZE 1
Read the text below and decide which answer best fits each space.
HELP IS ALWAYS AT HAND
A MOBILE IS A GIRL’S BEST FRIEND
If it fits inside a pocket, keeps you safe as well as in touch with your office, your mother and your
children, it is (1)…………. worth having. This is the (2)............ of the dwelling ranks of female mobile-phone
users who are beginning to (3)............ the consumer market.
Although Britain has been shown to be one of the most expensive places in the world to (4)............ a
mobile phone, both professional women and (5)………….mothers are undeterred. At first, the mobile phone
was a rich man’s plaything, or a businessman’s (6)………….symbol. Now women own almost as many
telephones as men do - but for very different reasons.
The main attraction for most women customers is that it (7)............ a form of communications back-
up, wherever they are, in case of emergency. James Tanner of Tancroft Communications says: "The (8)............
of people buying phones from us this year were women - often young women - or men who were buying for
their mothers, wives and their girlfriends. And it always seems to be a question of (9)............ of mind".
“Size is also (10)………….for women. They want something that will fit in a handbag,” said Mr.
Tanner, "The tiny phones coming in are having a very big impact. This year’s model are only half the size of
your hand."
1. A. totally B. certainly C. absolutely D. completely
2. A. swelling B. increasing C. boosting D. maximizing
3. A. master B. dominate C. overbear D. command
4. A. function B. drive C. work D. run
5. A. complete B. total C. fulktime D. absolute
6. A. prestige B. fame C. power D. status
7. A. supplies B. furnishes C. provides D. gives
8. A. most B. preponderance C. majority D. bulk
9. A. tranquility B. calmness C. serenity D. peace
10. A. crucial B. necessary C. urgent D. essential
185

Downloaded by Nhung Nguy?n (nhungnth2004@gmail.com)


lOMoARcPSD|20277325

GUIDED CLOZE 2
Read the text below and decide which answer best fits each space.
A popular character in the nation’s top television soap is (1) ………….for something of which she was
probably innocent. Having been found guilty of a series of fraudulent acts, she contemplates months of
incarceration. A good story-line, but wait! Within hours the television station is being (2)………….with calls
of protest. A national newspaper soon (3)…………. up a campaign to have her freed. Thousands of T-shirts
are printed with slogans (4)………….for her release. Offices and factory floors echo to the sounds of
animated debate. It is even mentioned in Parliament. It's easy to (5)…………., off such idiocịes a? 'a bit of
fun’, but there's surely a more serious side. A fair proportion of viewers were obviously (6)………….in by the
story to such an extent that their perception of fact and fiction was clearly (7)………….Everywhere, millions
will pore over their 7-day TV guide to get a preview of the week's soaps. If a character is deemed to be past
his or her sell- by date, and the decision has been taken to (8)…………. him or her out, possibly to have them
(9)………….off in spectacular fashion, viewing figures are likely to soar by up to 25%. A life-threatening fire
can be relied upon to add millions to the ratings. A major wedding can find half of Britain sitting (10)………
to the screen! It's all very strange.
1. A jailed B. prisoned C. sentenced D. charged
2. A bombed B. attacked C. streamed D. inundated
3. A opens B. starts C. puts D. establishes
4. A demanding B. calling C. insisting D. sounding
5. A laugh B. smile C. take D. put
6. A thrown B. carried C. indulged D. taken
7. A blurred B. hampered C. tampered D. glossed
8. A write B. cast C. sort D. work
9 A ridden B. taken C. driven D. killed
10. A swamped B. stuck C. glued D. paralysed
B. WRITTEN TEST
I. OPEN CLOZE TEST
OPEN CLOZE 1
Read the text below and think of the word which best fits each space. Use only ONE WORD for each
space.
Society does not encourage crying as a form of (1)………….expression. The shedding of tears makes
people uncomfortable and embarrassed. The (2) ………….race is the only species that sheds emotional tears.
What (3)………….does crying serve?
It has been suggested that emotional tears play a role in the (4)………….of stress. Scientific studies
have revealed that emotional tears contain a chemical which is one of the body’s natural (5)………….killers.
The chemical is not found in tears (6)………….as a response to slicing onions. Scientists think that tears may
(7) ………….to remove substances that accumulate in the body under stress. This would explain why many
people feel much better after crying. As a result of the view that it is not (8) .......... . ..to cry, men tend to hold
back their tears. This may explain why men develop more stress-related diseases than women.
(9) …………. emotion, be it a feeling of sadness or happiness, is stressful. Tears are therefore shed as
a response to sorrow, anger or joy. It is natural to cry. Indeed, it may be (10)………….to hold back tears.
OPEN CLOZE 2
Read the text below and think of the word which best fits each space. Use only ONE WORD for each
space.
Another critical factor that plays a part in susceptibility to colds is age. A study done by the University
of Michigan School of Public Health revealed particulars that seem to hold (1)………….for the general
population. Infants are the most cold-ridden group, averaging more than six colds in their first year. Boys have
more colds than girls up to age three. After the age of three, girls are more susceptible than boys, and teenage
girls average three colds a year (2) ………….boys’ two.
186

Downloaded by Nhung Nguy?n (nhungnth2004@gmail.com)


lOMoARcPSD|20277325

The general incidence of colds continues to decline into maturity. Elderly people who are in good
health have as (3)………….as one or two colds annually. One (4) …………. is found among people in their
twenties, especially women, who show a rise in cold infections, because people in this age group are most (5)
…………. to have young children. Adults who delay having children (6)………….their thirties and forties
experience the same sudden increase in cold infections.
The study also found that economics plays an important role. As income increases, the (7)………….at
which colds are reported in the family decreases. Families with the lowest income suffer about a third more
colds than families at the (8)………….end. Lower income generally forces people to live in more cramped
quarters than those typically occupied by wealthier people, and crowding increases the opportunities for the
cold virus to travel from person to person. Low income may also adversely influence diet. The degree (9)
…………. which poor nutrition affects susceptibility to colds is not yet clearly established, (10) …………. an
inadequate diet is suspected of lowering resistance generally.
III. WORD FORMATION
PART 1: Complete each sentence, using the correct form of the word in parentheses.
1. I don’t think their marriage will last long. They’re…………..quarrelling. (continue)
2. I’ve tried to advise my daughter against hitch-hiking around Europe alone, but she won’t listen to me. She’s
so…………..(head)
3. The book is full of scientific…………..(technique)
4. He must be drunk. He’s making ......................... remarks, (sense)
5. I’m going to quit my job, move abroad and start completely ............ (new)
6. I think saying that all black people are lazy is a very ................... remark, to be honest. (race)
7. I was surprised by his ................................to break the law. (prepare)
8. Hundreds of unemployed .......................could be pushed back towards crime by the closure of job training
programs. (prison)
9. The private school feared losing its .........................with the state’s university system. (credit)
10. The newspaper report before the trial may have had a ...................... effect on the verdict. (prejudice)
PART 2: Complete the passage with the correct forms from the words given in the box.
SKILLED / SUCCEED ! PHYSICAL / CRITICS / AESTHETE
ART / INUNDATION / VISIONARY / DUST / DISPOSITION

Native Americans probably arrived from Asia in (1)…………..waves over several millennia, crossing
a plain hundreds of miles wide that now lies (2) …………..by 160 feet of water released by melting glaciers.
For several periods of time, the first beginning around 60, 000 B.C. and the last ending around 7,000 B.C., this
land bridge was open. The first people traveled in the (3)…………..trails of the animals they hunted. They
brought with them not only their families, weapons, and tools but also a broad (4)…………..understanding,
sprung from dreams and (5)…………..and articulated in myth and song, which complemented their scientific
and historical knowledge of the lives of animals and of people. All this they shaped in a variety of languages,
bringing into being oral literatures of power and beauty.
Contemporary readers, forgetting the origins of western epic, lyric, and dramatic forms, are easily (6)
…………..to think of “literature” only as something written. But on reflection it becomes clear that the more
(7)…………..useful as well as the more frequently employed sense of the term concerns the (8)…………..
of the verbal creation, not its mode of presentation. Ultimately, literature is (9) ………….. valued,
regardless of language, culture, or mode of presentation, because some significant verbal achievement results
from the struggle in words between tradition and talent. Verbal art has the ability to shape out a compelling
inner vision in some (10)…………..crafted public verbal form.
III. ERROR IDENTIFICATION
The following passage contains 10 errors. Identify and correct them.
CHESS TOURNAMENTS
All tournament chess games are played with a chess clock - that is, two clocks attached together. When
187

Downloaded by Nhung Nguy?n (nhungnth2004@gmail.com)


lOMoARcPSD|20277325

one player does his move, he presses a button which stops his clock and starts his opponent clock. Whoever
fails to keep up the time limit, no matter what the position on the board, loses the game.
Weekend tournaments with a fast time limit and long sessions of play of up to twelve hours a day are
very strenuous and result from fatigue and time troubles. The play is quite sharp. Active, attacking chess is the
order of the day and it is difficult to maintain any sustained, precise defence against such play. A score of the
game must be kept as play goes on. Each move is written down on a score sheet, it has to be handed to the
tournament officials in the end of each round. The sorely thought in everybody’d head is to win. Talent and
youth- that’s what is needed for success at chess, with the emphasis on youth. Some approach the board with a
slow, purposeless manner without giving you a second glance- you simply don’t count. They seem to imply
that the outcome is a foregone conclusion for them; you only need to accept them with good grace.
IV. SENTENCE TRANSFORMATION
Rewrite the sentences with the given words or beginning in such a way that their meanings remain
unchanged.
1. The police are advising vigilance as there have been more robberies lately.
Due to an…………………………………………………………………………………………..
2. It was Nick’s advice that saved me from bankruptcy.
Had…………………………………………………………………………………………..bankrupt.
3. You may be dismissed if you fail to observe the company's dress code.
Failure …………………………………………………………………………………………..
4. "There is no point in writing it all out in longhand if you can use a typewriter, isn't there?”
She dissuaded ……………………………………………………………………………………
5. I tried to remember what has happened all those years before.
I cast……………………………………………………………………………………………..
6. David reckoned that his success was due to incredible luck. (PUT)
→.………………………………………………………………………………………………..
7. Don't forget those who are at work on this lovely, sunny day! (SPARE)
→.………………………………………………………………………………………………..
8. There is an enormous variety of tourist attractions in this part of the country. (MANNER)
→.………………………………………………………………………………………………..
9. Her latest novel isn't as good as her previous one. (PAR)
→.………………………………………………………………………………………………..
10. Students at the school are not allowed to go into the Rainbow Disco.(BOUNDS)
→The Rainbow Disco…………………………………………………………………………..
TRƯỜNG THPT CHUYÊN LƯƠNG VĂN CHÁNH - PHÚ YÊN
I. PHONOLOGY
A. Choose the word whose underlined part is pronounced differently from the others.
1. A. resume B. assume C. progressive D. release
2. A. anchor B. channel C. character D. chaotic
3. A. pamphlet B. tabloid C. applicable D. programme
4. A. exit. B. exile C. exchange D. exhale
5. A. therapy B.thesaurus C. mythology D. asthma
B. Choose the word whose stress pattern is different from the others.
1. A. experimental B. antiapartheid C. pasteurization D. immunology
2. A. revolutionize B. refrigerator C. multifunctional D. documentaries
3. A. popularity B. beneficial C. continental D. preparatory
4. A. dilemma B. standardize C. agree D. obstruct
5. A. silhouette B. presidential C. intellectual D. ambitious
II. WORD CHOICE
Choose the word which best completes each sentence.
188

Downloaded by Nhung Nguy?n (nhungnth2004@gmail.com)


lOMoARcPSD|20277325

1. The new teacher was taken advantage of by the students and often had to …………….her authority .
A. assert B. confirm C. inflict D. strike
2. He was given the France pọst even though his French is decidedly …………….
A. fluent B. untidy C. rusty D. disheveled
3. "Look, I don't know the best solution," he said …………….his shoulders and walking away from the table.
A. shaking B. shrouding C. shrugging D. bowing
4. Even though Ink Links is offering a larger discount, the Clear Image’s quality standards.…………….our
needs better.
A. suit B. fit C. match D. fix
5 After they vandalized the school, the teenagers…………….the scene.
A. flew B. flied C. fleed D.
fled
6.…………….by despair at her situation, she tried in vain to rob the local bank and ended up in prison for five
years.
A. Compelled B. Forced C. Desperate D. Driven
7. Bill and Mary resolved their problems after her brother got them to sit down and have a(n)…………….talk
with each other.
A. candid B. overt C. servile D. piteous
8. Four bedroom house for sale. Large garden. Close to…………….Reduced for quick sale.
A. facilities B. amenities C. equipment D. utilities
9. Jenkins! Yes, you …………….at the back of the class. If you find this whole episode so funny, why don’t
you come up to the front and explain to everyone exactly why!
A. frowning B. grinning C. glancing D. glimpsing
10. A bite of chocolate cake only…………….my desire for more.
A. whets B. spares C. lets D. changes
III. GRAMMAR AND STRUCTURES
Choose the answer that best completes the sentence.
1. That is Suzy’s car in the driveway. She.…………….at work!
A. shouldn't be B. couldn't be C. oughtn't be D. can't be
2. Only if I had known the difference…………….the more expensive car.
A. would I bought B. I would have bought
C. would I buy D. would I have bought
3. “John is really upset about losing his job.”- “Well,…………….once myself, I can understand.”
A. having been fired B. Fired C. having fired D. being fired
4……………., he remained optimistic.
A. Though badly wounded he was B. Badly wounded as he was
C. As he was badly wounded D. As badly wounded he was
5. We saw……………beautiful landscapes while on holiday.
A. quite a few B. quite much C. quite many D. quite some
6. This evidence should prove……………that he was telling the truth.
A. once and for all B. now and then C. over and above D. from time to time
7. A dividend is……………the only benefit a corporation can offer its shareholders.
A. no B. nor C. none D. not
8. I know it’s a big favour to ask but,……………I’ve done you plenty of favours in the past!
A. considering B. bearing in mind C. after all D. in the light
9. I will only sign the papers……………my solicitor.
A. within reach B. of the agreement of
C. under surveillance of D. in the presence of
10……………is more interested in sports than in music is apparent in his spending free time.
189

Downloaded by Nhung Nguy?n (nhungnth2004@gmail.com)


lOMoARcPSD|20277325

A. That Gerald Anderson B. Gerald Anderson, who


C. Gerald Anderson D. Because Gerald Anderson
IV. PREPOSITIONS AND PHRASAL VERBS
Choose the answer that best completes the sentence.
1.Harold's problems began when he fell ……………the wrong type of person and he ended up becoming a
criminal himself.
A. out with B. in with C. for D. back on
2. At every faculty meeting, Ms. Volatile always manages to put her foot in her mouth.
A. trip over, her big feet B. say the wrong thing C. move rapidly D. fall asleep
3. When you’re riding on the back of a motorbike, you need to……………tightly.
A. stick down B. hold up C. hang on D. sit up
4. Bert is a bit down in the……………He was sacked yesterday.
A. dumps B. sad C. low D. bad
5. Pull your socks……………or you won’t pass your exams.
A. down B. by C. over D. up
6. The company has a lot of problems but they all seem to ..................to money.
A. boil down B. cook up C. lead over D. reason out
7. The policeman shot off before anyone could stop him.
A. left in a hurry
B. let everyone know the truth about what was happening
C. fired his gun
D. closed all the doors
8. Sarah desperately wanted to be an actress, so when they offered her a part in the play, she..................it.
A. burst into B. seized on C. ran after D. jumped at
9. When the chairman ran off with his secretary, the Board tried to..................the matter.
A. switch off B. calm down C. hush up D. tuck away
10. Pm sorry to butt in, but I couldn’t help overhearing what you said.
A. interrupt you B. contradict you C. speak so rudely to you D. stop you
V. READING COMPREHENSIONS.
PASSAGE 1: Read the text below and choose the best answer (A,B, C or D) to each question.
1 Sharks have gained an unfair reputation for being fierce predators of large sea animals.
Humanity’s unfounded fear and hatred of these ancient creatures is leading to worldwide
slaughter that may result in the extinction of many
5 coastal shark species. The shark is the victim of a warped attitude of wildlife protection; we strive
only to protect the beautiful, non-threatening parts of our environment. And, in our efforts to
restore only non-threatening parts of our earth, we ignore other important parts.
A perfect illustration of this attitude is the contrasting attitude toward
10 another large sea animal, the dolphin. During the 1980s, environmentalists in the United States
protested the use of driftnets for tuna fishing in the Pacific Ocean since these nets also caught
dolphins. The environmentalists generated enough political and economic pressure to prevent tuna
companies from buying tuna that had been
15 caught in driftnets. In contrast to this effort on behalf of the dolphins, these same environmentalists
have done very little to help save the Pacific Ocean sharks whose population has decreased nearly to
the point of extinction. Sharks are among the oldest creatures on earth, having survived in the seas for
more than 350 million years. They are extremely
20 efficient animals, feeding on wounded or dying animals, thus performing an important role in nature of
weeding out the weaker animals in a species. Just the fact that species such as the Great White Shark
have managed to live in the oceans for so many millions of years is enough proof of their efficiency
and adaptability to changing environment. It is time for us humans, who may not survive another 1,000
190

Downloaded by Nhung Nguy?n (nhungnth2004@gmail.com)


lOMoARcPSD|20277325

years at the rate we are damaging the planet, to cast away our fears and begin considering the
protection of sharks as an important part of a program for protection of all our natural environment.
1. With which of the following topics is this passage primarily concerned?
A. Sharks are efficient creatures with bad reputations.
B. Sharks are some of the oldest creatures on earth.
C. Sharks illustrate a problem in wildlife protection.
D. The campaign to save dolphins was not extended to save sharks.
2. Which of the following is most similar to the meaning of the word “warped” in line 5?
A. distorted B. wasteful C. extravagant D. wanton
3. In the second paragraph, the word “generated” could be best replaced by
A. consumed B. absorbed C. designated D. produced
4. How did environmentalists mange to protect dolphins?
A. They prevented fishermen from selling them for meat.
B. They pressured fishermen into protecting dolphins by law.
C. They brought political pressure against tuna companies.
D. They created sanctuaries where dolphin fishing was not allowed
5. About how long have sharks live don the planet?
A. 25 million years B. 150 million years
C. 350 million years D. 500 million years
6. The author uses the phrase “weeding out” in line 21 to mean……………
A. strengthening something that is weak
B. feeding something that is hungiy
C. encouraging something that IS efficient
D. getting rid of something that is unwanted
7. The phrase “managed to live” in line 23 is used to infer that……………
A. surviving was difficult B. migration was common
C. procreation was expandingD. roaming was necessary
8. The word “proof’ in line 24 could be best replaced by which of the following?
A. characteristic B. evidence . C. praise D. customary
9. The phrase “to cast away” in line 26 means most nearly.…………….
A. to throw off B. to bring in C. to see through D. to set apart
10. What is the author’s tone in this passage?
A. explanatory B. accusatory C. gentle D. proud
PASSAGE 2: Read the passage and choose A,B,C or D to indicate the correct answer to each of the
questions.
1 Situated in the central mountains of Alaska, a peak named Denali
rises 20,320 feet above sea level. It is the highest peak in North America and the center of Denali
National park' One of America’s greatest wilderness areas, the park has had limited access to visitors,
but in spite
5 of this, tourism rose from 6000 visitors in 1950 to over 546,000 visitors in 1990. The increasing
popularity of this park is prompting serious discussions about the future use of Denali as well as how
to preserve wilderness areas in general.
One important issue of land use arises when parts of the National
10 Park are owned by individuals. In Denali, although most of the land in this vast tract of more than a
million acres is owned by the National Park Service, several thousand acres are still privately owned as
miring tracts. These mining tracts in Denali were once abundant sources of gold, but they also were
sources of heavy metals such as arsenic and lead that
15 polluted rivers and strearris.
Environmentalists were successful in getting the government to require mining companies to
191

Downloaded by Nhung Nguy?n (nhungnth2004@gmail.com)


lOMoARcPSD|20277325

submit statements showing the potential impact of a mining project before they are allowed to begin
mining. Because of this requirement, many individuals closed their mines and
20 some sold their land to the National Park Service. Some land, owners, however, are wondering if it is
better to sell their land to the government
or keep it for possible future use. Tourism in this previously remote area is bound to rise, as more
roads are built to provide easier access to the park. This increase in the number of visitors creates a
demand for hotels and other real estate development. The economic implications of this are of interest
to the land owners, but are dismaying to those interested in preserving the wilderness.
1. What is the primary focus of this passage?
A. Controversies over land use in Denali
B. Miners selling their property in Denali
C. Alaska building more roads to Denali
D. Limiting tourist access to Denali
2. The word “wilderness” in line 4 could be best replaced by the word…………..
A. dangerous B. natural C. rural D. pastoral
3. As used in the first paragraph, which of the following is more similar to the word “preserve”?
A. protect B. enclose C. investigate D. foster
4. The word “arises” in line 7 could be best replaced by…………..
A. surrenders B. occurs C. volunteers D. prospers
5. The word “tract” in line 11 refers to which of the following?
A. trail B. resort C. frontier D. expanse
6. What does the word “they” in line 14 refer to in this passage?
A. mining tracts B. gold C. million of acres D. sources
7. According to the passage, which of the following are pollutants in the Denali area?
A. gold B. pesticides C. human waste D. arsenic
8. Which of the following is closest in meaning to the phrase “potential impact” in line 17?
A. approximate cost B. expected value C. proposed size D. possible effects
9. The author infers that some mine owners might hesitate to sell their land to the Park Service for which the
following reasons?
A. They may be increasing demand for the ore of the mines.
B. They might want to move to the towns
C. They might receive more money selling their land to developers.
D. They might want to build a house on their property.
10. What is the author’s purpose in writing this passage?
A. To demonstrate the changes in Denali National Park.
B. To use Denali as an example of common park issues.
C. To introduce the wonders of the wilderness are of Denali
D. To explain the problems occurring in Denali Park
VI. CLOZE TEST
A. Read the following passage and then decide which answer (A, B, C or D) best fits each space.
There is no doubt at all that the Internet has made a huge difference to our lives. However, most
parents worry that their children spend too much time browsing the Internet or playing computer games,
hardly (1)………….doing anything else in their spare time. Naturally, parents want to know if these activities
are harmful to their children. What should they do if their children spend hours (2)………….a computer
screen?
Obviously, if children spend too much time (3)………….in some game instead of doing their
homework, then something is wrong. It is a good idea if parents and children decide together how much use
should be (4)………….of the Internet, and the child should (5)………….that it won’t interfere with
homework. If the child does not (6)………….to this arrangement, parents can take more drastic (7)...........
192

Downloaded by Nhung Nguy?n (nhungnth2004@gmail.com)


lOMoARcPSD|20277325

Any parent who is (8)........... alarmed about a child’s behavior should make an appointment to (9)
………….the matter with a teacher. Spending time in front of a computer screen does not (10) ………….
affect a child’s performance at school. Even if à youngster seems obsessed with the computer, he or she is
probably just going through a phase, and in a few months parents will have something else to worry about!
1. A. always B. ever C. never D.rare
2. A. peeping at B. glancing at C. staring at D. seeing
3. A. involved B. occupied C. taken D. absorbed
4. A. done B. had C. made D. taken
5. A. promise B. assure C.secure D. claim
6. A. commit B. stick C. follow D. hold
7. A. rules B. procedures C. steps D. regulations
8. A. actually B. heavily C. seriously D. urgently
9. A.speak B. discuss C. talk D. debate
10. A. possibly B. consequently C. probably D. necessarily
B. Read the following passage and then decide which answer (A,B,C, or D) best fits each space.
The ability to weep is a uniquely human form of emotional response. Some scientists have suggested
that human tears are (1)………….of an aquatic past-but this does not seem very likely. We cry from the
moment we enter this world, for a number of reasons. Helpless babies cry to persuade their parents that they
are ill, hungry or comfortable. As they (2)…………., they will also cry just to attract parental attention and
will often stop when they get it.
The idea that having a good cry do yoụ (3)………….is a very old one and now it has scientific validity
since recent research into tears has shown that they (4)………….a natural painkiller called enkaphalin. By
fighting sorrow and pain this chemical helps you feel better. Weeping can increase the quantities of enkaphalin
you (5)………….
Unfortunately, in our society we impose restrictions upon this naturally (6)………….activity. Because
some people still regard it as a (7)………….of weakness in men, boys in particular are admonished when they
cry. This kind of repression can only increase stress, both emotionally and physically.
Tears of emotion also help the body (8)………….itself of toxic chemical waste, for there is more
protein in them than tears resulting from cold winds or other irritants. Crying comforts, calms and can be very
enjoyable- (9)………….the popularity of highly emotional films which are commonly (10)………….
"weepies". It seems that people enjoy crying together almost as much as laughing together.
1. A. witness B. evidence C. result D. display
2. A. evolve B. change C. develop D. alter
3. A. better B. fine C. good D. well
4. A. contain B. retain C. hold D. keep
5. A. construct B. achieve C. provide D. produce
6. A. curing B. treating C. healing D. improving
7. A. hint B. symbol C. feature D. sign
8. A. release B. rid C. loosen D. expel
9. A. consider B. remark C. distinguish D. regard
10. A. named B. entitled C. subtitled D. called
VII. OPEN CLOZE TEST.
For questions 1-20, read the texts below and think of the word which best fits in each space. Use only
one word in each space.
Passage 1
GLOBAL ENGLISH
Global English exists (1) …………. a political and cultural reality. Many misguided theories attempt
to explain why the English language should have succeeded internationally, whilst (2)………….have not. Is it
because there is something inherently logical or beautiful about the structure of English? Does its simple
193

Downloaded by Nhung Nguy?n (nhungnth2004@gmail.com)


lOMoARcPSD|20277325

grammar make it easy to learn? Such ideas are misconceived. Latin was once a major international language,
(3)………….having a complicated grammatical structure, and English also presents learners with all manner
of real difficulties, (4)………….least its spelling system. Ease of learning, therefore, has little to do with
it. (5)………….all, children learn to speak their mother tongue in approximately the same period of time, (6)
………….of their language. English has spread not (7)………….much for linguistic reasons, but rather
because it has often found (8)………….in the right place, at the right time. Since the 1960s, two major
developments have contributed to strengthening this global status. Firstly, in a number of countries, English is
now (9)………….in addition to national or regional languages. As well as this, an electronic revolution has
taken place. It is estimated that (10)………….the region of 80% of worldwide electronic communication is
now in English
Passage 2
The cougar is an (11)………….animal. Its back legs are longer than its front legs, so it can jump
incredibly high; some cougars can jump up to six meters. It is extremely (12)…………., too, and can drag its
prey (which may be as (13)………….as the cougar itself) for long distances, until it finds a place where it can
eat its food (14)………….Its claws are very (15)…………., but they can be pulled back into its toes, so the
cougar can run (16)………….. Cougars live (17)………….they only meet with other cougars in order to
breed. Cougars can swim (18)…………., and climb trees. They used to live (19)………….in north and south
America, from Chile to the Yukon, but now they are quite (20)………….
VIII. WORD FORM.
A. Complete the sentences below with the correct form of the words at the end of the lines.
1. Despite the striking similarities between the two paintings, the court decided the artist was not guilty of
making a………….copy. fraud
2. The stray dog spent his days following tourists hoping to ………….one of them. friend
3. The best way to solve this dispute is to find a neutral,………….third party and follow his or her
suggestions. interest
4. His performance in the match today………….his reputation as a great player. lie
5. The rocks appear to be stationary but in the high winds that whip across this desert landscape, they are in
reality moving…………. perceive
6. The………….of any manned mission to the planets will be increased if a secure fuel supply can be found
beforehand. survival
7. So many vasteroids finish their days by crashing into Jupiter due to its huge………….pull. gravity
8. Are you sure the………….is working properly on this truck? It must have done more than twenty thousand.
mile
9. Far from being a drunkard, my father is actually a perfect example of………….. sober
10. After the cup final, the …………..team bussed back to the hotel parading the cup as they went. triumph
B. Use the correct form of the words given in the box tò complete the passage.
populate fall coast delete interfere
pollute inhere use limit reserve

The seas are considered man’s best hope for the future, the last frontier, and breath-space for the land
which is being 11…………. The sea is polluted by waste from ships, by rivers and even by the 12.………….
air. In 13.…………. areas, population pressures are increasing; populations move to coasts and they are
followed by industries. It is necessary to fight any threat to the sea.
Man causes marine pollution by introducing substances, which have 14………….. effects, into the
marine environment. This 15.…………. is a hazard to health and it hinders marine activities and impairs the
quality of sea water. Major 16.………….processes of the marine environment are seen as domestic sewage
and industrial waste. The ocean is considered by many as a perfect dump because of the great economy 17.
………….in the discharge of urban sewage and industrial waste. Its vast area and volume and its lack of
portability or 18.………….for domestic and most industrial purposes make it an 19.………….and most
194

Downloaded by Nhung Nguy?n (nhungnth2004@gmail.com)


lOMoARcPSD|20277325

attractive 20………….of waste assimilation. Because of this attitude, certain areas of the ocean are showing
signs of severe damage.
IX. ERROR RECOGNITION.
The passage below contains 10 mistakes. Identify and correct them
RAINFOREST CONCERN
The world’s rainforests represent a vast reservoir of knowledgement 1……………
and hold potentially for the discovery of new medicines and foods. 2……………
There is no doubt that large-scale deforestion alters the climate-intensifying 3……………
droughts in the dry season and floods in the rainy
season. The result is fewer animal and plant species, soil erosive, a 4……………
water supply which is inreliable and poorer health for the local 5……………
people. By joining Rainforest Concern and sponsoring acres of
threatening rainforest for the Choco-Andea Rainforest Corridor in 6……………
Ecuador, you’ll be protecting one of the world’s most important
ecological areas.
Within these forests lives an amazingly high number of 7……………
serious endangered species of animals, birds and plants. You will 8……………
also be helping to secure the survivors and the culture of the Awa 1……………
and Cayapas ingenious people, who still live in harmony with their 10……………
nature environment

X. SENTENCE TRANSFORMATION.
A. Rewrite the following sentences with the given words in such a way that the second sentence has the
same meaning as the first one. Do not change the form of the words in brackets.
1. Winning that prize has made him very conceited. HEAD
……………………………………………………………………………………………………..
2. The prisoner was recaptured as he rushed towards the gate. DASH
……………………………………………………………………………………………………..
3. Before you leave the office today, please look at the new contract. EYE
……………………………………………………………………………………………………..
4. The conflict between the two political parties will carry on for a long time. MEANS
……………………………………………………………………………………………………..
5. It is too complex to understand why you quit your job when you haven’t got another one. BEYOND
……………………………………………………………………………………………………..
B. Rewrite the following sentences in such a way that the second sentence has the same meaning as the
first one.
1. Since the company's methods were exposed in a newspaper, people have lost their good opinion of it.
Since the company's methods were exposed in a newspaper,…………………………………………….
2. If you need any help please don't hesitate to ask.
If I can……………………………………………………………………..please don't hesitate to ask.
3. He wouldn't have been interviewed by the police if he hadn’t seen the robbery.
The police wouldn't have interviewed him unless he…………………………………the robbery.
4. The audience suddenly started to applaud from the audience.
All………………………………………………………………………………from the audience.
5. Attendances at the exhibition have been down this year.
The exhibition………………………………………………………………………this year.

TRƯỜNG THPT CHUYÊN NGUYỄN BỈNH KHIÊM VĨNH LONG


195

Downloaded by Nhung Nguy?n (nhungnth2004@gmail.com)


lOMoARcPSD|20277325

A. MULTIPLE CHOICE QUESTIONS:


I. PHONOLOGY:
Choose the word whose underlined part is pronounced differently from that of the others.
1. A. Dutch B. butcher C. clutch D. crutch
2. A. post B. potent C. Poland D. polite
3. A. coverage B. massage C. heritage D. message
4. A. stone B. alone C. shone D. cyclone
5. A. champion B. change C. flame D. chamber
Choose the word that is stressed differently from the others in the list.
6. A. devastate B. departure C. desolate D. desperate
7. A. October B.radius C. octopus D. Occident
8. A. metallic B. momentum C. medieval D. monastery
9. A. supposedly B. curriculum C. surprisingly D. supernatural
10. A. invaluable B. investigate C. intimacy D. intensity
II. WORD CHOICE
Choose the option that best completes the blank.
1.1 cant eat this meat. It’s too………………
A. tough B. soggy C. strong D. tender
2. Many books were not available to the public because of government………………
A. omission B. inhibition C. compensation D. censorship
3. My car was so old that I could only sell it for………………
A. rubbish B. scrap C. debris D. waste
4. This isn't a ………………, is it? If.I lend you my car, you will bring it back, won’t you?
A. trick B. cheat C. rip off D. deceit
5. The woman accused of shoplifting was found not guilty and was ………………
A. excused B. liberated C. acquitted D. interned
6. I think we’ve kept everybody in the………………about the arrangements for far too long.
A. gloom B. dusk C. dark D. night
7. He was wounded in the………………stages of the battle
A. end B. middle C. intermediate D. closing
8. He is a (an)………………authority on the subject.
A. prominent B. expert C. eminent D. quality
9. You need to support your ideas with facts and....................
A. numbers B. informations C. figures D. material
10. Veronica broke the school rules so many times that the headmistress finally had no alternative but
to................ her
A. export B. deport C. expel D. eject.
III. STRUCTURES & GRAMMAR
Choose the best answer.
1. Although we have a large number of students, each one receives………………attention
A. alone B. only C. separate D. individual
2. No examples ...............relevant to this case.
A. are B. is C. has been D. was
3………………, he felt so unhappy and lonely.
A. Despite his wealthy B. Rich as was he
C. Rich as he was D. In spite of his being wealth
4………………have made communication faster and easier through the use of email and Internet is widely
recognized.
A. It is that computers B. That computers
196

Downloaded by Nhung Nguy?n (nhungnth2004@gmail.com)


lOMoARcPSD|20277325

C. Computers that D. That it is computers


5...................... gas tanks connected to welding equipment, one full of oxygen and the other full of acetylene.
A. It is two B. Of the two C. There are two D. Two
6. Although not as important as they once were, ................ a major form of transportation in North America.
A. there are still railroads B. railroads, which are still
C. railroads are still D. railroads still being
7………………team sports require cooperation.
A. Of all B. They are all C. All D. Why are all
8. Certain fish eggs contain droplets of oil, …………… to float on the surface of the water.
A. allowing them B. allows them C. they are allowed D. this allows them
9. Centuries of erosion have exposed……………rock surfaces in the Painted Desert of northern Arizona.
A. in colors of the rainbow B. colored like a rainbow
C. rainbow-colored D. a rainbow’s coloring
10………………make a good impression on her.
A. Only by doing so can I B. Only so doing can I
C. Only by so doing I can D. Only by doing so I can
IV. PREPOSITIONS AND PHRASAL VERBS
Choose the best answer.
1. Italy was knocked………………the World Cup
A. out of B. away from C. into D. forward to
2. The argument is centered………………whether or not to lower the age limit.
A. of B. towards C. on D. about
3. He wasn’t short, he wasn’t tall. He was………………average height.
A. in B. of C. at D. with
4. The teacher asked a difficult question, but finally Ted………………a good answer.
A. came up to B. came out at C. came up with D. came out of
5. The forecast has revealed that the world’s reserves of fossil fuel will have………………by 2015.
A. run out B. taken over C. caught up D. used off
6. My mother told me to ................ for electrician when her fan was out of order.
A. turn B. rent C. send D. write
7. My attention was drawn………………the picture on the far wall
A. with B. for C. to D. on
8. If I’m late for work again, I’ll be………………a severe warning from my boss.
A. up to B. in for C. onto D. alter
9. I'm going to stay at university and try to………………off getting a job for a few years!
A. stay B. put C. move D. set
10. He came………………a lot of criticism for the remarks he made in a television interview.
A. in for B. over C. out of D. off
V. READING COMPREHENSION
Read the passage and choose the best answer for each of the questions below.
Passage 1:
LETTER OF COMPLAINT
Dear Sirs,
Your shipment of twelve thousand ‘Smart’ watches was received by our company this morning.
However, we wish to make a number of complaints concerning the serious delay, in delivery and your failure
to carry out our explicit instructions with regard to this order.
It was stressed from the outset that the delivery date had to be less than six weeks from the initial
order, in order to comply with our own customer’s requirements. While we appreciate that delays in
production are occasionally inevitable, we must point out that the major reason why the order was placed with
197

Downloaded by Nhung Nguy?n (nhungnth2004@gmail.com)


lOMoARcPSD|20277325

your company was because we were assured by you of its straightforwardness, and that your existing stocks
were sufficiently high to ensure immediate shipment. Late delivery of the goods has caused us to have an
adverse effect on potential future orders.
The second complaint concerns the discrepancy in colour between the watches we ordered and those
delivered. It was stated clearly in the original order that watches in combinations of green/purple and
orange/purple only were required. However, only half the watches in the delivery received are of the colour
specified. Our Hong Kong agent assures us that she stressed to you the importance of following our
instructions precisely, since we consider there to be only a limited market in this country for watches of other
colour at present time. Any watches that are not of the specified colours will, of course, be returned to you.
We are also somewhat concerned about the rather poor quality of the goods received, since it is
apparent that the watches that finally arrived have been produced from inferior materials and have been
manufactured to a lower standard than those in the sample.
We have also found that a number of the watches do not appear to be functioning. Whether the latter
problem is due to poor manufacture, damage in transit or defective batteries is not yet clear, but we should like
to point out that we feel this matter to be entirely your responsibility.
As a result of the above problems, therefore, we feel that the most suitable course of action is to return
to you unpaid any of the goods considered unsatisfactory, and to deduct any costs incurred from our final
settlement. We shall also, of course, be forced to reconsider whether any further orders should be placed with
your company.
We look forward to your prompt reply.
1. How many of the watches will returned to the manufacturers?
A. 6,000. B. less than 6,000 C. more than 6,000
D. the passage doesn't give enough information to answer
2. The manufacturers of'Smart' watches were given the order because:
A. they were assured and there was sufficient space for immediate shipment.
B. the watches would be easy to make and the designs were already prepared.
C. they promised they could produce enough stocks quite quickly.
D. they claimed to order would be easy since the watches were already in stock.
3. Receiving watches in the wrong colors is a problem because:
A. the Hong Kong agent stressed the need to fulfil the order exactly.
B. these watches will be difficult to sell.
C. they will be not able to get into the limited market.
D. people will not buy the watches as presents.
4. Which of the following could best replace 'straightfowardness' in paragraph 2?
A. simplicity B. speed of delivery C. efficiency D. directness
5. ‘Prompt' in paragraph 6 is closest in meaning to……………
A. apologetic. B. explanatory C. expected. D. early
6. Which of the following does 'the letter problem' in paragraph 4 refer to……………
A. the poor quality of the goods
B. the use of inferior materials
C. the low standard of manufacture
D. some of the watches not working
7. The last sentence of paragraph 5 has been included in order to suggest that:
A. the receiving company may cease trading with the watch manufacturers
B. the company will probably reduce the number of watches it orders in future
C. the writers are afraid there company might go out of business soon
D. the company are probably willing to give the manufacturers anothẹr chance
8. Late delivery will have an 'adverse effect on future orders' in paragraph 2 because:
A. the company will no longer place orders with the manufacturers.
198

Downloaded by Nhung Nguy?n (nhungnth2004@gmail.com)


lOMoARcPSD|20277325

B. future orders will have to be delivered sooner.


C. the company is sure to lose some of its business with its customers.
D. the company will certainly have to advertise itself more effectively in future.
9. ‘Limited market in this country for watches of other colors' in paragraph 3 means:
A. not many shops are able to sell such watches.
B. other watch companies make it difficult for newcomers to enter the market.
C. market stalls don't seem to sell such watches here.
D. not many people are interested in buying such watches.
10. 'Discrepancy' in paragraph 3 is closest in meaning to……………
A. problem B. damge C. difference D. variety
Passage 2:
NUCLEAR WEAPONS
It is generally well known that in a number of particularly dangerous parts of the world, for example
the Middle East and the India/Pakistan border region, there are countries which either possess, or have the
technology to produce nuclear weapons. It is also worth remembering, however, that the country which
possesses more nuclear weapons than any other, the United States, is the only power ever to have used nuclear
weapons against people.
Nuclear weapons were first developed in the United States during the Second World War, to be used
against Germany. However, by the time the first bombs were ready for use, the war with Germany had ended
and, as a result, the decision was made to use the weapons against Japan instead. Hirosima and Nagasaki have
suffered the consequences of this decision to the present day.
The real reasons why bombs were dropped on two heavily populated cities are not altogether clear. A
number of people in 1944 and early 1945 argued that the use of nuclear weapons would be unnecessary, since
American Intelligence was aware that some of the most powerful and influential people in Japan had already
realized that the war was lost, and wanted to negotiate a Japanese surrender. It was also argued that, since
Japan has few natural resources, a blockade by the American navy would force it to surrender within a few
weeks, and the use of nuclear weapons would thus prove unnecessary. If a demonstration of force was
required to end the war, a bomb could be dropped over an unpopulated area of low population inside Japan,
such as a forest. Opting for this course of action might minimize the loss of further lives on all sides, while the
power of nuclear weapons would still be adequately demonstrated.
All of these arguments were rejected, however, and the general consensus was that the quickest way to
end the fighting would be to use nuclear weapons against centres of population inside Japan. In fact, two of
the more likely reasons why this decision was reached seem quite shocking to us now.
Since the beginning of the Second World War both Germany and Japan had adopted a policy of
genocide (i.e. killing as many people as possible, including civilians). Later on, even the US and Britain had
used the strategy of fire bombing cities (Dresden and Tokyo, for example) in order to kill, injure and
intimidate as many civilians as possible. Certainly, the general public in the West had become used to hearing
about the deaths of large numbers of people, so the deaths of another few thousand Japanese, who were the
enemy in any case, would not seem particularly unacceptable- a bit of ‘justifiable’ revenge for the Allies’ own
losses, perhaps.
The seconds reason is not much easier to comprehend. Some of the leading scientists in the world had
collaborated to develop nuclear weapons, and this development had resulted in a number of major advances in
technology and scientific knowledge. As a result, a lot of normal, intelligent people wanted to see nuclear
weapons used; they wanted to see just how destructive this new invention could be. It no doubt turned out to
be even more ‘effective’ than they had imagined.
1. In paragraph 1, the writer is suggesting that…………….
A. the United States should, if necessary, use nuclear weapons again
B. the United States is more likely them other countries to use nuclear weapons
C. the United States is one of several countries to have used nuclear weapons
199

Downloaded by Nhung Nguy?n (nhungnth2004@gmail.com)


lOMoARcPSD|20277325

D. the United States could potentially use nuclear weapons again


2. The writer refers to 'normal, intelligent'people in paragraph 6 because…………….
A. he wants to emphasize the fact that they were among the cleverest scientists
B. he feels the decision to use nuclear weapons against Japan was correct
C. he believes this make the decision to use nuclear weapons seem more shocking
D. he wants to show how many people wanted to observe the bombs being dropped
3. According to paragraph 3, a blockade would have been successful because…………….
A. Japan has not import most of its natural resources like coal and steel
B. Japan would not be resources enough to beat a blockade
C. an attack would probably destroy Japanese resources within a few weeks
D. the Americans could defeat Japan's navy since it was short of resources
4. It the last sentence of paragraph 6, the writer implies that …………….
A. he agrees with the decision to use nuclear weapons against Japan
B. he think decision to drop nuclear bombs on Japanese cities was wrong
C. nuclear weapons worked much better than the scientists probably expected
D. the weapons proved to be effective because Japan surrendered soon after
5. That first sentence ofparagraph 6 suggest,the writer believes that......
A. the decision were made by intelligent people and were difficult to follow
B. his presentation of the argument in paragraph 5 is difficult to understand
C. the reasons given for decision are hard for US to accept nowadays
D. the decisions were complex and made by highly intelligent people
6. According to paragraph 5, which of the following is true?
A. people in the West had got used to hearing the sounds of people dying.
B. it would probably not be wise to inform people in the West of the deaths.
C. scientists thought only a few thousand people would die if the bombs were used.
D. people in the West would accept that some people had to die to end the war.
7. How many reasons against using the weapons are given in paragraph 3?
A. two B. three C. foúr D. five
8. From the last sentence of paragraph 4, we can infer that…………….
A. the real reasons for the decision may never have been made clear
B. the writer probably expects US not to agree with his opinion
C. the writer has not done much research on this subject to establish the facts
D. the writer has attempted to present the facts as objectively as possible
9. Which of the following could be used instead if 'in any case' in paragraph 5?
A. all the time B. anyway C. anytime D. in this respect
10. Which of the following is closest in meaning to, 'blockade’ in paragraph3?
A. siege B. bombing C. attack D. defence
VI. CLOZE TEST
Read the text below and decide which answer (A, B,CorD) best fits each space. Cloze Test 1
In the match between Parkfield School and Greenport Football Club which took place at Greenport (1)
……………Saturday, the Parkfield team beat Greenport by three goals to two.
The last time the two (2) ……………met, Greenport won by three goals to nil, so Parkfield were
making a great (3) ……………to win this game.
In the first half, of the game, Greenport (4) …………… leading by two goals, and Parkfield’s position
looked hopeless. However, Greenport made a number of (5) ……………mistakes during the second half. This
was after their star player, Brown, missed the ball, slipped and (6) …………… heavily, injuring his left ankle.
Parkfield then scored two lucky (7) ……………, and in the few minutes before the finish, Prescott
shot the ball (8) ……………the net for them, bringing the score to three goals to two. Parkfield have (9) ……
well this season. And their fans have good reason to be proud. Greenport too, have had good results up to now.
200

Downloaded by Nhung Nguy?n (nhungnth2004@gmail.com)


lOMoARcPSD|20277325

Out of the last five games played, this is the first game they have (10)………….
1. A. next B. every C. once D. last
2. A. dates B. teams C. fights D. events
3. A. effort B. chance C. journey D. competition
4. A. had B. were C. seemed D. made
5. A. careless B. many C. generous D. deep
6. A. kicked B. bent C. fell D. ran
7. A. goals B. players C. matches D. teams
8. A. up B. through C. into D. beneath
9. A. been B. lost C. made D. played
10. A. won B. beaten C. missed D. lost
Cloze Test 2
MUSIC - A UNIVERSAL LANGUAGE
Music is universal - it is produced by all cultures. Some scientists believe that music came before
speech and (1) …………… as a development of mating calls. . In fact, there is (2) …………… theory that the
earliest languages were chanted or sung rather than spoken. Indeed, in some cultures, music is a form of (3)
…………… history. The Aboriginal Australians, for example, use music as a means to pass on stories of the
land and spirits to the next (4) ……………
New evidence suggests that music does not just (5) ……………thefeel-good factor but it is also good
for the brain. A study of intellectually (6) ……………children showed that they could recall more facts after it
was given to them in a song than it was read to them as a story.
Researchers also report that people (7) …………… better on a standard intelligent test after listening
to Mozart. The so-called “Mozart effect”, has also been (8) …………… by findings that rats (9) ……………
up on Mozart run faster through a complex network of paths or passages, known as a maze. Overall, it seems
that in most instances people who suffer from any form of mental (10) ……………benefit from listeningto
music.
1. A was B. swelled C. reacted D. arose
2. A. one B. every C. such D. that
3. A. enjoying B. making C. recording D. stating
4. A. children B. people C. tribe D.
generation
5. A. convince B. satisfy C. please D. prefer
6. A. disabled B. inactive C. incapable D. disordered
7. A. examine B. prepare C. score D. achieve
8. A. supported B. gi ven C. marked D. remembered
9. A. held B. brought C. stood D. set
10. A. badness B. hurt C. illness D. pain
B. WRITTEN TEST
I. OPEN CLOZE TEST
Fill each blank with ONE suitable word.
Cloze test 1
NOTHING’S NEW IN MEDICINE
Throughout the ages, disease has stalked our species. Prehistoric humans must quickly have learnt (1)
…………….could be eaten without danger, and how to avoid plants that-could (2) ......... about illness. They
found leaves, berries and the bark of different trees that could actually heal wounds and cure the sick, and (3)
…………….soon became a special skill to understand natural medicine.
Ever (4)…………….the dawn of history, medicine men and wise women have always been expert in
treating diseases and have dispensed medicine with ritual and magic. (5)…………….trial and error they
discovered treatments for almost any affliction prevalent at the time. The precious recipes for preparations
201

Downloaded by Nhung Nguy?n (nhungnth2004@gmail.com)


lOMoARcPSD|20277325

which could relieve pain, stop fits, sedate or stimulate were (6) …………….down from generation to
generation, although there was (7)…………….exact understanding of the way in which the medicines
worked. Nevertheless, (8)…………….the power of these primitive medicines, generations were still ravaged
by disease.
During the last 150 years, scientists and doctors, (9) ……………. work has focused on these early
medicines, have learnt that their power derived from certain chemicals which were found in herbal remedies
or could be synthesised in the laboratory. In just (10)……………. a way, advances in modem medicine
continue, aided by the discoveries made centuries ago by our ancestors.
Cloze test 2
FOOD FOR A FUTURE
Jon Wynne - Tyson was an original thinker whose best- known book “Food for a Future” was
published in 1975. In this classic work, a case was (1)…………….forward for what can only be described as a
more responsible and humane attitude towards the world’s food resources. It had gradually (2)…………….
clear to Wynne- Tyson that the economics and ecology of meat production did not (3) …………….sense.
What justification was (4)…………….he argued, for using seven tonnes of cereal to produce one tonne of
meat?
Even today, the book’s succinct style makes it compulsively readable. (5)…………….his approach is
basically an emotional one, Wynne- Tyson goes to great lengths to back up every statement with considerable
supporting evidence and statistical data. Thus, even those of us who are widely read (6)…………….the
subject of vegetarianism will gain fresh insights from this book. It is generally agreed that his most skilful
achievement is the slow revelation of his main thesis as the arguments unfold. The book concludes that a
move away from an animal- based diet to (7)…………….which is based on plant sources is inevitable in the
long term, in (8)…………….of the fact that there is no sound nutritional, medical or social justification for
meat- eating. Regardless of whether you agree with (9)…………….a conclusion or not, the book certainly
makes (10)…………….fascinating read.
II. WORD FORMS
1. Complete the following sentences with the correct forms of the words given
1. She is very .... She always manages to look good in photographs. (PHOTO)
2. There has been no …………….in this class- they have had a succession of different teacher.(CONTINUE)
3. You shouldn’t interrupt someone in……………. (SENTENCE)
4. A…………….friend is one who likes power. (DOMINATE)
5. Street- fights are an everyday…………….in this area of the city. (OCCUR)
6. Her leg has been ……………. for quite a while, and she’ll probably need surgery on it. (TROUBLE)
7. You are entitled to receive a government allowance for each ……………. who is living with you.
(DEPEND)
8. Mrs. Lan's shop is …………….from the others in the street. It’s hard to tell the difference.(DISTINGUISH)
9. This brochure will be ready for…………….in September. (PUBLISH)
10. Originally the builders gave me a price of $2000, but now they say they…………….and it’s going to be at
least $3000. (ESTIMATE)
2. Complete the following passage with the correct forms of the words given
EMERGE KNOW EXTEND CONSEQUENCE RIPE THREAT
SUPPLY MATURE COMPETE RECOGNISE

VANILLA
Thanks to the ubiquitous use of vanilla as a flavouring in ice creams and cakes the world over, its taste
is more (1)……………to the majority of people than the appearance of the plant.
The plant itself is actually a native of the tropical forests of Central America and is the only variety of
orchid to be grown on a commercial scale. Its delicate white flowers open in the early morning and, after
pollination by insects or humming birds, a narrow bean- like pod forms and (2) ……………,taking a period
202

Downloaded by Nhung Nguy?n (nhungnth2004@gmail.com)


lOMoARcPSD|20277325

of five to seven months to reach (3)……………It is this pod which is harvested to provide the food crop we
know as vanilla.
Despite its American origins, for decades it was only cultivated (4) ……………on the Indian Ocean
island of Madagascar, where it was introduced at the end of the nineteenth century. It soon became clear that
the vanilla grown there was of a quality (5)……………in other areas, and the island quickly became one of
the world’s major (6)……………
In recent year, however, new (7)……………have entered the vanilla market and, (8)……………,
Madagascar’s importance has started to slip. Of course, the (9) ……………of new producers means a smaller
market share, whilst the development of artificial substitutes is (10)……………to undermine demand for
the real thing.
III. ERROR RECOGNITION AND CORRECTION:
The passage below contains 10 errors. IDENTIFY and CORRECT them. (0) has been done as an
example. Write your answers in the numbered blanks below
After an absence in thirty years, I decided visiting my old school again.
I had expected to find changes, but no a completely different building.
As I walked up the school drive, I wondered for a moment if I had come to right address.
The grimy, red brick fortress with their tall windows that had looked up grimly on the playground and
playing fields had swept away. In its place stood a bright, modern block risen from the ground on great
concrete stilts.
A huge expanse of glass extending across the face of the building, and in front, there was a well-kept
lawn where previous there had been untidy gravel yard.
Answers: 0. in → of
IV. SENTENCE TRANSFORMATION
Rewrite the sentences with given words or beginning in such a way that their meanings remain
unchanged.
1. Be sure to say goodbye to your grandmother before you leave. (without)
→………………………………………………………………………………………………………..
2. They’ll have to take the dog on holiday with them, (behind)
→………………………………………………………………………………………………………..
3. In order to stay open, the charity shop needs at least four helpers, (enable)
→ No fewer ………………………………………………………………..the charity shop to stay.
4. The motorcyclist rounded the comer at a quite unbelievable speed. (which)
→………………………………………………………………………………………………………..
5. I’ll be thinking of you and hoping that you have good luck on the dạy of your interview. (fingers)
→………………………………………………………………………………………………………..
6. Don’t panic about something so trivial, (mountain)
→……………………………………………………………………………………………………….
7. The mistake in the accounts was not noticed until the figures were re-checked. (light)
→The mistake in the accounts only……………………………………the figures were re-checked.
8. The best solution was thought of by Sally. (came)
→………………………………………………………………………………………………………..
9. That jumper you knitted for my daughter no longer fits her. (grown)
→………………………………………………………………………………………………………..
10 They accused me of causing the accident. (blame)
→………………………………………………………………………………………………………..
TRƯỜNG THPT CHUYÊN VỊ THANH - HẬU GIANG
A. MULTIPLE CHOICE
I. PHONOLOGY
Choose the word whose underlined part is pronounced differently from the Other three.
203

Downloaded by Nhung Nguy?n (nhungnth2004@gmail.com)


lOMoARcPSD|20277325

1. A. rhinoceros B. vehicle C. whale D. uninhabitable


2. A. unconcernedly B. ragged C. sacred D. hiccupped
3. A. archaic B. archive C. choir D. archery
4. A. suggestion . B. congestion C. digestion D. devotion
5. A. dairy B. lair C. fair D. gait
Choose the word whose stress pattern is different from that of the other three.
6. A. apparently B. insurance C. photographer D. politician
7. A. vanity B. mechanize C. nonprofit D. micro wave
8. A. literature B. approximately C. museum D. maintain
9. A. fortunately B. entertain C. recommend D. disappear
10. A. survive B. fashionable C. palace D. memory
II. WORD CHOICE
Choose the best option to complete the following sentences.
1. In the modern area, the increased international movement of people has greatly…………..the destruction of
languages.
A. speeded B. urged C. accelerated D. hurried
2. They were…………..from their apartment because they hadn’t paid the rent.
A. evicted B. expelled C. deposed D. discarded
3. I’m afraid that this incident could put your career here…………..
A. beyond a joke B. in jeopardy C. in earnest D. at fault
4. Thomas…………..all the way to class because his teacher was absent.
A. mustn’t run B. didn’t need to run C. needn’t have run D. can’t have run
5. Mr. Henson's bitter comments on the management's mistakes gave…………..to the conflict which has
already lasted for four months.
A. cause B. ground C. goal D. rise
6…………..hedelivers the report, it will be sent to the headquarters
A. On the point B. At once C. Immediately D. Soon enough
7. Judy didn't…………..for a second to agree to Mike's proposal as she had been in love with the boy for a
long time.
A. decide B. linger C. hesitate D. await
8. Don't get so nervous about his coming late. When you get to know him better, you'll learn to take
it…………..
A. easy B. loose C. nice D. fine
9. The first thing for all of you to remember is that…………..your duties may result in an instant dismissal.
A. escaping B. neglecting C. resisting D. missing
10. It was the commission's job to decide whether the pilot was…………..for the crash that occurred right
after the take-off.
A. responsible B. prone C. guilty D. comprehensive
III. GRAMMAR AND STRUCTURES
Choose the best options to complete the following sentences.
1. My students, …………..are from the provinces, are having trouble finding accommodation.
A. most of whom B. many of them C. some of who D. all who
2. Mohandas K. Gandhi, …………..Mahatma, lived a noble life of poverty to work for independence.
A. that was called B. was called C. called D. calling
3.I tried to be diplomatic, but…………..I couldn’t help losing my temper.
A. in the end B. at the end C. by accident D. on purpose
4. Not until 1865…………..the first antiseptic treatment on a compound fracture.
A. when Joseph Lister tried B. when did Joseph Lister try
C. did Joseph Lister try D. that Joseph Lister tried
204

Downloaded by Nhung Nguy?n (nhungnth2004@gmail.com)


lOMoARcPSD|20277325

5. According to a recent report, the disease is rare, affecting …………..ten thousand adults.
A. out of every one B. the only one of
C. every one from D. only one out of every
6. Harriet was offered the job…………..her linguistic abilities.
A. as regard B. on account of C.owing to D. according to
7. Since they aren't answering the phone, they …………..
A. can’t have left B. should have left C. must have left D. need have left
8. Architects invóìved in big projects must study …………..about the outside orld before conceiving of an
idea.
A. clients think B. whether clients thoughts
C. how their clients think D. thoughts that clients
9. Having been served dinner,…………..
A. the problem was discussed by the members of the committee.
B. the committee members discussed the problem.
C. it was discussed by the committee members the problem.
9. a discussion of the problem was made by the members of the committee.
10. An artist………….. will do his best to express the innocence and inexperience in the child’s face.
A. portraying a child B. who portray a child
C. he portrays a child D. portrayed a child
IV. PHRASAL VERBS AND PREPOSITIONS
1. It is understood that his closest adviser will…………..as president.
A. take over B. get by C. take up D. come about
2. New peace proposals were…………..at the recent Middle East conference.
A. shown off B. spoken out C. put forward D. made up
3. I would be grateful if you kept the news ………….. yourself. Do not tell anyone about it.
A from B. to C. for D. at
4. He said he would contribute money, but later he backed…………..of it.
A. down B. away C. off D. out
5. I’m not surprised Margaret's ill. With all the voluntary work she's …………..she’s really been doing too
much.
A. taken off B. taken on C. taken in D. taken to
6. "You are ahead of ………….., Jim. The meeting doesn't start for another 30 minutes," Jack said.
A. pack B. class C. time D. game
7. Danny wishes his father could stay…………..good and all, but the man still had a few months of the
military service to do.
A. in B. for C. at D. with
8. The thieves decided to…………..and steal endangered animals as well as cars.
A. branch out B. go away C. hang out D. take over
9. David’s new album is expected to…………..at the end of the year.
A. come over B. come about C. come up D. come out
10. This is a sensitive matter, and we have to…………..dealing with it very carefully.
A. decide on B. catch on to C. set about D. run through
V. GUIDED CLOZE 1
Read the text below and decide which answer best fits each space.
In the early morning of 23 January, 2009, the most powerful storm for decads hit western France. With
wind speeds in (1)…………… of 120 miles per hour, it flattened forests, (2) ……………down power lines
and caused massive destruction to buildings and roads. But it also left behind an extraordinary creation. Seven
miles out to sea at the (3) ……………where the Atlantic Ocean meets the estuary of the River Gironde, a
small island had (4) ……………out of water. Locals soon gave it the name The Mysterious Isle. What was so
205

Downloaded by Nhung Nguy?n (nhungnth2004@gmail.com)


lOMoARcPSD|20277325

remarkable, (5) …………… its sudden apparition, was the fact that the island (6) ……………intact in what is
often quite a hostile sea environment. It could well become a permanent feature.
Scientists (7). …………… realised that the island’s appearance (8) …………… a unique opportunity
to study the creation and development of a new ecosystem. Within months, it had been colonised by seabirds,
insects and vegetation. Unfortunately, however, they were not alone in (9). …………… the island attractive. It
became increasingly difficult to (10) ……………the site from human visitors. In its first year, day trippers
came in power dinghies, a parachute club used it as a landing strip, a rave party was even held there one night.
1. A.surplus B. advance C. excess D. put
2. A. fetched B. brought C. carried D. sent
3. A. scene B. mark C. stage D. point
4. A. risen B. growth C. lifted D. surfaced
5. A. in spite of B. instead of C. apart from D on account of
6. A. prolonged B. remained C. resided D. preserved
7. A. quickly B. briskly C. hastily D. speedily
8. A. delivered B. awarded C. proposed D. offered
9. A. regarding B. finding C. seeking D. deciding
10. A. prevent B. preserve C. protect D. prohibit
VI. GUIDED CLOZE 2
Read the text below and decide which answer best fits each space.
Polar bears are in danger of dying out. (1) ……………some other endangered animals, it's not hunters
that are the problem, it's climate change. Since 1979, the ice cap at thẻ Arctic Circle where the polar bears live
has reduced in size. (2) ……………about 30 per cent. The temperature in the Arctic has slowly been
(3)…………. and this is (4) ……………the sea ice to melt, endangering the polar bears' home. The polar
bears' main (5) ……………of food are the different types of seals found in the Arctic. They catch them by
waiting next to the air holes seals have (6) ……………in the ice. (7) ……………the bears are very strong
swimmers, they could never catch seals in water. This means that the bears really do rely on the ice to hunt.
Polar bears also need sea ice to travel. They can ,(8) ……………a huge territory and often swim from
one part of the ice to another. They have been (9) ……………to swim up to 100 km, but when there is less
ice, they may have to swim further and this can (10…………… fatal to the bears. A number of bears have
drown in the last few years and scientists believe that it is because they were not able to reach more ice before,
they became too tired and couldn't swim any further.
1 . A. Different B. Compared C. Opposite D. Unlike
2. A. for B. by C. in D. within
3. A. lifting B. gaining C. rising D. advancing
4. A. causing B. resulting C. making D. turning
5. A. origins B. materials C. means D. sources
6. A. set B. placed C. made D. brought
7. A. Although B. As C. Despite D. Even
8. A. spread B. cover C. pass D. extend
9. A. known B. experienced C. learnt D. noticed
10. A. prove B. end C. come D. happen
VII. READING PASSAGE 1
Read the text below and choose the best answer to each question.
For many people who live in cities, parks are an important part of the landscape. They provide a place
for people to relax and play sports, as well as a refuge from the often harsh environment of a city. What people
often overlook is that parks also provide considerable environmental benefits.
One benefit of parks is that plants absorb carbon dioxide - a key pollutant - and emit oxygen, which
humans need to breathe. According to one study, an acre of trees can absorb the same amount of carbon
dioxide that a typical car emits in 11,000 miles of driving. Parks also make cities cooler. Scientists have long
206

Downloaded by Nhung Nguy?n (nhungnth2004@gmail.com)


lOMoARcPSD|20277325

noted what is called the Urban Heat Island Effect: building materials such as metal, concrete, and asphalt
absorb much more of the sun’s heat and release it much more quickly than organic surfaces like trees and
grass. Because city landscapes contain so much of these building materials, cities are usually warmer than
surrounding rural areas. Parks and other green spaces help to mitigate the Urban Heat Island Effect.
Unfortunately, many cities cannot easily create more parks because most land is already being used for
buildings, roads, parking lots, and other essential parts of the urban environment. However, cities could
benefit from many of the positive effects of parks by encouraging citizens to create another type of green
space: rooftop gardens. While most people would not think of starting a garden on their roof, human beings
have been planting gardens on rooftops for thousands of years. Some rooftop gardens are very complex and
require complicated engineering, but others are simple container gardens that anyone can create with the
investment of a few hundred dollars and a few hours of work.
Rooftop gardens provide many of the same benefits as other urban park and garden spaces, but without
taking up the much-needed land. Like parks, rooftop gardens help to replace carbon dioxide in the air with
nourishing oxygen. They also help to lessen the Urban Heat Island Effect, which can save people money. In
the summer, rooftop gardens prevent buildings from absorbing heat from the sun, which can significantly
reduce cooling bills. In the winter, gardens help hold in the heat that materials like brick and concrete radiate
so quickly, leading to savings on heating bills. Rooftop vegetable and herb gardens can also provide fresh food
for city dwellers, saving them money and making their diets healthier. Rooftop gardens are not only
something everyone can enjoy, they are also a smart environmental investment.
1. Based on its use in paragraph 2, it can be inferred that mitigate belongs to which of the following word
groups?
A. exacerbate, aggravate, intensity B. obliterate, destroy, annihilate
C. allay, alleviate, reduce D. absorb, intake, consume
2. Using the information in paragraph 2 as a guide, it can be inferred that…………….
A. cities with rooftop gardens are cooler than those without rooftop gardens
B. some plants are not suitable for growth in rooftop gardens
C. most people prefer parks to rooftop gardens
D. most people prefer life in the country over life in the city
3. According to the passage, the Urban Heat Island Effect is caused by the fact(s) that……………
a. cities are warmer than nearby rural areas
b. building materials absorb more of the sun’s heat than organic surfaces
c. building materials release the sun’s heat more quickly than organic surfaces
A. a. only B. a. and b. only C. b. and c. only D. a., b., and c.
4. Based on( the information in paragraph 3, which of the following best describes the main difference
between parks and rooftop gardens?
A. Parks are expensive to create while rooftop gardens are not.
B. Parks are public while rooftop gardens are private.
C. Parks absorb heat while rooftop gardens do not.
D. Parks require much space while rooftop gardens do not.
5. The author claims all of the following to be the benefits of rooftop gardens except…………...
A. increased space, for private relaxation
B. savings on heating and cooling costs
C. better food for city dwellers
D. improved air quality
6. According to the author, one advantage that rooftop gardens have over parks is that they …………….
A. decrease the Urban Heat Island Effect
B. replenish the air with nourishing oxygen
C. do not require the use of valuable urban land
D. are less expensive than traditional park spaces
207

Downloaded by Nhung Nguy?n (nhungnth2004@gmail.com)


lOMoARcPSD|20277325

7. The author’s tone in the passage is best described as. ……………


A. descriptive. B. passionate C. informative D. argumentative
8. Which of the following pieces of information would, if true, most weaken the author's claim that rooftop
gardens are good for the environment?
A. Parks have many benefits that rooftop gardens do not share.
B. More pollution is produced during rooftop garden construction than rooftop plants can remove from the air.
C. Extremely high winds atop tall city buildings can severely damage some plants.
D. The overall environmental benefits that result from driving less exceed those of planting a rooftop garden.
9. Which of the following best describes the organization of the passage?
A. A hypothesis is stated and then analyzed.
B. A proposal is evaluated and alternatives are explored.
C. A viewpoint is established and then defended.
D. A thesis is presented and then supported.
10. It can be inferred from the passage that the author would most likely endorse a program that…………….
A. permitted the construction of buildings in city park land provided they have rooftop gardens
B. extended discounts on plants to customers who use them to create rooftop gardens
C. offered free admission to schools willing to take their students on field trips to the city park
D. promised vacation getaways to cooler destinations for those trapped in the city at the peak of summer
VIII. READING PASSAGE 2
Read the text below and choose the best answer to each question.
Traditional education creates, in time, an incomplete image on family’s gender roles. From generation
to generation children interiorize theirs parent’s model known as “separated world’s myth” - a wife and a
mother only cooking, doing laundry and taking care of the child “contrasting” with a husband and a father
working or fixing different things. According to this mentality each partner has established responsibilities in
private and public life that the other one couldn’t take over.
Changing traditional gender roles seems to be difficult not because woman or man don’t have the
necessary skills for other one’s tasks but because everyone carries on a symbol of his “world”. Society’s
evolution especially on woman valorization redefines gender roles promoting a woman more active, ambitious
and independent and a man more emotional and involved in household tasks.
Women of our days have more liberty to choose between the “family model” - these ones are being
more obedient and anchored in tradition (the feminine type) and the “feminist model” - who’s priority is the
career (the anti-feminine type) or the middle model adopted by the “double career” type of woman -
profession and family. On the base of any of these choices are variables like social-status, race, education
level, culture’s values etc.
We are assisting to a reprioritization of woman needs and so professional success became more
interesting and even attractive than family one probably because it brings something new. Women are
motivated to choose the career by elements as the wish for financial independency, the need to prove her
capacities, to open and better integrate in modem society and the possibility to control and not being only
controlled. All this reasons offer her a psychological equilibrium.
Most of the men consider women’s career as a conflict source having arguments like: women will
neglect their family attributions, the imminent change of man’s authority, his incertitude concerning the
professional competition and the risk to deteriorate the marital relation. Even at first sight man and children
are felling the stress generated by the new status of woman, she’s living an interior conflict as well. Woman
need to understand her need for self-accomplishment but the remains of traditional mentality still affect her
modern vision.
So our children should make a difference on gender roles perspective but we are in a vicious circle
because every family is raising their kids for a passed time, influenced by their own mentality and resistance
to change. The main idea is that both women and men should consider this role’s exchange like a challenge
and also accept that a man as well as a woman can do anything with willing.
208

Downloaded by Nhung Nguy?n (nhungnth2004@gmail.com)


lOMoARcPSD|20277325

1. What does the reading passage mainly discuss?


A. Children’s perspective on the discrimination.
B. The domination of men’s roles over women’s.
C. Gender roles in modern society.
D. The interior conflicts in women’s roles over men’s.
2. In the first paragraph, the phrase “separated world’s myth” refers to…………….
A. children’s opinions on their world which is also their family
B. children’s false view on the different and unexchangeable roles of their parents
C. children’s thoughts on their mothers’ domination roles over their fathers.
D. children’s ideas on what responsibilities their parents should be take charge of
3 . What is the main idea of the second paragraph?
A. Men’s and women’s roles cannot be easily exchanged since their lack of necessary skills.
B. In the development of society, men becomes more involved in household chores while women are more
active and independent.
C. The exchange between men’s and women’s roles turns out challenging because of an invisible default.
D. Society’s evolutiqn is unlikely to happen due to the constancy of men’s and women’s roles.
4. Which of the following words can best replace the word “anchored in” in the third paragraph?
A. conflicted with B. attached to C. accustomed to D. connected with
5. What does the word “one” in paragraph four refer to?
A. reprioritization B. success C. woman D. need
6. Which of the followings is NOT mentioned as one of the women’s needs for reprioritization?
A. The need to manage their own lives on their own earnings.
B. The need to show their abilities in order to be recognized by the public.
C. The need of integration into modern society.
D. The need to possibly bring something new.
7. In the fifth paragraph, what does the author imply by saying “she’s living an interior conflict as well”?
A. The remains of traditional mentality still affect women’s modern vision.
B. Women still do not know how to manage the conflicts with men.
C. The opposition from their husbands and children is so strong that women cannot overcome it.
D. Men and children always feel the stress with the domination of women’s roles.
8. The word “deteriorate” in the fifth paragraph can be best replaced by .
A. worsen B. reduce C. devastate D. decline
9. What does the writer advise people to do by this reading passage?
A. Children should be taught about their particular roles when they are young.
B. Men and women have to understand their roles with satisfaction.
C. Men and women roles’ exchange is impossible and children should understand it with willing.
D. Children should learn that society’s evolution is no good for men at all.
10. Who may possibly write this passage?
A. An anthropologist B. A historian C. A professor D. A psychologist
B. WRITTEN TEST
I. CLOZE TEST
OPEN CLOZE 1
Read the text below and think of the word which best fits each space.
Are you looking forward to another busy week? You should be according to some experts. The argue
that the stress encountered in our daily lives is not only goodfor us, but essential to survival. They say that the
response to (1)……………,which creates a chemical called adrenalin, helps the mind and body to act quickly
(2) …………… emergencies. Animals and human beings use it to meet the hostile conditions which exist on
the planet.
Whilst nobody denies the pressures of everyday life, what is surprising is that we are yet to develop
209

Downloaded by Nhung Nguy?n (nhungnth2004@gmail.com)


lOMoARcPSD|20277325

successful ways of dealing with them. (3) ……………the experts consider the current strategies to be
inadequate and often dangerous. They believe that (4) …………… of trying to manage our response to stress
with drugs or relaxation techniques, we must exploit it. Apparently, research shows that people (5) …………
create conditions of stress for (6) ……………by doing exciting and risky sports or looking for challenges,
cope much better with life’s problems. Activities of this type have been shown to create a lot of emotions;
People may actually cry or feel extremely uncomfortable. But there is a point (7) …………… which they
realise they have succeeded and know that it was a positive experience. This is because we learn through
challenge and difficulty. That’s (8) ……………we get our wisdom. Few of us, unfortunately, understand this
fact. For example, many people believe they (9) ……………from stress at work, and take time off as a result.
Yet it has been found in some companies that by far (10) ……………healthiest people are those with the most
responsibility. So next time you are in a stressful situation, just remember that it will be a positive learning
experience and could also benefit your health!
OPEN CLOZE 2
Read the text below and think of the word which best fits each space.
Science has told us so much (1) ……………the moon that it is fairly easy to imagine what it would be
like to go (2) ……………. It is certainly not a friendly place. As there is no air or (3) ……………, there
can be no (4) …………….of any kind. There is no variety of scenery (5) ……………. For mile after mile
there are only flat plains of dust with mountains around them. Above, the sun and stars (6) ……………in a
black sky. If you step out of the mountains shadows, it will mean moving from severe cold into great heat.
These extreme (7) ……………continually break rocks away from the surface of the mountains. The moon is
also a very silent world for sound waves can only travel (8) ……………air. But beyond the horizon, you see a
friendly sight, our earth is shining more brightly (9) ……………the stars from this distance, it looks (10) ……
an immense ball, coloured blue and green and brown.
II. WORD FORMATION
Part 1: Complete each sentence, using the correct form of the word in parentheses.
1. These remote areas are still…………….to the Internet. (ACCESS)
2. A popular…………….is that whale is a type of fish. (CONCEPT)
3 …………….might have proved that the suspect was at the scene of the crime. (FINGER)
4. Her car needs…………….at the garage. (SERVICE)
5. …………….may take place when you don’t speak English correctly. (UNDERSTAND)
6. The president…………….drew his conclusion from that ill-founded evidence. (ERR)
7. Dolphins, …………….species, sometimes jump above the surface of the water. (MAMMAL)
8. Don't bother to visit that part of the city with just…………….high-rise apartment blocks. (FACE)
9. She felt her husband constantly…………….her achievements. (LITTLE)
10. Good…………….is partly about getting on well with the customers. (SELL)
PART 2: Complete the passage with appropriate forms from the words given in the box.
consistent influence conserve else orchestra
direct ordain week act differ
Antonio Vivaldi, an Italian composer and a violinist, was the most (11)……………of his age. He was
bom March 4, 1678, in Venice, and was trained by his father, a violinist at Sa Mark’s Cathedral. (12)
……………a priest in 1703, Vivaldi began teaching that year at the Ospedale della Pietà, a (13) ……………
for orphaned girls. He was associated with the Pietà, usually as music (14) ……………, until 1740, training
the students, composing concertos and oratorios for (15) ……………. concerts, and meanwhile establishing
an international reputation. From 1713 on, Vivaldi was also (16) ……………as an opera composer and
producer in Venice and traveled to Rome, Mantua, and (17) ……………to oversee performances of his
operas. In about 1740 he accepted a position at the court of Empire Charles VI in Vienna. He died in Vienna
on July 28, 1741.
Vivaldi’s concertos provided a model for this genre throughout Europe, affecting the style even of his
older contemporaries. Vivaldi was the first composer who (18) ……………used the ritornello form that
210

Downloaded by Nhung Nguy?n (nhungnth2004@gmail.com)


lOMoARcPSD|20277325

became standard for the fast movements of concertos. The ritornello was a section that recurred in (19)
…………… keys arid was played by the full orchestra. It alternated with soloist- dominated sections
(episodes) that in his works were often virtuosic in character. He virtually established the three-movement
format for the concerto and was among the first to introduce cadenzas for soloists. His opus 8 concertos
entitled The Four Seasons are early examples of (20) ……………program music. Like much of his music,
they are marked by vigorous rhythms and strong contrasts.
III. ERROR IDENTIFICATION
The following passage contains 10 errors. Identify and correct them.
Ever since human have inhabited the earth, they have made use of various forms of communication.
Generally, this expression of thoughts and feelings have been in the form of oral speech. When there is the
language barrier, communication is accomplished through sign language in which motions stand in letters,
words, and ideas. Tourists, the deaf, or the mute have had to resort to this form of expression. Many of these
symbols of the whole words are very picturesque and exactly and can be used international, spelling, however,
cannot.
Body language transmitted ideas or thoughts by certain actions, either intentionally or unintentionally.
A wink can be a way of flirting or indicating that the party is only joking. A nod signifies approval, while
shaking the head indicates a negative reaction.
Other form of linguistic language can be found in Braille (a system of raised dots read with the
fingertips), signal flags, Morse code, and smoke signals. Road maps and picture signs also guide, warn, and
instruct people:
While verbalization is best common form of language, other systems and techniques also express
human thoughts and feelings.
IV. SENTENCE TRANSFORMATION
Rewrite the following sentences using the words given.
1. Jack doesn’t know all the answers, though he pretends to.
→ Jack acts………………………………………………………………………………………………
2. It was an impressive building but it wasn’t to my taste.
→ Impressive……………………………………………………………………………………………
3. We had not expected that we should have so much difficulty in finding the place.
→ The place…………………………………………………………………………………………….
4. I hardly think he will agree to give you a pay rise.
→ There is………………………………………………………………………………………………
5. How likely is it she will pass the exam?
→ What…………………………………………………………………………………………………
6. They have discovered some interesting new information (LIGHT)
→…………………………………………………………………………………………………………
7. They suspended Jack for the next two matches. (BANNED)
→…………………………………………………………………………………………………………
8. I really want to see her again. (DYING)
→…………………………………………………………………………………………………………
9. She was so beautiful that I couldn’t stop looking at her. (EYES)
→…………………………………………………………………………………………………………
10. She is a student good at mathematics. (FIGURES)
→…………………………………………………………………………………………………………
ĐÁP ÁN
ĐỀ THI OLYMPIC TRUYỀN THỐNG NĂM 2014
ĐỀ THI CHÍNH THỨC
A. MULTIPLE CHOICE
I. PHONOLOGY
211

Downloaded by Nhung Nguy?n (nhungnth2004@gmail.com)


lOMoARcPSD|20277325

l.D 2. A 3. A 4.C 5.D 6.C 7.D 8.B 9. A 10. B


II. WORD CHOICE
11.B 12. B 13. C 14. D 15. C 16. B 17.C 18. D 19. A 20. D
III. GRAMMAR AND STRUCTURES
21.A 22. A 23. C 24. C 25. A 26. D 27. D 28. B 29. B 30. C
IV. PHRASAL VERBS AND PREPOSITIONS
31.D 32. C 33. D 34. B 35. D 36. D 37. A 38. C 39.C 40. A
V. GUIDED CLOZE 1
41. D 42. C 43. A 44. B 45. C 46. A 47. B 48. C 49. C 50 A
GUIDED CLOZE 2
51. B 52. D 53.A 54. B 55. A 56. D 57. A 58. B 59. A 60. D
VI. READING PASSAGE 1
61. C 62. B 63. A 64. D 65.0 66. B 67. B 68. D 69. C 70. B
READING PASSAGE 2
71. A 72. C 73. B 74. C 75. C 76. B 77. D 78. A 79. B 80. C
B. WRITTEN TEST
I. CLOZE TEST
OPEN CLOZE 1
1. down 2. there 3. caused 4. decades 5. why 6. another
7. inherited/hereditary 8. introduce 9. vaccines 10. industry
OPEN CLOZE 2
11. highest 12. limited 13. popularity
14. general 15.although/though 16. privately
17. heavy 18. allowed/ permitted 19. however/though
20. interest
II. WORD FORMATION
PART 1:
1. futuristic 2. imperturbable 3. compatriot 4. buoyancy
5. incapacitating 6. enumerates 7. Budgetwise 8. predecessors
9. uprooted 10. ingeniously
PART 2
11. migratory 12. southerly 13. conservationist 14. uphill
15. underlay 16. originally 17. ornamental 18. exploitation
19. ultimately 20. forefront
III. ERROR CORRECTION
Line Error Line Error
4 in → at intervals 15 Nobody → None
5 what → that many creatures 16 go → went
10 creations → creatures 16 while → matter
11 hugely → largely 18 coverage → surface
13 healthful → healthy 19 air → temperature

IV. SENTENCE TRANSFORMATION


1. Richard only took over the family business because his father decided to retire early.
→ Had it not been for his father’s early retirement, Richard wouldn’t have taken over the family business.
2' We would always take great care when flying at night. (WITS)
→ We always used to have/keep our wits about us when flying at night.
3. There haven’t been such long queues at the cinema since the release of the last blockbuster.
→ Not since the release of the last blockbuster have there been such long queues at the cinema,
212

Downloaded by Nhung Nguy?n (nhungnth2004@gmail.com)


lOMoARcPSD|20277325

4. I will only be satisfied if the manager apologizes fully.


→ Nothing short of a full apology from the manager will satisfy me.
5. She was concentrating so hard on her work that she didn’t notice when I came in. (WRAPPED)
→ She was so wrapped up in her work that she didn’t notice when I came in.
6. The price of the house has been reduced as much as possible because the owner needs some money.
(BONE)
→ The house has been cuưpared to the bone because the owner needs some money?
7. The permit expires at the end of the month.
→ The permit is not valid after the end of the month.
8. The first sign of the disease is blurred vision. (ONSET)
→ The onset of the disease is (signaled/marked by) blurred vision.
9. Don’t say anything negative about her hair because she’s very sensitive and might be offended by your
remarks. (OFFENCE)
→ Don’t say anything negative about her hair because she’s very sensitive and might take offence at your
remarks.
10. The new teacher was so nervous that the class reacted mischievously. (REACTION)
→ The new teacher’s nervousness drew mischievous reaction(s) from the class.

TRƯỜNG THPT CHUYÊN LÊ HỒNG PHONG - TP. HCM


A. MULTIPLE CHOICE
I. PHONOLOGY:
1.C 2.D 3.A 4.B 5.A 6.B 7.C 8.D 9.B 10. B
II. VOCABULARY AND STRUCTURE
11.B 12.C 13. D 14. A 15. D 16.A 17. C 18.B 19. A 20. D
21.A 22. B 23. D 24. B 25. A 26. D 27. B 28.A 29. B 30. A
31.D 32. C 33. D 34. D 35. A 36. B 37. C 38.C 39. D 40. B
III. READING COMPREHENSION
PASSAGE 1
l. B 2. A 3.D 4. A 5.B 6.D 7. B 8.B 9.B 10. D
PASSAGE 2
l.C 2. A 3.C 4. D 5. A 6.D 7. C 8. C 9. A 10. B
IV. GUIDED CLOZE:

GUIDED CLOZE 1:
l.A 2. D 3.C 4.B 5. A 6.D 7.D 8.B 9. C 10. B
GUIDED CLOZE TEST 2
l.B 2.C 3.C 4.C 5. A 6.B 7.C 8.B 9. D 10. A
B. WRITTEN TEST
I. CLOZE TEST:
PASSAGE 1
l.only 2. despite 3. with 4. fitting 5. teamed/joined
6. dressed 7. keyed 8. best 9. rate
10. doubt/doubting/question/denying
PASSAGE 2
1. expectancy 2. heightened 3. long 4. bind 5.role
6. where 7. conditions 8.demand 9. fetch 10. high
II. WORD FORMATION:
Part 1:
1. airborne 2. discontentedly 3. Precancerous 4. housebound
213

Downloaded by Nhung Nguy?n (nhungnth2004@gmail.com)


lOMoARcPSD|20277325

5. roadworthy 6. weather-beaten 7. well-groomed 8. hands-off


9. enmeshed 10. spendthrift
Part 2
1. heady 2. unknowns 3. operatic 4. backers
5. outgrow 6. wide-ranging 7. encompassing 8. focal
9. confrontation 10. finesse
III. ERROR IDENTIFICATION:
Line Correction Line Correction
1. 2 got → jumped 6. 12 to question → question
2. 5 supplying → supply 7. 13 famously → infamously/notoriously
3. 6 manufacturing → manufacture 8. 14 in → on
4. 8 usage → use 9. 14 a → the
5. 11 cynicals → cynics 10. 21 change → changing

IV. SENTENCE TRANSFORMATION:


1. Having common sense doesn’t mean that you are highly intelligent
2. The audience didn’t go down well with his performance.
3. Upset as she was, revenge was the last thing on her mind.
4. On no occasion should you leave the iron switched on for a Jong time.
5. You are at liberty to leave as you wish.
6. That jacket of yours has seen its better days - isn’t it time you bought a new one?
7. Joe waited for her under the pretence of tying his shoelaces.
8. She didn’t make a noise lest she wake her parents.
9. Injury put him out of contention for the title.
10. To the best of my knowledge, the project is nearing completion.

TRƯỜNG THPT CHUYÊN TRÀ VINH - TRÀ VINH


PART A : MULTIPLE CHOICE.
QUESTION 1: PHONOLOGY
1. C 2. B 3. A 4. C 5. A 6. A. 7. B 8.B 9. A 10. B
QUESTION 2: VOCABULARY
l. B 2. C 3.D 4. A 5. C 6. A 7. B 8. D 9. A 10. C
QUESTION III: GRAMMAR AND STRUCTURES
l.A 2. A 3.C 4. B 5. D 6. B 7.A 8. C 9. B 10. B
QUESTION IV: PHRASAL VERBS AND PREPOSITIONS
l.C 2.D 3.C 4. A 5. B 6. C 7. C 8. A 9.C 10. A
QUESTION V
Passage 1:
l.D 2.D 3. B 4. A 5.B 6. C 7.D 8. A 9.C 10. B
Passage 2 :
l.D 2. A 3.C 4.A 5.B 6.C 7.D 8.C 9.C 10. B
GUIDED CLOZE TEST
Passage 1
l.D 2. B 3.C 4. A 5.C 6.C 7.D 8.B 9.B 10. D
Passage 2
l.A 2.B 3. D 4. A 5. A 6.C 7.D 8.D 9. A 10. A
PART TWO: WRITTEN TEST
QUESTION 1: WORD FORMATION

214

Downloaded by Nhung Nguy?n (nhungnth2004@gmail.com)


lOMoARcPSD|20277325

Passage 1:
1. scarcity 2. counterproductive 3. preparedness
4. neighborly 5. Self- explanatory 6. indestructible
7. disciplinary 8. inconsiderate 9. differentiate 10. Fresh water
Passage 2:
11. invariably 12. pressure 3. excessive 14. essential 15. retailers
16. centrally 7. Illogical 18. unavoidable 19. critism(s)
20. efficiency
QUESTION 2: ERROR IDENTIFICATION
1. Line 1: to → for 2. Line 1: does → is
3. Line 2: a same → the same 4. Line 3: in that → in which
5. Line 5: many a → such a 6. Line 6: expressing → expressed
7. Line 8: requires → requiring 8. Line 10: interpreting that → interpreting what
9. Line 11: the one → the ones 10. Line 13: another → other.
QUESTION III: OPEN CLOZE TEST
Passage 1:
1. creatures 2. habitat 3.changed 4. land 5. harmless
6. actions 7. increased 8. process 9. exhaust 10. atmosphere
Passage 2
1. until 2. areas 3. ring 4. pacemaker 5. interfere
6. solve 7. passengers 8. silent 9. lose 10. turn
IV. SENTENCE TRANSFORMATION
1. At no time did the two sides look likely to reach an agreement.
Or at no time was there any likelihood of the two sides reạching an agreement.
2. Further information can be obtained by sending a self - addressed envelope to the above address.
3. But for his faher’s early retirement, Richard would not have taken over the family business.
4. With six children on her hands, she is extremely busy.
5. The use of the main college car park is restricted to final-year students.
6. The manager has come up against quite a few problems with the new computer systems.
7. We agreed that each of us would take turns to do this washing- up.
8. It’s high time you dressed/ got dressed yourself1
9. Could you put me up in Chicago for a few days John?
10. I wasn’t in the mood for doing / to do anything energetic.

TRƯỜNG THPT CHUYÊN HUỲNH MẪN ĐẠT - KIÊN GIANG


I. MULTIPLE CHOICE
1. PHONOLOGY
l.A 2. A 3. D 4. D 5. B 6. B 7. B 8. D 9. A 10. C
2. WORD CHOICE
11.A 12. D 13. A 14. B 15. B 16. A 17. C 18. D 19. C 20. A
3. STRUCTURES AND GRAMMAR
21.B 22. C 23.A 24. B 25. D 26. B 27. A 28. D 29. C 30. B
4. PREPOSITION AND PHRASAL VERB
31. A 32. A 33. D 34. A 35. D 36. A 37. B 38. A 39. A 40. A
5. READING COMPREHENSION
A. 41.D 42. B 43. B 44. A 45. B 46. A 47. C 48. C 49. A 50. D
B. 51. E 52. G 53. I 54. A 55. F 56. D 57. J 58. C 59. H 60. B
6. GUIDED CLOZE TEST
A. 61.C 62. D 63. B 64. C 65. B 66. C 67. C 68. A 69. D 70. A
215

Downloaded by Nhung Nguy?n (nhungnth2004@gmail.com)


lOMoARcPSD|20277325

B. 71.C 72. B 73. C 74. C 75. A 76. A 77. A 78. A 79. B 80. A
II. WRITTEN TEST
1. OPEN CLOZE TEST
A.
1. Brazil 2. participant 3. unwillingness 4. boycotted 5. Portuguese
6. absorbed 7. minority 8. Oceania 9. outside 10. semi-finalists
B.
1. characterized 2. cartilaginous 3. rays 4. lantemshark 5. largest
6. depths 7. seawater 8. denticles 9. replaceable 10. predator
2. WORD FORM
A.
1. chocoholic 2. matriarchy 3. cardiograph 4. appendectomy
5. equidistant 6. feet-first 7. Wellingtoniana 8. disheartened
9. non-appearance 10. maladministration
B.
1. mathematicians 2. length 3. thickness 4. reference 5. philosophical
6. particularly 7. geometry 8. dimensional 9. similarly 10. master
3. ERROR IDENTIFICATION
Error flopping on also in spite of in rest so
Correct flopping down nor in terms of at rest as
Error Having relaxed More than today feeling alone That
Correct Being relaxed rather than today’s let alone What
4. SENTENCE TRANSFORMATION
1. Were I not to have helped you, you would have been in trouble with your girlfriend.
2. Had he been present, she wouldn’t have been happy, nor would he have been welcomed.
3. It is not always thought essential that Miss World must have the great appearance.
4. These talkative girls must be chatting about my problem.
5. He is a thorn in my flesh.
6. I’m sorry, James, but I’m up to my neck in work.
7. You will fall on deaf ears if you continue to be stuck-up.
8. Our dependence on overseas trade puts drive and resourcefulness in export industries át a premium.
9. The audience ran away with the idea that economic recovery was close at hand.
10. I decided to go home and spruce myself up -to let everyone know that life had not got me down.
TRƯỜNG THPT CHUYÊN LÝ TỰ TRỌNG – CẦN THƠ
A. MULTIPLE CHOICE
I. PHONOLOGY
l.B 2. A 3.C 4.D 5. A 6.D 7.B 8.B 9.0 10. A
II. WORD CHOICE
1.D 2. A 3.B 4.D 5.D 6.D 7.B 8.D 9. D 10. B
III. GRAMMAR AND STRUCTURES
l.C 2.D 3.D 4. A 5.D 6.C 7.D 8.C 9. A 10. B
IV. PHRASAL VERBS AND PREPOSITIONS
1.A 2.B 3.D 4.B 5.B 6.C 7.C 8. A 9.B 10. A
V. GUIDED CLOZE 1
1.A 2.B 3.B 4.D 5.C 6.D 7.C 8.D 9.C 10. A
VI. GUIDED CLOZE 2
1.A 2.D 3.B 4.C 5. A 6.D 7.B 8.B 9.D 10. A
VII. READING PASSAGE 1
1. C 2. A 3.C 4.B 5.C 6.D 7.D 8. A 9. A 10. C
216

Downloaded by Nhung Nguy?n (nhungnth2004@gmail.com)


lOMoARcPSD|20277325

VIII. READING PASSAGE 2


l.D 2.D 3.C 4.C 5.D 6. C 7. B 8.D 9.B 10. A
B. WRITTEN TEST
1. CLOZE TEST
OPEN CLOZE 1
1. dealt 2. put 3. fair 4. it 5. on
6. breathing 7. pull 8. get 9. being 10. think
OPEN CLOZE 2
1.in 2. own 3. by 4. our 5. few
6. in 7. where 8. all 9. instead 10. among
II. WORD FORMS
Part 1:
1. incapacitated 2. decriminalise 3. ungovernable 4. demoralized
5. impersonating 6. unenviable 7. anticlimax. 8. unsubstantiated
9. apportion 10. desensitize
Part 2:
1. enduring 2. revolutionised 3. alternative 4. replacement
5. capability (ies) 6. processor 7. electricity 8. modernise
9. persistently 10. discontinued
III. ERROR CORRECTION
HOW TO BE A BETTER FRIEND Instead of
Instead giving endless advice, learn to listen more. Listening is an 1. underestimated
underestimating skill, and it is easy to forget when you are worrying about 2. people’s
other people problems. How many times do we say “Oh yeah, that happened 3.√
to me and...” before we give the other persons a chance to explain what 4. √
happen to them? Sometimes, a friend may just want to talk something about 5. happened
with someone else to sort things out in their mind - so listen hardly to what 6. over
they are saying, and try to offer advices only when you think they are asking 7. hard
for it 8. advice
We all feel we’ve been let down by a friend at some points in our lives. 9. √
Perhaps they let out a secret we trusted them to keep under wraps, or sudden 10. point
sided with the opposition during an argument. Nobody’s perfect, so try to 11. √
have realistic expectation. Friendships don’t develop overnight; they deepen 12. suddenly
over time as you begin to trust one other. Don’t place, unrealistic demands on 13. √
your friendship. 14. expectations
15. √
16. one another / each other
IV. SENTENCE TRANSFORMATION
1. Playing backgammon with Paul failed to/ didn’t take my mind off the exam.
2. It isn’t like Josh to tell lies,’ said Pete.
3. Retirement will free me up to spend mòre time with my grandchildren.
4. Oscar’s Russian degree qualifies him to teach the language.
5. I’m glad you have now come to your senses and agree that your parents are right.
6. I didn’t tell Angie out of consideration for Eddie.
7. You have to strike a balance between logic and lateral thinking in this job.
8. Ralph would hand in his resignation at the drop of a hat if he could find a better job.
9. That’s the girl whose parents are up in arms about her expulsion.
10. We always used to have/ keep our wits about us when flying at night.

217

Downloaded by Nhung Nguy?n (nhungnth2004@gmail.com)


lOMoARcPSD|20277325

TRƯỜNG THPT CHUYÊN THOẠI NGỌC HẦU - AN GIANG


PHẦN TRẮC NGHIỆM (Multiple choice questions)
CÂU 1: PHONOLOGY
l.D 2. C 3. A 4. B 5.D 6. A 7.D 8.C 9.B 10. D
CÂU 2: VOCABULARY
l.D 2. A 3. D 4. A 5.B 6.B 7.C 8. A 9. A 10. C
CÂU 3: GRAMMAR AND STRUCTURES
l.D 2.C 3.B 4.D 5. D 6.C 7.C 8.D 9.C 10. A
CÂU 4: PREPOSITIONS AND PHRASAL VERBS
l.B 2.C 3. B 4.B 5.C 6. A 7.B 8.C 9.B 10. A
CÂU 5: READING COMPREHENSION
Passage A:
l.B 2. A 3.C 4.C 5.B 6. A 7.C 8.D 9. D 10. C
Passage B:
l.A 2.C 3. A 4.B 5.D 6. A 7.C 8. A 9. A 10. C
CÂU 6: CLOZE TEST
Passage A:
l.C 2.D 3.D 4.C 5.D 6. A 7.B 8.C 9.C 10. A
Passage B:
l.B 2.C 3.B 4. D 5.D 6. A 7. A 8.C 9.B 10. C
PHẦN TỰ LUẬN (Written Questions)
CÂU 1:
Passage A
1. into 2. species 3. as 4. which 5. land
6. hive 7. called 8. eggs 9. Sting 10. eating
Passage B
11. time 12.or 13. it 14. language 15. mood.
16. chest 17. while 18.of 19. treated 20. influenced
CÂU 2:
A.
1. The government is taking care not to rush HEADLONG into another controversy. (head)
2. SUBCONSCIOUSLY, she was looking for the father she had never known. (conscious)
3. She added two SPOONFULS of sugar to the soup. (spoon)
4. All letters will be treated with complete CONFIDENTIALITY (confidential)
5. The V8 engines are all INTERCHANGEABLE with each other. (change)
6. The painting was sold to an UNIDENTIFIED American dealer. His name was not given. (identify)
7. They ưied to ensure UNIFORMITY across the different departments. (uniform)
8. The building did not fit to live in. It was totally UNINHABITABLE. (habitat)
9. This year saw a CONTINUATION in the upward trend in sales. (continue)
10. Claire has a wide circle of friends and ACQUAINTANCES (acquaint)
B.
11. beliefs 12. observations 13. concept 14. knowledgeable
15. continuing 16. belittle 17. documentation
18. planetary/ planet 19. discoveries 20. irreplaceable
CÂU 3:
1. Line 3: means → meant 2. Line 5: therefore → however
3. Line 7: has → have 4. Line 10: was seen → saw
5. Line 11: potentially → potential 6. Line 13: come → came
7. Line 17: considered → considerable 8. Line 18: with → over
218

Downloaded by Nhung Nguy?n (nhungnth2004@gmail.com)


lOMoARcPSD|20277325

9. Line 21: their → its 10. Line 25: adopting → to adopt


CÂU4:
A.
1. You cannot say anything that would persuade me to live overseas.
→ Nothing you soy would persuade me to live overseas.
2. The chairman’s leaving just before you re due to arrive.
→ By the time you arrive, the chairman will have (just) left.
3. It was difficult to understand her colleagues’ open hostility towards her proposal.
→ That her colleagues were so openly hostile towards her proposal was difficult to understand.
4. The club owner became a media celebrity, as well as extremely rich.
→ Not only did the club owner become a media celebrity, but he was also extremely rich.
→ Not only was the club owner extremely rich, but he also became a media celebrity.
5. David played the main role when the proposal was drafted.
→ David was instrumental in the drafting of/ in drafting the proposal.
B.
6. If you hadn’t changed our original agreement, everything would have been fine. (stuck to)
→ Had you stuck to/ by what we originally agreed, everything would have been fine.
Or: If you had stuck to…..
7. I think you should have some consideration for those who don’t have lives at privilege as yours. (spare)
→ I think you should spare a thought for those/ (the) people whose lives aren’t as privileged as yours.
Or: those who don’t have lives at privilege as yours.
8. We decided to stay longer because we were so thrilled by the place. (extend)
→ We decided to extend our stay as we were so thrilled by the place.
9. Mai felt entirely comfortable when her boss was around (ease)
→ Mai felt entirely at ease with/ in the presence of her boss.
10. John found it difficult to get used to the fact that he was fired, (terms)
→ John found it difficult to come to terms with the fact that he had lost the job.

TRƯỜNG THPT CHUYÊN THĂNG LONG - ĐÀ LẠT LÂM ĐỒNG


A. MULTIPLE CHOICE
I. PHONOLOGY
A. 1.C 2.D 3.D 4. B 5.A
B. 1.A 2.C 3.C 4.D 5.A
II. WORD CHOICE
1.A 2.D 3.B 4. B 5. B 6. A 7. A 8.B 9.D 10. C
III. GRAMMAR AND STRUCTURES
l. D 2. B 3. A 4.C 5.B 6. C 7. D 8.B 9.D 10. D
IV. PHRASAL VERBS AND PREPOSITIONS
l.D 2.C 3. D 4. A 5.B 6. A 7. D 8.B 9D 10. A
V. READING COMPREHENSION
Reading passage 1
1. B 2. B 3. A 4.C 5. D 6.C 7. B 8. A 9.D 10. D
Reading passage 2
1.D 2.C 3.C 4.A 5.C 6.B 7. B 8. C 9. A 10. D
VI. GUIDED CLOZE TEST
Guided cloze 1
l.B 2. A 3.B 4. D 5. C 6.D 7.C 8.C 9.D 10. A
Guided Cloze 2
l.A 2.D 3. B 4.B 5. A 6.D 7. A 8. A 9.D 10. C
219

Downloaded by Nhung Nguy?n (nhungnth2004@gmail.com)


lOMoARcPSD|20277325

B. WRITTEN TEST
I. OPEN CLOZE TEST
Open cloze 1
1. self 2. human 3. function 4. reduction 5. pain/ stress
6. shed 7. serve / help 8. mainly 9. intense 10. harmful
Open cloze 2
1. true 2. to 3. few 4. exception 5. likely
6. until 7. frequency 8. upper 9. to 10. but
III. WORD FORMATION
PART 1:
1. continually 2. headstrong 3. technicalities 4. nonsensical 5. anew
6. racist 7. preparedness 8. exprisoners 9. accreditation 10. prejudicial
PART 2:
1. successive 2. inundated 3. dusty 4. metaphysical 5. visions
6. disposed 7. critically 8. artfulness 9. aesthetically 10. skillfully
III. ERROR IDENTIFICATION
1. attached → joined 2. does → makes
3. opponent → opponent’s 4. up → to
5. from → in 6. it → which
7. in → at 8. sorely → only
9. purposeless → purposeful 10. them → it
IV. SENTENCE TRANSFORMATION
1. Due to an increase in the number of robberies, the police are advising vigilance.
2. Had it not been for Nick’s advice, I would have gone bankrupt.
3. Failure to observe the company's dress code could/ may/ might lead to/ result in your dismissal.
4. She dissuaded me from writing it all out in longhand (and told me to use a typewriter).
5. I cast my mind back to what has happened all those years before.
6. David put his success down to incredible luck.
7. Spare a/ some thought for those who are at work on this lovely, sunny day!
8. There are all manner of tourist attractions in this part of the country.
9. Her latest novel isn't on a par with her previous one.
10. The Rainbow Disco is out of bounds to students at the school.
TRƯỜNG THPT CHUYÊN LƯƠNG VĂN CHÁNH - PHÚ YÊN
I. PHONOLOGY
A. 1. A 2. B 3. C 4.C 5. D
B. 1. D 2. B 3. D 4. B 5. D
II. WORD CHOICE
1.A 2. C 3. C 4.A 5. D 6. D 7. A 8. B 9. B 10. A
III. GRAMMAR AND STRUCTURES
l.D 2. D 3. A 4. B 5. A 6. A 7. D 8.C 9. D 10. A
IV. PREPOSITIONS AND PHRASAL VERBS
1.B 2. B 3.C 4. A 5. D 6. A 7. A 8. D 9.C 10. A
V. READING COMPREHENSIONS.
Passage 1:
l.C 2. A 3.D 4. C 5.C 6. D 7. A 8. B 9. A 10. B
Passage 2:
1.A 2B 3. A 4.B 5. D 6. A 7, D 8. D 9.C 10. D
VI. CLOZE TEST.
A. 1.B 2. C 3.D 4.C 5. A 6. B 7. C 8. C 9. B 10. D
220

Downloaded by Nhung Nguy?n (nhungnth2004@gmail.com)


lOMoARcPSD|20277325

B. 1.B 2. C 3.C 4. A 5. D 6. C 7 D 8. B 9. A 10. D


VII. OPEN CLOZE TEST
Passage 1
1. as 2. others 3. despite 4. not 5. After
6. regardless/ irrespective 7. so 8. itself 9. used 10. in
Passage 2
11. amazing 12. strong 13. heavy 14. safely 15. sharp
16. silently 17. alone 18. well 19. everywhere 20. rare
VIII. WORD FORM
A.
1. fraudulent 2. befriend 3. disinterested 4. belied
5. imperceptibly 6. survivability 7. gravitational
8. mileometer/milometer 9. sobriety 10. triumphant
B.
11. overpopulated 12. fallout 13. coastal 14. deleterious
15. interference 16. polluting 17. inherent 18. usefulness
19. unlimited 20. reservoir
IX. ERROR RECOGNITION.
1. knowledgment → knowledge 2. potentially → potential
3. deforestion → deforestation 4. erosive → erosion
5. inreliable → unreliable 6. threatening → threatened
7. lives → live 8. serious → seriously
9. survivors → survival 10. ingenious → indigenous
X. SENTENCE TRANSFORMATION.
A.
1. Winning that prize has gone to his head.
2. The prisoner was recaptured as he made a dash for the gate.
3. Before you leave the office today, please cast an eye over the new contract
4. The conflict between the two political parties is by no means over.
5. It is beyond my comprehension why you quit your job when you haven’t got another one.
B.
1. Since the company’s methods were exposed in a newspaper, it has fallen into disrepute
2. If I can be of any assistance/ be of assistance, please don't hesitate to ask.
3. The police wouldn’t have interviewed him unless he had been witness to/had been a witness to.
4. All of a sudden there was loud applause from the audience.
5. The exhibition has not been so well attended this year.

TRƯỜNG THPT CHUYÊN NGUYỄN BỈNH KHIÊM


VĨNH LONG
A. MULTIPLE CHOICE QUESTIONS
I. PHONOLOGY:
1.B 2. D 3. B 4. C 5. A 6. B 7. A 8.D 9. D 10. C
II. WORD CHOICE
l.A 2. D 3. B 4. A 5.C 6. C 7.D 8.C 9. C 10. C
III. STRUCTURES & GRAMMAR
l.D 2. A 3.C 4.B 5.C 6.C 7.C 8. A 9.C 10. A
IV. PREPOSITIONS AND PHRASAL VERBS
l.A 2. C 3.B 4. C 5. A 6. C 7.C 8.B 9.B 10. A
221

Downloaded by Nhung Nguy?n (nhungnth2004@gmail.com)


lOMoARcPSD|20277325

V. READING COMPREHENSION
Passage 1:
l.A 2. D 3. B 4. A 5.D 6.D 7. A 8. C 9.D 10. C
Passage 2:
1.D 2.C 3. A 4. B 5.C 6. D 7.B 8.A 9.B 10. A
VI. CLOZE TEST
CLOZE TEST 1
1. D 2.B 3. A 4. B 5. A 6.C 7. A 8. B 9.D 10. C
CLOZE TEST 2
1.D 2. A 3. C 4.D 5. A 6. A 7.C 8.A 9.B 10. C
B. WRITTEN TEST
I. OPEN CLOZE TEST
Cloze test 1
1. what 2. bring 3.it 4. since
5. By/ Through 6. handed/ passed 7. no
8. despite/ notwithstanding 9. whose 10. such
Cloze test 2
1. put 2. become 3. make 4. there 5. Although/ Though/ While/ Whilst
6. on 7. one 8. view 9. such 10. a
II. WORD FORMS
1.
1. photogenic 2. continuity 3. mid- sentence 4. domineering
5. occurrence 6. troublesome 7. dependant 8. indistinguishable
9. publication 10. underestimated
2.
1. recognizable/recognisable 2. ripens 3. maturity
4. extensively 5. unknown 6. suppliers 7. competitors
8. consequently 9. emergence 10. threatening
III. ERROR RECOGNITION AND CORRECTION:
1. visiting → to visit 2. no → not
3. right address → the right address 4. their → its
5. up → down 6. had → had been
7. risen → raised 8. extending → extended
9. previous previoujy 10. untidy gravel yard → an untidy gravel yard
IV. SENTENCE TRANSFORMATION
1. Be sure not to leave without saying good bye to your grandmother.
2. They can’t leave the dog behind when they go on holiday.
3. No fewer than four helpers are needed to enable the charity shop to stay open.
4. The speed at which the motorcyclist rounded the comer was quite unbelievable.
5. I’ll keep my fingers crossed (for you) on the day of your interview.
6. Don’t make á mountin'out of a molehill.
7. The mistake in the accounts only came to light when / came to light after / came to light once the figures
were re-checked.
8. Sally came up with the best solution.
9. My daughter has grown out of that jumper you knitted for her.
10. They put the blame on me for the accident.

TRƯỜNG THPT CHUYÊN VỊ THANH - HẬU GIANG


A. MULTIPLE CHOICE
222

Downloaded by Nhung Nguy?n (nhungnth2004@gmail.com)


lOMoARcPSD|20277325

I. PHONOLOGY
l.D 2.D 3. D 4.D 5.D 6.D 7.C 8. A 9. A 10. A
II. WORD CHOICE
l.C 2. A 3. B 4.C 5.D 6.C 7.C 8. A 9. B 10. A
III. GRAMMAR AND STRUCTURES
l.A 2.C 3. A 4.C 5.D 6.B 7.C 8.C 9.B 10. A
IV. PHRASAL VERBS AND PREPOSITIONS
l.A 2.C 3.B 4.D 5.B 6.C 7. B 8. A 9.D 10. C
V. GUIDED CLOZE 1
l.C 2.B 3. D 4. A 5.C 6.B 7.A 8.D 9. B 10. C
VI. GUIDED CLOZE 2
1.D 2. B 3.C 4.A 5. D 6.C 7. A 8.B 9. A 10. A
VII. READING PASSAGE 1
l.C 2. A 3.C 4.D 5. A 6.C 7.C 8.B 9.C 10. B
VIII. READING PASSAGE 2
l.C 2.B 3.C 4.D 5.B 6. D 7. A 8. A 9. B 10. A
B. WRITTEN TEST
1. CLOZETEST
OPEN CLOZE 1
1. stress 2. in/during 3. Even 4. instead 5. who/that
6. themselves 7. at 8. how 9. suffer 10. the
OPEN CLOZE 2
1. about 2. there 3. water 4. life 5. either
6. shine 7. temperature 8. through 9. than 10. like
II. WORD FORMATION
PART 1:
1. inaccessible 2. misconception 3. fingerprint 4. servicing
5. Misunderstanding 6. erroneously 7. mammalian 8. faceless
9. belittled 10. salesmanship
PART 2:
11. influential 12. Ordained 13. conservatory 14. director 15. weekly
16. active 17. elsewhere 18. consistently 19. different 20. orchestral
III. ERROR IDENTIFICATION
L3: 1. have → has
L3: 2. there is the → there is a
L4 : 3. stand in stand for
L5 : 4. the deaf, or the mute → the deaf, and the mute
L7 : 5. exactly → exact, 6. international → internationally
L8: 7. transmitted → transmits.
L12: 8, 9 : form of linguistic → forms of nonlinguistic
L15: 10. best common → most common
IV. SENTENCE TRANSFORMATION
1. Jack acts as if / as though he knew all the answers.
2. Impressive as the building was/might be, it wasn’t to my taste.
3. The place was much more difficult to find than we had expected.
4. There is little likelihood of her giving you a pay rise.
5. What are her chances of passing the exam?
6. Some interesting new information has come to light.
7. Jack was banned from playing in the two matches.
223

Downloaded by Nhung Nguy?n (nhungnth2004@gmail.com)


lOMoARcPSD|20277325

8.I’m dying to see her again.


9. She was so beautiful that I couldn’t take my eyes off her.
10. She is a student having a head for figures.

224

Downloaded by Nhung Nguy?n (nhungnth2004@gmail.com)

You might also like